ΠšΠ°ΡΠ°Ρ‚Π΅Π»ΡŒΠ½Π°Ρ ΠΊ Π³Ρ€Π°Ρ„ΠΈΠΊΡƒ Ρ„ΡƒΠ½ΠΊΡ†ΠΈΠΈ ΠΏΠ°Ρ€Π°Π»Π»Π΅Π»ΡŒΠ½Π° прямой: ΠšΠ°ΡΠ°Ρ‚Π΅Π»ΡŒΠ½Π°Ρ ΠΏΠ°Ρ€Π°Π»Π»Π΅Π»ΡŒΠ½Π° прямой

Π‘ΠΎΠ΄Π΅Ρ€ΠΆΠ°Π½ΠΈΠ΅

ΠšΠ°ΡΠ°Ρ‚Π΅Π»ΡŒΠ½Π°Ρ. Π—Π°Π΄Π°Ρ‡ΠΈ Π½Π° ΠΊΠ°ΡΠ°Ρ‚Π΅Π»ΡŒΠ½ΡƒΡŽ | Π‘Ρ‚Π°Ρ‚ΡŒΡ Π² ΠΆΡƒΡ€Π½Π°Π»Π΅ «Молодой ΡƒΡ‡Π΅Π½Ρ‹ΠΉΒ»

Π§Ρ‚ΠΎΠ±Ρ‹ ΠΏΡ€Π°Π²ΠΈΠ»ΡŒΠ½ΠΎ ΠΈ Ρ€Π°Ρ†ΠΈΠΎΠ½Π°Π»ΡŒΠ½ΠΎ Ρ€Π΅ΡˆΠ°Ρ‚ΡŒ Π·Π°Π΄Π°Ρ‡ΠΈ, связанныС с ΡƒΡ€Π°Π²Π½Π΅Π½ΠΈΠ΅ΠΌ ΠΊΠ°ΡΠ°Ρ‚Π΅Π»ΡŒΠ½ΠΎΠΉ, Π½ΡƒΠΆΠ½ΠΎ Ρ‡Π΅Ρ‚ΠΊΠΎ ΠΏΠΎΠ½ΠΈΠΌΠ°Ρ‚ΡŒ, Ρ‡Ρ‚ΠΎ Ρ‚Π°ΠΊΠΎΠ΅ ΠΊΠ°ΡΠ°Ρ‚Π΅Π»ΡŒΠ½Π°Ρ, Π²Π»Π°Π΄Π΅Ρ‚ΡŒ Ρ‚Π΅Ρ…Π½ΠΈΠΊΠΎΠΉ составлСния уравнСния ΠΊΠ°ΡΠ°Ρ‚Π΅Π»ΡŒΠ½ΠΎΠΉ ΠΊ Π³Ρ€Π°Ρ„ΠΈΠΊΡƒ Ρ„ΡƒΠ½ΠΊΡ†ΠΈΠΈ ΠΈ ΠΏΡ€Π΅Π΄ΡΡ‚Π°Π²Π»ΡΡ‚ΡŒ сСбС, для Ρ€Π΅ΡˆΠ΅Π½ΠΈΡ ΠΊΠ°ΠΊΠΈΡ… Π·Π°Π΄Π°Ρ‡ (Π² Ρ‚ΠΎΠΌ числС ΠΈ Π·Π°Π΄Π°Ρ‡ с ΠΏΠ°Ρ€Π°ΠΌΠ΅Ρ‚Ρ€Π°ΠΌΠΈ) ΠΌΠΎΠΆΠ½ΠΎ ΠΈΡΠΏΠΎΠ»ΡŒΠ·ΠΎΠ²Π°Ρ‚ΡŒ ΠΌΠ΅Ρ‚ΠΎΠ΄ ΠΊΠ°ΡΠ°Ρ‚Π΅Π»ΡŒΠ½ΠΎΠΉ.

ΠžΠΏΡ€. 1. ΠšΠ°ΡΠ°Ρ‚Π΅Π»ΡŒΠ½ΠΎΠΉ ΠΊ Π³Ρ€Π°Ρ„ΠΈΠΊΡƒ Ρ„ΡƒΠ½ΠΊΡ†ΠΈΠΈ Ρƒ = f(x) называСтся ΠΏΡ€Π΅Π΄Π΅Π»ΡŒΠ½ΠΎΠ΅ ΠΏΠΎΠ»ΠΎΠΆΠ΅Π½ΠΈΠ΅ сСкущСй MN ΠΏΡ€ΠΈ (рис. 1).

Рис. 1

ΠšΠ°ΡΠ°Ρ‚Π΅Π»ΡŒΠ½Π°Ρ ΠΊ ΠΊΡ€ΠΈΠ²ΠΎΠΉ ΠΌΠΎΠΆΠ΅Ρ‚ ΠΈΠΌΠ΅Ρ‚ΡŒ с Π½Π΅ΠΉ нСсколько ΠΎΠ±Ρ‰ΠΈΡ… Ρ‚ΠΎΡ‡Π΅ΠΊ ΠΈΠ»ΠΈ ΠΏΠ΅Ρ€Π΅ΡΠ΅ΠΊΠ°Ρ‚ΡŒ Π΅Π΅. МоТно Π΄Π°Ρ‚ΡŒ ΠΈ Π΄Ρ€ΡƒΠ³ΠΎΠ΅ ΠΎΠΏΡ€Π΅Π΄Π΅Π»Π΅Π½ΠΈΠ΅ ΠΊΠ°ΡΠ°Ρ‚Π΅Π»ΡŒΠ½ΠΎΠΉ ΠΊ ΠΊΡ€ΠΈΠ²ΠΎΠΉ.

ΠžΠΏΡ€. 2. ΠšΠ°ΡΠ°Ρ‚Π΅Π»ΡŒΠ½ΠΎΠΉ ΠΊ Π³Ρ€Π°Ρ„ΠΈΠΊΡƒ Ρ„ΡƒΠ½ΠΊΡ†ΠΈΠΈ Ρƒ = f(x) Π² Ρ‚ΠΎΡ‡ΠΊΠ΅ A0(x0; f(x0)) называСтся прямая, проходящая Ρ‡Π΅Ρ€Π΅Π· Ρ‚ΠΎΡ‡ΠΊΡƒ

A0, ΡƒΠ³Π»ΠΎΠ²ΠΎΠΉ коэффициСнт ΠΊΠΎΡ‚ΠΎΡ€ΠΎΠΉ Ρ€Π°Π²Π΅Π½ Π·Π½Π°Ρ‡Π΅Π½ΠΈΡŽ ΠΏΡ€ΠΎΠΈΠ·Π²ΠΎΠ΄Π½ΠΎΠΉ Ρ„ΡƒΠ½ΠΊΡ†ΠΈΠΈ Ρƒ =f(x) Π² Ρ‚ΠΎΡ‡ΠΊΠ΅ с абсциссой x0.

Π£Ρ€Π°Π²Π½Π΅Π½ΠΈΠ΅ ΠΊΠ°ΡΠ°Ρ‚Π΅Π»ΡŒΠ½ΠΎΠΉ ΠΊ ΠΊΡ€ΠΈΠ²ΠΎΠΉ Ρƒ = f(x) Π² Ρ‚ΠΎΡ‡ΠΊΠ΅ с абсциссой Ρ…0ΠΈΠΌΠ΅Π΅Ρ‚ Π²ΠΈΠ΄: .

ΠœΠ΅ΠΆΠ΄Ρƒ понятиСм ΠΊΠ°ΡΠ°Ρ‚Π΅Π»ΡŒΠ½ΠΎΠΉ ΠΈ понятиС ΠΏΡ€ΠΎΠΈΠ·Π²ΠΎΠ΄Π½ΠΎΠΉ имССтся тСсная связь. ГСомСтричСский смысл ΠΏΡ€ΠΎΠΈΠ·Π²ΠΎΠ΄Π½ΠΎΠΉ ΠΌΠΎΠΆΠ½ΠΎ Π²Ρ‹Ρ€Π°Π·ΠΈΡ‚ΡŒ Ρ‚Π°ΠΊ: Ссли функция Ρƒ = f(x) Π² Ρ‚ΠΎΡ‡ΠΊΠ΅ Ρ…0 ΠΈΠΌΠ΅Π΅Ρ‚ ΠΏΡ€ΠΎΠΈΠ·Π²ΠΎΠ΄Π½ΡƒΡŽ, Ρ‚ΠΎ Π² Ρ‚ΠΎΡ‡ΠΊΠ΅ с этой абсциссой ΠΎΠΏΡ€Π΅Π΄Π΅Π»Π΅Π½Π° ΠΊΠ°ΡΠ°Ρ‚Π΅Π»ΡŒΠ½Π°Ρ ΠΊ Π³Ρ€Π°Ρ„ΠΈΠΊΡƒ Ρ„ΡƒΠ½ΠΊΡ†ΠΈΠΈ , ΠΏΡ€ΠΈΡ‡Π΅ΠΌ Π΅Π΅ ΡƒΠ³Π»ΠΎΠ²ΠΎΠΉ коэффициСнт Ρ€Π°Π²Π΅Π½ . Π’Ρ‹Π²ΠΎΠ΄: Ссли Π² Ρ‚ΠΎΡ‡ΠΊΠ΅ Ρ…0 Π΅ΡΡ‚ΡŒ производная Ρ„ΡƒΠ½ΠΊΡ†ΠΈΠΈ ,

Ρ‚ΠΎ Π² Ρ‚ΠΎΡ‡ΠΊΠ΅ с этой абсциссой Π΅ΡΡ‚ΡŒ ΠΊΠ°ΡΠ°Ρ‚Π΅Π»ΡŒΠ½Π°Ρ ΠΊ Π³Ρ€Π°Ρ„ΠΈΠΊΡƒ Ρ„ΡƒΠ½ΠΊΡ†ΠΈΠΈ ΠΈ Π½Π°ΠΎΠ±ΠΎΡ€ΠΎΡ‚; Ссли Π² Ρ‚ΠΎΡ‡ΠΊΠ΅ Ρ…0 Π½Π΅Ρ‚ ΠΏΡ€ΠΎΠΈΠ·Π²ΠΎΠ΄Π½ΠΎΠΉ Ρ„ΡƒΠ½ΠΊΡ†ΠΈΠΈ , Ρ‚ΠΎ Π² Ρ‚ΠΎΡ‡ΠΊΠ΅ с этой абсциссой Π½Π΅Ρ‚ ΠΊΠ°ΡΠ°Ρ‚Π΅Π»ΡŒΠ½ΠΎΠΉ ΠΊ Π³Ρ€Π°Ρ„ΠΈΠΊΡƒ Ρ„ΡƒΠ½ΠΊΡ†ΠΈΠΈ ΠΈ Π½Π°ΠΎΠ±ΠΎΡ€ΠΎΡ‚.

Π£ΠΊΠ°ΠΆΠ΅ΠΌ случаи, ΠΊΠΎΠ³Π΄Π° функция Π½Π΅ ΠΈΠΌΠ΅Π΅Ρ‚ Π² Ρ‚ΠΎΡ‡ΠΊΠ΅ ΠΊΠ°ΡΠ°Ρ‚Π΅Π»ΡŒΠ½ΠΎΠΉ, ΠΈ, ΡΠ»Π΅Π΄ΠΎΠ²Π°Ρ‚Π΅Π»ΡŒΠ½ΠΎ, Π½Π΅ ΠΈΠΌΠ΅Π΅Ρ‚ ΠΈ ΠΏΡ€ΠΎΠΈΠ·Π²ΠΎΠ΄Π½ΠΎΠΉ. Π’Π°ΠΊΠΈΡ… случаСв Ρ‚Ρ€ΠΈ: угловая Ρ‚ΠΎΡ‡ΠΊΠ°, Ρ‚ΠΎΡ‡ΠΊΠ° Π²ΠΎΠ·Π²Ρ€Π°Ρ‚Π°, узловая Ρ‚ΠΎΡ‡ΠΊΠ°
(рис. 2 Π°, Π±, Π²). Особо ΠΎΡ‚ΠΌΠ΅Ρ‚ΠΈΠΌ случай, ΠΊΠΎΠ³Π΄Π° Π² Ρ‚ΠΎΡ‡ΠΊΠ΅ функция ΠΈΠΌΠ΅Π΅Ρ‚ Π±Π΅ΡΠΊΠΎΠ½Π΅Ρ‡Π½ΡƒΡŽ ΠΏΡ€ΠΎΠΈΠ·Π²ΠΎΠ΄Π½ΡƒΡŽ (рис. 2 Π³).

угловая Ρ‚ΠΎΡ‡ΠΊΠ° Ρ‚ΠΎΡ‡ΠΊΠ° Π²ΠΎΠ·Π²Ρ€Π°Ρ‚Π° узловая Ρ‚ΠΎΡ‡ΠΊΠ°

Π°) Π±) Π²) Π³)

Рис. 2

Рассмотрим Ρ€Π΅ΡˆΠ΅Π½ΠΈΠ΅ Π½Π΅ΠΊΠΎΡ‚ΠΎΡ€Ρ‹Ρ… Π·Π°Π΄Π°Ρ‡.

Π—Π°Π΄Π°Ρ‡ΠΈ, связанныС с ΠΎΠΏΡ€Π΅Π΄Π΅Π»Π΅Π½ΠΈΠ΅ΠΌ Ρ‚ΠΎΠ³ΠΎ, являСтся Π»ΠΈ прямая Ρƒ

= kx + b ΠΊΠ°ΡΠ°Ρ‚Π΅Π»ΡŒΠ½ΠΎΠΉ ΠΊ Π³Ρ€Π°Ρ„ΠΈΠΊΡƒ Ρ„ΡƒΠ½ΠΊΡ†ΠΈΠΈ Ρƒ = f(x). МоТно ΡƒΠΊΠ°Π·Π°Ρ‚ΡŒ Π΄Π²Π° способа Ρ€Π΅ΡˆΠ΅Π½ΠΈΡ Ρ‚Π°ΠΊΠΈΡ… Π·Π°Π΄Π°Ρ‡.

  1. Находим ΠΎΠ±Ρ‰ΠΈΠ΅ Ρ‚ΠΎΡ‡ΠΊΠΈ Π³Ρ€Π°Ρ„ΠΈΠΊΠΎΠ², Ρ‚. Π΅. Ρ€Π΅ΡˆΠ°Π΅ΠΌ ΡƒΡ€Π°Π²Π½Π΅Π½ΠΈΠ΅ f(x) = kx + b, Π° Π·Π°Ρ‚Π΅ΠΌ для ΠΊΠ°ΠΆΠ΄ΠΎΠ³ΠΎ ΠΈΠ· Π΅Π³ΠΎ Ρ€Π΅ΡˆΠ΅Π½ΠΈΠΉ вычисляСм . Π’ Ρ‚Π΅Ρ… случаях, ΠΊΠΎΠ³Π΄Π° = k, ΠΈΠΌΠ΅Π΅Ρ‚ мСсто касаниС, Π² Π΄Ρ€ΡƒΠ³ΠΈΡ… β€” пСрСсСчСниС.

  2. Находим ΠΊΠΎΡ€Π½ΠΈ уравнСния = k ΠΈ для ΠΊΠ°ΠΆΠ΄ΠΎΠ³ΠΎ ΠΈΠ· Π½ΠΈΡ… провСряСм, выполняСтся Π»ΠΈ равСнство f(x) = kx + b. ΠŸΡ€ΠΈ Π΅Π³ΠΎ Π²Ρ‹ΠΏΠΎΠ»Π½Π΅Π½ΠΈΠΈ ΠΏΠΎΠ»ΡƒΡ‡Π°Π΅ΠΌ абсциссы Ρ‚ΠΎΡ‡Π΅ΠΊ касания.

ΠžΠ±ΠΎΠ±Ρ‰Π°Ρ ΠΎΠ±Π° способа, Π·Π°ΠΌΠ΅Ρ‚ΠΈΠΌ, Ρ‡Ρ‚ΠΎ для Ρ‚ΠΎΠ³ΠΎ Ρ‡Ρ‚ΠΎΠ±Ρ‹ прямая

Ρƒ = kx + b Π±Ρ‹Π»Π° ΠΊΠ°ΡΠ°Ρ‚Π΅Π»ΡŒΠ½ΠΎΠΉ ΠΊ Π³Ρ€Π°Ρ„ΠΈΠΊΡƒ Ρ„ΡƒΠ½ΠΊΡ†ΠΈΠΈ Ρƒ = f(x), Π½Π΅ΠΎΠ±Ρ…ΠΎΠ΄ΠΈΠΌΠΎ ΠΈ достаточно сущСствованиС хотя Π±Ρ‹ ΠΎΠ΄Π½ΠΎΠ³ΠΎ числа Ρ…0, для ΠΊΠΎΡ‚ΠΎΡ€ΠΎΠ³ΠΎ выполняСтся систСма

  1. ΠŸΡ€ΠΈ ΠΊΠ°ΠΊΠΈΡ… значСниях b прямая Ρƒ = 3Ρ… +b являСтся ΠΊΠ°ΡΠ°Ρ‚Π΅Π»ΡŒΠ½ΠΎΠΉ ΠΊ Π³Ρ€Π°Ρ„ΠΈΠΊΡƒ Ρ„ΡƒΠ½ΠΊΡ†ΠΈΠΈ Ρƒ =?

РСшСниС. Записав условиС касания ΠΏΠΎΠ»ΡƒΡ‡ΠΈΠΌ

ΠžΡ‚Π²Π΅Ρ‚: .

  1. ΠŸΡ€ΠΈ ΠΊΠ°ΠΊΠΈΡ… значСниях Π° прямая Ρƒ=Π°Ρ…+2 являСтся ΠΊΠ°ΡΠ°Ρ‚Π΅Π»ΡŒΠ½ΠΎΠΉ ΠΊ Π³Ρ€Π°Ρ„ΠΈΠΊΡƒ Ρ„ΡƒΠ½ΠΊΡ†ΠΈΠΈ

Π£ΠΊΠ°Π·Π°Π½ΠΈΠ΅.

ΠžΡ‚Π²Π΅Ρ‚: Π° = e-3

  1. ΠŸΡ€ΠΈ ΠΊΠ°ΠΊΠΈΡ… значСниях Π° прямая являСтся ΠΊΠ°ΡΠ°Ρ‚Π΅Π»ΡŒΠ½ΠΎΠΉ ΠΊ Π³Ρ€Π°Ρ„ΠΈΠΊΡƒ Ρ„ΡƒΠ½ΠΊΡ†ΠΈΠΈ

Π£ΠΊΠ°Π·Π°Π½ΠΈΠ΅.

ΠžΡ‚Π²Π΅Ρ‚: Π° = 7 ΠΈΠ»ΠΈ Π° = -1.

  1. ЯвляСтся Π»ΠΈ прямая ΠΊΠ°ΡΠ°Ρ‚Π΅Π»ΡŒΠ½ΠΎΠΉ ΠΊ Π³Ρ€Π°Ρ„ΠΈΠΊΡƒ Ρ„ΡƒΠ½ΠΊΡ†ΠΈΠΈ ? Если являСтся, Ρ‚ΠΎ Π½Π°ΠΉΡ‚ΠΈ ΠΊΠΎΠΎΡ€Π΄ΠΈΠ½Π°Ρ‚Ρ‹ Ρ‚ΠΎΡ‡ΠΊΠΈ касания.

РСшСниС. ΠŸΡƒΡΡ‚ΡŒ . Из условия слСдуСт, Ρ‡Ρ‚ΠΎ Π΄ΠΎΠ»ΠΆΠ½Ρ‹ Π²Ρ‹ΠΏΠΎΠ»Π½ΡΡ‚ΡŒΡΡ равСнство , Π³Π΄Π΅ - возмоТная абсцисса Ρ‚ΠΎΡ‡ΠΊΠΈ касания. ИмССм:

Если Ρ‚Π΅ΠΏΠ΅Ρ€ΡŒ ΡΠΎΡΡ‚Π°Π²ΠΈΡ‚ΡŒ ΡƒΡ€Π°Π²Π½Π΅Π½ΠΈΠ΅ ΠΊΠ°ΡΠ°Ρ‚Π΅Π»ΡŒΠ½ΠΎΠΉ ΠΊ Π³Ρ€Π°Ρ„ΠΈΠΊΡƒ Π·Π°Π΄Π°Π½Π½ΠΎΠΉ Ρ„ΡƒΠ½ΠΊΡ†ΠΈΠΈ Π² ΠΊΠ°ΠΆΠ΄ΠΎΠΉ ΠΈΠ· Π΄Π²ΡƒΡ… Π½Π°ΠΉΠ΄Π΅Π½Π½Ρ‹Ρ… Ρ‚ΠΎΡ‡Π΅ΠΊ, Ρ‚ΠΎ окаТСтся, Ρ‡Ρ‚ΠΎ Π² Ρ‚ΠΎΡ‡ΠΊΠ΅ ΠΊΠ°ΠΊ Ρ€Π°Π· ΠΈ получится . Π—Π½Π°Ρ‡ΠΈΡ‚, Ρ‚ΠΎΡ‡ΠΊΠ° касания ΠΈΠΌΠ΅Π΅Ρ‚ ΠΊΠΎΠΎΡ€Π΄ΠΈΠ½Π°Ρ‚Ρ‹ (1;-1).

  1. К Π³Ρ€Π°Ρ„ΠΈΠΊΡƒ Ρ„ΡƒΠ½ΠΊΡ†ΠΈΠΈ ΠΏΡ€ΠΎΠ²Π΅Π΄Π΅Π½Π° ΠΊΠ°ΡΠ°Ρ‚Π΅Π»ΡŒΠ½Π°Ρ, ΠΏΠ°Ρ€Π°Π»Π»Π΅Π»ΡŒΠ½Π°Ρ прямой . Найти ΠΎΡ€Π΄ΠΈΠ½Π°Ρ‚Ρƒ Ρ‚ΠΎΡ‡ΠΊΠΈ касания.

РСшСниС. . Абсцисса ΠΈΠ½Ρ‚Π΅Ρ€Π΅ΡΡƒΡŽΡ‰Π΅ΠΉ нас Ρ‚ΠΎΡ‡ΠΊΠΈ касания удовлСтворяСт ΡƒΡ€Π°Π²Π½Π΅Π½ΠΈΡŽ . ИмССм:

Π’Π°ΠΊΠΈΠΌ ΠΎΠ±Ρ€Π°Π·ΠΎΠΌ, . Π—Π½Π°Ρ‡ΠΈΡ‚, - абсцисса Ρ‚ΠΎΡ‡ΠΊΠΈ касания. Π§Ρ‚ΠΎΠ±Ρ‹ Π½Π°ΠΉΡ‚ΠΈ ΠΎΡ€Π΄ΠΈΠ½Π°Ρ‚Ρƒ Ρ‚ΠΎΡ‡ΠΊΠΈ касания ΠΏΡ€Π΅ΠΎΠ±Ρ€Π°Π·ΡƒΠ΅ΠΌ Π²Ρ‹Ρ€Π°ΠΆΠ΅Π½ΠΈΠ΅, Π·Π°Π΄Π°ΡŽΡ‰Π΅Π΅ Ρ„ΡƒΠ½ΠΊΡ†ΠΈΡŽ:

ΠžΡ‚Π²Π΅Ρ‚: 1.

  1. ΠΠ°ΠΏΠΈΡΠ°Ρ‚ΡŒ ΡƒΡ€Π°Π²Π½Π΅Π½ΠΈΠ΅ всСх ΠΊΠ°ΡΠ°Ρ‚Π΅Π»ΡŒΠ½Ρ‹Ρ… ΠΊ Π³Ρ€Π°Ρ„ΠΈΠΊΡƒ Ρ„ΡƒΠ½ΠΊΡ†ΠΈΠΈ , ΠΏΠ°Ρ€Π°Π»Π»Π΅Π»ΡŒΠ½Ρ‹Ρ… прямой .

РСшСниС. Π’Π°ΠΊ ΠΊΠ°ΠΊ ΠΊΠ°ΡΠ°Ρ‚Π΅Π»ΡŒΠ½Π°Ρ Π΄ΠΎΠ»ΠΆΠ½Π° Π±Ρ‹Ρ‚ΡŒ ΠΏΠ°Ρ€Π°Π»Π»Π΅Π»ΡŒΠ½Π° прямой , Ρ‚ΠΎ Π΅Π΅ ΡƒΠ³Π»ΠΎΠ²ΠΎΠΉ коэффициСнт, Ρ€Π°Π²Π½Ρ‹ΠΉ Ρƒ'(Ρ…0), Π³Π΄Π΅ Ρ…0 β€” абсцисса Ρ‚ΠΎΡ‡ΠΊΠΈ касания, совпадаСт с ΡƒΠ³Π»ΠΎΠ²Ρ‹ΠΌ коэффициСнтом Π΄Π°Π½Π½ΠΎΠΉ прямой, Ρ‚. Π΅. . ΠžΡ‚ΡΡŽΠ΄Π° ΠΈΠ»ΠΈ .

Π”Π°Π»Π΅Π΅ составляСм ΡƒΡ€Π°Π²Π½Π΅Π½ΠΈΠ΅ ΠΊΠ°ΡΠ°Ρ‚Π΅Π»ΡŒΠ½ΠΎΠΉ для ΠΊΠ°ΠΆΠ΄ΠΎΠΉ Ρ‚ΠΎΡ‡ΠΊΠΈ.

ΠžΡ‚Π²Π΅Ρ‚: ,.

  1. Найти всС значСния , ΠΏΡ€ΠΈ ΠΊΠ°ΠΆΠ΄ΠΎΠΌ ΠΈΠ· ΠΊΠΎΡ‚ΠΎΡ€Ρ‹Ρ… ΠΊΠ°ΡΠ°Ρ‚Π΅Π»ΡŒΠ½Π°Ρ ΠΊ Π³Ρ€Π°Ρ„ΠΈΠΊΠ°ΠΌ Ρ„ΡƒΠ½ΠΊΡ†ΠΈΠΉ ΠΈ Π² Ρ‚ΠΎΡ‡ΠΊΠ°Ρ… с абсциссой ΠΏΠ°Ρ€Π°Π»Π»Π΅Π»ΡŒΠ½Ρ‹.

РСшСниС. Π˜Π·Π²Π΅ΡΡ‚Π½ΠΎ, Ρ‡Ρ‚ΠΎ тангСнс ΡƒΠ³Π»Π° Π½Π°ΠΊΠ»ΠΎΠ½Π° ΠΊΠ°ΡΠ°Ρ‚Π΅Π»ΡŒΠ½ΠΎΠΉ ΠΊ Π³Ρ€Π°Ρ„ΠΈΠΊΡƒ Ρ„ΡƒΠ½ΠΊΡ†ΠΈΠΉ Π² Ρ‚ΠΎΡ‡ΠΊΠ΅ с абсциссой Ρ€Π°Π²Π΅Π½ . Π‘Π»Π΅Π΄ΠΎΠ²Π°Ρ‚Π΅Π»ΡŒΠ½ΠΎ, всС искомыС значСния Π±ΡƒΠ΄ΡƒΡ‚ корнями уравнСния , ΠΎΡ‚ΠΊΡƒΠ΄Π° . Π˜ΡΠΏΠΎΠ»ΡŒΠ·ΡƒΡ Ρ„ΠΎΡ€ΠΌΡƒΠ»Ρƒ разности синусов ΡƒΠ³Π»ΠΎΠ², Π±ΡƒΠ΄Π΅ΠΌ ΠΈΠΌΠ΅Ρ‚ΡŒ . РСшая ΠΏΠΎΠ»ΡƒΡ‡Π΅Π½Π½ΠΎΠ΅ ΡƒΡ€Π°Π²Π½Π΅Π½ΠΈΠ΅, ΠΏΠΎΠ»ΡƒΡ‡Π°Π΅ΠΌ

  1. Найти расстояниС ΠΌΠ΅ΠΆΠ΄Ρƒ ΠΊΠ°ΡΠ°Ρ‚Π΅Π»ΡŒΠ½Ρ‹ΠΌΠΈ ΠΊ Π³Ρ€Π°Ρ„ΠΈΠΊΡƒ Ρ„ΡƒΠ½ΠΊΡ†ΠΈΠΈ , располоТСнными ΠΏΠ°Ρ€Π°Π»Π»Π΅Π»ΡŒΠ½ΠΎ оси .

РСшСниС. НайдСм критичСскиС Ρ‚ΠΎΡ‡ΠΊΠΈ Π·Π°Π΄Π°Π½Π½ΠΎΠΉ Ρ„ΡƒΠ½ΠΊΡ†ΠΈΠΈ:

Π’Π°ΠΊ ΠΊΠ°ΠΊ, производная Π² Ρ‚ΠΎΡ‡ΠΊΠ°Ρ… ΠΈ Ρ€Π°Π²Π½Π° Π½ΡƒΠ»ΡŽ, Ρ‚ΠΎ ΠΊΠ°ΡΠ°Ρ‚Π΅Π»ΡŒΠ½Ρ‹Π΅, ΠΏΡ€ΠΎΠ²Π΅Π΄Π΅Π½Π½Ρ‹Π΅ ΠΊ ΠΊΡ€ΠΈΠ²ΠΎΠΉ Π² Ρ‚ΠΎΡ‡ΠΊΠ°Ρ… с этими абсциссами, ΠΏΠ°Ρ€Π°Π»Π»Π΅Π»ΡŒΠ½Ρ‹ оси .

НайдСм значСния Ρ„ΡƒΠ½ΠΊΡ†ΠΈΠΉ Π² этих Ρ‚ΠΎΡ‡ΠΊΠ°Ρ….

Π˜Ρ‚Π°ΠΊ, расстояниС d ΠΌΠ΅ΠΆΠ΄Ρƒ ΠΊΠ°ΡΠ°Ρ‚Π΅Π»ΡŒΠ½Ρ‹ΠΌΠΈ, ΠΏΠ°Ρ€Π°Π»Π»Π΅Π»ΡŒΠ½Ρ‹ΠΌΠΈ оси , Ρ€Π°Π²Π½ΠΎ

Π‘ составлСниСм уравнСния ΠΊΠ°ΡΠ°Ρ‚Π΅Π»ΡŒΠ½ΠΎΠΉ, ΠΏΠ°Ρ€Π°Π»Π»Π΅Π»ΡŒΠ½ΠΎΠΉ Π΄Π°Π½Π½ΠΎΠΉ прямой, связана Π·Π°Π΄Π°Ρ‡Π° ΠΎ Π½Π°Ρ…ΠΎΠΆΠ΄Π΅Π½ΠΈΠΈ ΠΊΡ€Π°Ρ‚Ρ‡Π°ΠΉΡˆΠ΅Π³ΠΎ расстояния ΠΌΠ΅ΠΆΠ΄Ρƒ Π³Ρ€Π°Ρ„ΠΈΠΊΠΎΠΌ Π½Π΅ΠΊΠΎΡ‚ΠΎΡ€ΠΎΠΉ Ρ„ΡƒΠ½ΠΊΡ†ΠΈΠΈ f(x) ΠΈ прямой .

Π’ΠΎ ΠΌΠ½ΠΎΠ³ΠΈΡ… случаях удаСтся Π½Π°ΠΉΡ‚ΠΈ ΠΊΠ°ΡΠ°Ρ‚Π΅Π»ΡŒΠ½ΡƒΡŽ ΠΊ Π³Ρ€Π°Ρ„ΠΈΠΊΡƒ , ΠΏΠ°Ρ€Π°Π»Π»Π΅Π»ΡŒΠ½ΡƒΡŽ Π΄Π°Π½Π½ΠΎΠΉ прямой ΠΈ Π΄Π΅Π»ΡΡ‰ΡƒΡŽ ΠΏΠ»ΠΎΡΠΊΠΎΡΡ‚ΡŒ Π½Π° Π΄Π²Π΅ части, Π² ΠΎΠ΄Π½ΠΎΠΉ ΠΈΠ· ΠΊΠΎΡ‚ΠΎΡ€Ρ‹Ρ… располоТСн Π³Ρ€Π°Ρ„ΠΈΠΊ Ρ„ΡƒΠ½ΠΊΡ†ΠΈΠΈ, Π° Π² Π΄Ρ€ΡƒΠ³ΠΎΠΉ β€” заданная прямая. Π’ΠΎΠ³Π΄Π° ΠΊΡ€Π°Ρ‚Ρ‡Π°ΠΉΡˆΠΈΠΌ расстояниСм ΠΌΠ΅ΠΆΠ΄Ρƒ Π³Ρ€Π°Ρ„ΠΈΠΊΠΎΠΌ Ρ„ΡƒΠ½ΠΊΡ†ΠΈΠΈ ΠΈ прямой являСтся расстояниС ΠΎΡ‚ Ρ‚ΠΎΡ‡ΠΊΠΈ М(Ρ…0; Ρƒ

0), Π² ΠΊΠΎΡ‚ΠΎΡ€ΠΎΠΉ ΠΏΡ€ΠΎΠ²Π΅Π΄Π΅Π½Π° ΠΏΠ°Ρ€Π°Π»Π»Π΅Π»ΡŒΠ½Π°Ρ ΠΊΠ°ΡΠ°Ρ‚Π΅Π»ΡŒΠ½Π°Ρ, Π΄ΠΎ Π·Π°Π΄Π°Π½Π½ΠΎΠΉ прямой Ρƒ = kx + b; это расстояниС ΠΌΠΎΠΆΠ½ΠΎ Π²Ρ‹Ρ‡ΠΈΡΠ»ΠΈΡ‚ΡŒ ΠΏΠΎ Ρ„ΠΎΡ€ΠΌΡƒΠ»Π΅

  1. Найти ΠΊΡ€Π°Ρ‚Ρ‡Π°ΠΉΡˆΠ΅Π΅ расстояниС ΠΌΠ΅ΠΆΠ΄Ρƒ ΠΏΠ°Ρ€Π°Π±ΠΎΠ»ΠΎΠΉ ΠΈ прямой

РСшСниС. УбСдившись, Ρ‡Ρ‚ΠΎ Π³Ρ€Π°Ρ„ΠΈΠΊΠΈ Π½Π΅ ΠΈΠΌΠ΅ΡŽΡ‚ ΠΎΠ±Ρ‰ΠΈΡ… Ρ‚ΠΎΡ‡Π΅ΠΊ (ΡƒΡ€Π°Π²Π½Π΅Π½ΠΈΠ΅ Π½Π΅ ΠΈΠΌΠ΅Π΅Ρ‚ Ρ€Π΅ΡˆΠ΅Π½ΠΈΠΉ), запишСм ΡƒΡ€Π°Π²Π½Π΅Π½ΠΈΠ΅ Ρ‚Π°ΠΊΠΎΠΉ ΠΊΠ°ΡΠ°Ρ‚Π΅Π»ΡŒΠ½ΠΎΠΉ ΠΊ Π³Ρ€Π°Ρ„ΠΈΠΊΡƒ Ρ„ΡƒΠ½ΠΊΡ†ΠΈΠΈ , которая ΠΏΠ°Ρ€Π°Π»Π»Π΅Π»ΡŒΠ½Π° прямой Π£Ρ€Π°Π²Π½Π΅Π½ΠΈΠ΅ ΠΊΠ°ΡΠ°Ρ‚Π΅Π»ΡŒΠ½ΠΎΠΉ ΠΈΠΌΠ΅Π΅Ρ‚ Π²ΠΈΠ΄ касаниС происходит Π² Ρ‚ΠΎΡ‡ΠΊΠ΅ ΠŸΡ€ΡΠΌΠ°Ρ Ρƒ = Ρ… – 2 ΠΈ ΠΏΠ°Ρ€Π°Π±ΠΎΠ»Π° Ρƒ = Ρ…2 располоТСны ΠΏΠΎ Ρ€Π°Π·Π½Ρ‹Π΅ стороны ΠΎΡ‚ ΠΊΠ°ΡΠ°Ρ‚Π΅Π»ΡŒΠ½ΠΎΠΉ. Π’Π°ΠΊΠΈΠΌ ΠΎΠ±Ρ€Π°Π·ΠΎΠΌ, ΠΊΡ€Π°Ρ‚Ρ‡Π°ΠΉΡˆΠ΅Π΅ расстояниС ΠΌΠ΅ΠΆΠ΄Ρƒ ΠΏΠ°Ρ€Π°Π±ΠΎΠ»ΠΎΠΉ ΠΈ прямой Ρ€Π°Π²Π½ΠΎ Ρ€Π°ΡΡΡ‚ΠΎΡΠ½ΠΈΡŽ ΠΎΡ‚ Ρ‚ΠΎΡ‡ΠΊΠΈ

М Π΄ΠΎ прямой .

ΠžΡ‚Π²Π΅Ρ‚:

Π”ΠΎΠ²ΠΎΠ»ΡŒΠ½ΠΎ слоТной являСтся Π·Π°Π΄Π°Ρ‡Π° составлСния уравнСния всСх ΠΊΠ°ΡΠ°Ρ‚Π΅Π»ΡŒΠ½Ρ‹Ρ… ΠΊ Π³Ρ€Π°Ρ„ΠΈΠΊΡƒ Ρ„ΡƒΠ½ΠΊΡ†ΠΈΠΈ Ρƒ = f(x), проходящих Ρ‡Π΅Ρ€Π΅Π· Π·Π°Π΄Π°Π½Π½ΡƒΡŽ Ρ‚ΠΎΡ‡ΠΊΡƒ М(Ρ…0; Ρƒ0), Π²ΠΎΠΎΠ±Ρ‰Π΅ говоря, Π½Π΅ Π»Π΅ΠΆΠ°Ρ‰ΡƒΡŽ Π½Π° Π³Ρ€Π°Ρ„ΠΈΠΊΠ΅. ΠŸΡ€ΠΈΠ²Π΅Π΄Π΅ΠΌ Π°Π»Π³ΠΎΡ€ΠΈΡ‚ΠΌ Ρ€Π΅ΡˆΠ΅Π½ΠΈΡ этой Π·Π°Π΄Π°Ρ‡ΠΈ.

1. БоставляСм ΡƒΡ€Π°Π²Π½Π΅Π½ΠΈΠ΅ ΠΊΠ°ΡΠ°Ρ‚Π΅Π»ΡŒΠ½ΠΎΠΉ ΠΊ Π³Ρ€Π°Ρ„ΠΈΠΊΡƒ Ρ„ΡƒΠ½ΠΊΡ†ΠΈΠΈ Ρƒ = f(x) Π² ΠΏΡ€ΠΎΠΈΠ·Π²ΠΎΠ»ΡŒΠ½ΠΎΠΉ Ρ‚ΠΎΡ‡ΠΊΠ΅ Π³Ρ€Π°Ρ„ΠΈΠΊΠ° с абсциссой t:

2. РСшаСм ΠΎΡ‚Π½ΠΎΡΠΈΡ‚Π΅Π»ΡŒΠ½ΠΎ t ΡƒΡ€Π°Π²Π½Π΅Π½ΠΈΠ΅ ΠΈ для ΠΊΠ°ΠΆΠ΄ΠΎΠ³ΠΎ Π΅Π³ΠΎ Ρ€Π΅ΡˆΠ΅Π½ΠΈΡ t записываСм ΡΠΎΠΎΡ‚Π²Π΅Ρ‚ΡΡ‚Π²ΡƒΡŽΡ‰ΡƒΡŽ ΠΊΠ°ΡΠ°Ρ‚Π΅Π»ΡŒΠ½ΡƒΡŽ Π² Π²ΠΈΠ΄Π΅ .

  1. ΠΠ°ΠΏΠΈΡΠ°Ρ‚ΡŒ ΡƒΡ€Π°Π²Π½Π΅Π½ΠΈΠ΅ всСх ΠΊΠ°ΡΠ°Ρ‚Π΅Π»ΡŒΠ½Ρ‹Ρ… ΠΊ Π³Ρ€Π°Ρ„ΠΈΠΊΡƒ Ρ„ΡƒΠ½ΠΊΡ†ΠΈΠΈ , проходящих Ρ‡Π΅Ρ€Π΅Π· Ρ‚ΠΎΡ‡ΠΊΡƒ

    М(2; -2).

Π£ΠΊΠ°Π·Π°Π½ΠΈΠ΅. Π£Ρ€Π°Π²Π½Π΅Π½ΠΈΠ΅ ΠΊΠ°ΡΠ°Ρ‚Π΅Π»ΡŒΠ½ΠΎΠΉ Π² Ρ‚ΠΎΡ‡ΠΊΠ΅ с абсциссой t ΠΈΠΌΠ΅Π΅Ρ‚ Π²ΠΈΠ΄ . Π’Π°ΠΊ ΠΊΠ°ΠΊ эта ΠΊΠ°ΡΠ°Ρ‚Π΅Π»ΡŒΠ½Π°Ρ ΠΏΡ€ΠΎΡ…ΠΎΠ΄ΠΈΡ‚ Ρ‡Π΅Ρ€Π΅Π· Ρ‚ΠΎΡ‡ΠΊΡƒ (2; -2), Ρ‚ΠΎ
, ΠΎΡ‚ΠΊΡƒΠ΄Π° .

ΠžΡ‚Π²Π΅Ρ‚: .

  1. Найти ΠΏΠ»ΠΎΡ‰Π°Π΄ΡŒ Ρ‚Ρ€Π΅ΡƒΠ³ΠΎΠ»ΡŒΠ½ΠΈΠΊΠ°, ΠΎΠ±Ρ€Π°Π·ΠΎΠ²Π°Π½Π½ΠΎΠ³ΠΎ ΠΊΠ°ΡΠ°Ρ‚Π΅Π»ΡŒΠ½Ρ‹ΠΌΠΈ, ΠΏΡ€ΠΎΠ²Π΅Π΄Π΅Π½Π½Ρ‹ΠΌΠΈ ΠΊ Π³Ρ€Π°Ρ„ΠΈΠΊΡƒ Ρ„ΡƒΠ½ΠΊΡ†ΠΈΠΈ Ρ‡Π΅Ρ€Π΅Π· Ρ‚ΠΎΡ‡ΠΊΡƒ ΠΈ сСкущСй, проходящСй Ρ‡Π΅Ρ€Π΅Π· Ρ‚ΠΎΡ‡ΠΊΠΈ касания.

Π£ΠΊΠ°Π·Π°Π½ΠΈΠ΅. Π£Ρ€Π°Π²Π½Π΅Π½ΠΈΠ΅ Π΄Π°Π΅Ρ‚ Π΄Π²Π° Ρ€Π΅ΡˆΠ΅Π½ΠΈΡ: t1 = 1, t2 = 4. Π’Π°ΠΊΠΈΠΌ ΠΎΠ±Ρ€Π°Π·ΠΎΠΌ, Ρ‚ΠΎΡ‡ΠΊΠΈ K1 (1;1) ΠΈ K2(4;2) ΡΠ²Π»ΡΡŽΡ‚ΡΡ Ρ‚ΠΎΡ‡ΠΊΠ°ΠΌΠΈ касания.

ΠžΡ‚Π²Π΅Ρ‚: 0,25.

Говорят, Ρ‡Ρ‚ΠΎ прямая являСтся ΠΎΠ±Ρ‰Π΅ΠΉ ΠΊΠ°ΡΠ°Ρ‚Π΅Π»ΡŒΠ½ΠΎΠΉ Π³Ρ€Π°Ρ„ΠΈΠΊΠΎΠ² Ρ„ΡƒΠ½ΠΊΡ†ΠΈΠΈ
ΠΈ , Ссли ΠΎΠ½Π° касаСтся ΠΊΠ°ΠΊ ΠΎΠ΄Π½ΠΎΠ³ΠΎ, Ρ‚Π°ΠΊ ΠΈ Π΄Ρ€ΡƒΠ³ΠΎΠ³ΠΎ Π³Ρ€Π°Ρ„ΠΈΠΊΠΎΠ² (Π½ΠΎ ΡΠΎΠ²Π΅Ρ€ΡˆΠ΅Π½Π½ΠΎ Π½Π΅ ΠΎΠ±ΡΠ·Π°Ρ‚Π΅Π»ΡŒΠ½ΠΎ Π² ΠΎΠ΄Π½ΠΎΠΉ ΠΈ Ρ‚ΠΎΠΉ ΠΆΠ΅ Ρ‚ΠΎΡ‡ΠΊΠ΅). НапримСр, прямая являСтся ΠΎΠ±Ρ‰Π΅ΠΉ ΠΊΠ°ΡΠ°Ρ‚Π΅Π»ΡŒΠ½ΠΎΠΉ Π³Ρ€Π°Ρ„ΠΈΠΊΠΎΠ² Ρ„ΡƒΠ½ΠΊΡ†ΠΈΠΉ (Π² Ρ‚ΠΎΡ‡ΠΊΠ΅ М(2; 5) ΠΈ (Π² Ρ‚ΠΎΡ‡ΠΊΠ΅ K(0,5; -1)). Π—Π°ΠΌΠ΅Ρ‚ΠΈΠΌ, Ρ‡Ρ‚ΠΎ Π³Ρ€Π°Ρ„ΠΈΠΊΠΈ Ρ„ΡƒΠ½ΠΊΡ†ΠΈΠΉ ΠΈ ΠΈΠΌΠ΅ΡŽΡ‚ Π² Ρ‚ΠΎΡ‡ΠΊΠ΅ ΠΈΡ… пСрСсСчСния М(Ρ…0; Ρƒ0) ΠΎΠ±Ρ‰ΡƒΡŽ Π½Π΅Π²Π΅Ρ€Ρ‚ΠΈΠΊΠ°Π»ΡŒΠ½ΡƒΡŽ ΠΊΠ°ΡΠ°Ρ‚Π΅Π»ΡŒΠ½ΡƒΡŽ Ρ‚ΠΎΠ³Π΄Π° ΠΈ Ρ‚ΠΎΠ»ΡŒΠΊΠΎ Ρ‚ΠΎΠ³Π΄Π°, ΠΊΠΎΠ³Π΄Π° .

  1. Π”ΠΎΠΊΠ°Π·Π°Ρ‚ΡŒ, Ρ‡Ρ‚ΠΎ ΠΏΠ°Ρ€Π°Π±ΠΎΠ»Ρ‹ ΠΈ ΠΈΠΌΠ΅ΡŽΡ‚ Π² ΠΈΡ… ΠΎΠ±Ρ‰Π΅ΠΉ Ρ‚ΠΎΡ‡ΠΊΠ΅ ΠΎΠ±Ρ‰ΡƒΡŽ ΠΊΠ°ΡΠ°Ρ‚Π΅Π»ΡŒΠ½ΡƒΡŽ. Найти ΡƒΡ€Π°Π²Π½Π΅Π½ΠΈΠ΅ этой ΠΎΠ±Ρ‰Π΅ΠΉ ΠΊΠ°ΡΠ°Ρ‚Π΅Π»ΡŒΠ½ΠΎΠΉ. РСшСниС. Π£Ρ€Π°Π²Π½Π΅Π½ΠΈΠ΅ ΠΈΠΌΠ΅Π΅Ρ‚ СдинствСнный ΠΊΠΎΡ€Π΅Π½ΡŒ Ρ…=2, Ρ‚. Π΅. ΠΏΠ°Ρ€Π°Π±ΠΎΠ»Ρ‹ ΠΈΠΌΠ΅ΡŽΡ‚ Π΅Π΄ΠΈΠ½ΡΡ‚Π²Π΅Π½Π½ΡƒΡŽ ΠΎΠ±Ρ‰ΡƒΡŽ Ρ‚ΠΎΡ‡ΠΊΡƒ М(2;0). УбСдимся, Ρ‡Ρ‚ΠΎ значСния ΠΏΡ€ΠΎΠΈΠ·Π²ΠΎΠ΄Π½Ρ‹Ρ… для ΠΎΠ±Π΅ΠΈΡ… Ρ„ΡƒΠ½ΠΊΡ†ΠΈΠΉ Π² Ρ‚ΠΎΡ‡ΠΊΠ΅ Ρ… = 2 Ρ€Π°Π²Π½Ρ‹; Π΄Π΅ΠΉΡΡ‚Π²ΠΈΡ‚Π΅Π»ΡŒΠ½ΠΎ, ΠΈ . Π”Π°Π»Π΅Π΅ составляСм ΡƒΡ€Π°Π²Π½Π΅Π½ΠΈΠ΅ ΠΊΠ°ΡΠ°Ρ‚Π΅Π»ΡŒΠ½ΠΎΠΉ.

ΠžΡ‚Π²Π΅Ρ‚:.

Π’ Π·Π°Π²Π΅Ρ€ΡˆΠ΅Π½ΠΈΠΈ рассмотрим Ρ€Π΅ΡˆΠ΅Π½ΠΈΠ΅ Π΅Ρ‰Π΅ Π½Π΅ΡΠΊΠΎΠ»ΡŒΠΊΠΈΡ… Π·Π°Π΄Π°Ρ‡ Π½Π° ΠΊΠ°ΡΠ°Ρ‚Π΅Π»ΡŒΠ½ΡƒΡŽ с ΠΏΠ°Ρ€Π°ΠΌΠ΅Ρ‚Ρ€ΠΎΠΌ.

  1. ΠŸΡ€ΠΈ ΠΊΠ°ΠΊΠΈΡ… значСниях ΠΏΠ°Ρ€Π°ΠΌΠ΅Ρ‚Ρ€Π° ΠΊΠ°ΡΠ°Ρ‚Π΅Π»ΡŒΠ½Π°Ρ ΠΊ Π³Ρ€Π°Ρ„ΠΈΠΊΡƒ Ρ„ΡƒΠ½ΠΊΡ†ΠΈΠΈ Π² Ρ‚ΠΎΡ‡ΠΊΠ΅ ΠΏΡ€ΠΎΡ…ΠΎΠ΄ΠΈΡ‚ Ρ‡Π΅Ρ€Π΅Π· Ρ‚ΠΎΡ‡ΠΊΡƒ (2;3)?

РСшСниС. Боставим ΡƒΡ€Π°Π²Π½Π΅Π½ΠΈΠ΅ ΠΊΠ°ΡΠ°Ρ‚Π΅Π»ΡŒΠ½ΠΎΠΉ ΠΊ Π³Ρ€Π°Ρ„ΠΈΠΊΡƒ Π·Π°Π΄Π°Π½Π½ΠΎΠΉ Ρ„ΡƒΠ½ΠΊΡ†ΠΈΠΈ Π² Ρ‚ΠΎΡ‡ΠΊΠ΅ : Π’Π°ΠΊ ΠΊΠ°ΠΊ эта прямая ΠΏΡ€ΠΎΡ…ΠΎΠ΄ΠΈΡ‚ Ρ‡Π΅Ρ€Π΅Π· Ρ‚ΠΎΡ‡ΠΊΡƒ (2;3), Ρ‚ΠΎ ΠΈΠΌΠ΅Π΅Ρ‚ мСсто равСнство , ΠΎΡ‚ΠΊΡƒΠ΄Π° Π½Π°Ρ…ΠΎΠ΄ΠΈΠΌ: .

  1. ΠœΠΎΠΆΠ΅Ρ‚ Π»ΠΈ ΠΊΠ°ΡΠ°Ρ‚Π΅Π»ΡŒΠ½Π°Ρ ΠΊ ΠΊΡ€ΠΈΠ²ΠΎΠΉ Π² ΠΊΠ°ΠΊΠΎΠΉ-Π»ΠΈΠ±ΠΎ Π΅Π΅ Ρ‚ΠΎΡ‡ΠΊΠ΅ ΡΠΎΡΡ‚Π°Π²Π»ΡΡ‚ΡŒ острый ΡƒΠ³ΠΎΠ» с ΠΏΠΎΠ»ΠΎΠΆΠΈΡ‚Π΅Π»ΡŒΠ½Ρ‹ΠΌ Π½Π°ΠΏΡ€Π°Π²Π»Π΅Π½ΠΈΠ΅ΠΌ оси ?

РСшСниС. НайдСм ΠΏΡ€ΠΎΠΈΠ·Π²ΠΎΠ΄Π½ΡƒΡŽ Ρ„ΡƒΠ½ΠΊΡ†ΠΈΠΈ . Π’ любой Ρ‚ΠΎΡ‡ΠΊΠ΅, Π² ΠΊΠΎΡ‚ΠΎΡ€ΠΎΠΉ функция ΠΎΠΏΡ€Π΅Π΄Π΅Π»Π΅Π½Π°, производная ΠΎΡ‚Ρ€ΠΈΡ†Π°Ρ‚Π΅Π»ΡŒΠ½Π°. Но производная Π΅ΡΡ‚ΡŒ тангСнс ΡƒΠ³Π»Π° Π½Π°ΠΊΠ»ΠΎΠ½Π° ΠΊΠ°ΡΠ°Ρ‚Π΅Π»ΡŒΠ½ΠΎΠΉ, Π° Ρ‚Π°ΠΊ ΠΊΠ°ΠΊ ΠΎΠ½ ΠΎΡ‚Ρ€ΠΈΡ†Π°Ρ‚Π΅Π»Π΅Π½, Ρ‚ΠΎ ΡƒΠ³ΠΎΠ» Ρ‚ΡƒΠΏΠΎΠΉ.

ΠžΡ‚Π²Π΅Ρ‚: НС ΠΌΠΎΠΆΠ΅Ρ‚.

  1. Найти Π·Π½Π°Ρ‡Π΅Π½ΠΈΠ΅ ΠΏΠ°Ρ€Π°ΠΌΠ΅Ρ‚Ρ€Π° , ΠΏΡ€ΠΈ ΠΊΠΎΡ‚ΠΎΡ€ΠΎΠΌ ΠΊΠ°ΡΠ°Ρ‚Π΅Π»ΡŒΠ½Π°Ρ ΠΊ Π³Ρ€Π°Ρ„ΠΈΠΊΡƒ Ρ„ΡƒΠ½ΠΊΡ†ΠΈΠΈ Π² Ρ‚ΠΎΡ‡ΠΊΠ΅ ΠΏΡ€ΠΎΡ…ΠΎΠ΄ΠΈΡ‚ Ρ‡Π΅Ρ€Π΅Π· Ρ‚ΠΎΡ‡ΠΊΡƒ М(1;7).

РСшСниС. ΠŸΡƒΡΡ‚ΡŒ Ρ‚ΠΎΠ³Π΄Π° . Боставим ΡƒΡ€Π°Π²Π½Π΅Π½ΠΈΠ΅ ΠΊΠ°ΡΠ°Ρ‚Π΅Π»ΡŒΠ½ΠΎΠΉ:

По ΡƒΡΠ»ΠΎΠ²ΠΈΡŽ эта ΠΊΠ°ΡΠ°Ρ‚Π΅Π»ΡŒΠ½Π°Ρ ΠΏΡ€ΠΎΡ…ΠΎΠ΄ΠΈΡ‚ Ρ‡Π΅Ρ€Π΅Π· Ρ‚ΠΎΡ‡ΠΊΡƒ М(1;7), Π·Π½Π°Ρ‡ΠΈΡ‚, , ΠΎΡ‚ΠΊΡƒΠ΄Π° ΠΏΠΎΠ»ΡƒΡ‡Π°Π΅ΠΌ:

  1. ΠŸΡ€ΠΈ ΠΊΠ°ΠΊΠΈΡ… значСниях ΠΏΠ°Ρ€Π°ΠΌΠ΅Ρ‚Ρ€Π° прямая являСтся ΠΊΠ°ΡΠ°Ρ‚Π΅Π»ΡŒΠ½ΠΎΠΉ ΠΊ Π³Ρ€Π°Ρ„ΠΈΠΊΡƒ Ρ„ΡƒΠ½ΠΊΡ†ΠΈΠΈ ?

РСшСниС. Из условия слСдуСт, Ρ‡Ρ‚ΠΎ Π΄ΠΎΠ»ΠΆΠ½ΠΎ выполнятся равСнство Π³Π΄Π΅ абсцисса Ρ‚ΠΎΡ‡ΠΊΠΈ касания. Π—Π½Π°Ρ‡ΠΈΡ‚, ΠΈ связаны ΠΌΠ΅ΠΆΠ΄Ρƒ собой равСнством (1). Боставим ΡƒΡ€Π°Π²Π½Π΅Π½ΠΈΠ΅ ΠΊΠ°ΡΠ°Ρ‚Π΅Π»ΡŒΠ½ΠΎΠΉ ΠΊ Π³Ρ€Π°Ρ„ΠΈΠΊΡƒ Π·Π°Π΄Π°Π½Π½ΠΎΠΉ Ρ„ΡƒΠ½ΠΊΡ†ΠΈΠΈ Π² Ρ‚ΠΎΡ‡ΠΊΠ΅

Из условия слСдуСт, Ρ‡Ρ‚ΠΎ Π΄ΠΎΠ»ΠΆΠ½ΠΎ Π²Ρ‹ΠΏΠΎΠ»Π½ΡΡ‚ΡŒΡΡ равСнство . РСшив это ΡƒΡ€Π°Π²Π½Π΅Π½ΠΈΠ΅, ΠΏΠΎΠ»ΡƒΡ‡ΠΈΠΌ . Π’ΠΎΠ³Π΄Π° ΠΈΠ· (1) ΠΏΠΎΠ»ΡƒΡ‡Π°Π΅ΠΌ, Ρ‡Ρ‚ΠΎ .

  1. ΠŸΡ€ΠΈ ΠΊΠ°ΠΊΠΎΠΌ Π·Π½Π°Ρ‡Π΅Π½ΠΈΠΈ прямая являСтся ΠΊΠ°ΡΠ°Ρ‚Π΅Π»ΡŒΠ½ΠΎΠΉ Ρƒ Π³Ρ€Π°Ρ„ΠΈΠΊΡƒ ?

РСшСниС. Π’Π°ΠΊ ΠΊΠ°ΠΊ прямая являСтся ΠΊΠ°ΡΠ°Ρ‚Π΅Π»ΡŒΠ½ΠΎΠΉ ΠΊ Π³Ρ€Π°Ρ„ΠΈΠΊΡƒ Ρ„ΡƒΠ½ΠΊΡ†ΠΈΠΈ , Ρ‚ΠΎ Π² Ρ‚ΠΎΡ‡ΠΊΠ΅ касания ΡƒΠ³Π»ΠΎΠ²ΠΎΠΉ коэффициСнт ΠΊΠ°ΡΠ°Ρ‚Π΅Π»ΡŒΠ½ΠΎΠΉ Ρ€Π°Π²Π΅Π½ 3. Но ΡƒΠ³Π»ΠΎΠ²ΠΎΠΉ коэффициСнт ΠΊΠ°ΡΠ°Ρ‚Π΅Π»ΡŒΠ½ΠΎΠΉ Ρ€Π°Π²Π΅Π½ Π·Π½Π°Ρ‡Π΅Π½ΠΈΡŽ ΠΏΡ€ΠΎΠΈΠ·Π²ΠΎΠ΄Π½ΠΎΠΉ Ρ„ΡƒΠ½ΠΊΡ†ΠΈΠΈ Π² этой Ρ‚ΠΎΡ‡ΠΊΠ΅, Ρ‚ΠΎ Π΅ΡΡ‚ΡŒ , ΠΎΡ‚ΠΊΡƒΠ΄Π° , ΡΠ»Π΅Π΄ΠΎΠ²Π°Ρ‚Π΅Π»ΡŒΠ½ΠΎ, - абсцисса Ρ‚ΠΎΡ‡ΠΊΠΈ касания. НайдСм Ρ‚Π΅ΠΏΠ΅Ρ€ΡŒ ΠΈΠ· условия равСнства Π·Π½Π°Ρ‡Π΅Π½ΠΈΠΉ Ρ„ΡƒΠ½ΠΊΡ†ΠΈΠΉ ΠΈ ΠΏΡ€ΠΈ . ИмССм , ΠΎΡ‚ΠΊΡƒΠ΄Π° .

  1. ΠŸΡ€ΠΈ ΠΊΠ°ΠΊΠΈΡ… значСниях ΠΏΠ°Ρ€Π°ΠΌΠ΅Ρ‚Ρ€Π° Π° ΠΊΠ°ΡΠ°Ρ‚Π΅Π»ΡŒΠ½Ρ‹Π΅ ΠΊ Π³Ρ€Π°Ρ„ΠΈΠΊΡƒ Ρ„ΡƒΠ½ΠΊΡ†ΠΈΠΈ , ΠΏΡ€ΠΎΠ²Π΅Π΄Π΅Π½Π½Ρ‹Π΅ Π² Ρ‚ΠΎΡ‡ΠΊΠ°Ρ… Π΅Π³ΠΎ пСрСсСчСния с осью ΠΎx, ΠΎΠ±Ρ€Π°Π·ΡƒΡŽΡ‚ ΠΌΠ΅ΠΆΠ΄Ρƒ собой ΡƒΠ³ΠΎΠ» 60ΠΎ?

РСшСниС. Π’ этой Π·Π°Π΄Π°Ρ‡Π΅, ΠΊΠ°ΠΊ ΠΈ Π² ΠΏΡ€Π΅Π΄Ρ‹Π΄ΡƒΡ‰ΠΈΡ…, Ρ€Π΅Ρ‡ΡŒ ΠΈΠ΄Π΅Ρ‚ ΠΎ ΠΊΠ°ΡΠ°Ρ‚Π΅Π»ΡŒΠ½Ρ‹Ρ… ΠΊ Π³Ρ€Π°Ρ„ΠΈΠΊΡƒ Ρ„ΡƒΠ½ΠΊΡ†ΠΈΠΈ. Π‘ΠΎΡΡ‚Π°Π²Π»ΡΡ‚ΡŒ ΡƒΡ€Π°Π²Π½Π΅Π½ΠΈΠ΅ ΠΊΠ°ΡΠ°Ρ‚Π΅Π»ΡŒΠ½ΠΎΠΉ Π½Π΅ Π½Π°Π΄ΠΎ, достаточно ΠΈΡΠΏΠΎΠ»ΡŒΠ·ΠΎΠ²Π°Ρ‚ΡŒ гСомСтричСский смысл ΠΏΡ€ΠΎΠΈΠ·Π²ΠΎΠ΄Π½ΠΎΠΉ, Ρ‚ΠΎ Π΅ΡΡ‚ΡŒ ΡƒΠ³Π»ΠΎΠ²Ρ‹Π΅ коэффициСнты ΠΊΠ°ΡΠ°Ρ‚Π΅Π»ΡŒΠ½Ρ‹Ρ…. Π“Ρ€Π°Ρ„ΠΈΠΊΠΎΠΌ Π΄Π°Π½Π½ΠΎΠΉ Ρ„ΡƒΠ½ΠΊΡ†ΠΈΠΈ являСтся ΠΏΠ°Ρ€Π°Π±ΠΎΠ»Π° с вСтвями, Π½Π°ΠΏΡ€Π°Π²Π»Π΅Π½Π½Ρ‹ΠΌΠΈ Π²Π²Π΅Ρ€Ρ…, ΠΏΠ΅Ρ€Π΅ΡΠ΅ΠΊΠ°ΡŽΡ‰Π°Ρ ось ΠΎx Π² Π΄Π²ΡƒΡ… Ρ‚ΠΎΡ‡ΠΊΠ°Ρ… (случай Π°=0 нас Π½Π΅ устраиваСт): ΠΈ ΡƒΡ‡ΠΈΡ‚Ρ‹Π²Π°Π΅ΠΌ, Ρ‡Ρ‚ΠΎ Ρ…2>0 (рис. 3)

Рис. 3

ΠšΠ°ΡΠ°Ρ‚Π΅Π»ΡŒΠ½Ρ‹Π΅ АМ ΠΈ Π’Πœ ΠΏΠ΅Ρ€Π΅ΡΠ΅ΠΊΠ°ΡŽΡ‚ΡΡ ΠΏΠΎΠ΄ ΡƒΠ³Π»ΠΎΠΌ 60ΠΎ Π² Ρ‚ΠΎΡ‡ΠΊΠ΅ М, Π»Π΅ΠΆΠ°Ρ‰Π΅ΠΉ Π½Π° оси ΠΏΠ°Ρ€Π°Π±ΠΎΠ»Ρ‹, ΠΏΡ€ΠΈΡ‡Π΅ΠΌ Π²ΠΎΠ·ΠΌΠΎΠΆΠ½Ρ‹ Π΄Π²Π° случая: Π»ΠΈΠ±ΠΎ , Π»ΠΈΠ±ΠΎ смСТный ΡƒΠ³ΠΎΠ» Ρ€Π°Π²Π΅Π½ 60ΠΎ. Π² ΠΏΠ΅Ρ€Π²ΠΎΠΌ случаС ΡƒΠ³ΠΎΠ» ΠΌΠ΅ΠΆΠ΄Ρƒ ΠΊΠ°ΡΠ°Ρ‚Π΅Π»ΡŒΠ½ΠΎΠΉ АО ΠΈ осью Ρ… Ρ€Π°Π²Π΅Π½ 120ΠΎ, ΡΠ»Π΅Π΄ΠΎΠ²Π°Ρ‚Π΅Π»ΡŒΠ½ΠΎ, ΡƒΠ³ΠΎΠ» коэффициСнта ΠΊΠ°ΡΠ°Ρ‚Π΅Π»ΡŒΠ½ΠΎΠΉ Ρ€Π°Π²Π΅Π½ tg120o, Ρ‚ΠΎ Π΅ΡΡ‚ΡŒ Ρ€Π°Π²Π΅Π½ Π”Π°Π»Π΅Π΅ ΠΈΠΌΠ΅Π΅ΠΌ: . Π’Π°ΠΊΠΈΠΌ ΠΎΠ±Ρ€Π°Π·ΠΎΠΌ, ΠΏΠΎΠ»ΡƒΡ‡Π°Π΅ΠΌ, Ρ‡Ρ‚ΠΎ , Ρ‚ΠΎ . Π’ΠΎ Π²Ρ‚ΠΎΡ€ΠΎΠΌ случаС , поэтому ΡƒΠ³ΠΎΠ» ΠΌΠ΅ΠΆΠ΄Ρƒ ΠΊΠ°ΡΠ°Ρ‚Π΅Π»ΡŒΠ½ΠΎΠΉ АО ΠΈ ΠΎΡΡ‚ΡŒΡŽ ΠΎΡ… Ρ€Π°Π²Π΅Π½ 150ΠΎ. Π—Π½Π°Ρ‡ΠΈΡ‚, ΡƒΠ³Π»ΠΎΠ²ΠΎΠΉ коэффициСнт ΠΊΠ°ΡΠ°Ρ‚Π΅Π»ΡŒΠ½ΠΎΠΉ Ρ€Π°Π²Π΅Π½ tg150o , Ρ‚ΠΎ Π΅ΡΡ‚ΡŒ ΠΎΠ½ Ρ€Π°Π²Π΅Π½ . Π’Π°ΠΊΠΈΠΌ ΠΎΠ±Ρ€Π°Π·ΠΎΠΌ, ΠΏΠΎΠ»ΡƒΡ‡Π°Π΅ΠΌ, Ρ‡Ρ‚ΠΎ , Ρ‚ΠΎ Π΅ΡΡ‚ΡŒ

ΠžΡ‚Π²Π΅Ρ‚: .

Π›ΠΈΡ‚Π΅Ρ€Π°Ρ‚ΡƒΡ€Π°:

  1. Π”Π°Π»ΠΈΠ½Π³Π΅Ρ€, Π’.А. Начала матСматичСского Π°Π½Π°Π»ΠΈΠ·Π° Π² Π·Π°Π΄Π°Ρ‡Π°Ρ… [ВСкст]: ΡƒΡ‡Π΅Π±Π½ΠΎΠ΅ пособиС / Π’.А. Π”Π°Π»ΠΈΠ½Π³Π΅Ρ€. – Омск: Изд-Π²ΠΎ Π“ΠžΠ£ ΠžΠœΠ“ΠŸΠ£, 2009. – 312 с.

  2. Π—Π²Π°Π²ΠΈΡ‡, Π›.И. АлгСбра ΠΈ Π½Π°Ρ‡Π°Π»Π° Π°Π½Π°Π»ΠΈΠ·Π°. 8-11 ΠΊΠ». [ВСкст]: пособиС для школ ΠΈ классов с ΡƒΠ³Π»ΡƒΠ±Π». ΠΈΠ·ΡƒΡ‡Π΅Π½ΠΈΠ΅ΠΌ ΠΌΠ°Ρ‚Π΅ΠΌΠ°Ρ‚ΠΈΠΊΠΈ / Π›. И. Π—Π²Π°Π²ΠΈΡ‡, Π›.Π―. Шляпочник, М.Π’. Π§ΠΈΠ½ΠΊΠΈΠ½Π°.– М.: Π”Ρ€ΠΎΡ„Π°, 1999. – 352 с.

ΠšΠ°ΡΠ°Ρ‚Π΅Π»ΡŒΠ½Π°Ρ ΠΊ Π³Ρ€Π°Ρ„ΠΈΠΊΡƒ Ρ„ΡƒΠ½ΠΊΡ†ΠΈΠΈ Π² Ρ‚ΠΎΡ‡ΠΊΠ΅. Π£Ρ€Π°Π²Π½Π΅Π½ΠΈΠ΅ ΠΊΠ°ΡΠ°Ρ‚Π΅Π»ΡŒΠ½ΠΎΠΉ. ГСомСтричСский смысл ΠΏΡ€ΠΎΠΈΠ·Π²ΠΎΠ΄Π½ΠΎΠΉ

Π‘Ρ‚Π°Ρ‚ΡŒΡ Π΄Π°Π΅Ρ‚ ΠΏΠΎΠ΄Ρ€ΠΎΠ±Π½ΠΎΠ΅ Ρ€Π°Π·ΡŠΡΡΠ½Π΅Π½ΠΈΠ΅ ΠΎΠΏΡ€Π΅Π΄Π΅Π»Π΅Π½ΠΈΠΉ, гСомСтричСского смысла ΠΏΡ€ΠΎΠΈΠ·Π²ΠΎΠ΄Π½ΠΎΠΉ с графичСскими обозначСниями. Π‘ΡƒΠ΄Π΅Ρ‚ рассмотрСно ΡƒΡ€Π°Π²Π½Π΅Π½ΠΈΠ΅ ΠΊΠ°ΡΠ°Ρ‚Π΅Π»ΡŒΠ½ΠΎΠΉ прямой с ΠΏΡ€ΠΈΠ²Π΅Π΄Π΅Π½ΠΈΠ΅ΠΌ ΠΏΡ€ΠΈΠΌΠ΅Ρ€ΠΎΠ², Π½Π°ΠΉΠ΄Π΅Π½ΠΎ уравнСния ΠΊΠ°ΡΠ°Ρ‚Π΅Π»ΡŒΠ½ΠΎΠΉ ΠΊ ΠΊΡ€ΠΈΠ²Ρ‹ΠΌΒ 2 порядка.

ΠžΠΏΡ€Π΅Π΄Π΅Π»Π΅Π½ΠΈΡ ΠΈ понятия

ΠžΠΏΡ€Π΅Π΄Π΅Π»Π΅Π½ΠΈΠ΅ 1

Π£Π³ΠΎΠ» Π½Π°ΠΊΠ»ΠΎΠ½Π° прямой y=kx+b называСтся  ΡƒΠ³ΠΎΠ» Ξ±, ΠΊΠΎΡ‚ΠΎΡ€Ρ‹ΠΉ отсчитываСтся ΠΎΡ‚ ΠΏΠΎΠ»ΠΎΠΆΠΈΡ‚Π΅Π»ΡŒΠ½ΠΎΠ³ΠΎ направлСния оси ΠΎΡ… ΠΊ прямой y=kx+b Π² ΠΏΠΎΠ»ΠΎΠΆΠΈΡ‚Π΅Π»ΡŒΠ½ΠΎΠΌ Π½Π°ΠΏΡ€Π°Π²Π»Π΅Π½ΠΈΠΈ.

На рисункС Π½Π°ΠΏΡ€Π°Π²Π»Π΅Π½ΠΈΠ΅ ΠΎΡ… обозначаСтся ΠΏΡ€ΠΈ ΠΏΠΎΠΌΠΎΡ‰ΠΈ Π·Π΅Π»Π΅Π½ΠΎΠΉ стрСлки ΠΈ Π² Π²ΠΈΠ΄Π΅ Π·Π΅Π»Π΅Π½ΠΎΠΉ Π΄ΡƒΠ³ΠΈ, Π° ΡƒΠ³ΠΎΠ» Π½Π°ΠΊΠ»ΠΎΠ½Π° ΠΏΡ€ΠΈ ΠΏΠΎΠΌΠΎΡ‰ΠΈ красной Π΄ΡƒΠ³ΠΈ. Биняя линия относится ΠΊ прямой.

ΠžΠΏΡ€Π΅Π΄Π΅Π»Π΅Π½ΠΈΠ΅ 2

Π£Π³Π»ΠΎΠ²ΠΎΠΉ коэффициСнт прямой y=kx+b Π½Π°Π·Ρ‹Π²Π°ΡŽΡ‚ числовым коэффициСнтом k.

Π£Π³Π»ΠΎΠ²ΠΎΠΉ коэффициСнт равняСтся тангСнсу Π½Π°ΠΊΠ»ΠΎΠ½Π° прямой, ΠΈΠ½Π°Ρ‡Π΅ говоря k=tgΒ Ξ±.

  • Π£Π³ΠΎΠ» Π½Π°ΠΊΠ»ΠΎΠ½Π° прямой равняСтся 0 Ρ‚ΠΎΠ»ΡŒΠΊΠΎ ΠΏΡ€ΠΈ ΠΏΠ°Ρ€Π°Π»Π»Π΅Π»ΡŒΠ½ΠΎΡΡ‚ΠΈ ΠΎΡ… ΠΈΒ  ΡƒΠ³Π»ΠΎΠ²ΠΎΠΌ коэффициСнтС, Ρ€Π°Π²Π½ΠΎΠΌΡƒ Π½ΡƒΠ»ΡŽ, ΠΏΠΎΡ‚ΠΎΠΌΡƒ ΠΊΠ°ΠΊ тангСнс нуля Ρ€Π°Π²Π΅Π½ 0. Π—Π½Π°Ρ‡ΠΈΡ‚, Π²ΠΈΠ΄ уравнСния Π±ΡƒΠ΄Π΅Ρ‚ y=b.
  • Если ΡƒΠ³ΠΎΠ» Π½Π°ΠΊΠ»ΠΎΠ½Π° прямой y=kx+b острый, Ρ‚ΠΎΠ³Π΄Π° Π²Ρ‹ΠΏΠΎΠ»Π½ΡΡŽΡ‚ΡΡ условия 0<Ξ±<Ο€2Β ΠΈΠ»ΠΈ 0Β°<Ξ±<90Β°. ΠžΡ‚ΡΡŽΠ΄Π° ΠΈΠΌΠ΅Π΅ΠΌ, Ρ‡Ρ‚ΠΎ Π·Π½Π°Ρ‡Π΅Π½ΠΈΠ΅ ΡƒΠ³Π»ΠΎΠ²ΠΎΠ³ΠΎ коэффициСнта k считаСтся ΠΏΠΎΠ»ΠΎΠΆΠΈΡ‚Π΅Π»ΡŒΠ½Ρ‹ΠΌ числом, ΠΏΠΎΡ‚ΠΎΠΌΡƒ ΠΊΠ°ΠΊ Π·Π½Π°Ρ‡Π΅Π½ΠΈΠ΅ тангСнс удовлСтворяСт ΡƒΡΠ»ΠΎΠ²ΠΈΡŽ tgΒ Ξ±>0, ΠΏΡ€ΠΈΡ‡Π΅ΠΌ имССтся возрастаниС Π³Ρ€Π°Ρ„ΠΈΠΊΠ°.
  • Если Ξ±=Ο€2, Ρ‚ΠΎΠ³Π΄Π° располоТСниС прямой пСрпСндикулярно ΠΎΡ…. РавСнство задаСтся ΠΏΡ€ΠΈ ΠΏΠΎΠΌΠΎΡ‰ΠΈ равСнства x=c со Π·Π½Π°Ρ‡Π΅Π½ΠΈΠ΅ΠΌ с, ΡΠ²Π»ΡΡŽΡ‰ΠΈΠΌΡΡ Π΄Π΅ΠΉΡΡ‚Π²ΠΈΡ‚Π΅Π»ΡŒΠ½Ρ‹ΠΌ числом.
  • Если ΡƒΠ³ΠΎΠ» Π½Π°ΠΊΠ»ΠΎΠ½Π° прямой y=kx+b Ρ‚ΡƒΠΏΠΎΠΉ, Ρ‚ΠΎ соотвСтствуСт условиям Ο€2<Ξ±<π или 90Β°<Ξ±<180Β°, Π·Π½Π°Ρ‡Π΅Π½ΠΈΠ΅ ΡƒΠ³Π»ΠΎΠ²ΠΎΠ³ΠΎ коэффициСнта k ΠΏΡ€ΠΈΠ½ΠΈΠΌΠ°Π΅Ρ‚ ΠΎΡ‚Ρ€ΠΈΡ†Π°Ρ‚Π΅Π»ΡŒΠ½ΠΎΠ΅ Π·Π½Π°Ρ‡Π΅Π½ΠΈΠ΅, Π° Π³Ρ€Π°Ρ„ΠΈΠΊ ΡƒΠ±Ρ‹Π²Π°Π΅Ρ‚.
ΠžΠΏΡ€Π΅Π΄Π΅Π»Π΅Π½ΠΈΠ΅ 3

Π‘Π΅ΠΊΡƒΡ‰Π΅ΠΉ Π½Π°Π·Ρ‹Π²Π°ΡŽΡ‚ ΠΏΡ€ΡΠΌΡƒΡŽ, которая ΠΏΡ€ΠΎΡ…ΠΎΠ΄ΠΈΡ‚ Ρ‡Π΅Ρ€Π΅Π· 2 Ρ‚ΠΎΡ‡ΠΊΠΈ Ρ„ΡƒΠ½ΠΊΡ†ΠΈΠΈ f(x). Π˜Π½Π°Ρ‡Π΅ говоря, сСкущая – это прямая, которая проводится Ρ‡Π΅Ρ€Π΅Π· Π»ΡŽΠ±Ρ‹Π΅ Π΄Π²Π΅ Ρ‚ΠΎΡ‡ΠΊΠΈ Π³Ρ€Π°Ρ„ΠΈΠΊΠ° Π·Π°Π΄Π°Π½Π½ΠΎΠΉ Ρ„ΡƒΠ½ΠΊΡ†ΠΈΠΈ.

По рисунку Π²ΠΈΠ΄Π½ΠΎ, Ρ‡Ρ‚ΠΎ АВ являСтся сСкущСй,Β Π° f(x) – чСрная кривая, Ξ±Β — красная Π΄ΡƒΠ³Π°, ΠΎΠ·Π½Π°Ρ‡Π°ΡŽΡ‰Π°Ρ ΡƒΠ³ΠΎΠ» Π½Π°ΠΊΠ»ΠΎΠ½Π° сСкущСй.

Когда ΡƒΠ³Π»ΠΎΠ²ΠΎΠΉ коэффициСнт прямой равняСтся тангСнсу ΡƒΠ³Π»Π° Π½Π°ΠΊΠ»ΠΎΠ½Π°, Ρ‚ΠΎ Π²ΠΈΠ΄Π½ΠΎ, Ρ‡Ρ‚ΠΎ тангСнс ΠΈΠ· ΠΏΡ€ΡΠΌΠΎΡƒΠ³ΠΎΠ»ΡŒΠ½ΠΎΠ³ΠΎ Ρ‚Ρ€Π΅ΡƒΠ³ΠΎΠ»ΡŒΠ½ΠΈΠΊΠ° АВБ ΠΌΠΎΠΆΠ½ΠΎ Π½Π°ΠΉΡ‚ΠΈ ΠΏΠΎ ΠΎΡ‚Π½ΠΎΡˆΠ΅Π½ΠΈΡŽ ΠΏΡ€ΠΎΡ‚ΠΈΠ²ΠΎΠ»Π΅ΠΆΠ°Ρ‰Π΅Π³ΠΎ ΠΊΠ°Ρ‚Π΅Ρ‚Π° ΠΊ ΠΏΡ€ΠΈΠ»Π΅ΠΆΠ°Ρ‰Π΅ΠΌΡƒ.

ΠžΠΏΡ€Π΅Π΄Π΅Π»Π΅Π½ΠΈΠ΅ 4

ΠŸΠΎΠ»ΡƒΡ‡Π°Π΅ΠΌ Ρ„ΠΎΡ€ΠΌΡƒΠ»Ρƒ для нахоТдСния сСкущСй Π²ΠΈΠ΄Π°:

k=tgΒ Ξ±=BCAC=f(xB)-fxAxB-xA, Π³Π΄Π΅ абсциссами Ρ‚ΠΎΡ‡Π΅ΠΊ А ΠΈ Π’ ΡΠ²Π»ΡΡŽΡ‚ΡΡ значСния xA,Β xB, Π° f(xA),Β f(xB)Β — это значСния Ρ„ΡƒΠ½ΠΊΡ†ΠΈΠΈ Π² этих Ρ‚ΠΎΡ‡ΠΊΠ°Ρ….

ΠžΡ‡Π΅Π²ΠΈΠ΄Π½ΠΎ, Ρ‡Ρ‚ΠΎ ΡƒΠ³Π»ΠΎΠ²ΠΎΠΉ коэффициСнт сСкущСй ΠΎΠΏΡ€Π΅Π΄Π΅Π»Π΅Π½ ΠΏΡ€ΠΈ ΠΏΠΎΠΌΠΎΡ‰ΠΈ равСнства k=f(xB)-f(xA)xB-xAΒ ΠΈΠ»ΠΈ k=f(xA)-f(xB)xA-xB, ΠΏΡ€ΠΈΡ‡Π΅ΠΌ ΡƒΡ€Π°Π²Π½Π΅Π½ΠΈΠ΅ Π½Π΅ΠΎΠ±Ρ…ΠΎΠ΄ΠΈΠΌΠΎ Π·Π°ΠΏΠΈΡΠ°Ρ‚ΡŒ ΠΊΠ°ΠΊ y=f(xB)-f(xA)xB-xAΒ·x-xA+f(xA)Β ΠΈΠ»ΠΈ
y=f(xA)-f(xB)xA-xBΒ·x-xB+f(xB).

БСкущая Π΄Π΅Π»ΠΈΡ‚ Π³Ρ€Π°Ρ„ΠΈΠΊ Π²ΠΈΠ·ΡƒΠ°Π»ΡŒΠ½ΠΎ Π½Π° 3 части: слСва ΠΎΡ‚ Ρ‚ΠΎΡ‡ΠΊΠΈ А, ΠΎΡ‚ А Π΄ΠΎ Π’, справа ΠΎΡ‚ Π’. На располагаСмом Π½ΠΈΠΆΠ΅ рисункС Π²ΠΈΠ΄Π½ΠΎ, Ρ‡Ρ‚ΠΎ ΠΈΠΌΠ΅ΡŽΡ‚ΡΡ Ρ‚Ρ€ΠΈ сСкущиС, ΠΊΠΎΡ‚ΠΎΡ€Ρ‹Π΅ ΡΡ‡ΠΈΡ‚Π°ΡŽΡ‚ΡΡ ΡΠΎΠ²ΠΏΠ°Π΄Π°ΡŽΡ‰ΠΈΠΌΠΈ, Ρ‚ΠΎ Π΅ΡΡ‚ΡŒ Π·Π°Π΄Π°ΡŽΡ‚ΡΡ ΠΏΡ€ΠΈ ΠΏΠΎΠΌΠΎΡ‰ΠΈ Π°Π½Π°Π»ΠΎΠ³ΠΈΡ‡Π½ΠΎΠ³ΠΎ уравнСния.

По ΠΎΠΏΡ€Π΅Π΄Π΅Π»Π΅Π½ΠΈΡŽ Π²ΠΈΠ΄Π½ΠΎ, Ρ‡Ρ‚ΠΎ прямая ΠΈ Π΅Π΅ сСкущая Π² Π΄Π°Π½Π½ΠΎΠΌ случаС ΡΠΎΠ²ΠΏΠ°Π΄Π°ΡŽΡ‚.

БСкущая ΠΌΠΎΠΆΠ΅Ρ‚ мноТСствСнно Ρ€Π°Π· ΠΏΠ΅Ρ€Π΅ΡΠ΅ΠΊΠ°Ρ‚ΡŒ Π³Ρ€Π°Ρ„ΠΈΠΊ Π·Π°Π΄Π°Π½Π½ΠΎΠΉ Ρ„ΡƒΠ½ΠΊΡ†ΠΈΠΈ. Если имССтся ΡƒΡ€Π°Π²Π½Π΅Π½ΠΈΠ΅ Π²ΠΈΠ΄Π° Ρƒ=0 для сСкущСй, Ρ‚ΠΎΠ³Π΄Π° количСство Ρ‚ΠΎΡ‡Π΅ΠΊ пСрСсСчСния с синусоидой бСсконСчно.

ΠžΠΏΡ€Π΅Π΄Π΅Π»Π΅Π½ΠΈΠ΅ 5

ΠšΠ°ΡΠ°Ρ‚Π΅Π»ΡŒΠ½Π°Ρ ΠΊ Π³Ρ€Π°Ρ„ΠΈΠΊΡƒ Ρ„ΡƒΠ½ΠΊΡ†ΠΈΠΈ f(x) Π² Ρ‚ΠΎΡ‡ΠΊΠ΅ x0;Β f(x0) называСтся прямая, проходящая Ρ‡Π΅Ρ€Π΅Π· Π·Π°Π΄Π°Π½Π½ΡƒΡŽ Ρ‚ΠΎΡ‡ΠΊΡƒ x0;Β f(x0),Β  с Π½Π°Π»ΠΈΡ‡ΠΈΠ΅ΠΌ ΠΎΡ‚Ρ€Π΅Π·ΠΊΠ°, ΠΊΠΎΡ‚ΠΎΡ€Ρ‹ΠΉ ΠΈΠΌΠ΅Π΅Ρ‚ мноТСство Π·Π½Π°Ρ‡Π΅Π½ΠΈΠΉ Ρ…, Π±Π»ΠΈΠ·ΠΊΠΈΡ… ΠΊ x0.

ΠŸΡ€ΠΈΠΌΠ΅Ρ€ 1

Рассмотрим ΠΏΠΎΠ΄Ρ€ΠΎΠ±Π½ΠΎ Π½Π° Π½ΠΈΠΆΠ΅ ΠΏΡ€ΠΈΠ²Π΅Π΄Π΅Π½Π½ΠΎΠΌ ΠΏΡ€ΠΈΠΌΠ΅Ρ€Π΅. Π’ΠΎΠ³Π΄Π° Π²ΠΈΠ΄Π½ΠΎ, Ρ‡Ρ‚ΠΎ прямая, заданная Ρ„ΡƒΠ½ΠΊΡ†ΠΈΠ΅ΠΉ y=x+1, считаСтся ΠΊΠ°ΡΠ°Ρ‚Π΅Π»ΡŒΠ½ΠΎΠΉ ΠΊ y=2xΒ Π² Ρ‚ΠΎΡ‡ΠΊΠ΅  с ΠΊΠΎΠΎΡ€Π΄ΠΈΠ½Π°Ρ‚Π°ΠΌΠΈ (1;Β 2). Для наглядности, Π½Π΅ΠΎΠ±Ρ…ΠΎΠ΄ΠΈΠΌΠΎ Ρ€Π°ΡΡΠΌΠΎΡ‚Ρ€Π΅Ρ‚ΡŒ Π³Ρ€Π°Ρ„ΠΈΠΊΠΈ с ΠΏΡ€ΠΈΠ±Π»ΠΈΠΆΠ΅Π½Π½Ρ‹ΠΌΠΈ ΠΊ (1;Β 2) значСниями. Ѐункция y=2xΒ ΠΎΠ±ΠΎΠ·Π½Π°Ρ‡Π΅Π½Π° Ρ‡Π΅Ρ€Π½Ρ‹ΠΌ Ρ†Π²Π΅Ρ‚ΠΎΠΌ, синяя линия – ΠΊΠ°ΡΠ°Ρ‚Π΅Π»ΡŒΠ½Π°Ρ, красная Ρ‚ΠΎΡ‡ΠΊΠ° – Ρ‚ΠΎΡ‡ΠΊΠ° пСрСсСчСния.

ΠžΡ‡Π΅Π²ΠΈΠ΄Π½ΠΎ, Ρ‡Ρ‚ΠΎ y=2x сливаСтся с прямой Ρƒ=Ρ…+1.

Для опрСдСлСния ΠΊΠ°ΡΠ°Ρ‚Π΅Π»ΡŒΠ½ΠΎΠΉ слСдуСт Ρ€Π°ΡΡΠΌΠΎΡ‚Ρ€Π΅Ρ‚ΡŒ ΠΏΠΎΠ²Π΅Π΄Π΅Π½ΠΈΠ΅ ΠΊΠ°ΡΠ°Ρ‚Π΅Π»ΡŒΠ½ΠΎΠΉ АВ ΠΏΡ€ΠΈ бСсконСчном ΠΏΡ€ΠΈΠ±Π»ΠΈΠΆΠ΅Π½ΠΈΠΈ Ρ‚ΠΎΡ‡ΠΊΠΈ Π’ ΠΊ Ρ‚ΠΎΡ‡ΠΊΠ΅ А. Для наглядности ΠΏΡ€ΠΈΠ²Π΅Π΄Π΅ΠΌ рисунок.

БСкущая АВ, обозначСнная ΠΏΡ€ΠΈ ΠΏΠΎΠΌΠΎΡ‰ΠΈ синСй Π»ΠΈΠ½ΠΈΠΈ, стрСмится ΠΊ полоТСнию самой ΠΊΠ°ΡΠ°Ρ‚Π΅Π»ΡŒΠ½ΠΎΠΉ, Π° ΡƒΠ³ΠΎΠ» Π½Π°ΠΊΠ»ΠΎΠ½Π° сСкущСй Ξ±Β Π½Π°Ρ‡Π½Π΅Ρ‚ ΡΡ‚Ρ€Π΅ΠΌΠΈΡ‚ΡŒΡΡ ΠΊ ΡƒΠ³Π»Ρƒ Π½Π°ΠΊΠ»ΠΎΠ½Π° самой ΠΊΠ°ΡΠ°Ρ‚Π΅Π»ΡŒΠ½ΠΎΠΉ Ξ±x.

ΠžΠΏΡ€Π΅Π΄Π΅Π»Π΅Π½ΠΈΠ΅ 6

ΠšΠ°ΡΠ°Ρ‚Π΅Π»ΡŒΠ½ΠΎΠΉ ΠΊ Π³Ρ€Π°Ρ„ΠΈΠΊΡƒ Ρ„ΡƒΠ½ΠΊΡ†ΠΈΠΈ y=f(x) Π² Ρ‚ΠΎΡ‡ΠΊΠ΅ А считаСтся ΠΏΡ€Π΅Π΄Π΅Π»ΡŒΠ½ΠΎΠ΅ ΠΏΠΎΠ»ΠΎΠΆΠ΅Π½ΠΈΠ΅ сСкущСй АВ ΠΏΡ€ΠΈ Π’ стрСмящСйся ΠΊ А, Ρ‚ΠΎ Π΅ΡΡ‚ΡŒ Bβ†’A.

Π’Π΅ΠΏΠ΅Ρ€ΡŒ ΠΏΠ΅Ρ€Π΅ΠΉΠ΄Π΅ΠΌ ΠΊ Ρ€Π°ΡΡΠΌΠΎΡ‚Ρ€Π΅Π½ΠΈΡŽ гСомСтричСского смысла ΠΏΡ€ΠΎΠΈΠ·Π²ΠΎΠ΄Π½ΠΎΠΉ Ρ„ΡƒΠ½ΠΊΡ†ΠΈΠΈ Π² Ρ‚ΠΎΡ‡ΠΊΠ΅.

ГСомСтричСский смысл ΠΏΡ€ΠΎΠΈΠ·Π²ΠΎΠ΄Π½ΠΎΠΉ Ρ„ΡƒΠ½ΠΊΡ†ΠΈΠΈ Π² Ρ‚ΠΎΡ‡ΠΊΠ΅

ΠŸΠ΅Ρ€Π΅ΠΉΠ΄Π΅ΠΌ ΠΊ Ρ€Π°ΡΡΠΌΠΎΡ‚Ρ€Π΅Π½ΠΈΡŽ сСкущСй АВ для Ρ„ΡƒΠ½ΠΊΡ†ΠΈΠΈ f(x), Π³Π΄Π΅ А ΠΈ В с ΠΊΠΎΠΎΡ€Π΄ΠΈΠ½Π°Ρ‚Π°ΠΌΠΈ x0,Β f(x0)Β ΠΈ x0+βˆ†x,Β f(x0+βˆ†x), Π° βˆ†xΒ ΠΎΠ±ΠΎΠ·Π½Π°Ρ‡Π°Π΅ΠΌ ΠΊΠ°ΠΊ ΠΏΡ€ΠΈΡ€Π°Ρ‰Π΅Π½ΠΈΠ΅ Π°Ρ€Π³ΡƒΠΌΠ΅Π½Ρ‚Π°. Π’Π΅ΠΏΠ΅Ρ€ΡŒ функция ΠΏΡ€ΠΈΠΌΠ΅Ρ‚ Π²ΠΈΠ΄ βˆ†y=βˆ†f(x)=f(x0+βˆ†x)-f(βˆ†x). Для наглядности ΠΏΡ€ΠΈΠ²Π΅Π΄Π΅ΠΌ Π² ΠΏΡ€ΠΈΠΌΠ΅Ρ€ рисунок.

Рассмотрим ΠΏΠΎΠ»ΡƒΡ‡Π΅Π½Π½Ρ‹ΠΉ ΠΏΡ€ΡΠΌΠΎΡƒΠ³ΠΎΠ»ΡŒΠ½Ρ‹ΠΉ Ρ‚Ρ€Π΅ΡƒΠ³ΠΎΠ»ΡŒΠ½ΠΈΠΊ АВБ. Π˜ΡΠΏΠΎΠ»ΡŒΠ·ΡƒΠ΅ΠΌ ΠΎΠΏΡ€Π΅Π΄Π΅Π»Π΅Π½ΠΈΠ΅ тангСнса для Ρ€Π΅ΡˆΠ΅Π½ΠΈΡ, Ρ‚ΠΎ Π΅ΡΡ‚ΡŒ ΠΏΠΎΠ»ΡƒΡ‡ΠΈΠΌ ΠΎΡ‚Π½ΠΎΡˆΠ΅Π½ΠΈΠ΅ βˆ†yβˆ†x=tgΒ Ξ±. Из опрСдСлСния ΠΊΠ°ΡΠ°Ρ‚Π΅Π»ΡŒΠ½ΠΎΠΉ слСдуСт, Ρ‡Ρ‚ΠΎ limβˆ†xβ†’0βˆ†yβˆ†x=tgΒ Ξ±x. По ΠΏΡ€Π°Π²ΠΈΠ»Ρƒ ΠΏΡ€ΠΎΠΈΠ·Π²ΠΎΠ΄Π½ΠΎΠΉ Π² Ρ‚ΠΎΡ‡ΠΊΠ΅ ΠΈΠΌΠ΅Π΅ΠΌ, Ρ‡Ρ‚ΠΎ ΠΏΡ€ΠΎΠΈΠ·Π²ΠΎΠ΄Π½ΡƒΡŽ f(x) Π² Ρ‚ΠΎΡ‡ΠΊΠ΅ x0Β Π½Π°Π·Ρ‹Π²Π°ΡŽΡ‚ ΠΏΡ€Π΅Π΄Π΅Π»ΠΎΠΌ ΠΎΡ‚Π½ΠΎΡˆΠ΅Π½ΠΈΠΉ приращСния Ρ„ΡƒΠ½ΠΊΡ†ΠΈΠΈ ΠΊ ΠΏΡ€ΠΈΡ€Π°Ρ‰Π΅Π½ΠΈΡŽ Π°Ρ€Π³ΡƒΠΌΠ΅Π½Ρ‚Π°, Π³Π΄Π΅ βˆ†xβ†’0, Ρ‚ΠΎΠ³Π΄Π° ΠΎΠ±ΠΎΠ·Π½Π°Ρ‡ΠΈΠΌ ΠΊΠ°ΠΊ f(x0)=limβˆ†xβ†’0βˆ†yβˆ†x.

ΠžΡ‚ΡΡŽΠ΄Π° слСдуСт, Ρ‡Ρ‚ΠΎ f'(x0)=limβˆ†xβ†’0βˆ†yβˆ†x=tgΒ Ξ±x=kx, Π³Π΄Π΅ kxΒ ΠΎΠ±ΠΎΠ·Π½Π°Ρ‡Π°ΡŽΡ‚ Π² качСствС ΡƒΠ³Π»ΠΎΠ²ΠΎΠ³ΠΎ коэффициСнта ΠΊΠ°ΡΠ°Ρ‚Π΅Π»ΡŒΠ½ΠΎΠΉ.

Π’ΠΎ Π΅ΡΡ‚ΡŒ ΠΏΠΎΠ»ΡƒΡ‡Π°Π΅ΠΌ, Ρ‡Ρ‚ΠΎ f’(x) ΠΌΠΎΠΆΠ΅Ρ‚ ΡΡƒΡ‰Π΅ΡΡ‚Π²ΠΎΠ²Π°Ρ‚ΡŒΒ  Π² Ρ‚ΠΎΡ‡ΠΊΠ΅ x0Β ΠΏΡ€ΠΈΡ‡Π΅ΠΌ ΠΊΠ°ΠΊ ΠΈ ΠΊΠ°ΡΠ°Ρ‚Π΅Π»ΡŒΠ½Π°Ρ ΠΊ Π·Π°Π΄Π°Π½Π½ΠΎΠΌΡƒ Π³Ρ€Π°Ρ„ΠΈΠΊΡƒ Ρ„ΡƒΠ½ΠΊΡ†ΠΈΠΈ Π² Ρ‚ΠΎΡ‡ΠΊΠ΅ касания Ρ€Π°Π²Π½ΠΎΠΉ x0,Β f0(x0), Π³Π΄Π΅ Π·Π½Π°Ρ‡Π΅Π½ΠΈΠ΅ ΡƒΠ³Π»ΠΎΠ²ΠΎΠ³ΠΎ коэффициСнта ΠΊΠ°ΡΠ°Ρ‚Π΅Π»ΡŒΠ½ΠΎΠΉΒ  Π² Ρ‚ΠΎΡ‡ΠΊΠ΅ равняСтся ΠΏΡ€ΠΎΠΈΠ·Π²ΠΎΠ΄Π½ΠΎΠΉΒ  Π² Ρ‚ΠΎΡ‡ΠΊΠ΅ x0. Π’ΠΎΠ³Π΄Π° ΠΏΠΎΠ»ΡƒΡ‡Π°Π΅ΠΌ, Ρ‡Ρ‚ΠΎ kx=f'(x0).

ГСомСтричСский смысл ΠΏΡ€ΠΎΠΈΠ·Π²ΠΎΠ΄Π½ΠΎΠΉ Ρ„ΡƒΠ½ΠΊΡ†ΠΈΠΈ Π² Ρ‚ΠΎΡ‡ΠΊΠ΅ Π² Ρ‚ΠΎΠΌ, Ρ‡Ρ‚ΠΎ даСтся понятиС сущСствования ΠΊΠ°ΡΠ°Ρ‚Π΅Π»ΡŒΠ½ΠΎΠΉ ΠΊ Π³Ρ€Π°Ρ„ΠΈΠΊΡƒ Π² этой ΠΆΠ΅ Ρ‚ΠΎΡ‡ΠΊΠ΅.

Π£Ρ€Π°Π²Π½Π΅Π½ΠΈΠ΅ ΠΊΠ°ΡΠ°Ρ‚Π΅Π»ΡŒΠ½ΠΎΠΉ прямой

Π§Ρ‚ΠΎΠ±Ρ‹ Π·Π°ΠΏΠΈΡΠ°Ρ‚ΡŒ ΡƒΡ€Π°Π²Π½Π΅Π½ΠΈΠ΅ любой прямой Π½Π° плоскости, Π½Π΅ΠΎΠ±Ρ…ΠΎΠ΄ΠΈΠΌΠΎ ΠΈΠΌΠ΅Ρ‚ΡŒ ΡƒΠ³Π»ΠΎΠ²ΠΎΠΉ коэффициСнт с Ρ‚ΠΎΡ‡ΠΊΠΎΠΉ, Ρ‡Π΅Ρ€Π΅Π· ΠΊΠΎΡ‚ΠΎΡ€ΡƒΡŽ ΠΎΠ½Π° ΠΏΡ€ΠΎΡ…ΠΎΠ΄ΠΈΡ‚. Π•Π³ΠΎ ΠΎΠ±ΠΎΠ·Π½Π°Ρ‡Π΅Π½ΠΈΠ΅ принимаСтся ΠΊΠ°ΠΊ x0Β ΠΏΡ€ΠΈ пСрСсСчСнии.

Π£Ρ€Π°Π²Π½Π΅Π½ΠΈΠ΅ ΠΊΠ°ΡΠ°Ρ‚Π΅Π»ΡŒΠ½ΠΎΠΉ ΠΊ Π³Ρ€Π°Ρ„ΠΈΠΊΡƒ Ρ„ΡƒΠ½ΠΊΡ†ΠΈΠΈ y=f(x) Π² Ρ‚ΠΎΡ‡ΠΊΠ΅ x0,Β f0(x0)Β ΠΏΡ€ΠΈΠ½ΠΈΠΌΠ°Π΅Ρ‚ Π²ΠΈΠ΄ y=f'(x0)Β·x-x0+f(x0).

Π˜ΠΌΠ΅Π΅Ρ‚ΡΡ Π² Π²ΠΈΠ΄Ρƒ, Ρ‡Ρ‚ΠΎ ΠΊΠΎΠ½Π΅Ρ‡Π½Ρ‹ΠΌ Π·Π½Π°Ρ‡Π΅Π½ΠΈΠ΅ΠΌ ΠΏΡ€ΠΎΠΈΠ·Π²ΠΎΠ΄Π½ΠΎΠΉ f'(x0)Β ΠΌΠΎΠΆΠ½ΠΎ ΠΎΠΏΡ€Π΅Π΄Π΅Π»ΠΈΡ‚ΡŒ ΠΏΠΎΠ»ΠΎΠΆΠ΅Π½ΠΈΠ΅ ΠΊΠ°ΡΠ°Ρ‚Π΅Π»ΡŒΠ½ΠΎΠΉ, Ρ‚ΠΎ Π΅ΡΡ‚ΡŒ Π²Π΅Ρ€Ρ‚ΠΈΠΊΠ°Π»ΡŒΠ½ΠΎ ΠΏΡ€ΠΈ условии limxβ†’x0+0f'(x)=∞ и limxβ†’x0-0f'(x)=∞ или отсутствиС вовсС ΠΏΡ€ΠΈ условии limxβ†’x0+0f'(x)β‰ limxβ†’x0-0f'(x).

РасполоТСниС ΠΊΠ°ΡΠ°Ρ‚Π΅Π»ΡŒΠ½ΠΎΠΉ зависит ΠΎΡ‚ значСния Π΅Π΅ ΡƒΠ³Π»ΠΎΠ²ΠΎΠ³ΠΎ коэффициСнта kx=f'(x0). ΠŸΡ€ΠΈ ΠΏΠ°Ρ€Π°Π»Π»Π΅Π»ΡŒΠ½ΠΎΡΡ‚ΠΈ ΠΊ оси ΠΎΡ… ΠΏΠΎΠ»ΡƒΡ‡Π°Π΅ΠΌ, Ρ‡Ρ‚ΠΎ kk=0, ΠΏΡ€ΠΈ ΠΏΠ°Ρ€Π°Π»Π»Π΅Π»ΡŒΠ½ΠΎΡΡ‚ΠΈ ΠΊ ΠΎΡƒ — kx=∞, ΠΏΡ€ΠΈΡ‡Π΅ΠΌ Π²ΠΈΠ΄ уравнСния ΠΊΠ°ΡΠ°Ρ‚Π΅Π»ΡŒΠ½ΠΎΠΉ x=x0 возрастаСт ΠΏΡ€ΠΈ kx>0, ΡƒΠ±Ρ‹Π²Π°Π΅Ρ‚ ΠΏΡ€ΠΈ kx<0.

ΠŸΡ€ΠΈΠΌΠ΅Ρ€ 2

ΠŸΡ€ΠΎΠΈΠ·Π²Π΅ΡΡ‚ΠΈ составлСниС уравнСния ΠΊΠ°ΡΠ°Ρ‚Π΅Π»ΡŒΠ½ΠΎΠΉ ΠΊ Π³Ρ€Π°Ρ„ΠΈΠΊΡƒ Ρ„ΡƒΠ½ΠΊΡ†ΠΈΠΈ y=ex+1+x33-6-33x-17-33Β Π² Ρ‚ΠΎΡ‡ΠΊΠ΅  с ΠΊΠΎΠΎΡ€Π΄ΠΈΠ½Π°Ρ‚Π°ΠΌΠΈ (1;Β 3) с ΠΎΠΏΡ€Π΅Π΄Π΅Π»Π΅Π½ΠΈΠ΅ΠΌ ΡƒΠ³Π»Π° Π½Π°ΠΊΠ»ΠΎΠ½Π°.

РСшСниС

По ΡƒΡΠ»ΠΎΠ²ΠΈΡŽ ΠΈΠΌΠ΅Π΅ΠΌ, Ρ‡Ρ‚ΠΎ функция опрСдСляСтся для всСх Π΄Π΅ΠΉΡΡ‚Π²ΠΈΡ‚Π΅Π»ΡŒΠ½Ρ‹Ρ… чисСл. ΠŸΠΎΠ»ΡƒΡ‡Π°Π΅ΠΌ, Ρ‡Ρ‚ΠΎ Ρ‚ΠΎΡ‡ΠΊΠ° с ΠΊΠΎΠΎΡ€Π΄ΠΈΠ½Π°Ρ‚Π°ΠΌΠΈ, Π·Π°Π΄Π°Π½Π½Ρ‹ΠΌΠΈ ΠΏΠΎ ΡƒΡΠ»ΠΎΠ²ΠΈΡŽ, (1;Β 3) являСтся Ρ‚ΠΎΡ‡ΠΊΠΎΠΉ касания, Ρ‚ΠΎΠ³Π΄Π° x0=-1,Β f(x0)=-3.

НСобходимо Π½Π°ΠΉΡ‚ΠΈ ΠΏΡ€ΠΎΠΈΠ·Π²ΠΎΠ΄Π½ΡƒΡŽ Π² Ρ‚ΠΎΡ‡ΠΊΠ΅ со Π·Π½Π°Ρ‡Π΅Π½ΠΈΠ΅ΠΌ -1. ΠŸΠΎΠ»ΡƒΡ‡Π°Π΅ΠΌ, Ρ‡Ρ‚ΠΎ

y’=ex+1+x33-6-33x-17-33’==ex+1’+x33′-6-33x’-17-33’=ex+1+x2-6-33y'(x0)=y'(-1)=e-1+1+-12-6-33=33

Π—Π½Π°Ρ‡Π΅Π½ΠΈΠ΅ f’(x) Π² Ρ‚ΠΎΡ‡ΠΊΠ΅ касания являСтся Β ΡƒΠ³Π»ΠΎΠ²Ρ‹ΠΌ коэффициСнтом ΠΊΠ°ΡΠ°Ρ‚Π΅Π»ΡŒΠ½ΠΎΠΉ, ΠΊΠΎΡ‚ΠΎΡ€Ρ‹ΠΉ равняСтся тангСнсу Π½Π°ΠΊΠ»ΠΎΠ½Π°.

Π’ΠΎΠ³Π΄Π°Β kx=tgΒ Ξ±x=y'(x0)=33

ΠžΡ‚ΡΡŽΠ΄Π° слСдуСт, Ρ‡Ρ‚ΠΎ Ξ±x=arctg33=Ο€6

ΠžΡ‚Π²Π΅Ρ‚:Β ΡƒΡ€Π°Π²Π½Π΅Π½ΠΈΠ΅ ΠΊΠ°ΡΠ°Ρ‚Π΅Π»ΡŒΠ½ΠΎΠΉ ΠΏΡ€ΠΈΠΎΠ±Ρ€Π΅Ρ‚Π°Π΅Ρ‚ Π²ΠΈΠ΄

y=f'(x0)Β·x-x0+f(x0)y=33(x+1)-3y=33x-9-33

Для наглядности ΠΏΡ€ΠΈΠ²Π΅Π΄Π΅ΠΌ ΠΏΡ€ΠΈΠΌΠ΅Ρ€ Π² графичСской ΠΈΠ»Π»ΡŽΡΡ‚Ρ€Π°Ρ†ΠΈΠΈ.

Π§Π΅Ρ€Π½Ρ‹ΠΉ Ρ†Π²Π΅Ρ‚ ΠΈΡΠΏΠΎΠ»ΡŒΠ·ΡƒΠ΅Ρ‚ΡΡ для Π³Ρ€Π°Ρ„ΠΈΠΊΠ° исходной Ρ„ΡƒΠ½ΠΊΡ†ΠΈΠΈ, синий Ρ†Π²Π΅Ρ‚ – ΠΈΠ·ΠΎΠ±Ρ€Π°ΠΆΠ΅Π½ΠΈΠ΅ ΠΊΠ°ΡΠ°Ρ‚Π΅Π»ΡŒΠ½ΠΎΠΉ, красная Ρ‚ΠΎΡ‡ΠΊΠ° – Ρ‚ΠΎΡ‡ΠΊΠ° касания. Рисунок, располагаСмый справа, ΠΏΠΎΠΊΠ°Π·Ρ‹Π²Π°Π΅Ρ‚Β  Π² ΡƒΠ²Π΅Π»ΠΈΡ‡Π΅Π½Π½ΠΎΠΌ Π²ΠΈΠ΄Π΅.

ΠŸΡ€ΠΈΠΌΠ΅Ρ€ 3

Π’Ρ‹ΡΡΠ½ΠΈΡ‚ΡŒ Π½Π°Π»ΠΈΡ‡ΠΈΠ΅ сущСствования ΠΊΠ°ΡΠ°Ρ‚Π΅Π»ΡŒΠ½ΠΎΠΉ ΠΊ Π³Ρ€Π°Ρ„ΠΈΠΊΡƒ Π·Π°Π΄Π°Π½Π½ΠΎΠΉ Ρ„ΡƒΠ½ΠΊΡ†ΠΈΠΈ
y=3Β·x-15+1Β Π² Ρ‚ΠΎΡ‡ΠΊΠ΅ с ΠΊΠΎΠΎΡ€Π΄ΠΈΠ½Π°Ρ‚Π°ΠΌΠΈ (1;1). Π‘ΠΎΡΡ‚Π°Π²ΠΈΡ‚ΡŒ ΡƒΡ€Π°Π²Π½Π΅Π½ΠΈΠ΅ ΠΈ ΠΎΠΏΡ€Π΅Π΄Π΅Π»ΠΈΡ‚ΡŒ ΡƒΠ³ΠΎΠ» Π½Π°ΠΊΠ»ΠΎΠ½Π°.

РСшСниС

По ΡƒΡΠ»ΠΎΠ²ΠΈΡŽ ΠΈΠΌΠ΅Π΅ΠΌ, Ρ‡Ρ‚ΠΎ ΠΎΠ±Π»Π°ΡΡ‚ΡŒΡŽ опрСдСлСния Π·Π°Π΄Π°Π½Π½ΠΎΠΉ Ρ„ΡƒΠ½ΠΊΡ†ΠΈΠΈ считаСтся мноТСство всСх Π΄Π΅ΠΉΡΡ‚Π²ΠΈΡ‚Π΅Π»ΡŒΠ½Ρ‹Ρ… чисСл.

ΠŸΠ΅Ρ€Π΅ΠΉΠ΄Π΅ΠΌ ΠΊ Π½Π°Ρ…ΠΎΠΆΠ΄Π΅Π½ΠΈΡŽ ΠΏΡ€ΠΎΠΈΠ·Π²ΠΎΠ΄Π½ΠΎΠΉ

y’=3Β·x-15+1’=3Β·15Β·(x-1)15-1=35Β·1(x-1)45

Если x0=1, Ρ‚ΠΎΠ³Π΄Π° f’(x) Π½Π΅ ΠΎΠΏΡ€Π΅Π΄Π΅Π»Π΅Π½Π°, Π½ΠΎ ΠΏΡ€Π΅Π΄Π΅Π»Ρ‹ Π·Π°ΠΏΠΈΡΡ‹Π²Π°ΡŽΡ‚ΡΡ ΠΊΠ°ΠΊ Β limxβ†’1+035Β·1(x-1)45=35Β·1(+0)45=35Β·1+0=+∞ и limxβ†’1-035Β·1(x-1)45=35Β·1(-0)45=35Β·1+0=+∞, Ρ‡Ρ‚ΠΎ ΠΎΠ·Π½Π°Ρ‡Π°Π΅Ρ‚ сущСствованиС Π²Π΅Ρ€Ρ‚ΠΈΠΊΠ°Π»ΡŒΠ½ΠΎΠΉ ΠΊΠ°ΡΠ°Ρ‚Π΅Π»ΡŒΠ½ΠΎΠΉ Π² Ρ‚ΠΎΡ‡ΠΊΠ΅ (1;1).

ΠžΡ‚Π²Π΅Ρ‚:Β ΡƒΡ€Π°Π²Π½Π΅Π½ΠΈΠ΅ ΠΏΡ€ΠΈΠΌΠ΅Ρ‚ Π²ΠΈΠ΄ Ρ…=1, Π³Π΄Π΅ ΡƒΠ³ΠΎΠ» Π½Π°ΠΊΠ»ΠΎΠ½Π° Π±ΡƒΠ΄Π΅Ρ‚ Ρ€Π°Π²Π΅Π½ Ο€2.

Для наглядности ΠΈΠ·ΠΎΠ±Ρ€Π°Π·ΠΈΠΌ графичСски.

ΠŸΡ€ΠΈΠΌΠ΅Ρ€ 4

Найти Ρ‚ΠΎΡ‡ΠΊΠΈ Π³Ρ€Π°Ρ„ΠΈΠΊΠ° Ρ„ΡƒΠ½ΠΊΡ†ΠΈΠΈ y=115x+23-45×2-165x-265+3x+2, Π³Π΄Π΅

  1. ΠšΠ°ΡΠ°Ρ‚Π΅Π»ΡŒΠ½Π°Ρ Π½Π΅ сущСствуСт;
  2. ΠšΠ°ΡΠ°Ρ‚Π΅Π»ΡŒΠ½Π°Ρ располагаСтся ΠΏΠ°Ρ€Π°Π»Π»Π΅Π»ΡŒΠ½ΠΎ ΠΎΡ…;
  3. ΠšΠ°ΡΠ°Ρ‚Π΅Π»ΡŒΠ½Π°Ρ ΠΏΠ°Ρ€Π°Π»Π»Π΅Π»ΡŒΠ½Π° прямой y=85x+4.

РСшСниС

НСобходимо ΠΎΠ±Ρ€Π°Ρ‚ΠΈΡ‚ΡŒ Π²Π½ΠΈΠΌΠ°Π½ΠΈΠ΅ Π½Π° ΠΎΠ±Π»Π°ΡΡ‚ΡŒ опрСдСлСния. По ΡƒΡΠ»ΠΎΠ²ΠΈΡŽ ΠΈΠΌΠ΅Π΅ΠΌ, Ρ‡Ρ‚ΠΎ функция ΠΎΠΏΡ€Π΅Π΄Π΅Π»Π΅Π½Π° Π½Π° мноТСствС всСх Π΄Π΅ΠΉΡΡ‚Π²ΠΈΡ‚Π΅Π»ΡŒΠ½Ρ‹Ρ… чисСл. РаскрываСм ΠΌΠΎΠ΄ΡƒΠ»ΡŒΒ ΠΈ Ρ€Π΅ΡˆΠ°Π΅ΠΌ систСму с ΠΏΡ€ΠΎΠΌΠ΅ΠΆΡƒΡ‚ΠΊΠ°ΠΌΠΈ x∈-∞;Β 2Β ΠΈ [-2;Β +∞). ΠŸΠΎΠ»ΡƒΡ‡Π°Π΅ΠΌ, Ρ‡Ρ‚ΠΎ

y=-115×3+18×2+105x+176,Β x∈-∞;Β -2115×3-6×2+9x+12,Β x∈[-2;Β +∞)

НСобходимо ΠΏΡ€ΠΎΠ΄ΠΈΡ„Ρ„Π΅Ρ€Π΅Π½Ρ†ΠΈΡ€ΠΎΠ²Π°Ρ‚ΡŒ Ρ„ΡƒΠ½ΠΊΡ†ΠΈΡŽ. ИмССм, Ρ‡Ρ‚ΠΎ

y’=-115×3+18×2+105x+176′,Β x∈-∞;Β -2115×3-6×2+9x+12′,Β x∈[-2;Β +∞)⇔y’=-15(x2+12x+35),Β x∈-∞;Β -215×2-4x+3,Β x∈[-2;Β +∞)

Когда Ρ…=-2, Ρ‚ΠΎΠ³Π΄Π° производная Π½Π΅ сущСствуСт, ΠΏΠΎΡ‚ΠΎΠΌΡƒ Ρ‡Ρ‚ΠΎ односторонниС ΠΏΡ€Π΅Π΄Π΅Π»Ρ‹ Π½Π΅ Ρ€Π°Π²Π½Ρ‹ Π² этой Ρ‚ΠΎΡ‡ΠΊΠ΅:

limx→-2-0y'(x)=limx→-2-0-15(x2+12x+35=-15(-2)2+12(-2)+35=-3limx→-2+0y'(x)=limx→-2+015(x2-4x+3)=15-22-4-2+3=3

ВычисляСм Π·Π½Π°Ρ‡Π΅Π½ΠΈΠ΅ Ρ„ΡƒΠ½ΠΊΡ†ΠΈΠΈ Π² Ρ‚ΠΎΡ‡ΠΊΠ΅ Ρ…=-2, Π³Π΄Π΅ ΠΏΠΎΠ»ΡƒΡ‡Π°Π΅ΠΌ, Ρ‡Ρ‚ΠΎ

  1. y(-2)=115-2+23-45(-2)2-165(-2)-265+3-2+2=-2, Ρ‚ΠΎ Π΅ΡΡ‚ΡŒ ΠΊΠ°ΡΠ°Ρ‚Π΅Π»ΡŒΠ½Π°Ρ Π² Ρ‚ΠΎΡ‡ΠΊΠ΅ (-2;-2) Π½Π΅ Π±ΡƒΠ΄Π΅Ρ‚ ΡΡƒΡ‰Π΅ΡΡ‚Π²ΠΎΠ²Π°Ρ‚ΡŒ.
  2. ΠšΠ°ΡΠ°Ρ‚Π΅Π»ΡŒΠ½Π°Ρ ΠΏΠ°Ρ€Π°Π»Π»Π΅Π»ΡŒΠ½Π° ΠΎΡ…, ΠΊΠΎΠ³Π΄Π° ΡƒΠ³Π»ΠΎΠ²ΠΎΠΉ коэффициСнт равняСтся Π½ΡƒΠ»ΡŽ. Π’ΠΎΠ³Π΄Π° kx=tgΒ Ξ±x=f'(x0). Π’ΠΎ Π΅ΡΡ‚ΡŒ Π½Π΅ΠΎΠ±Ρ…ΠΎΠ΄ΠΈΠΌΠΎ Π½Π°ΠΉΡ‚ΠΈ значСния Ρ‚Π°ΠΊΠΈΡ… Ρ…, ΠΊΠΎΠ³Π΄Π° производная Ρ„ΡƒΠ½ΠΊΡ†ΠΈΠΈΒ  ΠΎΠ±Ρ€Π°Ρ‰Π°Π΅Ρ‚ Π΅Π΅ Π² ноль. Π’ΠΎ Π΅ΡΡ‚ΡŒ значСния f’(x)Β ΠΈ Π±ΡƒΠ΄ΡƒΡ‚ ΡΠ²Π»ΡΡ‚ΡŒΡΡ Ρ‚ΠΎΡ‡ΠΊΠ°ΠΌΠΈ касания, Π³Π΄Π΅ ΠΊΠ°ΡΠ°Ρ‚Π΅Π»ΡŒΠ½Π°Ρ являСтся ΠΏΠ°Ρ€Π°Π»Π»Π΅Π»ΡŒΠ½ΠΎΠΉ ΠΎΡ….

Когда x∈-∞;Β -2, Ρ‚ΠΎΠ³Π΄Π° -15(x2+12x+35)=0, Π° ΠΏΡ€ΠΈ x∈(-2;Β +∞)Β ΠΏΠΎΠ»ΡƒΡ‡Π°Π΅ΠΌ 15(x2-4x+3)=0.

РСшим:

-15(x2+12x+35)=0D=122-4Β·35=144-140=4×1=-12+42=-5∈-∞;Β -2×2=-12-42=-7∈-∞;Β -2Β  Β 15(x2-4x+3)=0D=42-4Β·3=4×3=4-42=1∈-2;Β +∞x4=4+42=3∈-2;Β +∞

ВычисляСм ΡΠΎΠΎΡ‚Π²Π΅Ρ‚ΡΡ‚Π²ΡƒΡŽΡ‰ΠΈΠ΅ значСния Ρ„ΡƒΠ½ΠΊΡ†ΠΈΠΈ

y1=y-5=115-5+23-45-52-165-5-265+3-5+2=85y2=y(-7)=115-7+23-45(-7)2-165-7-265+3-7+2=43y3=y(1)=1151+23-45Β·12-165Β·1-265+31+2=85y4=y(3)=1153+23-45Β·32-165Β·3-265+33+2=43

ΠžΡ‚ΡΡŽΠ΄Π° -5;Β 85,Β -4;Β 43,Β 1;Β 85,Β 3;Β 43Β ΡΡ‡ΠΈΡ‚Π°ΡŽΡ‚ΡΡ искомыми Ρ‚ΠΎΡ‡ΠΊΠ°ΠΌΠΈ Π³Ρ€Π°Ρ„ΠΈΠΊΠ° Ρ„ΡƒΠ½ΠΊΡ†ΠΈΠΈ.

Рассмотрим графичСскоС ΠΈΠ·ΠΎΠ±Ρ€Π°ΠΆΠ΅Π½ΠΈΠ΅ Ρ€Π΅ΡˆΠ΅Π½ΠΈΡ.

ЧСрная линия – Π³Ρ€Π°Ρ„ΠΈΠΊ Ρ„ΡƒΠ½ΠΊΡ†ΠΈΠΈ, красныС Ρ‚ΠΎΡ‡ΠΊΠΈ – Ρ‚ΠΎΡ‡ΠΊΠΈ касания.

  1. Когда прямыС Ρ€Π°ΡΠΏΠΎΠ»Π°Π³Π°ΡŽΡ‚ΡΡ ΠΏΠ°Ρ€Π°Π»Π»Π΅Π»ΡŒΠ½ΠΎ, Ρ‚ΠΎ ΡƒΠ³Π»ΠΎΠ²Ρ‹Π΅ коэффициСнты Ρ€Π°Π²Π½Ρ‹. Π’ΠΎΠ³Π΄Π° Π½Π΅ΠΎΠ±Ρ…ΠΎΠ΄ΠΈΠΌΠΎ Π·Π°Π½ΡΡ‚ΡŒΡΡ поиском Ρ‚ΠΎΡ‡Π΅ΠΊ Π³Ρ€Π°Ρ„ΠΈΠΊΠ° Ρ„ΡƒΠ½ΠΊΡ†ΠΈΠΈ, Π³Π΄Π΅ ΡƒΠ³Π»ΠΎΠ²ΠΎΠΉ коэффициСнт Π±ΡƒΠ΄Π΅Ρ‚ Ρ€Π°Π²Π½ΡΡ‚ΡŒΡΡ Π·Π½Π°Ρ‡Π΅Π½ΠΈΡŽ 85 . Для этого Π½ΡƒΠΆΠ½ΠΎ Ρ€Π΅ΡˆΠΈΡ‚ΡŒ ΡƒΡ€Π°Π²Π½Π΅Π½ΠΈΠ΅ Π²ΠΈΠ΄Π° y'(x)=85. Π’ΠΎΠ³Π΄Π°, Ссли x∈-∞;Β -2, ΠΏΠΎΠ»ΡƒΡ‡Π°Π΅ΠΌ, Ρ‡Ρ‚ΠΎ -15(x2+12x+35)=85, Π° Ссли x∈(-2;Β +∞), Ρ‚ΠΎΠ³Π΄Π° 15(x2-4x+3)=85.

ΠŸΠ΅Ρ€Π²ΠΎΠ΅ ΡƒΡ€Π°Π²Π½Π΅Π½ΠΈΠ΅ Π½Π΅ ΠΈΠΌΠ΅Π΅Ρ‚ ΠΊΠΎΡ€Π½Π΅ΠΉ, Ρ‚Π°ΠΊ ΠΊΠ°ΠΊ дискриминант мСньшС нуля. Π—Π°ΠΏΠΈΡˆΠ΅ΠΌ, Ρ‡Ρ‚ΠΎ

-15×2+12x+35=85×2+12x+43=0D=122-4Β·43=-28<0

Π”Ρ€ΡƒΠ³ΠΎΠ΅ ΡƒΡ€Π°Π²Π½Π΅Π½ΠΈΠ΅ ΠΈΠΌΠ΅Π΅Ρ‚ Π΄Π²Π° Π΄Π΅ΠΉΡΡ‚Π²ΠΈΡ‚Π΅Π»ΡŒΠ½Ρ‹Ρ… корня, Ρ‚ΠΎΠ³Π΄Π°

15(x2-4x+3)=85×2-4x-5=0D=42-4Β·(-5)=36×1=4-362=-1∈-2;Β +∞x2=4+362=5∈-2;Β +∞

ΠŸΠ΅Ρ€Π΅ΠΉΠ΄Π΅ΠΌ ΠΊ Π½Π°Ρ…ΠΎΠΆΠ΄Π΅Π½ΠΈΡŽ Π·Π½Π°Ρ‡Π΅Π½ΠΈΠΉ Ρ„ΡƒΠ½ΠΊΡ†ΠΈΠΈ. ΠŸΠΎΠ»ΡƒΡ‡Π°Π΅ΠΌ, Ρ‡Ρ‚ΠΎ

y1=y(-1)=115-1+23-45(-1)2-165(-1)-265+3-1+2=415y2=y(5)=1155+23-45Β·52-165Β·5-265+35+2=83

Π’ΠΎΡ‡ΠΊΠΈ со значСниями -1;Β 415,Β 5;Β 83Β ΡΠ²Π»ΡΡŽΡ‚ΡΡ Ρ‚ΠΎΡ‡ΠΊΠ°ΠΌΠΈ, Π² ΠΊΠΎΡ‚ΠΎΡ€Ρ‹Ρ… ΠΊΠ°ΡΠ°Ρ‚Π΅Π»ΡŒΠ½Ρ‹Π΅ ΠΏΠ°Ρ€Π°Π»Π»Π΅Π»ΡŒΠ½Ρ‹ прямой y=85x+4.

ΠžΡ‚Π²Π΅Ρ‚: чСрная линия – Π³Ρ€Π°Ρ„ΠΈΠΊ Ρ„ΡƒΠ½ΠΊΡ†ΠΈΠΈ, красная линия – Π³Ρ€Π°Ρ„ΠΈΠΊ y=85x+4, синяя линия – ΠΊΠ°ΡΠ°Ρ‚Π΅Π»ΡŒΠ½Ρ‹Π΅Β  Π² Ρ‚ΠΎΡ‡ΠΊΠ°Ρ… -1;Β 415,Β 5;Β 83.

Π’ΠΎΠ·ΠΌΠΎΠΆΠ½ΠΎ сущСствованиС бСсконСчного количСства ΠΊΠ°ΡΠ°Ρ‚Π΅Π»ΡŒΠ½Ρ‹Ρ… для Π·Π°Π΄Π°Π½Π½Ρ‹Ρ… Ρ„ΡƒΠ½ΠΊΡ†ΠΈΠΉ.

ΠŸΡ€ΠΈΠΌΠ΅Ρ€ 5

ΠΠ°ΠΏΠΈΡΠ°Ρ‚ΡŒ уравнСния всСх ΠΈΠΌΠ΅ΡŽΡ‰ΠΈΡ…ΡΡ ΠΊΠ°ΡΠ°Ρ‚Π΅Π»ΡŒΠ½Ρ‹Ρ… Ρ„ΡƒΠ½ΠΊΡ†ΠΈΠΈ y=3cos32x-Ο€4-13, ΠΊΠΎΡ‚ΠΎΡ€Ρ‹Π΅ Ρ€Π°ΡΠΏΠΎΠ»Π°Π³Π°ΡŽΡ‚ΡΡ пСрпСндикулярно прямой y=-2x+12.

РСшСниС

Для составлСния уравнСния ΠΊΠ°ΡΠ°Ρ‚Π΅Π»ΡŒΠ½ΠΎΠΉ Π½Π΅ΠΎΠ±Ρ…ΠΎΠ΄ΠΈΠΌΠΎ Π½Π°ΠΉΡ‚ΠΈ коэффициСнт ΠΈ ΠΊΠΎΠΎΡ€Π΄ΠΈΠ½Π°Ρ‚Ρ‹ Ρ‚ΠΎΡ‡ΠΊΠΈ касания, исходя ΠΈΠ· условия пСрпСндикулярности прямых. ΠžΠΏΡ€Π΅Π΄Π΅Π»Π΅Π½ΠΈΠ΅ Π·Π²ΡƒΡ‡ΠΈΡ‚ Ρ‚Π°ΠΊ: ΠΏΡ€ΠΎΠΈΠ·Π²Π΅Π΄Π΅Π½ΠΈΠ΅ ΡƒΠ³Π»ΠΎΠ²Ρ‹Ρ… коэффициСнтов, ΠΊΠΎΡ‚ΠΎΡ€Ρ‹Π΅ пСрпСндикулярны прямым, равняСтся -1, Ρ‚ΠΎ Π΅ΡΡ‚ΡŒ записываСтся ΠΊΠ°ΠΊ kxΒ·kβŠ₯=-1. Из условия ΠΈΠΌΠ΅Π΅ΠΌ, Ρ‡Ρ‚ΠΎ ΡƒΠ³Π»ΠΎΠ²ΠΎΠΉ коэффициСнт располагаСтся пСрпСндикулярно прямой  ΠΈ равняСтся kβŠ₯=-2, Ρ‚ΠΎΠ³Π΄Π° kx=-1kβŠ₯=-1-2=12.

Π’Π΅ΠΏΠ΅Ρ€ΡŒ Π½Π΅ΠΎΠ±Ρ…ΠΎΠ΄ΠΈΠΌΠΎ Π½Π°ΠΉΡ‚ΠΈ ΠΊΠΎΠΎΡ€Π΄ΠΈΠ½Π°Ρ‚Ρ‹ Ρ‚ΠΎΡ‡Π΅ΠΊ касания. НуТно Π½Π°ΠΉΡ‚ΠΈ Ρ…, послС Ρ‡Π΅Π³ΠΎ Π΅Π³ΠΎ Π·Π½Π°Ρ‡Π΅Π½ΠΈΠ΅ для Π·Π°Π΄Π°Π½Π½ΠΎΠΉ Ρ„ΡƒΠ½ΠΊΡ†ΠΈΠΈ. ΠžΡ‚ΠΌΠ΅Ρ‚ΠΈΠΌ, Ρ‡Ρ‚ΠΎ ΠΈΠ· гСомСтричСского смысла ΠΏΡ€ΠΎΠΈΠ·Π²ΠΎΠ΄Π½ΠΎΠΉΒ  Π² Ρ‚ΠΎΡ‡ΠΊΠ΅
x0Β ΠΏΠΎΠ»ΡƒΡ‡Π°Π΅ΠΌ, Ρ‡Ρ‚ΠΎ kx=y'(x0).Β  Из Π΄Π°Π½Π½ΠΎΠ³ΠΎ равСнства Π½Π°ΠΉΠ΄Π΅ΠΌ значСния Ρ… для Ρ‚ΠΎΡ‡Π΅ΠΊ касания.

ΠŸΠΎΠ»ΡƒΡ‡Π°Π΅ΠΌ, Ρ‡Ρ‚ΠΎ

y'(x0)=3cos32x0-Ο€4-13’=3Β·-sin32x0-Ο€4Β·32×0-Ο€4’==-3Β·sin32x0-Ο€4Β·32=-92Β·sin32x0-Ο€4β‡’kx=y'(x0)⇔-92Β·sin32x0-Ο€4=12β‡’sin32x0-Ο€4=-19

Π­Ρ‚ΠΎ тригономСтричСскоС ΡƒΡ€Π°Π²Π½Π΅Π½ΠΈΠ΅ Π±ΡƒΠ΄Π΅Ρ‚ использовано для вычислСния ΠΎΡ€Π΄ΠΈΠ½Π°Ρ‚ Ρ‚ΠΎΡ‡Π΅ΠΊ касания.

32×0-Ο€4=arcsin-19+2Ο€kΒ ΠΈΠ»ΠΈΒ 32×0-Ο€4=Ο€-arcsin-19+2Ο€k

32×0-Ο€4=-arcsin19+2Ο€kΒ ΠΈΠ»ΠΈΒ 32×0-Ο€4=Ο€+arcsin19+2Ο€k

x0=23Ο€4-arcsin19+2Ο€kΒ ΠΈΠ»ΠΈΒ x0=235Ο€4+arcsin19+2Ο€k,Β k∈Z

Z- мноТСство Ρ†Π΅Π»Ρ‹Ρ… чисСл.

НайдСны Ρ… Ρ‚ΠΎΡ‡Π΅ΠΊ касания. Π’Π΅ΠΏΠ΅Ρ€ΡŒ Π½Π΅ΠΎΠ±Ρ…ΠΎΠ΄ΠΈΠΌΠΎ ΠΏΠ΅Ρ€Π΅ΠΉΡ‚ΠΈ ΠΊ поиску Π·Π½Π°Ρ‡Π΅Π½ΠΈΠΉ Ρƒ:

y0=3cos32x0-Ο€4-13

y0=3Β·1-sin232x0-Ο€4-13Β ΠΈΠ»ΠΈΒ y0=3Β·-1-sin232x0-Ο€4-13

y0=3Β·1—192-13Β ΠΈΠ»ΠΈΒ y0=3Β·-1—192-13

y0=45-13Β ΠΈΠ»ΠΈΒ y0=-45+13

ΠžΡ‚ΡΡŽΠ΄Π° ΠΏΠΎΠ»ΡƒΡ‡Π°Π΅ΠΌ, Ρ‡Ρ‚ΠΎ 23Ο€4-arcsin19+2Ο€k;Β 45-13,Β 235Ο€4+arcsin19+2Ο€k;Β -45+13Β ΡΠ²Π»ΡΡŽΡ‚ΡΡ Ρ‚ΠΎΡ‡ΠΊΠ°ΠΌΠΈ касания.

ΠžΡ‚Π²Π΅Ρ‚: Π½Π΅ΠΎΠ±Ρ…ΠΎΠ΄ΠΈΠΌΡ‹ уравнСния Π·Π°ΠΏΠΈΡˆΡƒΡ‚ΡΡ ΠΊΠ°ΠΊ

y=12x-23Ο€4-arcsin19+2Ο€k+45-13,y=12x-235Ο€4+arcsin19+2Ο€k-45+13,Β k∈Z

Для наглядного изобраТСния рассмотрим Ρ„ΡƒΠ½ΠΊΡ†ΠΈΡŽ ΠΈ ΠΊΠ°ΡΠ°Ρ‚Π΅Π»ΡŒΠ½ΡƒΡŽ Π½Π° ΠΊΠΎΠΎΡ€Π΄ΠΈΠ½Π°Ρ‚Π½ΠΎΠΉ прямой.

Рисунок ΠΏΠΎΠΊΠ°Π·Ρ‹Π²Π°Π΅Ρ‚, Ρ‡Ρ‚ΠΎ располоТСниС Ρ„ΡƒΠ½ΠΊΡ†ΠΈΠΈ ΠΈΠ΄Π΅Ρ‚ Π½Π° ΠΏΡ€ΠΎΠΌΠ΅ΠΆΡƒΡ‚ΠΊΠ΅ [-10;10], Π³Π΄Π΅ чСрная прямя – Π³Ρ€Π°Ρ„ΠΈΠΊ Ρ„ΡƒΠ½ΠΊΡ†ΠΈΠΈ, синиС Π»ΠΈΠ½ΠΈΠΈ – ΠΊΠ°ΡΠ°Ρ‚Π΅Π»ΡŒΠ½Ρ‹Π΅, ΠΊΠΎΡ‚ΠΎΡ€Ρ‹Π΅ Ρ€Π°ΡΠΏΠΎΠ»Π°Π³Π°ΡŽΡ‚ΡΡ пСрпСндикулярно Π·Π°Π΄Π°Π½Π½ΠΎΠΉ прямой Π²ΠΈΠ΄Π° y=-2x+12. ΠšΡ€Π°ΡΠ½Ρ‹Π΅ Ρ‚ΠΎΡ‡ΠΊΠΈ – это Ρ‚ΠΎΡ‡ΠΊΠΈ касания.

ΠšΠ°ΡΠ°Ρ‚Π΅Π»ΡŒΠ½Π°Ρ ΠΊ окруТности, эллипсу, Π³ΠΈΠΏΠ΅Ρ€Π±ΠΎΠ»Π΅, ΠΏΠ°Ρ€Π°Π±ΠΎΠ»Π΅

ΠšΠ°Π½ΠΎΠ½ΠΈΡ‡Π΅ΡΠΊΠΈΠ΅ уравнСния ΠΊΡ€ΠΈΠ²Ρ‹Ρ… 2 порядка Π½Π΅ ΡΠ²Π»ΡΡŽΡ‚ΡΡ ΠΎΠ΄Π½ΠΎΠ·Π½Π°Ρ‡Π½Ρ‹ΠΌΠΈ функциями. УравнСния ΠΊΠ°ΡΠ°Ρ‚Π΅Π»ΡŒΠ½Ρ‹Ρ… для Π½ΠΈΡ… ΡΠΎΡΡ‚Π°Π²Π»ΡΡŽΡ‚ΡΡ ΠΏΠΎ извСстным схСмам.

ΠšΠ°ΡΠ°Ρ‚Π΅Π»ΡŒΠ½Π°Ρ ΠΊ окруТности

Для задания окруТности  с Ρ†Π΅Π½Ρ‚Ρ€ΠΎΠΌΒ  Π² Ρ‚ΠΎΡ‡ΠΊΠ΅ xcenter;Β ycenterΒ ΠΈ радиусом R примСняСтся Ρ„ΠΎΡ€ΠΌΡƒΠ»Π° x-xcenter2+y-ycenter2=R2.

Π”Π°Π½Π½ΠΎΠ΅ равСнство ΠΌΠΎΠΆΠ΅Ρ‚ Π±Ρ‹Ρ‚ΡŒ записано ΠΊΠ°ΠΊ объСдинСниС Π΄Π²ΡƒΡ… Ρ„ΡƒΠ½ΠΊΡ†ΠΈΠΉ:

y=R2-x-xcenter2+ycentery=-R2-x-xcenter2+ycenter

ΠŸΠ΅Ρ€Π²Π°Ρ функция располагаСтся Π²Π²Π΅Ρ€Ρ…Ρƒ, Π° вторая Π²Π½ΠΈΠ·Ρƒ, ΠΊΠ°ΠΊ ΠΏΠΎΠΊΠ°Π·Π°Π½ΠΎ Π½Π° рисункС.

Для составлСния уравнСния окруТности  Π² Ρ‚ΠΎΡ‡ΠΊΠ΅ x0;Β y0, которая располагаСтся  Π² Π²Π΅Ρ€Ρ…Π½Π΅ΠΉ ΠΈΠ»ΠΈ Π½ΠΈΠΆΠ½Π΅ΠΉ полуокруТности, слСдуСт Π½Π°ΠΉΡ‚ΠΈ ΡƒΡ€Π°Π²Π½Π΅Π½ΠΈΠ΅ Π³Ρ€Π°Ρ„ΠΈΠΊΠ° Ρ„ΡƒΠ½ΠΊΡ†ΠΈΠΈ Π²ΠΈΠ΄Π° y=R2-x-xcenter2+ycenterΒ ΠΈΠ»ΠΈ y=-R2-x-xcenter2+ycenterΒ Π² ΡƒΠΊΠ°Π·Π°Π½Π½ΠΎΠΉ Ρ‚ΠΎΡ‡ΠΊΠ΅.

Когда Π² Ρ‚ΠΎΡ‡ΠΊΠ°Ρ… xcenter;Β ycenter+RΒ ΠΈ xcenter;Β ycenter-RΒ ΠΊΠ°ΡΠ°Ρ‚Π΅Π»ΡŒΠ½Ρ‹Π΅ ΠΌΠΎΠ³ΡƒΡ‚ Π±Ρ‹Ρ‚ΡŒ Π·Π°Π΄Π°Π½Ρ‹ уравнСниями y=ycenter+RΒ ΠΈ y=ycenter-R, Π°Β  Π² Ρ‚ΠΎΡ‡ΠΊΠ°Ρ… xcenter+R;Β ycenterΒ ΠΈ
xcenter-R;Β ycenterΒ Π±ΡƒΠ΄ΡƒΡ‚ ΡΠ²Π»ΡΡ‚ΡŒΡΡ ΠΏΠ°Ρ€Π°Π»Π»Π΅Π»ΡŒΠ½Ρ‹ΠΌΠΈ ΠΎΡƒ, Ρ‚ΠΎΠ³Π΄Π° ΠΏΠΎΠ»ΡƒΡ‡ΠΈΠΌ уравнСния Π²ΠΈΠ΄Π° x=xcenter+RΒ ΠΈ x=xcenter-R.

ΠšΠ°ΡΠ°Ρ‚Π΅Π»ΡŒΠ½Π°Ρ ΠΊ эллипсу

Когда эллипс ΠΈΠΌΠ΅Π΅Ρ‚ Ρ†Π΅Π½Ρ‚Ρ€Β  Π² Ρ‚ΠΎΡ‡ΠΊΠ΅ xcenter;Β ycenter с полуосями a ΠΈ b, Ρ‚ΠΎΠ³Π΄Π° ΠΎΠ½ ΠΌΠΎΠΆΠ΅Ρ‚ Π±Ρ‹Ρ‚ΡŒ Π·Π°Π΄Π°Π½ ΠΏΡ€ΠΈ ΠΏΠΎΠΌΠΎΡ‰ΠΈ уравнСния x-xcenter2a2+y-ycenter2b2=1.

Эллипс ΠΈ ΠΎΠΊΡ€ΡƒΠΆΠ½ΠΎΡΡ‚ΡŒ ΠΌΠΎΠ³ΡƒΡ‚ Π±Ρ‹Ρ‚ΡŒ ΠΎΠ±ΠΎΠ·Π½Π°Ρ‡Π°Ρ‚ΡŒΡΡ ΠΏΡ€ΠΈ ΠΏΠΎΠΌΠΎΡ‰ΠΈ объСдинСния Π΄Π²ΡƒΡ… Ρ„ΡƒΠ½ΠΊΡ†ΠΈΠΉ, Π° ΠΈΠΌΠ΅Π½Π½ΠΎ: Π²Π΅Ρ€Ρ…Π½Π΅Π³ΠΎ ΠΈ Π½ΠΈΠΆΠ½Π΅Π³ΠΎ полуэллипса. Π’ΠΎΠ³Π΄Π° ΠΏΠΎΠ»ΡƒΡ‡Π°Π΅ΠΌ, Ρ‡Ρ‚ΠΎ

y=baΒ·a2-(x-xcenter)2+ycentery=-baΒ·a2-(x-xcenter)2+ycenter

Если  ΠΊΠ°ΡΠ°Ρ‚Π΅Π»ΡŒΠ½Ρ‹Π΅ Ρ€Π°ΡΠΏΠΎΠ»Π°Π³Π°ΡŽΡ‚ΡΡ Π½Π° Π²Π΅Ρ€ΡˆΠΈΠ½Π°Ρ… эллипса, Ρ‚ΠΎΠ³Π΄Π° ΠΎΠ½ΠΈ ΠΏΠ°Ρ€Π°Π»Π»Π΅Π»ΡŒΠ½Ρ‹ ΠΎΡ… ΠΈΠ»ΠΈ ΠΎΡƒ. НиТС для наглядности рассмотрим рисунок.

ΠŸΡ€ΠΈΠΌΠ΅Ρ€ 6

ΠΠ°ΠΏΠΈΡΠ°Ρ‚ΡŒ ΡƒΡ€Π°Π²Π½Π΅Π½ΠΈΠ΅ ΠΊΠ°ΡΠ°Ρ‚Π΅Π»ΡŒΠ½ΠΎΠΉ ΠΊ эллипсу x-324+y-5225=1Β Π² Ρ‚ΠΎΡ‡ΠΊΠ°Ρ… со значСниями x Ρ€Π°Π²Π½ΠΎΠ³ΠΎ Ρ…=2.

РСшСниС

НСобходимо Π½Π°ΠΉΡ‚ΠΈ Ρ‚ΠΎΡ‡ΠΊΠΈ касания, ΠΊΠΎΡ‚ΠΎΡ€Ρ‹Π΅ ΡΠΎΠΎΡ‚Π²Π΅Ρ‚ΡΡ‚Π²ΡƒΡŽΡ‚ Π·Π½Π°Ρ‡Π΅Π½ΠΈΡŽΒ Ρ…=2. ΠŸΡ€ΠΎΠΈΠ·Π²ΠΎΠ΄ΠΈΠΌ подстановку Π² ΠΈΠΌΠ΅ΡŽΡ‰Π΅Π΅ΡΡ ΡƒΡ€Π°Π²Π½Π΅Π½ΠΈΠ΅ эллипса ΠΈ ΠΏΠΎΠ»ΡƒΡ‡Π°Π΅ΠΌ, Ρ‡Ρ‚ΠΎ

x-324x=2+y-5225=114+y-5225=1β‡’y-52=34Β·25β‡’y=Β±532+5

Π’ΠΎΠ³Π΄Π° 2;Β 532+5Β ΠΈ 2;Β -532+5Β ΡΠ²Π»ΡΡŽΡ‚ΡΡ Ρ‚ΠΎΡ‡ΠΊΠ°ΠΌΠΈ касания, ΠΊΠΎΡ‚ΠΎΡ€Ρ‹Π΅ ΠΏΡ€ΠΈΠ½Π°Π΄Π»Π΅ΠΆΠ°Ρ‚ Π²Π΅Ρ€Ρ…Π½Π΅ΠΌΡƒ ΠΈ Π½ΠΈΠΆΠ½Π΅ΠΌΡƒ полуэллипсу.

ΠŸΠ΅Ρ€Π΅ΠΉΠ΄Π΅ΠΌ ΠΊ Π½Π°Ρ…ΠΎΠΆΠ΄Π΅Π½ΠΈΡŽ ΠΈ Ρ€Π°Π·Ρ€Π΅ΡˆΠ΅Π½ΠΈΡŽ уравнСния эллипса ΠΎΡ‚Π½ΠΎΡΠΈΡ‚Π΅Π»ΡŒΠ½ΠΎ y. ΠŸΠΎΠ»ΡƒΡ‡ΠΈΠΌ, Ρ‡Ρ‚ΠΎ

x-324+y-5225=1y-5225=1-x-324(y-5)2=25Β·1-x-324y-5=Β±5Β·1-x-324y=5Β±524-x-32

ΠžΡ‡Π΅Π²ΠΈΠ΄Π½ΠΎ, Ρ‡Ρ‚ΠΎ Π²Π΅Ρ€Ρ…Π½ΠΈΠΉ полуэллипс задаСтся с ΠΏΠΎΠΌΠΎΡ‰ΡŒΡŽ Ρ„ΡƒΠ½ΠΊΡ†ΠΈΠΈ Π²ΠΈΠ΄Π° y=5+524-x-32, Π° Π½ΠΈΠΆΠ½ΠΈΠΉ y=5-524-x-32.

ΠŸΡ€ΠΈΠΌΠ΅Π½ΠΈΠΌ стандартный Π°Π»Π³ΠΎΡ€ΠΈΡ‚ΠΌ для Ρ‚ΠΎΠ³ΠΎ, Ρ‡Ρ‚ΠΎΠ±Ρ‹ ΡΠΎΡΡ‚Π°Π²ΠΈΡ‚ΡŒ ΡƒΡ€Π°Π²Π½Π΅Π½ΠΈΠ΅ ΠΊΠ°ΡΠ°Ρ‚Π΅Π»ΡŒΠ½ΠΎΠΉ ΠΊ Π³Ρ€Π°Ρ„ΠΈΠΊΡƒ Ρ„ΡƒΠ½ΠΊΡ†ΠΈΠΈ Π² Ρ‚ΠΎΡ‡ΠΊΠ΅. Π—Π°ΠΏΠΈΡˆΠ΅ΠΌ, Ρ‡Ρ‚ΠΎ ΡƒΡ€Π°Π²Π½Π΅Π½ΠΈΠ΅ для ΠΏΠ΅Ρ€Π²ΠΎΠΉ ΠΊΠ°ΡΠ°Ρ‚Π΅Π»ΡŒΠ½ΠΎΠΉ Π² Ρ‚ΠΎΡ‡ΠΊΠ΅ 2;Β 532+5Β Π±ΡƒΠ΄Π΅Ρ‚ ΠΈΠΌΠ΅Ρ‚ΡŒ Π²ΠΈΠ΄

y’=5+524-x-32’=52Β·124-(x-3)2Β·4-(x-3)2’==-52Β·x-34-(x-3)2β‡’y'(x0)=y'(2)=-52Β·2-34-(2-3)2=523β‡’y=y'(x0)Β·x-x0+y0⇔y=523(x-2)+532+5

ΠŸΠΎΠ»ΡƒΡ‡Π°Π΅ΠΌ, Ρ‡Ρ‚ΠΎ ΡƒΡ€Π°Π²Π½Π΅Π½ΠΈΠ΅ Π²Ρ‚ΠΎΡ€ΠΎΠΉ ΠΊΠ°ΡΠ°Ρ‚Π΅Π»ΡŒΠ½ΠΎΠΉ со Π·Π½Π°Ρ‡Π΅Π½ΠΈΠ΅ΠΌΒ Π² Ρ‚ΠΎΡ‡ΠΊΠ΅
2;Β -532+5Β ΠΏΡ€ΠΈΠ½ΠΈΠΌΠ°Π΅Ρ‚ Π²ΠΈΠ΄

y’=5-524-(x-3)2’=-52Β·124-(x-3)2Β·4-(x-3)2’==52Β·x-34-(x-3)2β‡’y'(x0)=y'(2)=52Β·2-34-(2-3)2=-523β‡’y=y'(x0)Β·x-x0+y0⇔y=-523(x-2)-532+5

ГрафичСски ΠΊΠ°ΡΠ°Ρ‚Π΅Π»ΡŒΠ½Ρ‹Π΅ ΠΎΠ±ΠΎΠ·Π½Π°Ρ‡Π°ΡŽΡ‚ΡΡ Β Ρ‚Π°ΠΊ:

ΠšΠ°ΡΠ°Ρ‚Π΅Π»ΡŒΠ½Π°Ρ ΠΊ Π³ΠΈΠΏΠ΅Ρ€Π±ΠΎΠ»Π΅

Когда Π³ΠΈΠΏΠ΅Ρ€Π±ΠΎΠ»Π° ΠΈΠΌΠ΅Π΅Ρ‚ Ρ†Π΅Π½Ρ‚Ρ€ Π² Ρ‚ΠΎΡ‡ΠΊΠ΅ xcenter;Β ycenterΒ ΠΈ Π²Π΅Ρ€ΡˆΠΈΠ½Ρ‹ xcenter+Ξ±;Β ycenterΒ ΠΈ xcenter-Ξ±;Β ycenter, ΠΈΠΌΠ΅Π΅Ρ‚ мСсто Π·Π°Π΄Π°Π½ΠΈΠ΅ нСравСнства x-xcenter2Ξ±2-y-ycenter2b2=1, Ссли с Π²Π΅Ρ€ΡˆΠΈΠ½Π°ΠΌΠΈΒ xcenter;Β ycenter+bΒ ΠΈ xcenter;Β ycenter-b, Ρ‚ΠΎΠ³Π΄Π° задаСтся ΠΏΡ€ΠΈ ΠΏΠΎΠΌΠΎΡ‰ΠΈ нСравСнства x-xcenter2Ξ±2-y-ycenter2b2=-1.

Π“ΠΈΠΏΠ΅Ρ€Π±ΠΎΠ»Π° ΠΌΠΎΠΆΠ΅Ρ‚ Π±Ρ‹Ρ‚ΡŒ прСдставлСна Π² Π²ΠΈΠ΄Π΅ Π΄Π²ΡƒΡ… ΠΎΠ±ΡŠΠ΅Π΄ΠΈΠ½Π΅Π½Π½Ρ‹Ρ… Ρ„ΡƒΠ½ΠΊΡ†ΠΈΠΉ Π²ΠΈΠ΄Π°

y=baΒ·(x-xcenter)2-a2+ycentery=-baΒ·(x-xcenter)2-a2+ycenterΒ ΠΈΠ»ΠΈ y=baΒ·(x-xcenter)2+a2+ycentery=-baΒ·(x-xcenter)2+a2+ycenter

Π’ ΠΏΠ΅Ρ€Π²ΠΎΠΌ случаС ΠΈΠΌΠ΅Π΅ΠΌ, Ρ‡Ρ‚ΠΎ ΠΊΠ°ΡΠ°Ρ‚Π΅Π»ΡŒΠ½Ρ‹Π΅ ΠΏΠ°Ρ€Π°Π»Π»Π΅Π»ΡŒΠ½Ρ‹ ΠΎΡƒ, Π° Π²ΠΎ Π²Ρ‚ΠΎΡ€ΠΎΠΌ ΠΏΠ°Ρ€Π°Π»Π»Π΅Π»ΡŒΠ½Ρ‹ ΠΎΡ….

ΠžΡ‚ΡΡŽΠ΄Π° слСдуСт, Ρ‡Ρ‚ΠΎ для Ρ‚ΠΎΠ³ΠΎ, Ρ‡Ρ‚ΠΎΠ±Ρ‹ Π½Π°ΠΉΡ‚ΠΈ ΡƒΡ€Π°Π²Π½Π΅Π½ΠΈΠ΅ ΠΊΠ°ΡΠ°Ρ‚Π΅Π»ΡŒΠ½ΠΎΠΉ ΠΊ Π³ΠΈΠΏΠ΅Ρ€Π±ΠΎΠ»Π΅, Π½Π΅ΠΎΠ±Ρ…ΠΎΠ΄ΠΈΠΌΠΎ Π²Ρ‹ΡΡΠ½ΠΈΡ‚ΡŒ, ΠΊΠ°ΠΊΠΎΠΉ Ρ„ΡƒΠ½ΠΊΡ†ΠΈΠΈ ΠΏΡ€ΠΈΠ½Π°Π΄Π»Π΅ΠΆΠΈΡ‚ Ρ‚ΠΎΡ‡ΠΊΠ° касания. Π§Ρ‚ΠΎΠ±Ρ‹ ΠΎΠΏΡ€Π΅Π΄Π΅Π»ΠΈΡ‚ΡŒ это, Π½Π΅ΠΎΠ±Ρ…ΠΎΠ΄ΠΈΠΌΠΎ произвСсти подстановку Π² уравнСния ΠΈ ΠΏΡ€ΠΎΠ²Π΅Ρ€ΠΈΡ‚ΡŒ ΠΈΡ… Π½Π° Ρ‚ΠΎΠΆΠ΄Π΅ΡΡ‚Π²Π΅Π½Π½ΠΎΡΡ‚ΡŒ.

ΠŸΡ€ΠΈΠΌΠ΅Ρ€ 7

Π‘ΠΎΡΡ‚Π°Π²ΠΈΡ‚ΡŒ ΡƒΡ€Π°Π²Π½Π΅Π½ΠΈΠ΅ ΠΊΠ°ΡΠ°Ρ‚Π΅Π»ΡŒΠ½ΠΎΠΉ ΠΊ Π³ΠΈΠΏΠ΅Ρ€Π±ΠΎΠ»Π΅ x-324-y+329=1Β Π² Ρ‚ΠΎΡ‡ΠΊΠ΅ 7;Β -33-3.

РСшСниС

НСобходимо ΠΏΡ€Π΅ΠΎΠ±Ρ€Π°Π·ΠΎΠ²Π°Ρ‚ΡŒ запись Ρ€Π΅ΡˆΠ΅Π½ΠΈΡ нахоТдСния Π³ΠΈΠΏΠ΅Ρ€Π±ΠΎΠ»Ρ‹ ΠΏΡ€ΠΈ ΠΏΠΎΠΌΠΎΡ‰ΠΈ 2 Ρ„ΡƒΠ½ΠΊΡ†ΠΈΠΉ. ΠŸΠΎΠ»ΡƒΡ‡ΠΈΠΌ, Ρ‡Ρ‚ΠΎ

x-324-y+329=1β‡’y+329=x-324-1β‡’y+32=9Β·x-324-1β‡’y+3=32Β·x-32-4Β ΠΈΠ»ΠΈΒ y+3=-32Β·x-32-4β‡’y=32Β·x-32-4-3y=-32Β·x-32-4-3

НСобходимо Π²Ρ‹ΡΠ²ΠΈΡ‚ΡŒ, ΠΊ ΠΊΠ°ΠΊΠΎΠΉ Ρ„ΡƒΠ½ΠΊΡ†ΠΈΠΈ ΠΏΡ€ΠΈΠ½Π°Π΄Π»Π΅ΠΆΠΈΡ‚ заданная Ρ‚ΠΎΡ‡ΠΊΠ° с ΠΊΠΎΠΎΡ€Π΄ΠΈΠ½Π°Ρ‚Π°ΠΌΠΈ 7;Β -33-3.

ΠžΡ‡Π΅Π²ΠΈΠ΄Π½ΠΎ, Ρ‡Ρ‚ΠΎ для ΠΏΡ€ΠΎΠ²Π΅Ρ€ΠΊΠΈ ΠΏΠ΅Ρ€Π²ΠΎΠΉ Ρ„ΡƒΠ½ΠΊΡ†ΠΈΠΈ Π½Π΅ΠΎΠ±Ρ…ΠΎΠ΄ΠΈΠΌΠΎ y(7)=32Β·(7-3)2-4-3=33-3β‰ -33-3, Ρ‚ΠΎΠ³Π΄Π° Ρ‚ΠΎΡ‡ΠΊΠ° Π³Ρ€Π°Ρ„ΠΈΠΊΡƒ Π½Π΅ ΠΏΡ€ΠΈΠ½Π°Π΄Π»Π΅ΠΆΠΈΡ‚, Ρ‚Π°ΠΊ ΠΊΠ°ΠΊ равСнство Π½Π΅ выполняСтся.

Для Π²Ρ‚ΠΎΡ€ΠΎΠΉ Ρ„ΡƒΠ½ΠΊΡ†ΠΈΠΈ ΠΈΠΌΠ΅Π΅ΠΌ, Ρ‡Ρ‚ΠΎ y(7)=-32Β·(7-3)2-4-3=-33-3β‰ -33-3, Π·Π½Π°Ρ‡ΠΈΡ‚, Ρ‚ΠΎΡ‡ΠΊΠ° ΠΏΡ€ΠΈΠ½Π°Π΄Π»Π΅ΠΆΠΈΡ‚ Π·Π°Π΄Π°Π½Π½ΠΎΠΌΡƒ Π³Ρ€Π°Ρ„ΠΈΠΊΡƒ. ΠžΡ‚ΡΡŽΠ΄Π° слСдуСт Π½Π°ΠΉΡ‚ΠΈ ΡƒΠ³Π»ΠΎΠ²ΠΎΠΉ коэффициСнт.

ΠŸΠΎΠ»ΡƒΡ‡Π°Π΅ΠΌ, Ρ‡Ρ‚ΠΎ

y’=-32Β·(x-3)2-4-3’=-32Β·x-3(x-3)2-4β‡’kx=y'(x0)=-32Β·x0-3×0-32-4×0=7=-32Β·7-37-32-4=-3

ΠžΡ‚Π²Π΅Ρ‚: ΡƒΡ€Π°Π²Π½Π΅Π½ΠΈΠ΅ ΠΊΠ°ΡΠ°Ρ‚Π΅Π»ΡŒΠ½ΠΎΠΉ ΠΌΠΎΠΆΠ½ΠΎ ΠΏΡ€Π΅Π΄ΡΡ‚Π°Π²ΠΈΡ‚ΡŒ ΠΊΠ°ΠΊ

y=-3Β·x-7-33-3=-3Β·x+43-3

Наглядно изобраТаСтся Ρ‚Π°ΠΊ:

ΠšΠ°ΡΠ°Ρ‚Π΅Π»ΡŒΠ½Π°Ρ ΠΊ ΠΏΠ°Ρ€Π°Π±ΠΎΠ»Π΅

Π§Ρ‚ΠΎΠ±Ρ‹ ΡΠΎΡΡ‚Π°Π²ΠΈΡ‚ΡŒ ΡƒΡ€Π°Π²Π½Π΅Π½ΠΈΠ΅ ΠΊΠ°ΡΠ°Ρ‚Π΅Π»ΡŒΠ½ΠΎΠΉ ΠΊ ΠΏΠ°Ρ€Π°Π±ΠΎΠ»Π΅ y=ax2+bx+cΒ Π² Ρ‚ΠΎΡ‡ΠΊΠ΅ x0,Β y(x0), Π½Π΅ΠΎΠ±Ρ…ΠΎΠ΄ΠΈΠΌΠΎ ΠΈΡΠΏΠΎΠ»ΡŒΠ·ΠΎΠ²Π°Ρ‚ΡŒ стандартный Π°Π»Π³ΠΎΡ€ΠΈΡ‚ΠΌ, Ρ‚ΠΎΠ³Π΄Π° ΡƒΡ€Π°Π²Π½Π΅Π½ΠΈΠ΅ ΠΏΡ€ΠΈΠΌΠ΅Ρ‚ Π²ΠΈΠ΄ y=y'(x0)Β·x-x0+y(x0). Вакая ΠΊΠ°ΡΠ°Ρ‚Π΅Π»ΡŒΠ½Π°Ρ Π² Π²Π΅Ρ€ΡˆΠΈΠ½Π΅ ΠΏΠ°Ρ€Π°Π»Π»Π΅Π»ΡŒΠ½Π° ΠΎΡ….

Π‘Π»Π΅Π΄ΡƒΠ΅Ρ‚ Π·Π°Π΄Π°Ρ‚ΡŒ ΠΏΠ°Ρ€Π°Π±ΠΎΠ»Ρƒ x=ay2+by+cΒ ΠΊΠ°ΠΊ объСдинСниС Π΄Π²ΡƒΡ… Ρ„ΡƒΠ½ΠΊΡ†ΠΈΠΉ. ΠŸΠΎΡΡ‚ΠΎΠΌΡƒ Π½ΡƒΠΆΠ½ΠΎ Ρ€Π°Π·Ρ€Π΅ΡˆΠΈΡ‚ΡŒ ΡƒΡ€Π°Π²Π½Π΅Π½ΠΈΠ΅ ΠΎΡ‚Π½ΠΎΡΠΈΡ‚Π΅Π»ΡŒΠ½ΠΎ Ρƒ. ΠŸΠΎΠ»ΡƒΡ‡Π°Π΅ΠΌ, Ρ‡Ρ‚ΠΎ

x=ay2+by+c⇔ay2+by+c-x=0D=b2-4a(c-x)y=-b+b2-4a(c-x)2ay=-b-b2-4a(c-x)2a

ГрафичСски ΠΈΠ·ΠΎΠ±Ρ€Π°Π·ΠΈΠΌ ΠΊΠ°ΠΊ:

Для выяснСния принадлСТности Ρ‚ΠΎΡ‡ΠΊΠΈ x0,Β y(x0)Β Ρ„ΡƒΠ½ΠΊΡ†ΠΈΠΈ, Π½Π΅ΠΆΠ½ΠΎ Π΄Π΅ΠΉΡΡ‚Π²ΠΎΠ²Π°Ρ‚ΡŒ ΠΏΠΎ стандартному Π°Π»Π³ΠΎΡ€ΠΈΡ‚ΠΌΡƒ. Вакая ΠΊΠ°ΡΠ°Ρ‚Π΅Π»ΡŒΠ½Π°Ρ Π±ΡƒΠ΄Π΅Ρ‚ ΠΏΠ°Ρ€Π°Π»Π»Π΅Π»ΡŒΠ½Π° ΠΎΡƒ ΠΎΡ‚Π½ΠΎΡΠΈΡ‚Π΅Π»ΡŒΠ½ΠΎ ΠΏΠ°Ρ€Π°Π±ΠΎΠ»Ρ‹.

ΠŸΡ€ΠΈΠΌΠ΅Ρ€ 8

ΠΠ°ΠΏΠΈΡΠ°Ρ‚ΡŒ ΡƒΡ€Π°Π²Π½Π΅Π½ΠΈΠ΅ ΠΊΠ°ΡΠ°Ρ‚Π΅Π»ΡŒΠ½ΠΎΠΉ ΠΊ Π³Ρ€Π°Ρ„ΠΈΠΊΡƒ x-2y2-5y+3, ΠΊΠΎΠ³Π΄Π° ΠΈΠΌΠ΅Π΅ΠΌ ΡƒΠ³ΠΎΠ» Π½Π°ΠΊΠ»ΠΎΠ½Π° ΠΊΠ°ΡΠ°Ρ‚Π΅Π»ΡŒΠ½ΠΎΠΉ 150Β°.

РСшСниС

НачинаСм Ρ€Π΅ΡˆΠ΅Π½ΠΈΠ΅ с прСдставлСния ΠΏΠ°Ρ€Π°Π±ΠΎΠ»Ρ‹ Π² качСствС Π΄Π²ΡƒΡ… Ρ„ΡƒΠ½ΠΊΡ†ΠΈΠΉ. ΠŸΠΎΠ»ΡƒΡ‡ΠΈΠΌ, Ρ‡Ρ‚ΠΎ

-2y2-5y+3-x=0D=(-5)2-4Β·(-2)Β·(3-x)=49-8xy=5+49-8x-4y=5-49-8x-4

Π—Π½Π°Ρ‡Π΅Π½ΠΈΠ΅ ΡƒΠ³Π»ΠΎΠ²ΠΎΠ³ΠΎ коэффициСнта равняСтся Π·Π½Π°Ρ‡Π΅Π½ΠΈΡŽ ΠΏΡ€ΠΎΠΈΠ·Π²ΠΎΠ΄Π½ΠΎΠΉ Π² Ρ‚ΠΎΡ‡ΠΊΠ΅ x0 этой Ρ„ΡƒΠ½ΠΊΡ†ΠΈΠΈ ΠΈ равняСтся тангСнсу ΡƒΠ³Π»Π° Π½Π°ΠΊΠ»ΠΎΠ½Π°.

ΠŸΠΎΠ»ΡƒΡ‡Π°Π΅ΠΌ:

kx=y'(x0)=tgΒ Ξ±x=tgΒ 150Β°=-13

ΠžΡ‚ΡΡŽΠ΄Π° ΠΎΠΏΡ€Π΅Π΄Π΅Π»ΠΈΠΌ Π·Π½Π°Ρ‡Π΅Π½ΠΈΠ΅ Ρ… для Ρ‚ΠΎΡ‡Π΅ΠΊ касания.

ΠŸΠ΅Ρ€Π²Π°Ρ функция Π·Π°ΠΏΠΈΡˆΠ΅Ρ‚ΡΡ ΠΊΠ°ΠΊ

y’=5+49-8x-4’=149-8xβ‡’y'(x0)=149-8×0=-13⇔49-8×0=-3

ΠžΡ‡Π΅Π²ΠΈΠ΄Π½ΠΎ, Ρ‡Ρ‚ΠΎ Π΄Π΅ΠΉΡΡ‚Π²ΠΈΡ‚Π΅Π»ΡŒΠ½Ρ‹Ρ… ΠΊΠΎΡ€Π½Π΅ΠΉ Π½Π΅Ρ‚, Ρ‚Π°ΠΊ ΠΊΠ°ΠΊ ΠΏΠΎΠ»ΡƒΡ‡ΠΈΠ»ΠΈ ΠΎΡ‚Ρ€ΠΈΡ†Π°Ρ‚Π΅Π»ΡŒΠ½ΠΎΠ΅ Π·Π½Π°Ρ‡Π΅Π½ΠΈΠ΅. Π”Π΅Π»Π°Π΅ΠΌ Π²Ρ‹Π²ΠΎΠ΄, Ρ‡Ρ‚ΠΎ ΠΊΠ°ΡΠ°Ρ‚Π΅Π»ΡŒΠ½ΠΎΠΉ с ΡƒΠ³Π»ΠΎΠΌ 150° для Ρ‚Π°ΠΊΠΎΠΉ Ρ„ΡƒΠ½ΠΊΡ†ΠΈΠΈ Π½Π΅ сущСствуСт.

Вторая функция Π·Π°ΠΏΠΈΡˆΠ΅Ρ‚ΡΡ ΠΊΠ°ΠΊ

y’=5-49-8x-4’=-149-8xβ‡’y'(x0)=-149-8×0=-13⇔49-8×0=-3×0=234β‡’y(x0)=5-49-8Β·234-4=-5+34

ИмССм, Ρ‡Ρ‚ΠΎ Ρ‚ΠΎΡ‡ΠΊΠΈ касания —Β 234;Β -5+34.

ΠžΡ‚Π²Π΅Ρ‚: ΡƒΡ€Π°Π²Π½Π΅Π½ΠΈΠ΅ ΠΊΠ°ΡΠ°Ρ‚Π΅Π»ΡŒΠ½ΠΎΠΉ ΠΏΡ€ΠΈΠ½ΠΈΠΌΠ°Π΅Ρ‚ Π²ΠΈΠ΄

y=-13Β·x-234+-5+34

ГрафичСски ΠΈΠ·ΠΎΠ±Ρ€Π°Π·ΠΈΠΌ это Ρ‚Π°ΠΊΠΈΠΌ ΠΎΠ±Ρ€Π°Π·ΠΎΠΌ:

РСшСниС Π·Π°Π΄Π°Ρ‡ ΠΎΡ‚Β 1 дня / ΠΎΡ‚Β 150Β Ρ€. ΠšΡƒΡ€ΡΠΎΠ²Π°Ρ Ρ€Π°Π±ΠΎΡ‚Π° ΠΎΡ‚Β 5Β Π΄Π½Π΅ΠΉ / ΠΎΡ‚Β 1800Β Ρ€. Π Π΅Ρ„Π΅Ρ€Π°Ρ‚ ΠΎΡ‚Β 1 дня / ΠΎΡ‚Β 700Β Ρ€.

ΠšΠ°ΡΠ°Ρ‚Π΅Π»ΡŒΠ½Π°Ρ ΠΊ Π³Ρ€Π°Ρ„ΠΈΠΊΡƒ

ΠžΠΏΡ€Π΅Π΄Π΅Π»Π΅Π½ΠΈΡ и понятия.

ΠžΠΏΡ€Π΅Π΄Π΅Π»Π΅Π½ΠΈΠ΅.

Π£Π³Π»ΠΎΠΌ Π½Π°ΠΊΠ»ΠΎΠ½Π°Β  прямой y=kx+bΒ Π½Π°Π·Ρ‹Π²Π°ΡŽΡ‚ ΡƒΠ³ΠΎΠ»Β  , отсчитываСмый ΠΎΡ‚ ΠΏΠΎΠ»ΠΎΠΆΠΈΡ‚Π΅Π»ΡŒΠ½ΠΎΠ³ΠΎ направлСния оси абсцисс Π΄ΠΎ прямой y=kx+bΒ Π² ΠΏΠΎΠ»ΠΎΠΆΠΈΡ‚Π΅Π»ΡŒΠ½ΠΎΠΌ Π½Π°ΠΏΡ€Π°Π²Π»Π΅Π½ΠΈΠΈ (Ρ‚ΠΎ Π΅ΡΡ‚ΡŒ, ΠΏΡ€ΠΎΡ‚ΠΈΠ² часовой стрСлки).

ΠΠ°Β Ρ€ΠΈΡΡƒΠ½ΠΊΠ΅Β ΠΏΠΎΠ»ΠΎΠΆΠΈΡ‚Π΅Π»ΡŒΠ½ΠΎΠ΅Β  направлСниС оси абсцисс показано Π³ΠΎΡ€ΠΈΠ·ΠΎΠ½Ρ‚Π°Π»ΡŒΠ½ΠΎΠΉ зСлСной стрСлочкой, ΠΏΠΎΠ»ΠΎΠΆΠΈΡ‚Π΅Π»ΡŒΠ½ΠΎΠ΅Β Π½Π°ΠΏΡ€Π°Π²Π»Π΅Π½ΠΈΠ΅Β ΠΎΡ‚ΡΡ‡Π΅Ρ‚Π°Β  ΡƒΠ³Π»Π° ΠΈΠ·ΠΎΠ±Ρ€Π°ΠΆΠ΅Π½ΠΎ Π·Π΅Π»Π΅Π½ΠΎΠΉΒ Π΄ΡƒΠ³ΠΎΠΉ, прямая ΠΏΠΎΠΊΠ°Π·Π°Π½Π° синСй Π»ΠΈΠ½ΠΈΠ΅ΠΉ, Π° ΡƒΠ³ΠΎΠ» Π½Π°ΠΊΠ»ΠΎΠ½Π° прямой — красной Π΄ΡƒΠ³ΠΎΠΉ.

ΠžΠΏΡ€Π΅Π΄Π΅Π»Π΅Π½ΠΈΠ΅.

Π£Π³Π»ΠΎΠ²Ρ‹ΠΌ коэффициСнтом прямой y=kx+bΒ Π½Π°Π·Ρ‹Π²Π°ΡŽΡ‚ числовой коэффициСнт k.

Π£Π³Π»ΠΎΠ²ΠΎΠΉ коэффициСнт прямой равСн тангСнсу ΡƒΠ³Π»Π° наклона прямой, Ρ‚ΠΎ Π΅ΡΡ‚ΡŒ,Β  .

  • Π£Π³ΠΎΠ» Π½Π°ΠΊΠ»ΠΎΠ½Π° прямой Ρ€Π°Π²Π΅Π½ Π½ΡƒΠ»ΡŽ, ΠΊΠΎΠ³Π΄Π° прямая ΠΏΠ°Ρ€Π°Π»Π»Π΅Π»ΡŒΠ½Π° оси абсцисс. Π’ этом случаС Π½ΡƒΠ»ΡŽ Ρ€Π°Π²Π΅Π½ ΠΈ ΡƒΠ³Π»ΠΎΠ²ΠΎΠΉ коэффициСнт, Ρ‚Π°ΠΊ ΠΊΠ°ΠΊ тангСнс нуля Π΅ΡΡ‚ΡŒ ноль. Π‘Π»Π΅Π΄ΠΎΠ²Π°Ρ‚Π΅Π»ΡŒΠ½ΠΎ, ΡƒΡ€Π°Π²Π½Π΅Π½ΠΈΠ΅ прямой Π±ΡƒΠ΄Π΅Ρ‚ ΠΈΠΌΠ΅Ρ‚ΡŒ Π²ΠΈΠ΄Β y=b.
  • Когда ΡƒΠ³ΠΎΠ» Π½Π°ΠΊΠ»ΠΎΠ½Π° прямой y=kx+b являСтся острым ( Β ΠΈΠ»ΠΈΒ  ), Ρ‚ΠΎ ΡƒΠ³Π»ΠΎΠ²ΠΎΠΉ коэффициСнт k являСтся ΠΏΠΎΠ»ΠΎΠΆΠΈΡ‚Π΅Π»ΡŒΠ½Ρ‹ΠΌ числом (Ρ‚Π°ΠΊ ΠΊΠ°ΠΊ тангСнс острого ΡƒΠ³Π»Π°Β  Β ΠΏΡ€ΠΈΠ½ΠΈΠΌΠ°Π΅Ρ‚ ΠΏΠΎΠ»ΠΎΠΆΠΈΡ‚Π΅Π»ΡŒΠ½Ρ‹Π΅ значСния  ) ΠΈ ΡƒΠΊΠ°Π·Ρ‹Π²Π°Π΅Ρ‚ Π½Π° возрастаниС Π³Ρ€Π°Ρ„ΠΈΠΊΠ° прямой.
  • Π’ случаС, ΠΊΠΎΠ³Π΄Π°Β   прямая располагаСтся пСрпСндикулярно оси абсцисс (ΠΏΠ°Ρ€Π°Π»Π»Π΅Π»ΡŒΠ½ΠΎ оси ΠΎΡ€Π΄ΠΈΠ½Π°Ρ‚) ΠΈ задаСтся равСнством x=c, Π³Π΄Π΅Β cΒ — Π½Π΅ΠΊΠΎΡ‚ΠΎΡ€ΠΎΠ΅ Π΄Π΅ΠΉΡΡ‚Π²ΠΈΡ‚Π΅Π»ΡŒΠ½ΠΎΠ΅ число.
  • Когда ΡƒΠ³ΠΎΠ» Π½Π°ΠΊΠ»ΠΎΠ½Π° прямой y=kx+b являСтся Ρ‚ΡƒΠΏΡ‹ΠΌ ( Β ΠΈΠ»ΠΈΒ  ), Ρ‚ΠΎ ΡƒΠ³Π»ΠΎΠ²ΠΎΠΉ коэффициСнт k являСтся ΠΎΡ‚Ρ€ΠΈΡ†Π°Ρ‚Π΅Π»ΡŒΠ½Ρ‹ΠΌ числом ΠΈ ΡƒΠΊΠ°Π·Ρ‹Π²Π°Π΅Ρ‚ Π½Π° ΡƒΠ±Ρ‹Π²Π°Π½ΠΈΠ΅ Π³Ρ€Π°Ρ„ΠΈΠΊΠ° прямой.

ΠžΠΏΡ€Π΅Π΄Π΅Π»Π΅Π½ΠΈΠ΅.

ΠŸΡ€ΡΠΌΡƒΡŽΒ AB, ΠΏΡ€ΠΎΠ²Π΅Π΄Π΅Π½Π½ΡƒΡŽ Ρ‡Π΅Ρ€Π΅Π· Π΄Π²Π΅ Ρ‚ΠΎΡ‡ΠΊΠΈ Π³Ρ€Π°Ρ„ΠΈΠΊΠ° Ρ„ΡƒΠ½ΠΊΡ†ΠΈΠΈΒ y=f(x), Π½Π°Π·Ρ‹Π²Π°ΡŽΡ‚Β ΡΠ΅ΠΊΡƒΡ‰Π΅ΠΉ. Π”Ρ€ΡƒΠ³ΠΈΠΌΠΈ словами, сСкущая – это прямая, проходящая Ρ‡Π΅Ρ€Π΅Π· Π΄Π²Π΅ Ρ‚ΠΎΡ‡ΠΊΠΈ Π³Ρ€Π°Ρ„ΠΈΠΊΠ° Ρ„ΡƒΠ½ΠΊΡ†ΠΈΠΈ.

На рисункС сСкущая  прямая ABΒ ΠΈΠ·ΠΎΠ±Ρ€Π°ΠΆΠ΅Π½Π° синСй Π»ΠΈΠ½ΠΈΠ΅ΠΉ, Π³Ρ€Π°Ρ„ΠΈΠΊ Ρ„ΡƒΠ½ΠΊΡ†ΠΈΠΈΒ y=f(x)Β — Ρ‡Π΅Ρ€Π½ΠΎΠΉ ΠΊΡ€ΠΈΠ²ΠΎΠΉ, ΡƒΠ³ΠΎΠ» Π½Π°ΠΊΠ»ΠΎΠ½Π° сСкущСй  Β — красной Π΄ΡƒΠ³ΠΎΠΉ.

Π•ΡΠ»ΠΈΒ ΠΏΡ€ΠΈΠ½ΠΈΠΌΠ°Ρ‚ΡŒΒ  Π²ΠΎ Π²Π½ΠΈΠΌΠ°Π½ΠΈΠ΅, что угловой коэффициСнт  прямой равСн тангСнсу ΡƒΠ³Π»Π° Π½Π°ΠΊΠ»ΠΎΠ½Π° (ΠΎΠ± этом Π³ΠΎΠ²ΠΎΡ€ΠΈΠ»ΠΈ Π²Ρ‹ΡˆΠ΅), и тангСнс  ΡƒΠ³Π»Π° Π²Β ΠΏΡ€ΡΠΌΠΎΡƒΠ³ΠΎΠ»ΡŒΠ½ΠΎΠΌΒ Ρ‚Ρ€Π΅ΡƒΠ³ΠΎΠ»ΡŒΠ½ΠΈΠΊΠ΅Β ABCΒ Π΅ΡΡ‚ΡŒ ΠΎΡ‚Π½ΠΎΡˆΠ΅Π½ΠΈΠ΅ ΠΏΡ€ΠΎΡ‚ΠΈΠ²ΠΎΠ»Π΅ΠΆΠ°Ρ‰Π΅Π³ΠΎ ΠΊΠ°Ρ‚Π΅Ρ‚Π° ΠΊ ΠΏΡ€ΠΈΠ»Π΅ΠΆΠ°Ρ‰Π΅ΠΌΡƒ (это ΠΎΠΏΡ€Π΅Π΄Π΅Π»Π΅Π½ΠΈΠ΅ тангСнса ΡƒΠ³Π»Π°), Ρ‚ΠΎ для нашСй сСкущСй Π±ΡƒΠ΄Π΅Ρ‚ справСдлива сСрия равСнств  , Π³Π΄Π΅Β  Β — абсциссы точСк А и В,Β  Β — ΡΠΎΠΎΡ‚Π²Π΅Ρ‚ΡΡ‚Π²ΡƒΡŽΡ‰ΠΈΠ΅ значСния Ρ„ΡƒΠ½ΠΊΡ†ΠΈΠΈ.

Π’ΠΎΒ Π΅ΡΡ‚ΡŒ,Β ΡƒΠ³Π»ΠΎΠ²ΠΎΠΉ коэффициСнт сСкущСй опрСдСляСтся равСнством  Β ΠΈΠ»ΠΈΒ  , Π°Β ΡƒΡ€Π°Π²Π½Π΅Π½ΠΈΠ΅ сСкущСй записываСтся Π² Π²ΠΈΠ΄Π΅Β  Β ΠΈΠ»ΠΈΒ  Β (ΠΏΡ€ΠΈ нСобходимости ΠΎΠ±Ρ€Π°Ρ‰Π°ΠΉΡ‚Π΅ΡΡŒ ΠΊ Ρ€Π°Π·Π΄Π΅Π»ΡƒΒ ΡƒΡ€Π°Π²Π½Π΅Π½ΠΈΠ΅ прямой с ΡƒΠ³Π»ΠΎΠ²Ρ‹ΠΌ коэффициСнтом, проходящСй Ρ‡Π΅Ρ€Π΅Π· Π·Π°Π΄Π°Π½Π½ΡƒΡŽ Ρ‚ΠΎΡ‡ΠΊΡƒ).

БСкущая прямая Ρ€Π°Π·Π±ΠΈΠ²Π°Π΅Ρ‚Β  Π³Ρ€Π°Ρ„ΠΈΠΊ Ρ„ΡƒΠ½ΠΊΡ†ΠΈΠΈΒ Π½Π° три части: слСва  ΠΎΡ‚ точки А, от А до В и справа ΠΎΡ‚ Ρ‚ΠΎΡ‡ΠΊΠΈΒ Π’, хотя ΠΌΠΎΠΆΠ΅Ρ‚ ΠΈΠΌΠ΅Ρ‚ΡŒ Π±ΠΎΠ»Π΅Π΅ Ρ‡Π΅ΠΌ Π΄Π²Π΅ ΠΎΠ±Ρ‰ΠΈΡ… Ρ‚ΠΎΡ‡ΠΊΠΈ с Π³Ρ€Π°Ρ„ΠΈΠΊΠΎΠΌ Ρ„ΡƒΠ½ΠΊΡ†ΠΈΠΈ.

На рисункС ниТС привСдСны три фактичСски Ρ€Π°Π·Π½Ρ‹Ρ… сСкущих (точки А и В различны), Π½ΠΎ ΠΎΠ½ΠΈ ΡΠΎΠ²ΠΏΠ°Π΄Π°ΡŽΡ‚ ΠΈ Π·Π°Π΄Π°ΡŽΡ‚ΡΡ ΠΎΠ΄Π½ΠΈΠΌ ΡƒΡ€Π°Π²Π½Π΅Π½ΠΈΠ΅ΠΌ.

Нам Π½ΠΈ Ρ€Π°Π·Ρƒ Π½Π΅ Π²ΡΡ‚Ρ€Π΅Ρ‡Π°Π»ΠΈΡΡŒΒ  разговоры о сСкущСй прямой для прямой. Но всС ΠΆΠ΅, Ссли ΠΎΡ‚Ρ‚Π°Π»ΠΊΠΈΠ²Π°Ρ‚ΡŒΡΡ ΠΎΡ‚ опрСдСлСния, Ρ‚ΠΎ прямая ΠΈ Π΅Π΅ сСкущая прямая ΡΠΎΠ²ΠΏΠ°Π΄Π°ΡŽΡ‚.

Π’ нСкоторых случаях  ΡΠ΅ΠΊΡƒΡ‰Π°ΡΒ ΠΌΠΎΠΆΠ΅Ρ‚Β ΠΈΠΌΠ΅Ρ‚ΡŒΒ ΡΒ Π³Ρ€Π°Ρ„ΠΈΠΊΠΎΠΌ функции бСсконСчноС число точСк  пСрСсСчСния. НапримСр, сСкущая, опрСдСляСмая ΡƒΡ€Π°Π²Π½Π΅Π½ΠΈΠ΅ΠΌΒ y=0, ΠΈΠΌΠ΅Π΅Ρ‚ бСсконСчноС число ΠΎΠ±Ρ‰ΠΈΡ… Ρ‚ΠΎΡ‡Π΅ΠΊ с синусоидой.

ΠžΠΏΡ€Π΅Π΄Π΅Π»Π΅Π½ΠΈΠ΅.

ΠšΠ°ΡΠ°Ρ‚Π΅Π»ΡŒΠ½ΠΎΠΉ ΠΊΒ Π³Ρ€Π°Ρ„ΠΈΠΊΡƒΒ Ρ„ΡƒΠ½ΠΊΡ†ΠΈΠΈΒ y=f(x)Β Π² Ρ‚ΠΎΡ‡ΠΊΠ΅Β  Β Π½Π°Π·Ρ‹Π²Π°ΡŽΡ‚ ΠΏΡ€ΡΠΌΡƒΡŽ, ΠΏΡ€ΠΎΡ…ΠΎΠ΄ΡΡ‰ΡƒΡŽ Ρ‡Π΅Ρ€Π΅Π· Ρ‚ΠΎΡ‡ΠΊΡƒΒ  , с ΠΎΡ‚Ρ€Π΅Π·ΠΊΠΎΠΌ ΠΊΠΎΡ‚ΠΎΡ€ΠΎΠΉ практичСски сливаСтся Π³Ρ€Π°Ρ„ΠΈΠΊ Ρ„ΡƒΠ½ΠΊΡ†ΠΈΠΈ ΠΏΡ€ΠΈ Π·Π½Π°Ρ‡Π΅Π½ΠΈΡΡ…Β Ρ…Β ΡΠΊΠΎΠ»ΡŒ ΡƒΠ³ΠΎΠ΄Π½ΠΎ Π±Π»ΠΈΠ·ΠΊΠΈΡ… ΠΊΒ  .

Поясним это опрСдСлСниС  Π½Π° ΠΏΡ€ΠΈΠΌΠ΅Ρ€Π΅. ПокаТСм, что прямая y = x+1 являСтся ΠΊΠ°ΡΠ°Ρ‚Π΅Π»ΡŒΠ½ΠΎΠΉ ΠΊ Π³Ρ€Π°Ρ„ΠΈΠΊΡƒ Ρ„ΡƒΠ½ΠΊΡ†ΠΈΠΈΒ  Β Π² Ρ‚ΠΎΡ‡ΠΊΠ΅Β (1; 2). Для этого ΠΏΠΎΠΊΠ°ΠΆΠ΅ΠΌ Π³Ρ€Π°Ρ„ΠΈΠΊΠΈ этих Ρ„ΡƒΠ½ΠΊΡ†ΠΈΠΉ ΠΏΡ€ΠΈ ΠΏΡ€ΠΈΠ±Π»ΠΈΠΆΠ΅Π½ΠΈΠΈ ΠΊ Ρ‚ΠΎΡ‡ΠΊΠ΅ касания (1; 2). Π§Π΅Ρ€Π½Ρ‹ΠΌ Ρ†Π²Π΅Ρ‚ΠΎΠΌ ΠΏΠΎΠΊΠ°Π·Π°Π½ Π³Ρ€Π°Ρ„ΠΈΠΊ Ρ„ΡƒΠ½ΠΊΡ†ΠΈΠΈΒ  , ΠΊΠ°ΡΠ°Ρ‚Π΅Π»ΡŒΠ½Π°Ρ прямая ΠΏΠΎΠΊΠ°Π·Π°Π½Π° синСй Π»ΠΈΠ½ΠΈΠ΅ΠΉ, Ρ‚ΠΎΡ‡ΠΊΠ° касания ΠΈΠ·ΠΎΠ±Ρ€Π°ΠΆΠ΅Π½Π° красной Ρ‚ΠΎΡ‡ΠΊΠΎΠΉ.

ΠšΠ°ΠΆΠ΄Ρ‹ΠΉΒ ΠΏΠΎΡΠ»Π΅Π΄ΡƒΡŽΡ‰ΠΈΠΉΒ  рисунок являСтся ΡƒΠ²Π΅Π»ΠΈΡ‡Π΅Π½Π½ΠΎΠΉΒ ΠΎΠ±Π»Π°ΡΡ‚ΡŒΡŽ ΠΏΡ€Π΅Π΄Ρ‹Π΄ΡƒΡ‰Π΅Π³ΠΎ (эти области выдСлСны красными ΠΊΠ²Π°Π΄Ρ€Π°Ρ‚Π°ΠΌΠΈ).

Π₯ΠΎΡ€ΠΎΡˆΠΎΒ Π²ΠΈΠ΄Π½ΠΎ, Ρ‡Ρ‚ΠΎΒ  Π²Π±Π»ΠΈΠ·ΠΈ точки касания график Ρ„ΡƒΠ½ΠΊΡ†ΠΈΠΈΒ   практичСски сливаСтся с ΠΊΠ°ΡΠ°Ρ‚Π΅Π»ΡŒΠ½ΠΎΠΉ прямой y=x+1.

А сСйчас ΠΏΠ΅Ρ€Π΅ΠΉΠ΄Π΅ΠΌ ΠΊΒ Π±ΠΎΠ»Π΅Π΅Β Π·Π½Π°Ρ‡ΠΈΠΌΠΎΠΌΡƒΒ ΠΎΠΏΡ€Π΅Π΄Π΅Π»Π΅Π½ΠΈΡŽΒ  ΠΊΠ°ΡΠ°Ρ‚Π΅Π»ΡŒΠ½ΠΎΠΉ.

Для этого покаТСм, Ρ‡Ρ‚ΠΎΒ Π±ΡƒΠ΄Π΅Ρ‚Β ΠΏΡ€ΠΎΠΈΡΡ…ΠΎΠ΄ΠΈΡ‚ΡŒΒ ΡΒ ΡΠ΅ΠΊΡƒΡ‰Π΅ΠΉΒ ΠΠ’, Ссли точку В бСсконСчно ΠΏΡ€ΠΈΠ±Π»ΠΈΠΆΠ°Ρ‚ΡŒ ΠΊ точкС А.

Рисунок Π½ΠΈΠΆΠ΅ ΠΈΠ»Π»ΡŽΡΡ‚Ρ€ΠΈΡ€ΡƒΠ΅Ρ‚ этот процСсс.

БСкущая АВ (ΠΏΠΎΠΊΠ°Π·Π°Π½Π° синСй ΠΏΡƒΠ½ΠΊΡ‚ΠΈΡ€Π½ΠΎΠΉ прямой) Π±ΡƒΠ΄Π΅Ρ‚ ΡΡ‚Ρ€Π΅ΠΌΠΈΡ‚ΡŒΡΡ Π·Π°Π½ΡΡ‚ΡŒ ΠΏΠΎΠ»ΠΎΠΆΠ΅Π½ΠΈΠ΅ ΠΊΠ°ΡΠ°Ρ‚Π΅Π»ΡŒΠ½ΠΎΠΉ прямой (ΠΏΠΎΠΊΠ°Π·Π°Π½Π° синСй сплошной Π»ΠΈΠ½ΠΈΠ΅ΠΉ), ΡƒΠ³ΠΎΠ» Π½Π°ΠΊΠ»ΠΎΠ½Π° сСкущСй  Β (ΠΏΠΎΠΊΠ°Π·Π°Π½ красной прСрывистой Π΄ΡƒΠ³ΠΎΠΉ) Π±ΡƒΠ΄Π΅Ρ‚ ΡΡ‚Ρ€Π΅ΠΌΠΈΡ‚ΡŒΡΡ ΠΊ ΡƒΠ³Π»Ρƒ Π½Π°ΠΊΠ»ΠΎΠ½Π° ΠΊΠ°ΡΠ°Ρ‚Π΅Π»ΡŒΠ½ΠΎΠΉΒ  Β (ΠΈΠ·ΠΎΠ±Ρ€Π°ΠΆΠ΅Π½ красной сплошной Π΄ΡƒΠ³ΠΎΠΉ).

ΠžΠΏΡ€Π΅Π΄Π΅Π»Π΅Π½ΠΈΠ΅.

Π’Π°ΠΊΠΈΠΌ ΠΎΠ±Ρ€Π°Π·ΠΎΠΌ,Β ΠΊΠ°ΡΠ°Ρ‚Π΅Π»ΡŒΠ½Π°Ρ ΠΊ Π³Ρ€Π°Ρ„ΠΈΠΊΡƒ Ρ„ΡƒΠ½ΠΊΡ†ΠΈΠΈΒ y=f(x)Β Π² точкС А – это ΠΏΡ€Π΅Π΄Π΅Π»ΡŒΠ½ΠΎΠ΅ ΠΏΠΎΠ»ΠΎΠΆΠ΅Π½ΠΈΠ΅ сСкущСй ABΒ ΠΏΡ€ΠΈΒ  .

Π’ΠΎΡ‚ Ρ‚Π΅ΠΏΠ΅Ρ€ΡŒ ΠΌΠΎΠΆΠ½ΠΎΒ  ΠΏΠ΅Ρ€Π΅Ρ…ΠΎΠ΄ΠΈΡ‚ΡŒ к оописанию гСомСтричСского смысла ΠΏΡ€ΠΎΠΈΠ·Π²ΠΎΠ΄Π½ΠΎΠΉ Ρ„ΡƒΠ½ΠΊΡ†ΠΈΠΈ Π² Ρ‚ΠΎΡ‡ΠΊΠ΅.

К Π½Π°Ρ‡Π°Π»Ρƒ страницы

ГСомСтричСский  смысл производной функции в  Ρ‚ΠΎΡ‡ΠΊΠ΅.

Π Π°ΡΡΠΌΠΎΡ‚Ρ€ΠΈΠΌΒ ΡΠ΅ΠΊΡƒΡ‰ΡƒΡŽΒ ΠΠ’Β Π³Ρ€Π°Ρ„ΠΈΠΊΠ° Ρ„ΡƒΠ½ΠΊΡ†ΠΈΠΈΒ y=f(x)Β Ρ‚Π°ΠΊΡƒΡŽ, Ρ‡Ρ‚ΠΎ Ρ‚ΠΎΡ‡ΠΊΠΈΒ ΠΒ ΠΈΒ Π’Β ΠΈΠΌΠ΅ΡŽΡ‚ соотвСтствСнно ΠΊΠΎΠΎΡ€Π΄ΠΈΠ½Π°Ρ‚Ρ‹Β  Β ΠΈΒ  , Π³Π΄Π΅Β  Β — ΠΏΡ€ΠΈΡ€Π°Ρ‰Π΅Π½ΠΈΠ΅ Π°Ρ€Π³ΡƒΠΌΠ΅Π½Ρ‚Π°. ΠžΠ±ΠΎΠ·Π½Π°Ρ‡ΠΈΠΌ Ρ‡Π΅Ρ€Π΅Π·Β  Β ΠΏΡ€ΠΈΡ€Π°Ρ‰Π΅Π½ΠΈΠ΅ Ρ„ΡƒΠ½ΠΊΡ†ΠΈΠΈ. ΠžΡ‚ΠΌΠ΅Ρ‚ΠΈΠΌ всС Π½Π° Ρ‡Π΅Ρ€Ρ‚Π΅ΠΆΠ΅:

Π˜Π·Β ΠΏΡ€ΡΠΌΠΎΡƒΠ³ΠΎΠ»ΡŒΠ½ΠΎΠ³ΠΎ Ρ‚Ρ€Π΅ΡƒΠ³ΠΎΠ»ΡŒΠ½ΠΈΠΊΠ°Β ΠΠ’Π‘Β ΠΈΠΌΠ΅Π΅ΠΌΒ  . Π’Π°ΠΊ ΠΊΠ°ΠΊ ΠΏΠΎ ΠΎΠΏΡ€Π΅Π΄Π΅Π»Π΅Π½ΠΈΡŽ ΠΊΠ°ΡΠ°Ρ‚Π΅Π»ΡŒΠ½Π°Ρ – это ΠΏΡ€Π΅Π΄Π΅Π»ΡŒΠ½ΠΎΠ΅ ΠΏΠΎΠ»ΠΎΠΆΠ΅Π½ΠΈΠ΅ сСкущСй, Ρ‚ΠΎΒ  .

Вспомним опрСдСлСниС ΠΏΡ€ΠΎΠΈΠ·Π²ΠΎΠ΄Π½ΠΎΠΉ Ρ„ΡƒΠ½ΠΊΡ†ΠΈΠΈ Π² Ρ‚ΠΎΡ‡ΠΊΠ΅: ΠΏΡ€ΠΎΠΈΠ·Π²ΠΎΠ΄Π½ΠΎΠΉ Ρ„ΡƒΠ½ΠΊΡ†ΠΈΠΈΒ y=f(x)Β Π² Ρ‚ΠΎΡ‡ΠΊΠ΅Β  называСтся ΠΏΡ€Π΅Π΄Π΅Π» ΠΎΡ‚Π½ΠΎΡˆΠ΅Π½ΠΈΡ приращСния Ρ„ΡƒΠ½ΠΊΡ†ΠΈΠΈ ΠΊ ΠΏΡ€ΠΈΡ€Π°Ρ‰Π΅Π½ΠΈΡŽ Π°Ρ€Π³ΡƒΠΌΠ΅Π½Ρ‚Π° ΠΏΡ€ΠΈΒ  , обозначаСтся  .

Π‘Π»Π΅Π΄ΠΎΠ²Π°Ρ‚Π΅Π»ΡŒΠ½ΠΎ,Β  , Π³Π΄Π΅Β  Β — ΡƒΠ³Π»ΠΎΠ²ΠΎΠΉ коэффициСнт ΠΊΠ°ΡΠ°Ρ‚Π΅Π»ΡŒΠ½ΠΎΠΉ.

Π’Π°ΠΊΠΈΠΌ ΠΎΠ±Ρ€Π°Π·ΠΎΠΌ, сущСствованиС  ΠΏΡ€ΠΎΠΈΠ·Π²ΠΎΠ΄Π½ΠΎΠΉΒ Ρ„ΡƒΠ½ΠΊΡ†ΠΈΠΈΒ y=f(x)Β Π² Ρ‚ΠΎΡ‡ΠΊΠ΅Β   эквивалСнтно ΡΡƒΡ‰Π΅ΡΡ‚Π²ΠΎΠ²Π°Π½ΠΈΡŽ ΠΊΠ°ΡΠ°Ρ‚Π΅Π»ΡŒΠ½ΠΎΠΉ ΠΊ Π³Ρ€Π°Ρ„ΠΈΠΊΡƒ Ρ„ΡƒΠ½ΠΊΡ†ΠΈΠΈΒ y=f(x)Β Π² Ρ‚ΠΎΡ‡ΠΊΠ΅ касания  , ΠΏΡ€ΠΈΡ‡Π΅ΠΌΡƒΠ³Π»ΠΎΠ²ΠΎΠΉ коэффициСнт ΠΊΠ°ΡΠ°Ρ‚Π΅Π»ΡŒΠ½ΠΎΠΉ Ρ€Π°Π²Π΅Π½ Π·Π½Π°Ρ‡Π΅Π½ΠΈΡŽ ΠΏΡ€ΠΎΠΈΠ·Π²ΠΎΠ΄Π½ΠΎΠΉ Π² Ρ‚ΠΎΡ‡ΠΊΠ΅Β  , Ρ‚ΠΎ Π΅ΡΡ‚ΡŒΒ  .

Π—Π°ΠΊΠ»ΡŽΡ‡Π°Π΅ΠΌ: гСомСтричСский смысл ΠΏΡ€ΠΎΠΈΠ·Π²ΠΎΠ΄Π½ΠΎΠΉ Ρ„ΡƒΠ½ΠΊΡ†ΠΈΠΈ Π² точкС состоит Π² сущСствовании ΠΊΠ°ΡΠ°Ρ‚Π΅Π»ΡŒΠ½ΠΎΠΉ ΠΊ Π³Ρ€Π°Ρ„ΠΈΠΊΡƒ Ρ„ΡƒΠ½ΠΊΡ†ΠΈΠΈ Π² этой Ρ‚ΠΎΡ‡ΠΊΠ΅.

К Π½Π°Ρ‡Π°Π»Ρƒ страницы

Π£Ρ€Π°Π²Π½Π΅Π½ΠΈΠ΅ ΠΊΠ°ΡΠ°Ρ‚Π΅Π»ΡŒΠ½ΠΎΠΉΒ ΠΏΡ€ΡΠΌΠΎΠΉ.

Для записи уравнСния  Π»ΡŽΠ±ΠΎΠΉΒ ΠΏΡ€ΡΠΌΠΎΠΉ Π½Π° плоскости достаточно Π·Π½Π°Ρ‚ΡŒΒ Π΅Π΅ угловой коэффициСнт  ΠΈΒ Ρ‚ΠΎΡ‡ΠΊΡƒ, Ρ‡Π΅Ρ€Π΅Π·Β ΠΊΠΎΡ‚ΠΎΡ€ΡƒΡŽΒ ΠΎΠ½Π°Β  ΠΏΡ€ΠΎΡ…ΠΎΠ΄ΠΈΡ‚. ΠšΠ°ΡΠ°Ρ‚Π΅Π»ΡŒΠ½Π°ΡΒ ΠΏΡ€ΡΠΌΠ°Ρ ΠΏΡ€ΠΎΡ…ΠΎΠ΄ΠΈΡ‚ чСрСз точку касания и СС ΡƒΠ³Π»ΠΎΠ²ΠΎΠΉΒ  коэффициСнт для диффСрСнцируСмой Ρ„ΡƒΠ½ΠΊΡ†ΠΈΠΈΒ Ρ€Π°Π²Π΅Π½Β Π·Π½Π°Ρ‡Π΅Π½ΠΈΡŽ ΠΏΡ€ΠΎΠΈΠ·Π²ΠΎΠ΄Π½ΠΎΠΉΒ  Π²Β Ρ‚ΠΎΡ‡ΠΊΠ΅Β  . Π’ΠΎ Π΅ΡΡ‚ΡŒ, ΠΈΠ· пункта гСомСтричСский смысл ΠΏΡ€ΠΎΠΈΠ·Π²ΠΎΠ΄Π½ΠΎΠΉ Ρ„ΡƒΠ½ΠΊΡ†ΠΈΠΈ Π² Ρ‚ΠΎΡ‡ΠΊΠ΅Β ΠΌΡ‹ ΠΌΠΎΠΆΠ΅ΠΌ Π²Π·ΡΡ‚ΡŒ всС Π΄Π°Π½Π½Ρ‹Π΅ для записи уравнСния ΠΊΠ°ΡΠ°Ρ‚Π΅Π»ΡŒΠ½ΠΎΠΉ прямой.

Π£Ρ€Π°Π²Π½Π΅Π½ΠΈΠ΅ ΠΊΠ°ΡΠ°Ρ‚Π΅Π»ΡŒΠ½ΠΎΠΉΒ ΠΊΒ Π³Ρ€Π°Ρ„ΠΈΠΊΡƒΒ Ρ„ΡƒΠ½ΠΊΡ†ΠΈΠΈΒ y = f(x)Β Π² Ρ‚ΠΎΡ‡ΠΊΠ΅Β  Β ΠΈΠΌΠ΅Π΅Ρ‚ Π²ΠΈΠ΄Β  .

ΠœΡ‹Β ΠΏΠΎΠ΄Ρ€Π°Π·ΡƒΠΌΠ΅Π²Π°Π΅ΠΌ, что сущСствуСт ΠΊΠΎΠ½Π΅Ρ‡Π½ΠΎΠ΅ Π·Π½Π°Ρ‡Π΅Π½ΠΈΠ΅ ΠΏΡ€ΠΎΠΈΠ·Π²ΠΎΠ΄Π½ΠΎΠΉΒ  , Π² ΠΏΡ€ΠΎΡ‚ΠΈΠ²Π½ΠΎΠΌ случаС ΠΊΠ°ΡΠ°Ρ‚Π΅Π»ΡŒΠ½Π°Ρ прямая Π»ΠΈΠ±ΠΎ Π²Π΅Ρ€Ρ‚ΠΈΠΊΠ°Π»ΡŒΠ½Π° (Ссли  Β ΠΈΒ  ), Π»ΠΈΠ±ΠΎ Π½Π΅ сущСствуСт (Ссли  ).

Π’ зависимости от ΡƒΠ³Π»ΠΎΠ²ΠΎΠ³ΠΎ коэффициСнта  , ΠΊΠ°ΡΠ°Ρ‚Π΅Π»ΡŒΠ½Π°Ρ ΠΌΠΎΠΆΠ΅Ρ‚ Π±Ρ‹Ρ‚ΡŒ ΠΏΠ°Ρ€Π°Π»Π»Π΅Π»ΡŒΠ½Π° оси абсцисс ( ), ΠΏΠ°Ρ€Π°Π»Π»Π΅Π»ΡŒΠ½Π° оси ΠΎΡ€Π΄ΠΈΠ½Π°Ρ‚ ( Β Π² этом случаС ΡƒΡ€Π°Π²Π½Π΅Π½ΠΈΠ΅ ΠΊΠ°ΡΠ°Ρ‚Π΅Π»ΡŒΠ½ΠΎΠΉ Π±ΡƒΠ΄Π΅Ρ‚ ΠΈΠΌΠ΅Ρ‚ΡŒ Π²ΠΈΠ΄Β  ), Π²ΠΎΠ·Ρ€Π°ΡΡ‚Π°Ρ‚ΡŒ ( ) ΠΈΠ»ΠΈ ΡƒΠ±Ρ‹Π²Π°Ρ‚ΡŒ ( ).

Π‘Π°ΠΌΠΎΠ΅ врСмя привСсти Π½Π΅ΡΠΊΠΎΠ»ΡŒΠΊΠΎΒ ΠΏΡ€ΠΈΠΌΠ΅Ρ€ΠΎΠ² для пояснСния.

ΠŸΡ€ΠΈΠΌΠ΅Ρ€.

Π‘ΠΎΡΡ‚Π°Π²ΠΈΡ‚ΡŒ ΡƒΡ€Π°Π²Π½Π΅Π½ΠΈΠ΅Β  ΠΊΠ°ΡΠ°Ρ‚Π΅Π»ΡŒΠ½ΠΎΠΉΒ ΠΊΒ Π³Ρ€Π°Ρ„ΠΈΠΊΡƒΒ Ρ„ΡƒΠ½ΠΊΡ†ΠΈΠΈΒ  Β Π² Ρ‚ΠΎΡ‡ΠΊΠ΅Β (-1;-3)Β ΠΈ ΠΎΠΏΡ€Π΅Π΄Π΅Π»ΠΈΡ‚ΡŒ ΡƒΠ³ΠΎΠ» Π½Π°ΠΊΠ»ΠΎΠ½Π°.

РСшСниС.

Ѐункция ΠΎΠΏΡ€Π΅Π΄Π΅Π»Π΅Π½Π° для всСх Π΄Π΅ΠΉΡΡ‚Π²ΠΈΡ‚Π΅Π»ΡŒΠ½Ρ‹Ρ… чисСл (ΠΏΡ€ΠΈΒ  Π½Π΅ΠΎΠ±Ρ…ΠΎΠ΄ΠΈΠΌΠΎΡΡ‚ΠΈΒ ΠΎΠ±Ρ€Π°Ρ‰Π°ΠΉΡ‚Π΅ΡΡŒΒ ΠΊΒ  ΡΡ‚Π°Ρ‚ΡŒΠ΅Β ΠΎΠ±Π»Π°ΡΡ‚ΡŒ опрСдСлСния Ρ„ΡƒΠ½ΠΊΡ†ΠΈΠΈ). Π’Π°ΠΊ ΠΊΠ°ΠΊΒ (-1;-3) – Ρ‚ΠΎΡ‡ΠΊΠ° касания, Ρ‚ΠΎΒ  .

Находим ΠΏΡ€ΠΎΠΈΠ·Π²ΠΎΠ΄Π½ΡƒΡŽ (Π΄Π»ΡΒ ΡΡ‚ΠΎΠ³ΠΎΒ ΠΌΠΎΠΆΠ΅Ρ‚Β ΠΏΡ€ΠΈΠ³ΠΎΠ΄ΠΈΡ‚ΡŒΡΡΒ  ΠΌΠ°Ρ‚Π΅Ρ€ΠΈΠ°Π» ΡΡ‚Π°Ρ‚ΡŒΠΈΠ΄ΠΈΡ„Ρ„Π΅Ρ€Π΅Π½Ρ†ΠΈΡ€ΠΎΠ²Π°Π½ΠΈΠ΅ Ρ„ΡƒΠ½ΠΊΡ†ΠΈΠΈ, Π½Π°Ρ…ΠΎΠΆΠ΄Π΅Π½ΠΈΠ΅ ΠΏΡ€ΠΎΠΈΠ·Π²ΠΎΠ΄Π½ΠΎΠΉ) ΠΈ вычисляСм Π΅Π΅ Π·Π½Π°Ρ‡Π΅Π½ΠΈΠ΅ Π² Ρ‚ΠΎΡ‡ΠΊΠ΅Β  :Β 

Π’Π°ΠΊ ΠΊΠ°ΠΊΒ Π·Π½Π°Ρ‡Π΅Π½ΠΈΠ΅ производной в точкС касания  Π΅ΡΡ‚ΡŒ ΡƒΠ³Π»ΠΎΠ²ΠΎΠΉΒ ΠΊΠΎΡΡ„Ρ„ΠΈΡ†ΠΈΠ΅Π½Ρ‚Β ΠΊΠ°ΡΠ°Ρ‚Π΅Π»ΡŒΠ½ΠΎΠΉ, Π°Β ΠΎΠ½ равСн тангСнсу ΡƒΠ³Π»Π° Π½Π°ΠΊΠ»ΠΎΠ½Π°, Ρ‚ΠΎΒ  .

Π‘Π»Π΅Π΄ΠΎΠ²Π°Ρ‚Π΅Π»ΡŒΠ½ΠΎ, ΡƒΠ³ΠΎΠ» Π½Π°ΠΊΠ»ΠΎΠ½Π°Β ΠΊΠ°ΡΠ°Ρ‚Π΅Π»ΡŒΠ½ΠΎΠΉΒ Ρ€Π°Π²Π΅Π½Β  , Π° ΡƒΡ€Π°Π²Π½Π΅Π½ΠΈΠ΅ ΠΊΠ°ΡΠ°Ρ‚Π΅Π»ΡŒΠ½ΠΎΠΉ прямой ΠΈΠΌΠ΅Π΅Ρ‚ Π²ΠΈΠ΄Β 

ГрафичСская ΠΈΠ»Π»ΡŽΡΡ‚Ρ€Π°Ρ†ΠΈΡ.

Π§Π΅Ρ€Π½Ρ‹ΠΌΒ Ρ†Π²Π΅Ρ‚ΠΎΠΌ ΠΏΠΎΠΊΠ°Π·Π°Π½Β  Π³Ρ€Π°Ρ„ΠΈΠΊ исходной Ρ„ΡƒΠ½ΠΊΡ†ΠΈΠΈ, ΠΊΠ°ΡΠ°Ρ‚Π΅Π»ΡŒΠ½Π°ΡΒ  прямая ΠΈΠ·ΠΎΠ±Ρ€Π°ΠΆΠ΅Π½Π° синСй Π»ΠΈΠ½ΠΈΠ΅ΠΉ, Ρ‚ΠΎΡ‡ΠΊΠ° касания — красной Ρ‚ΠΎΡ‡ΠΊΠΎΠΉ. Рисунок справа прСдставляСт собой ΡƒΠ²Π΅Π»ΠΈΡ‡Π΅Π½Π½ΡƒΡŽ ΠΎΠ±Π»Π°ΡΡ‚ΡŒ, ΠΎΠ±ΠΎΠ·Π½Π°Ρ‡Π΅Π½Π½ΡƒΡŽ красным ΠΏΡƒΠ½ΠΊΡ‚ΠΈΡ€Π½Ρ‹ΠΌ ΠΊΠ²Π°Π΄Ρ€Π°Ρ‚ΠΎΠΌ Π½Π° рисункС слСва.

ΠŸΡ€ΠΈΠΌΠ΅Ρ€.

Π’Ρ‹ΡΡΠ½ΠΈΡ‚ΡŒ, сущСствуСт Π»ΠΈ ΠΊΠ°ΡΠ°Ρ‚Π΅Π»ΡŒΠ½Π°ΡΒ ΠΊΒ Π³Ρ€Π°Ρ„ΠΈΠΊΡƒΒ Ρ„ΡƒΠ½ΠΊΡ†ΠΈΠΈΒ  Β Π² Ρ‚ΠΎΡ‡ΠΊΠ΅Β (1; 1), Ссли Π΄Π°, Ρ‚ΠΎ ΡΠΎΡΡ‚Π°Π²ΠΈΡ‚ΡŒ Π΅Π΅ ΡƒΡ€Π°Π²Π½Π΅Π½ΠΈΠ΅ ΠΈ ΠΎΠΏΡ€Π΅Π΄Π΅Π»ΠΈΡ‚ΡŒ ΡƒΠ³ΠΎΠ» Π΅Π΅ Π½Π°ΠΊΠ»ΠΎΠ½Π°.

РСшСниС.

ΠžΠ±Π»Π°ΡΡ‚ΡŒΡŽΒ ΠΎΠΏΡ€Π΅Π΄Π΅Π»Π΅Π½ΠΈΡΒ  функции являСтся всС мноТСство  Π΄Π΅ΠΉΡΡ‚Π²ΠΈΡ‚Π΅Π»ΡŒΠ½Ρ‹Ρ… чисСл.

Находим ΠΏΡ€ΠΎΠΈΠ·Π²ΠΎΠ΄Π½ΡƒΡŽ:Β 

ΠŸΡ€ΠΈΒ   производная Π½Π΅ ΠΎΠΏΡ€Π΅Π΄Π΅Π»Π΅Π½Π°, Π½ΠΎΒ  Β ΠΈΒ  , ΡΠ»Π΅Π΄ΠΎΠ²Π°Ρ‚Π΅Π»ΡŒΠ½ΠΎ, Π² Ρ‚ΠΎΡ‡ΠΊΠ΅Β (1;1) сущСствуСт Π²Π΅Ρ€Ρ‚ΠΈΠΊΠ°Π»ΡŒΠ½Π°Ρ ΠΊΠ°ΡΠ°Ρ‚Π΅Π»ΡŒΠ½Π°Ρ, Π΅Π΅ ΡƒΡ€Π°Π²Π½Π΅Π½ΠΈΠ΅ ΠΈΠΌΠ΅Π΅Ρ‚ Π²ΠΈΠ΄Β x = 1, Π° ΡƒΠ³ΠΎΠ» Π½Π°ΠΊΠ»ΠΎΠ½Π° Ρ€Π°Π²Π΅Π½Β  .

ГрафичСская ΠΈΠ»Π»ΡŽΡΡ‚Ρ€Π°Ρ†ΠΈΡ.

ΠŸΡ€ΠΈΠΌΠ΅Ρ€.

Найти всС точки  Π³Ρ€Π°Ρ„ΠΈΠΊΠ°Β Ρ„ΡƒΠ½ΠΊΡ†ΠΈΠΈΒ  , Π² ΠΊΠΎΡ‚ΠΎΡ€Ρ‹Ρ…:Β 
a)Β ΠΊΠ°ΡΠ°Ρ‚Π΅Π»ΡŒΠ½Π°Ρ Π½Π΅ сущСствуСт;Β b)Β ΠΊΠ°ΡΠ°Ρ‚Π΅Π»ΡŒΠ½Π°Ρ ΠΏΠ°Ρ€Π°Π»Π»Π΅Π»ΡŒΠ½Π° оси абсцисс;Β c)Β ΠΊΠ°ΡΠ°Ρ‚Π΅Π»ΡŒΠ½Π°Ρ ΠΏΠ°Ρ€Π°Π»Π»Π΅Π»ΡŒΠ½Π° прямой  .

РСшСниС.

Как всСгда Π½Π°Ρ‡ΠΈΠ½Π°Π΅ΠΌ с области опрСдСлСния функции. Π’Β Π½Π°ΡˆΠ΅ΠΌΒ ΠΏΡ€ΠΈΠΌΠ΅Ρ€Π΅Β Ρ„ΡƒΠ½ΠΊΡ†ΠΈΡΒ ΠΎΠΏΡ€Π΅Π΄Π΅Π»Π΅Π½Π° Π½Π° всСм ΠΌΠ½ΠΎΠΆΠ΅ΡΡ‚Π²Π΅Β Π΄Π΅ΠΉΡΡ‚Π²ΠΈΡ‚Π΅Π»ΡŒΠ½Ρ‹Ρ… чисСл. РаскроСм Π·Π½Π°ΠΊ модуля, для этого  рассмотрим Π΄Π²Π°Β ΠΏΡ€ΠΎΠΌΠ΅ΠΆΡƒΡ‚ΠΊΠ°Β  Β ΠΈΒ  :Β 

ΠŸΡ€ΠΎΠ΄ΠΈΡ„Ρ„Π΅Ρ€Π΅Π½Ρ†ΠΈΡ€ΡƒΠ΅ΠΌ Ρ„ΡƒΠ½ΠΊΡ†ΠΈΡŽ:Β 

ΠŸΡ€ΠΈΒ x=-2 производная Π½Π΅ сущСствуСт, Ρ‚Π°ΠΊ ΠΊΠ°ΠΊ односторонниС ΠΏΡ€Π΅Π΄Π΅Π»Ρ‹ Π² этой Ρ‚ΠΎΡ‡ΠΊΠ΅ Π½Π΅ Ρ€Π°Π²Π½Ρ‹:Β 

Π’Π°ΠΊΠΈΠΌ ΠΎΠ±Ρ€Π°Π·ΠΎΠΌ, вычислив Π·Π½Π°Ρ‡Π΅Π½ΠΈΠ΅ Ρ„ΡƒΠ½ΠΊΡ†ΠΈΠΈΒ ΠΏΡ€ΠΈΒ x=-2, ΠΌΡ‹ ΠΌΠΎΠΆΠ΅ΠΌ Π΄Π°Ρ‚ΡŒ ΠΎΡ‚Π²Π΅Ρ‚ Π½Π° ΠΏΡƒΠ½ΠΊΡ‚Π°):Β  , ΠΊΠ°ΡΠ°Ρ‚Π΅Π»ΡŒΠ½Π°Ρ ΠΊ Π³Ρ€Π°Ρ„ΠΈΠΊΡƒ Ρ„ΡƒΠ½ΠΊΡ†ΠΈΠΈ Π½Π΅ сущСствуСт Π² Ρ‚ΠΎΡ‡ΠΊΠ΅Β (-2;-2).

b)Β ΠšΠ°ΡΠ°Ρ‚Π΅Π»ΡŒΠ½Π°Ρ ΠΏΠ°Ρ€Π°Π»Π»Π΅Π»ΡŒΠ½Π° оси абсцисс, Ссли Π΅Π΅ ΡƒΠ³Π»ΠΎΠ²ΠΎΠΉ коэффициСнт Ρ€Π°Π²Π΅Π½ Π½ΡƒΠ»ΡŽ (тангСнс ΡƒΠ³Π»Π° Π½Π°ΠΊΠ»ΠΎΠ½Π° Ρ€Π°Π²Π΅Π½ Π½ΡƒΠ»ΡŽ). Π’Π°ΠΊ ΠΊΠ°ΠΊΒ  , Ρ‚ΠΎ Π½Π°ΠΌ Π½ΡƒΠΆΠ½ΠΎ Π½Π°ΠΉΡ‚ΠΈ всС значСния х, ΠΏΡ€ΠΈ ΠΊΠΎΡ‚ΠΎΡ€Ρ‹Ρ… производная Ρ„ΡƒΠ½ΠΊΡ†ΠΈΠΈ обращаСтся Π² ноль. Π­Ρ‚ΠΈ значСния ΠΈ Π±ΡƒΠ΄ΡƒΡ‚ абсциссами Ρ‚ΠΎΡ‡Π΅ΠΊ касания, Π² ΠΊΠΎΡ‚ΠΎΡ€Ρ‹Ρ… ΠΊΠ°ΡΠ°Ρ‚Π΅Π»ΡŒΠ½Π°Ρ ΠΏΠ°Ρ€Π°Π»Π»Π΅Π»ΡŒΠ½Π° оси Ox.

ΠŸΡ€ΠΈΒ  Β Ρ€Π΅ΡˆΠ°Π΅ΠΌ ΡƒΡ€Π°Π²Π½Π΅Π½ΠΈΠ΅Β  , Π° ΠΏΡ€ΠΈΒ  Β — ΡƒΡ€Π°Π²Π½Π΅Π½ΠΈΠ΅Β  :Β 

ΠžΡΡ‚Π°Π»ΠΎΡΡŒΒ Π²Ρ‹Ρ‡ΠΈΡΠ»ΠΈΡ‚ΡŒΒ  ΡΠΎΠΎΡ‚Π²Π΅Ρ‚ΡΡ‚Π²ΡƒΡŽΡ‰ΠΈΠ΅Β Π·Π½Π°Ρ‡Π΅Π½ΠΈΡ Ρ„ΡƒΠ½ΠΊΡ†ΠΈΠΈ:Β 

ΠŸΠΎΡΡ‚ΠΎΠΌΡƒ,Β  Β — искомыС Ρ‚ΠΎΡ‡ΠΊΠΈ Π³Ρ€Π°Ρ„ΠΈΠΊΠ° Ρ„ΡƒΠ½ΠΊΡ†ΠΈΠΈ.

ГрафичСская ΠΈΠ»Π»ΡŽΡΡ‚Ρ€Π°Ρ†ΠΈΡ.

График исходной Ρ„ΡƒΠ½ΠΊΡ†ΠΈΠΈΒ ΠΈΠ·ΠΎΠ±Ρ€Π°ΠΆΠ΅Π½Β Ρ‡Π΅Ρ€Π½ΠΎΠΉ Π»ΠΈΠ½ΠΈΠ΅ΠΉ, красными Ρ‚ΠΎΡ‡ΠΊΠ°ΠΌΠΈΒ ΠΎΡ‚ΠΌΠ΅Ρ‡Π΅Π½Ρ‹ Π½Π°ΠΉΠ΄Π΅Π½Π½Ρ‹Π΅Β  Ρ‚ΠΎΡ‡ΠΊΠΈ, Π²Β ΠΊΠΎΡ‚ΠΎΡ€Ρ‹Ρ…Β ΠΊΠ°ΡΠ°Ρ‚Π΅Π»ΡŒΠ½Ρ‹Π΅Β  ΠΏΠ°Ρ€Π°Π»Π»Π΅Π»ΡŒΠ½Ρ‹Β ΠΎΡΠΈΒ Π°Π±ΡΡ†ΠΈΡΡ.

c) Если Π΄Π²Π΅ прямыС Π½Π° плоскости ΠΏΠ°Ρ€Π°Π»Π»Π΅Π»ΡŒΠ½Ρ‹, Ρ‚ΠΎ ΠΈΡ… ΡƒΠ³Π»ΠΎΠ²Ρ‹Π΅ коэффициСнты Ρ€Π°Π²Π½Ρ‹ (ΠΎΠ± этом написано Π² ΡΡ‚Π°Ρ‚ΡŒΠ΅Β ΠΏΠ°Ρ€Π°Π»Π»Π΅Π»ΡŒΠ½Ρ‹Π΅ прямыС, ΠΏΠ°Ρ€Π°Π»Π»Π΅Π»ΡŒΠ½ΠΎΡΡ‚ΡŒ прямых). Π˜ΡΡ…ΠΎΠ΄Ρ ΠΈΠ· этого утвСрТдСния, Π½Π°ΠΌ Π½ΡƒΠΆΠ½ΠΎ Π½Π°ΠΉΡ‚ΠΈ всС Ρ‚ΠΎΡ‡ΠΊΠΈ Π³Ρ€Π°Ρ„ΠΈΠΊΠ° Ρ„ΡƒΠ½ΠΊΡ†ΠΈΠΈ, Π² ΠΊΠΎΡ‚ΠΎΡ€Ρ‹Ρ… ΡƒΠ³Π»ΠΎΠ²ΠΎΠΉ коэффициСнт ΠΊΠ°ΡΠ°Ρ‚Π΅Π»ΡŒΠ½ΠΎΠΉ Ρ€Π°Π²Π΅Π½ восьми пятым. Π’ΠΎ Π΅ΡΡ‚ΡŒ, Π½Π°ΠΌ Π½ΡƒΠΆΠ½ΠΎ Ρ€Π΅ΡˆΠΈΡ‚ΡŒ ΡƒΡ€Π°Π²Π½Π΅Π½ΠΈΠ΅Β  . Π’Π°ΠΊΠΈΠΌ ΠΎΠ±Ρ€Π°Π·ΠΎΠΌ, ΠΏΡ€ΠΈΒ  Β Ρ€Π΅ΡˆΠ°Π΅ΠΌ ΡƒΡ€Π°Π²Π½Π΅Π½ΠΈΠ΅Β  , Π° ΠΏΡ€ΠΈΒ  Β — ΡƒΡ€Π°Π²Π½Π΅Π½ΠΈΠ΅Β  .

Дискриминант пСрвого  уравнСния отрицатСлСн, ΡΠ»Π΅Π΄ΠΎΠ²Π°Ρ‚Π΅Π»ΡŒΠ½ΠΎ, ΠΎΠ½ΠΎΒ Π½Π΅ ΠΈΠΌΠ΅Π΅Ρ‚Β Π΄Π΅ΠΉΡΡ‚Π²ΠΈΡ‚Π΅Π»ΡŒΠ½Ρ‹Ρ… ΠΊΠΎΡ€Π½Π΅ΠΉ:Β 

Π’Ρ‚ΠΎΡ€ΠΎΠ΅Β ΡƒΡ€Π°Π²Π½Π΅Π½ΠΈΠ΅Β  ΠΈΠΌΠ΅Π΅Ρ‚Β Π΄Π²Π°Β Π΄Π΅ΠΉΡΡ‚Π²ΠΈΡ‚Π΅Π»ΡŒΠ½Ρ‹Ρ… корня:Β 

Находим ΡΠΎΠΎΡ‚Π²Π΅Ρ‚ΡΡ‚Π²ΡƒΡŽΡ‰ΠΈΠ΅Β  значСния Ρ„ΡƒΠ½ΠΊΡ†ΠΈΠΈ:Β 

Π’ Ρ‚ΠΎΡ‡ΠΊΠ°Ρ…Β  Β ΠΊΠ°ΡΠ°Ρ‚Π΅Π»ΡŒΠ½Ρ‹Π΅ ΠΊ Π³Ρ€Π°Ρ„ΠΈΠΊΡƒ Ρ„ΡƒΠ½ΠΊΡ†ΠΈΠΈ ΠΏΠ°Ρ€Π°Π»Π»Π΅Π»ΡŒΠ½Ρ‹ прямой  .

ГрафичСская ΠΈΠ»Π»ΡŽΡΡ‚Ρ€Π°Ρ†ΠΈΡ.

Π“Ρ€Π°Ρ„ΠΈΠΊΒ Ρ„ΡƒΠ½ΠΊΡ†ΠΈΠΈΒ  ΠΈΠ·ΠΎΠ±Ρ€Π°ΠΆΠ΅Π½Β Ρ‡Π΅Ρ€Π½ΠΎΠΉ Π»ΠΈΠ½ΠΈΠ΅ΠΉ, красной  Π»ΠΈΠ½ΠΈΠ΅ΠΉ ΠΏΠΎΠΊΠ°Π·Π°Π½Β Π³Ρ€Π°Ρ„ΠΈΠΊ прямой  , синими линиями ΠΏΠΎΠΊΠ°Π·Π°Π½Ρ‹ ΠΊΠ°ΡΠ°Ρ‚Π΅Π»ΡŒΠ½Ρ‹Π΅ ΠΊ Π³Ρ€Π°Ρ„ΠΈΠΊΡƒ Ρ„ΡƒΠ½ΠΊΡ†ΠΈΠΈ Π² Ρ‚ΠΎΡ‡ΠΊΠ°Ρ…Β  .

Для тригономСтричСских функций в силу ΠΈΡ… пСриодичности, ΠΌΠΎΠΆΠ΅Ρ‚Β ΡΡƒΡ‰Π΅ΡΡ‚Π²ΠΎΠ²Π°Ρ‚ΡŒ бСсконСчно ΠΌΠ½ΠΎΠ³ΠΎΒ  ΠΊΠ°ΡΠ°Ρ‚Π΅Π»ΡŒΠ½Ρ‹Ρ…Β ΠΏΡ€ΡΠΌΡ‹Ρ…, ΠΈΠΌΠ΅ΡŽΡ‰ΠΈΡ…Β ΠΎΠ΄ΠΈΠ½ ΡƒΠ³ΠΎΠ» Π½Π°ΠΊΠ»ΠΎΠ½Π° (ΠΎΠ΄ΠΈΠ½Π°ΠΊΠΎΠ²Ρ‹ΠΉ угловой коэффициСнт).

ΠŸΡ€ΠΈΠΌΠ΅Ρ€.

ΠΠ°ΠΏΠΈΡΠ°Ρ‚ΡŒΒ ΡƒΡ€Π°Π²Π½Π΅Π½ΠΈΡΒ  всСх ΠΊΠ°ΡΠ°Ρ‚Π΅Π»ΡŒΠ½Ρ‹Ρ…Β ΠΊΒ Π³Ρ€Π°Ρ„ΠΈΠΊΡƒΒ Ρ„ΡƒΠ½ΠΊΡ†ΠΈΠΈΒ  , ΠΊΠΎΡ‚ΠΎΡ€Ρ‹Π΅ пСрпСндикулярны прямой  .

РСшСниС.

Π§Ρ‚ΠΎΠ±Ρ‹ ΡΠΎΡΡ‚Π°Π²ΠΈΡ‚ΡŒΒ  ΡƒΡ€Π°Π²Π½Π΅Π½ΠΈΠ΅Β ΠΊΠ°ΡΠ°Ρ‚Π΅Π»ΡŒΠ½ΠΎΠΉΒ ΠΊΒ Π³Ρ€Π°Ρ„ΠΈΠΊΡƒΒ  функции нам достаточно Π·Π½Π°Ρ‚ΡŒΒ  Π΅Π΅ угловой коэффициСнт и координаты точки касания.

Π£Π³Π»ΠΎΠ²ΠΎΠΉ коэффициСнт  ΠΊΠ°ΡΠ°Ρ‚Π΅Π»ΡŒΠ½Ρ‹Ρ…Β  Β Π½Π°ΠΉΠ΄Π΅ΠΌ из условия пСрпСндикулярности прямых: ΠΏΡ€ΠΎΠΈΠ·Π²Π΅Π΄Π΅Π½ΠΈΠ΅ ΡƒΠ³Π»ΠΎΠ²Ρ‹Ρ… коэффициСнтов пСрпСндикулярных прямых Ρ€Π°Π²Π½ΠΎ минус Π΅Π΄ΠΈΠ½ΠΈΡ†Π΅, Ρ‚ΠΎ Π΅ΡΡ‚ΡŒΒ  . Π’Π°ΠΊ ΠΊΠ°ΠΊ ΠΏΠΎ ΡƒΡΠ»ΠΎΠ²ΠΈΡŽ ΡƒΠ³Π»ΠΎΠ²ΠΎΠΉ коэффициСнт пСрпСндикулярной прямой Ρ€Π°Π²Π΅Π½Β  , Ρ‚ΠΎΒ  .

ΠŸΡ€ΠΈΡΡ‚ΡƒΠΏΠΈΠΌ ΠΊΒ Π½Π°Ρ…ΠΎΠΆΠ΄Π΅Π½ΠΈΡŽ координат точСк касания. Для  Π½Π°Ρ‡Π°Π»Π° Π½Π°ΠΉΠ΄Π΅ΠΌ абсциссы, затСм вычислим ΡΠΎΠΎΡ‚Π²Π΅Ρ‚ΡΡ‚Π²ΡƒΡŽΡ‰ΠΈΠ΅Β Π·Π½Π°Ρ‡Π΅Π½ΠΈΡ Ρ„ΡƒΠ½ΠΊΡ†ΠΈΠΈΒ  – это будут ординаты Ρ‚ΠΎΡ‡Π΅ΠΊΒ  касания.

ΠŸΡ€ΠΈ описании гСомСтричСского  смысла ΠΏΡ€ΠΎΠΈΠ·Π²ΠΎΠ΄Π½ΠΎΠΉΒ Ρ„ΡƒΠ½ΠΊΡ†ΠΈΠΈΒ Π²Β  Ρ‚ΠΎΡ‡ΠΊΠ΅Β  Β ΠΌΡ‹ ΠΎΡ‚ΠΌΠ΅Ρ‚ΠΈΠ»ΠΈ, Ρ‡Ρ‚ΠΎΒ  . Из этого равСнства Π½Π°ΠΉΠ΄Π΅ΠΌ абсциссы Ρ‚ΠΎΡ‡Π΅ΠΊ касания.Β 

ΠœΡ‹Β ΠΏΡ€ΠΈΡˆΠ»ΠΈ к тригономСтричСскому  ΡƒΡ€Π°Π²Π½Π΅Π½ΠΈΡŽ. ΠŸΡ€ΠΎΡΠΈΠΌ ΠΎΠ±Ρ€Π°Ρ‚ΠΈΡ‚ΡŒ Π½Π° Π½Π΅Π³ΠΎ Π²Π½ΠΈΠΌΠ°Π½ΠΈΠ΅, Ρ‚Π°ΠΊΒ ΠΊΠ°ΠΊΒ ΠΏΠΎΠ·ΠΆΠ΅Β ΠΌΡ‹ Π΅Π³ΠΎΒ ΠΈΡΠΏΠΎΠ»ΡŒΠ·ΡƒΠ΅ΠΌ при вычислСнии ΠΎΡ€Π΄ΠΈΠ½Π°Ρ‚Β Ρ‚ΠΎΡ‡Π΅ΠΊΒ  касания. РСшаСм Π΅Π³ΠΎ (при затруднСниях ΠΎΠ±Ρ€Π°Ρ‰Π°ΠΉΡ‚Π΅ΡΡŒΒ ΠΊΒ Ρ€Π°Π·Π΄Π΅Π»ΡƒΒ Ρ€Π΅ΡˆΠ΅Π½ΠΈΠ΅ тригономСтричСских ΡƒΡ€Π°Π²Π½Π΅Π½ΠΈΠΉ):Β 

Абсциссы точСк  касания найдСны, вычислим ΡΠΎΠΎΡ‚Π²Π΅Ρ‚ΡΡ‚Π²ΡƒΡŽΡ‰ΠΈΠ΅Β  ΠΎΡ€Π΄ΠΈΠ½Π°Ρ‚Ρ‹ (Π·Π΄Π΅ΡΡŒΒ ΠΈΡΠΏΠΎΠ»ΡŒΠ·ΡƒΠ΅ΠΌ равСнство, Π½Π° ΠΊΠΎΡ‚ΠΎΡ€ΠΎΠ΅Β ΠΌΡ‹ ΠΏΡ€ΠΎΡΠΈΠ»ΠΈΒ ΠΎΠ±Ρ€Π°Ρ‚ΠΈΡ‚ΡŒ Π²Π½ΠΈΠΌΠ°Π½ΠΈΠ΅ Ρ‡ΡƒΡ‚ΡŒ Π²Ρ‹ΡˆΠ΅):Β 

Π’Π°ΠΊΠΈΠΌ ΠΎΠ±Ρ€Π°Π·ΠΎΠΌ,Β  — всС Ρ‚ΠΎΡ‡ΠΊΠΈ касания. Π‘Π»Π΅Π΄ΠΎΠ²Π°Ρ‚Π΅Π»ΡŒΠ½ΠΎ, искомыС уравнСния ΠΊΠ°ΡΠ°Ρ‚Π΅Π»ΡŒΠ½Ρ‹Ρ… ΠΈΠΌΠ΅ΡŽΡ‚ Π²ΠΈΠ΄:Β 

ГрафичСская ΠΈΠ»Π»ΡŽΡΡ‚Ρ€Π°Ρ†ΠΈΡ.

На рисункС чСрной ΠΊΡ€ΠΈΠ²ΠΎΠΉ ΠΏΠΎΠΊΠ°Π·Π°Π½Β Π³Ρ€Π°Ρ„ΠΈΠΊ исходной Ρ„ΡƒΠ½ΠΊΡ†ΠΈΠΈΒ  Π½Π° ΠΎΡ‚Ρ€Π΅Π·ΠΊΠ΅Β [-10;10], синими линиями ΠΈΠ·ΠΎΠ±Ρ€Π°ΠΆΠ΅Π½Ρ‹ ΠΊΠ°ΡΠ°Ρ‚Π΅Π»ΡŒΠ½Ρ‹Π΅ прямыС. Π₯ΠΎΡ€ΠΎΡˆΠΎ Π²ΠΈΠ΄Π½ΠΎ, Ρ‡Ρ‚ΠΎ ΠΎΠ½ΠΈ пСрпСндикулярны красной прямой  . Π’ΠΎΡ‡ΠΊΠΈ касания ΠΎΡ‚ΠΌΠ΅Ρ‡Π΅Π½Ρ‹ красными Ρ‚ΠΎΡ‡ΠΊΠ°ΠΌΠΈ.

К Π½Π°Ρ‡Π°Π»Ρƒ страницы

ΠšΠ°ΡΠ°Ρ‚Π΅Π»ΡŒΠ½Π°Ρ к окруТности, эллипсу, Π³ΠΈΠΏΠ΅Ρ€Π±ΠΎΠ»Π΅, ΠΏΠ°Ρ€Π°Π±ΠΎΠ»Π΅.

До этого момСнта  ΠΌΡ‹ занимались Π½Π°Ρ…ΠΎΠΆΠ΄Π΅Π½ΠΈΠ΅ΠΌΒ ΡƒΡ€Π°Π²Π½Π΅Π½ΠΈΠΉΒ  ΠΊΠ°ΡΠ°Ρ‚Π΅Π»ΡŒΠ½Ρ‹Ρ…Β ΠΊΒ Π³Ρ€Π°Ρ„ΠΈΠΊΠ°ΠΌ ΠΎΠ΄Π½ΠΎΠ·Π½Π°Ρ‡Π½Ρ‹Ρ…Β  Ρ„ΡƒΠ½ΠΊΡ†ΠΈΠΉΒ Π²ΠΈΠ΄Π°Β y = f(x)Β Π² Ρ€Π°Π·Π»ΠΈΡ‡Π½Ρ‹Ρ… Ρ‚ΠΎΡ‡ΠΊΠ°Ρ…. ΠšΠ°Π½ΠΎΠ½ΠΈΡ‡Π΅ΡΠΊΠΈΠ΅ уравнСния ΠΊΡ€ΠΈΠ²Ρ‹Ρ… Π²Ρ‚ΠΎΡ€ΠΎΠ³ΠΎ порядка Π½Π΅ ΡΠ²Π»ΡΡŽΡ‚ΡΡ ΠΎΠ΄Π½ΠΎΠ·Π½Π°Ρ‡Π½Ρ‹ΠΌΠΈ функциями. Но ΠΎΠΊΡ€ΡƒΠΆΠ½ΠΎΡΡ‚ΡŒ, эллипс, Π³ΠΈΠΏΠ΅Ρ€Π±ΠΎΠ»Ρƒ ΠΈ ΠΏΠ°Ρ€Π°Π±ΠΎΠ»Ρƒ ΠΌΡ‹ ΠΌΠΎΠΆΠ΅ΠΌ ΠΏΡ€Π΅Π΄ΡΡ‚Π°Π²ΠΈΡ‚ΡŒ ΠΊΠΎΠΌΠ±ΠΈΠ½Π°Ρ†ΠΈΠ΅ΠΉ Π΄Π²ΡƒΡ… ΠΎΠ΄Π½ΠΎΠ·Π½Π°Ρ‡Π½Ρ‹Ρ… Ρ„ΡƒΠ½ΠΊΡ†ΠΈΠΉ ΠΈ ΡƒΠΆΠ΅ послС этого ΡΠΎΡΡ‚Π°Π²Π»ΡΡ‚ΡŒ уравнСния ΠΊΠ°ΡΠ°Ρ‚Π΅Π»ΡŒΠ½Ρ‹Ρ… ΠΏΠΎ извСстной схСмС.

ΠšΠ°ΡΠ°Ρ‚Π΅Π»ΡŒΠ½Π°Ρ к окруТности.

ΠžΠΊΡ€ΡƒΠΆΠ½ΠΎΡΡ‚ΡŒΒ ΡΒ  Ρ†Π΅Π½Ρ‚Ρ€ΠΎΠΌΒ Π²Β Ρ‚ΠΎΡ‡ΠΊΠ΅Β  Β ΠΈ радиусом R задаСтся равСнством  .

Π—Π°ΠΏΠΈΡˆΠ΅ΠΌ это равСнство  Π²Β Π²ΠΈΠ΄Π΅ ΠΎΠ±ΡŠΠ΅Π΄ΠΈΠ½Π΅Π½ΠΈΡΒ Π΄Π²ΡƒΡ… Ρ„ΡƒΠ½ΠΊΡ†ΠΈΠΉ:Β 

Π—Π΄Π΅ΡΡŒ пСрвая функция  соотвСтствуСт вСрхнСй полуокруТности, вторая — Π½ΠΈΠΆΠ½Π΅ΠΉ.

Π’Π°ΠΊΠΈΠΌ ΠΎΠ±Ρ€Π°Π·ΠΎΠΌ, Ρ‡Ρ‚ΠΎΠ±Ρ‹Β  ΡΠΎΡΡ‚Π°Π²ΠΈΡ‚ΡŒΒ ΡƒΡ€Π°Π²Π½Π΅Π½ΠΈΠ΅Β ΠΊΠ°ΡΠ°Ρ‚Π΅Π»ΡŒΠ½ΠΎΠΉΒ  к окруТности Π²Β Ρ‚ΠΎΡ‡ΠΊΠ΅Β  , ΠΏΡ€ΠΈΠ½Π°Π΄Π»Π΅ΠΆΠ°Ρ‰Π΅ΠΉ Π²Π΅Ρ€Ρ…Π½Π΅ΠΉ (ΠΈΠ»ΠΈ Π½ΠΈΠΆΠ½Π΅ΠΉ) полуокруТности, ΠΌΡ‹ Π½Π°Ρ…ΠΎΠ΄ΠΈΠΌ ΡƒΡ€Π°Π²Π½Π΅Π½ΠΈΠ΅ ΠΊΠ°ΡΠ°Ρ‚Π΅Π»ΡŒΠ½ΠΎΠΉ ΠΊ Π³Ρ€Π°Ρ„ΠΈΠΊΡƒ Ρ„ΡƒΠ½ΠΊΡ†ΠΈΠΈΒ  Β (ΠΈΠ»ΠΈΒ  ) Π² ΡƒΠΊΠ°Π·Π°Π½Π½ΠΎΠΉ Ρ‚ΠΎΡ‡ΠΊΠ΅.

Π›Π΅Π³ΠΊΠΎ ΠΏΠΎΠΊΠ°Π·Π°Ρ‚ΡŒ, Ρ‡Ρ‚ΠΎΒ  Π²Β Ρ‚ΠΎΡ‡ΠΊΠ°Ρ… окруТности с координатами  Β ΠΈΒ  ΠΊΠ°ΡΠ°Ρ‚Π΅Π»ΡŒΠ½Ρ‹Π΅ ΠΏΠ°Ρ€Π°Π»Π»Π΅Π»ΡŒΠ½Ρ‹ оси абсцисс ΠΈ Π·Π°Π΄Π°ΡŽΡ‚ΡΡ уравнСниями  Β ΠΈΒ  соотвСтствСнно (Π½Π° рисункС Π½ΠΈΠΆΠ΅ ΠΎΠ½ΠΈ ΠΏΠΎΠΊΠ°Π·Π°Π½Ρ‹ синими Ρ‚ΠΎΡ‡ΠΊΠ°ΠΌΠΈ ΠΈ синими прямыми), Π° Π² Ρ‚ΠΎΡ‡ΠΊΠ°Ρ…Β  Β ΠΈΒ  Β — ΠΏΠ°Ρ€Π°Π»Π»Π΅Π»ΡŒΠ½Ρ‹ оси ΠΎΡ€Π΄ΠΈΠ½Π°Ρ‚ ΠΈ ΠΈΠΌΠ΅ΡŽΡ‚ уравнСния  Β ΠΈΒ   соотвСтствСнно (Π½Π° рисункС Π½ΠΈΠΆΠ΅ ΠΎΠ½ΠΈ ΠΎΡ‚ΠΌΠ΅Ρ‡Π΅Π½Ρ‹ красными Ρ‚ΠΎΡ‡ΠΊΠ°ΠΌΠΈ ΠΈ красными прямыми).

Π£Ρ€Π°Π²Π½Π΅Π½ΠΈΠ΅ ΠΊΠ°ΡΠ°Ρ‚Π΅Π»ΡŒΠ½ΠΎΠΉ. Π£Ρ€Π°Π²Π½Π΅Π½ΠΈΠ΅ ΠΊΠ°ΡΠ°Ρ‚Π΅Π»ΡŒΠ½ΠΎΠΉ ΠΊ Π³Ρ€Π°Ρ„ΠΈΠΊΡƒ Ρ„ΡƒΠ½ΠΊΡ†ΠΈΠΈ

ΠŸΡ€ΠΈΠΌΠ΅Ρ€ 1. Π”Π°Π½Π° функция f (x ) = 3x 2 + 4x – 5. НапишСм ΡƒΡ€Π°Π²Π½Π΅Π½ΠΈΠ΅ ΠΊΠ°ΡΠ°Ρ‚Π΅Π»ΡŒΠ½ΠΎΠΉ ΠΊ Π³Ρ€Π°Ρ„ΠΈΠΊΡƒ Ρ„ΡƒΠ½ΠΊΡ†ΠΈΠΈ f (x ) Π² Ρ‚ΠΎΡ‡ΠΊΠ΅ Π³Ρ€Π°Ρ„ΠΈΠΊΠ° с абсциссой x 0 = 1.

РСшСниС. ΠŸΡ€ΠΎΠΈΠ·Π²ΠΎΠ΄Π½Π°Ρ Ρ„ΡƒΠ½ΠΊΡ†ΠΈΠΈ f (x ) сущСствуСт для любого x R . НайдСм Π΅Π΅:

= (3x 2 + 4x – 5)β€² = 6x + 4.

Π’ΠΎΠ³Π΄Π° f (x 0) = f (1) = 2; (x 0) = = 10. Π£Ρ€Π°Π²Π½Π΅Π½ΠΈΠ΅ ΠΊΠ°ΡΠ°Ρ‚Π΅Π»ΡŒΠ½ΠΎΠΉ ΠΈΠΌΠ΅Π΅Ρ‚ Π²ΠΈΠ΄:

y = (x 0) (x – x 0) + f (x 0),

y = 10(x – 1) + 2,

y = 10x – 8.

ΠžΡ‚Π²Π΅Ρ‚. y = 10x – 8.

ΠŸΡ€ΠΈΠΌΠ΅Ρ€ 2. Π”Π°Π½Π° функция f (x ) = x 3 – 3x 2 + 2x + 5. НапишСм ΡƒΡ€Π°Π²Π½Π΅Π½ΠΈΠ΅ ΠΊΠ°ΡΠ°Ρ‚Π΅Π»ΡŒΠ½ΠΎΠΉ ΠΊ Π³Ρ€Π°Ρ„ΠΈΠΊΡƒ Ρ„ΡƒΠ½ΠΊΡ†ΠΈΠΈ f (x ), ΠΏΠ°Ρ€Π°Π»Π»Π΅Π»ΡŒΠ½ΠΎΠΉ прямой y = 2x – 11.

РСшСниС. ΠŸΡ€ΠΎΠΈΠ·Π²ΠΎΠ΄Π½Π°Ρ Ρ„ΡƒΠ½ΠΊΡ†ΠΈΠΈ f (x ) сущСствуСт для любого x R . НайдСм Π΅Π΅:

= (x 3 – 3x 2 + 2x + 5)β€² = 3x 2 – 6x + 2.

Π’Π°ΠΊ ΠΊΠ°ΠΊ ΠΊΠ°ΡΠ°Ρ‚Π΅Π»ΡŒΠ½Π°Ρ ΠΊ Π³Ρ€Π°Ρ„ΠΈΠΊΡƒ Ρ„ΡƒΠ½ΠΊΡ†ΠΈΠΈ f (x ) Π² Ρ‚ΠΎΡ‡ΠΊΠ΅ с абсциссой x 0 ΠΏΠ°Ρ€Π°Π»Π»Π΅Π»ΡŒΠ½Π° прямой y = 2x – 11, Ρ‚ΠΎ Π΅Π΅ ΡƒΠ³Π»ΠΎΠ²ΠΎΠΉ коэффициСнт Ρ€Π°Π²Π΅Π½ 2, Ρ‚. Π΅. (x 0) = 2. НайдСм эту абсциссу ΠΈΠ· условия, Ρ‡Ρ‚ΠΎ 3x – 6x 0 + 2 = 2. Π­Ρ‚ΠΎ равСнство справСдливо лишь ΠΏΡ€ΠΈ x 0 = 0 ΠΈ ΠΏΡ€ΠΈ x 0 = 2. Π’Π°ΠΊ ΠΊΠ°ΠΊ Π² Ρ‚ΠΎΠΌ ΠΈ Π² Π΄Ρ€ΡƒΠ³ΠΎΠΌ случаС f (x 0) = 5, Ρ‚ΠΎ прямая y = 2x + b касаСтся Π³Ρ€Π°Ρ„ΠΈΠΊΠ° Ρ„ΡƒΠ½ΠΊΡ†ΠΈΠΈ ΠΈΠ»ΠΈ Π² Ρ‚ΠΎΡ‡ΠΊΠ΅ (0; 5), ΠΈΠ»ΠΈ Π² Ρ‚ΠΎΡ‡ΠΊΠ΅ (2; 5).

Π’ ΠΏΠ΅Ρ€Π²ΠΎΠΌ случаС Π²Π΅Ρ€Π½ΠΎ числовоС равСнство 5 = 2Γ—0 + b , ΠΎΡ‚ΠΊΡƒΠ΄Π° b = 5, Π° Π²ΠΎ Π²Ρ‚ΠΎΡ€ΠΎΠΌ случаС Π²Π΅Ρ€Π½ΠΎ числовоС равСнство 5 = 2Γ—2 + b , ΠΎΡ‚ΠΊΡƒΠ΄Π° b = 1.

Π˜Ρ‚Π°ΠΊ, сущСствуСт Π΄Π²Π΅ ΠΊΠ°ΡΠ°Ρ‚Π΅Π»ΡŒΠ½Ρ‹Π΅ y = 2x + 5 ΠΈ y = 2x + 1 ΠΊ Π³Ρ€Π°Ρ„ΠΈΠΊΡƒ Ρ„ΡƒΠ½ΠΊΡ†ΠΈΠΈ f (x ), ΠΏΠ°Ρ€Π°Π»Π»Π΅Π»ΡŒΠ½Ρ‹Π΅ прямой y = 2x – 11.

ΠžΡ‚Π²Π΅Ρ‚. y = 2x + 5, y = 2x + 1.

ΠŸΡ€ΠΈΠΌΠ΅Ρ€ 3. Π”Π°Π½Π° функция f (x ) = x 2 – 6x + 7. НапишСм ΡƒΡ€Π°Π²Π½Π΅Π½ΠΈΠ΅ ΠΊΠ°ΡΠ°Ρ‚Π΅Π»ΡŒΠ½ΠΎΠΉ ΠΊ Π³Ρ€Π°Ρ„ΠΈΠΊΡƒ Ρ„ΡƒΠ½ΠΊΡ†ΠΈΠΈ f (x ), проходящСй Ρ‡Π΅Ρ€Π΅Π· Ρ‚ΠΎΡ‡ΠΊΡƒ A (2; –5).

РСшСниС. Π’Π°ΠΊ ΠΊΠ°ΠΊ f (2) –5, Ρ‚ΠΎ Ρ‚ΠΎΡ‡ΠΊΠ° A Π½Π΅ ΠΏΡ€ΠΈΠ½Π°Π΄Π»Π΅ΠΆΠΈΡ‚ Π³Ρ€Π°Ρ„ΠΈΠΊΡƒ Ρ„ΡƒΠ½ΠΊΡ†ΠΈΠΈ f (x ). ΠŸΡƒΡΡ‚ΡŒ x 0 — абсцисса Ρ‚ΠΎΡ‡ΠΊΠΈ касания.

ΠŸΡ€ΠΎΠΈΠ·Π²ΠΎΠ΄Π½Π°Ρ Ρ„ΡƒΠ½ΠΊΡ†ΠΈΠΈ f (x ) сущСствуСт для любого x R . НайдСм Π΅Π΅:

= (x 2 – 6x + 1)β€² = 2x – 6.

Π’ΠΎΠ³Π΄Π° f (x 0) = x – 6x 0 + 7; (x 0) = 2x 0 – 6. Π£Ρ€Π°Π²Π½Π΅Π½ΠΈΠ΅ ΠΊΠ°ΡΠ°Ρ‚Π΅Π»ΡŒΠ½ΠΎΠΉ ΠΈΠΌΠ΅Π΅Ρ‚ Π²ΠΈΠ΄:

y = (2x 0 – 6)(x – x 0) + x – 6x + 7,

y = (2x 0 – 6)x – x + 7.

Π’Π°ΠΊ ΠΊΠ°ΠΊ Ρ‚ΠΎΡ‡ΠΊΠ° A ΠΏΡ€ΠΈΠ½Π°Π΄Π»Π΅ΠΆΠΈΡ‚ ΠΊΠ°ΡΠ°Ρ‚Π΅Π»ΡŒΠ½ΠΎΠΉ, Ρ‚ΠΎ справСдливо числовоС равСнство

–5 = (2x 0 – 6)Γ—2– x + 7,

ΠΎΡ‚ΠΊΡƒΠ΄Π° x 0 = 0 ΠΈΠ»ΠΈ x 0 = 4. Π­Ρ‚ΠΎ ΠΎΠ·Π½Π°Ρ‡Π°Π΅Ρ‚, Ρ‡Ρ‚ΠΎ Ρ‡Π΅Ρ€Π΅Π· Ρ‚ΠΎΡ‡ΠΊΡƒ A ΠΌΠΎΠΆΠ½ΠΎ провСсти Π΄Π²Π΅ ΠΊΠ°ΡΠ°Ρ‚Π΅Π»ΡŒΠ½Ρ‹Π΅ ΠΊ Π³Ρ€Π°Ρ„ΠΈΠΊΡƒ Ρ„ΡƒΠ½ΠΊΡ†ΠΈΠΈ f (x ).

Если x 0 = 0, Ρ‚ΠΎ ΡƒΡ€Π°Π²Π½Π΅Π½ΠΈΠ΅ ΠΊΠ°ΡΠ°Ρ‚Π΅Π»ΡŒΠ½ΠΎΠΉ ΠΈΠΌΠ΅Π΅Ρ‚ Π²ΠΈΠ΄ y = –6x + 7. Если x 0 = 4, Ρ‚ΠΎ ΡƒΡ€Π°Π²Π½Π΅Π½ΠΈΠ΅ ΠΊΠ°ΡΠ°Ρ‚Π΅Π»ΡŒΠ½ΠΎΠΉ ΠΈΠΌΠ΅Π΅Ρ‚ Π²ΠΈΠ΄ y = 2x – 9.

ΠžΡ‚Π²Π΅Ρ‚. y = –6x + 7, y = 2x – 9.

ΠŸΡ€ΠΈΠΌΠ΅Ρ€ 4. Π”Π°Π½Ρ‹ Ρ„ΡƒΠ½ΠΊΡ†ΠΈΠΈ f (x ) = x 2 – 2x + 2 ΠΈ g (x ) = –x 2 – 3. НапишСм ΡƒΡ€Π°Π²Π½Π΅Π½ΠΈΠ΅ ΠΎΠ±Ρ‰Π΅ΠΉ ΠΊΠ°ΡΠ°Ρ‚Π΅Π»ΡŒΠ½ΠΎΠΉ ΠΊ Π³Ρ€Π°Ρ„ΠΈΠΊΠ°ΠΌ этих Ρ„ΡƒΠ½ΠΊΡ†ΠΈΠΈ.

РСшСниС. ΠŸΡƒΡΡ‚ΡŒ x 1 — абсцисса Ρ‚ΠΎΡ‡ΠΊΠΈ касания искомой прямой с Π³Ρ€Π°Ρ„ΠΈΠΊΠΎΠΌ Ρ„ΡƒΠ½ΠΊΡ†ΠΈΠΈ f (x ), Π° x 2 — абсцисса Ρ‚ΠΎΡ‡ΠΊΠΈ касания Ρ‚ΠΎΠΉ ΠΆΠ΅ прямой с Π³Ρ€Π°Ρ„ΠΈΠΊΠΎΠΌ Ρ„ΡƒΠ½ΠΊΡ†ΠΈΠΈ g (x ).

ΠŸΡ€ΠΎΠΈΠ·Π²ΠΎΠ΄Π½Π°Ρ Ρ„ΡƒΠ½ΠΊΡ†ΠΈΠΈ f (x ) сущСствуСт для любого x R . НайдСм Π΅Π΅:

= (x 2 – 2x + 2)β€² = 2x – 2.

Π’ΠΎΠ³Π΄Π° f (x 1) = x – 2x 1 + 2; (x 1) = 2 x 1 – 2. Π£Ρ€Π°Π²Π½Π΅Π½ΠΈΠ΅ ΠΊΠ°ΡΠ°Ρ‚Π΅Π»ΡŒΠ½ΠΎΠΉ ΠΈΠΌΠ΅Π΅Ρ‚ Π²ΠΈΠ΄:

y = (2x 1 – 2)(x – x 1) + x – 2x 1 + 2,

y = (2x 1 – 2)x – x + 2. (1)

НайдСм ΠΏΡ€ΠΎΠΈΠ·Π²ΠΎΠ΄Π½ΡƒΡŽ Ρ„ΡƒΠ½ΠΊΡ†ΠΈΠΈ g (x ):

= (–x 2 – 3)β€² = –2x .

ΠŸΡƒΡΡ‚ΡŒ Π΄Π°Π½Π° функция f , которая Π² Π½Π΅ΠΊΠΎΡ‚ΠΎΡ€ΠΎΠΉ Ρ‚ΠΎΡ‡ΠΊΠ΅ x 0 ΠΈΠΌΠ΅Π΅Ρ‚ ΠΊΠΎΠ½Π΅Ρ‡Π½ΡƒΡŽ ΠΏΡ€ΠΎΠΈΠ·Π²ΠΎΠ΄Π½ΡƒΡŽ f (x 0). Π’ΠΎΠ³Π΄Π° прямая, проходящая Ρ‡Π΅Ρ€Π΅Π· Ρ‚ΠΎΡ‡ΠΊΡƒ (x 0 ; f (x 0)), ΠΈΠΌΠ΅ΡŽΡ‰Π°Ρ ΡƒΠ³Π»ΠΎΠ²ΠΎΠΉ коэффициСнт f ’(x 0), называСтся ΠΊΠ°ΡΠ°Ρ‚Π΅Π»ΡŒΠ½ΠΎΠΉ.

А Ρ‡Ρ‚ΠΎ Π±ΡƒΠ΄Π΅Ρ‚, Ссли производная Π² Ρ‚ΠΎΡ‡ΠΊΠ΅ x 0 Π½Π΅ сущСствуСт? Π’ΠΎΠ·ΠΌΠΎΠΆΠ½Ρ‹ Π΄Π²Π° Π²Π°Ρ€ΠΈΠ°Π½Ρ‚Π°:

  1. ΠšΠ°ΡΠ°Ρ‚Π΅Π»ΡŒΠ½Π°Ρ ΠΊ Π³Ρ€Π°Ρ„ΠΈΠΊΡƒ Ρ‚ΠΎΠΆΠ΅ Π½Π΅ сущСствуСт. ΠšΠ»Π°ΡΡΠΈΡ‡Π΅ΡΠΊΠΈΠΉ ΠΏΡ€ΠΈΠΌΠ΅Ρ€ — функция y = |x | Π² Ρ‚ΠΎΡ‡ΠΊΠ΅ (0; 0).
  2. ΠšΠ°ΡΠ°Ρ‚Π΅Π»ΡŒΠ½Π°Ρ становится Π²Π΅Ρ€Ρ‚ΠΈΠΊΠ°Π»ΡŒΠ½ΠΎΠΉ. Π­Ρ‚ΠΎ Π²Π΅Ρ€Π½ΠΎ, ΠΊ ΠΏΡ€ΠΈΠΌΠ΅Ρ€Ρƒ, для Ρ„ΡƒΠ½ΠΊΡ†ΠΈΠΈ y = arcsin x Π² Ρ‚ΠΎΡ‡ΠΊΠ΅ (1; Ο€ /2).

Π£Ρ€Π°Π²Π½Π΅Π½ΠΈΠ΅ ΠΊΠ°ΡΠ°Ρ‚Π΅Π»ΡŒΠ½ΠΎΠΉ

Всякая Π½Π΅Π²Π΅Ρ€Ρ‚ΠΈΠΊΠ°Π»ΡŒΠ½Π°Ρ прямая задаСтся ΡƒΡ€Π°Π²Π½Π΅Π½ΠΈΠ΅ΠΌ Π²ΠΈΠ΄Π° y = kx + b , Π³Π΄Π΅ k — ΡƒΠ³Π»ΠΎΠ²ΠΎΠΉ коэффициСнт. ΠšΠ°ΡΠ°Ρ‚Π΅Π»ΡŒΠ½Π°Ρ — Π½Π΅ ΠΈΡΠΊΠ»ΡŽΡ‡Π΅Π½ΠΈΠ΅, ΠΈ Ρ‡Ρ‚ΠΎΠ±Ρ‹ ΡΠΎΡΡ‚Π°Π²ΠΈΡ‚ΡŒ Π΅Π΅ ΡƒΡ€Π°Π²Π½Π΅Π½ΠΈΠ΅ Π² Π½Π΅ΠΊΠΎΡ‚ΠΎΡ€ΠΎΠΉ Ρ‚ΠΎΡ‡ΠΊΠ΅ x 0 , достаточно Π·Π½Π°Ρ‚ΡŒ Π·Π½Π°Ρ‡Π΅Π½ΠΈΠ΅ Ρ„ΡƒΠ½ΠΊΡ†ΠΈΠΈ ΠΈ ΠΏΡ€ΠΎΠΈΠ·Π²ΠΎΠ΄Π½ΠΎΠΉ Π² этой Ρ‚ΠΎΡ‡ΠΊΠ΅.

Π˜Ρ‚Π°ΠΊ, ΠΏΡƒΡΡ‚ΡŒ Π΄Π°Π½Π° функция y = f (x ), которая ΠΈΠΌΠ΅Π΅Ρ‚ ΠΏΡ€ΠΎΠΈΠ·Π²ΠΎΠ΄Π½ΡƒΡŽ y = f ’(x ) Π½Π° ΠΎΡ‚Ρ€Π΅Π·ΠΊΠ΅ . Π’ΠΎΠ³Π΄Π° Π² любой Ρ‚ΠΎΡ‡ΠΊΠ΅ x 0 ∈ (a ; b ) ΠΊ Π³Ρ€Π°Ρ„ΠΈΠΊΡƒ этой Ρ„ΡƒΠ½ΠΊΡ†ΠΈΠΈ ΠΌΠΎΠΆΠ½ΠΎ провСсти ΠΊΠ°ΡΠ°Ρ‚Π΅Π»ΡŒΠ½ΡƒΡŽ, которая задаСтся ΡƒΡ€Π°Π²Π½Π΅Π½ΠΈΠ΅ΠΌ:

y = f ’(x 0) Β· (x βˆ’ x 0) + f (x 0)

Π—Π΄Π΅ΡΡŒ f ’(x 0) — Π·Π½Π°Ρ‡Π΅Π½ΠΈΠ΅ ΠΏΡ€ΠΎΠΈΠ·Π²ΠΎΠ΄Π½ΠΎΠΉ Π² Ρ‚ΠΎΡ‡ΠΊΠ΅ x 0 , Π° f (x 0) — Π·Π½Π°Ρ‡Π΅Π½ΠΈΠ΅ самой Ρ„ΡƒΠ½ΠΊΡ†ΠΈΠΈ.

Π—Π°Π΄Π°Ρ‡Π°. Π”Π°Π½Π° функция y = x 3 . Π‘ΠΎΡΡ‚Π°Π²ΠΈΡ‚ΡŒ ΡƒΡ€Π°Π²Π½Π΅Π½ΠΈΠ΅ ΠΊΠ°ΡΠ°Ρ‚Π΅Π»ΡŒΠ½ΠΎΠΉ ΠΊ Π³Ρ€Π°Ρ„ΠΈΠΊΡƒ этой Ρ„ΡƒΠ½ΠΊΡ†ΠΈΠΈ Π² Ρ‚ΠΎΡ‡ΠΊΠ΅ x 0 = 2.

Π£Ρ€Π°Π²Π½Π΅Π½ΠΈΠ΅ ΠΊΠ°ΡΠ°Ρ‚Π΅Π»ΡŒΠ½ΠΎΠΉ: y = f ’(x 0) Β· (x βˆ’ x 0) + f (x 0). Π’ΠΎΡ‡ΠΊΠ° x 0 = 2 Π½Π°ΠΌ Π΄Π°Π½Π°, Π° Π²ΠΎΡ‚ значСния f (x 0) ΠΈ f ’(x 0) придСтся Π²Ρ‹Ρ‡ΠΈΡΠ»ΡΡ‚ΡŒ.

Для Π½Π°Ρ‡Π°Π»Π° Π½Π°ΠΉΠ΄Π΅ΠΌ Π·Π½Π°Ρ‡Π΅Π½ΠΈΠ΅ Ρ„ΡƒΠ½ΠΊΡ†ΠΈΠΈ. Π’ΡƒΡ‚ всС Π»Π΅Π³ΠΊΠΎ: f (x 0) = f (2) = 2 3 = 8;
Π’Π΅ΠΏΠ΅Ρ€ΡŒ Π½Π°ΠΉΠ΄Π΅ΠΌ ΠΏΡ€ΠΎΠΈΠ·Π²ΠΎΠ΄Π½ΡƒΡŽ: f ’(x ) = (x 3)’ = 3x 2 ;
ΠŸΠΎΠ΄ΡΡ‚Π°Π²Π»ΡΠ΅ΠΌ Π² ΠΏΡ€ΠΎΠΈΠ·Π²ΠΎΠ΄Π½ΡƒΡŽ x 0 = 2: f ’(x 0) = f ’(2) = 3 Β· 2 2 = 12;
Π˜Ρ‚ΠΎΠ³ΠΎ ΠΏΠΎΠ»ΡƒΡ‡Π°Π΅ΠΌ: y = 12 Β· (x βˆ’ 2) + 8 = 12x βˆ’ 24 + 8 = 12x βˆ’ 16.
Π­Ρ‚ΠΎ ΠΈ Π΅ΡΡ‚ΡŒ ΡƒΡ€Π°Π²Π½Π΅Π½ΠΈΠ΅ ΠΊΠ°ΡΠ°Ρ‚Π΅Π»ΡŒΠ½ΠΎΠΉ.

Π—Π°Π΄Π°Ρ‡Π°. Π‘ΠΎΡΡ‚Π°Π²ΠΈΡ‚ΡŒ ΡƒΡ€Π°Π²Π½Π΅Π½ΠΈΠ΅ ΠΊΠ°ΡΠ°Ρ‚Π΅Π»ΡŒΠ½ΠΎΠΉ ΠΊ Π³Ρ€Π°Ρ„ΠΈΠΊΡƒ Ρ„ΡƒΠ½ΠΊΡ†ΠΈΠΈ f (x ) = 2sin x + 5 Π² Ρ‚ΠΎΡ‡ΠΊΠ΅ x 0 = Ο€ /2.

Π’ этот Ρ€Π°Π· Π½Π΅ Π±ΡƒΠ΄Π΅ΠΌ ΠΏΠΎΠ΄Ρ€ΠΎΠ±Π½ΠΎ Ρ€Π°ΡΠΏΠΈΡΡ‹Π²Π°Ρ‚ΡŒ ΠΊΠ°ΠΆΠ΄ΠΎΠ΅ дСйствиС — ΡƒΠΊΠ°ΠΆΠ΅ΠΌ лишь ΠΊΠ»ΡŽΡ‡Π΅Π²Ρ‹Π΅ шаги. ИмССм:

f (x 0) = f (Ο€ /2) = 2sin (Ο€ /2) + 5 = 2 + 5 = 7;
f ’(x ) = (2sin x + 5)’ = 2cos x ;
f ’(x 0) = f ’(Ο€ /2) = 2cos (Ο€ /2) = 0;

Π£Ρ€Π°Π²Π½Π΅Π½ΠΈΠ΅ ΠΊΠ°ΡΠ°Ρ‚Π΅Π»ΡŒΠ½ΠΎΠΉ:

y = 0 Β· (x βˆ’ Ο€ /2) + 7 β‡’ y = 7

Π’ послСднСм случаС прямая оказалась Π³ΠΎΡ€ΠΈΠ·ΠΎΠ½Ρ‚Π°Π»ΡŒΠ½ΠΎΠΉ, Ρ‚.ΠΊ. Π΅Π΅ ΡƒΠ³Π»ΠΎΠ²ΠΎΠΉ коэффициСнт k = 0. НичСго ΡΡ‚Ρ€Π°ΡˆΠ½ΠΎΠ³ΠΎ Π² этом Π½Π΅Ρ‚ — просто ΠΌΡ‹ Π½Π°Ρ‚ΠΊΠ½ΡƒΠ»ΠΈΡΡŒ Π½Π° Ρ‚ΠΎΡ‡ΠΊΡƒ экстрСмума.

Рассмотрим ΡΠ»Π΅Π΄ΡƒΡŽΡ‰ΠΈΠΉ рисунок:

На Π½Π΅ΠΌ ΠΈΠ·ΠΎΠ±Ρ€Π°ΠΆΠ΅Π½Π° нСкоторая функция y = f(x), которая Π΄ΠΈΡ„Ρ„Π΅Ρ€Π΅Π½Ρ†ΠΈΡ€ΡƒΠ΅ΠΌΠ° Π² Ρ‚ΠΎΡ‡ΠΊΠ΅ a. ΠžΡ‚ΠΌΠ΅Ρ‡Π΅Π½Π° Ρ‚ΠΎΡ‡ΠΊΠ° М с ΠΊΠΎΠΎΡ€Π΄ΠΈΠ½Π°Ρ‚Π°ΠΌΠΈ (Π°; f(a)). Π§Π΅Ρ€Π΅Π· ΠΏΡ€ΠΎΠΈΠ·Π²ΠΎΠ»ΡŒΠ½ΡƒΡŽ Ρ‚ΠΎΡ‡ΠΊΡƒ Π (a + βˆ†x; f(a + βˆ†x)) Π³Ρ€Π°Ρ„ΠΈΠΊΠ° ΠΏΡ€ΠΎΠ²Π΅Π΄Π΅Π½Π° сСкущая МР.

Если Ρ‚Π΅ΠΏΠ΅Ρ€ΡŒ Ρ‚ΠΎΡ‡ΠΊΡƒ Π  ΡΠ΄Π²ΠΈΠ³Π°Ρ‚ΡŒ ΠΏΠΎ Π³Ρ€Π°Ρ„ΠΈΠΊΡƒ ΠΊ Ρ‚ΠΎΡ‡ΠΊΠ΅ М, Ρ‚ΠΎ прямая МР Π±ΡƒΠ΄Π΅Ρ‚ ΠΏΠΎΠ²ΠΎΡ€Π°Ρ‡ΠΈΠ²Π°Ρ‚ΡŒΡΡ Π²ΠΎΠΊΡ€ΡƒΠ³ Ρ‚ΠΎΡ‡ΠΊΠΈ М. ΠŸΡ€ΠΈ этом βˆ†Ρ… Π±ΡƒΠ΄Π΅Ρ‚ ΡΡ‚Ρ€Π΅ΠΌΠΈΡ‚ΡŒΡΡ ΠΊ Π½ΡƒΠ»ΡŽ. ΠžΡ‚ΡΡŽΠ΄Π° ΠΌΠΎΠΆΠ½ΠΎ ΡΡ„ΠΎΡ€ΠΌΡƒΠ»ΠΈΡ€ΠΎΠ²Π°Ρ‚ΡŒ ΠΎΠΏΡ€Π΅Π΄Π΅Π»Π΅Π½ΠΈΠ΅ ΠΊΠ°ΡΠ°Ρ‚Π΅Π»ΡŒΠ½ΠΎΠΉ ΠΊ Π³Ρ€Π°Ρ„ΠΈΠΊΡƒ Ρ„ΡƒΠ½ΠΊΡ†ΠΈΠΈ.

ΠšΠ°ΡΠ°Ρ‚Π΅Π»ΡŒΠ½Π°Ρ ΠΊ Π³Ρ€Π°Ρ„ΠΈΠΊΡƒ Ρ„ΡƒΠ½ΠΊΡ†ΠΈΠΈ

ΠšΠ°ΡΠ°Ρ‚Π΅Π»ΡŒΠ½Π°Ρ ΠΊ Π³Ρ€Π°Ρ„ΠΈΠΊΡƒ Ρ„ΡƒΠ½ΠΊΡ†ΠΈΠΈ Π΅ΡΡ‚ΡŒ ΠΏΡ€Π΅Π΄Π΅Π»ΡŒΠ½ΠΎΠ΅ ΠΏΠΎΠ»ΠΎΠΆΠ΅Π½ΠΈΠ΅ сСкущСй ΠΏΡ€ΠΈ стрСмлСнии приращСния Π°Ρ€Π³ΡƒΠΌΠ΅Π½Ρ‚Π° ΠΊ Π½ΡƒΠ»ΡŽ. Π‘Π»Π΅Π΄ΡƒΠ΅Ρ‚ ΠΏΠΎΠ½ΠΈΠΌΠ°Ρ‚ΡŒ, Ρ‡Ρ‚ΠΎ сущСствованиС ΠΏΡ€ΠΎΠΈΠ·Π²ΠΎΠ΄Π½ΠΎΠΉ Ρ„ΡƒΠ½ΠΊΡ†ΠΈΠΈ f Π² Ρ‚ΠΎΡ‡ΠΊΠ΅ Ρ…0, ΠΎΠ·Π½Π°Ρ‡Π°Π΅Ρ‚, Ρ‡Ρ‚ΠΎ Π² этой Ρ‚ΠΎΡ‡ΠΊΠ΅ Π³Ρ€Π°Ρ„ΠΈΠΊΠ° сущСствуСт ΠΊΠ°ΡΠ°Ρ‚Π΅Π»ΡŒΠ½Π°Ρ ΠΊ Π½Π΅ΠΌΡƒ.

ΠŸΡ€ΠΈ этом ΡƒΠ³Π»ΠΎΠ²ΠΎΠΉ коэффициСнт ΠΊΠ°ΡΠ°Ρ‚Π΅Π»ΡŒΠ½ΠΎΠΉ Π±ΡƒΠ΄Π΅Ρ‚ Ρ€Π°Π²Π΅Π½ ΠΏΡ€ΠΎΠΈΠ·Π²ΠΎΠ΄Π½ΠΎΠΉ этой Ρ„ΡƒΠ½ΠΊΡ†ΠΈΠΈ Π² этой Ρ‚ΠΎΡ‡ΠΊΠ΅ f’(x0). Π’ этом Π·Π°ΠΊΠ»ΡŽΡ‡Π°Π΅Ρ‚ΡΡ гСомСтричСский смысл ΠΏΡ€ΠΎΠΈΠ·Π²ΠΎΠ΄Π½ΠΎΠΉ. ΠšΠ°ΡΠ°Ρ‚Π΅Π»ΡŒΠ½Π°Ρ ΠΊ Π³Ρ€Π°Ρ„ΠΈΠΊΡƒ Π΄ΠΈΡ„Ρ„Π΅Ρ€Π΅Π½Ρ†ΠΈΡ€ΡƒΠ΅ΠΌΠΎΠΉ Π² Ρ‚ΠΎΡ‡ΠΊΠ΅ Ρ…0 Ρ„ΡƒΠ½ΠΊΡ†ΠΈΠΈ f — это нСкоторая прямая, проходящая Ρ‡Π΅Ρ€Π΅Π· Ρ‚ΠΎΡ‡ΠΊΡƒ (x0;f(x0)) ΠΈ ΠΈΠΌΠ΅ΡŽΡ‰Π°Ρ ΡƒΠ³Π»ΠΎΠ²ΠΎΠΉ коэффициСнт f’(x0).

Π£Ρ€Π°Π²Π½Π΅Π½ΠΈΠ΅ ΠΊΠ°ΡΠ°Ρ‚Π΅Π»ΡŒΠ½ΠΎΠΉ

ΠŸΠΎΠΏΡ‹Ρ‚Π°Π΅ΠΌΡΡ ΠΏΠΎΠ»ΡƒΡ‡ΠΈΡ‚ΡŒ ΡƒΡ€Π°Π²Π½Π΅Π½ΠΈΠ΅ ΠΊΠ°ΡΠ°Ρ‚Π΅Π»ΡŒΠ½ΠΎΠΉ ΠΊ Π³Ρ€Π°Ρ„ΠΈΠΊΡƒ Π½Π΅ΠΊΠΎΡ‚ΠΎΡ€ΠΎΠΉ Ρ„ΡƒΠ½ΠΊΡ†ΠΈΠΈ f Π² Ρ‚ΠΎΡ‡ΠΊΠ΅ А(x0; f(x0)). Π£Ρ€Π°Π²Π½Π΅Π½ΠΈΠ΅ прямой с ΡƒΠ³Π»ΠΎΠ²Ρ‹ΠΌ коэффициСнтом k ΠΈΠΌΠ΅Π΅Ρ‚ ΡΠ»Π΅Π΄ΡƒΡŽΡ‰ΠΈΠΉ Π²ΠΈΠ΄:

Π’Π°ΠΊ ΠΊΠ°ΠΊ Ρƒ нас ΡƒΠ³Π»ΠΎΠ²ΠΎΠΉ коэффициСнт Ρ€Π°Π²Π΅Π½ ΠΏΡ€ΠΎΠΈΠ·Π²ΠΎΠ΄Π½ΠΎΠΉ f’(x0) , Ρ‚ΠΎ ΡƒΡ€Π°Π²Π½Π΅Π½ΠΈΠ΅ ΠΏΡ€ΠΈΠΌΠ΅Ρ‚ ΡΠ»Π΅Π΄ΡƒΡŽΡ‰ΠΈΠΉ Π²ΠΈΠ΄: y = f’(x0) *x + b.

Π’Π΅ΠΏΠ΅Ρ€ΡŒ вычислим Π·Π½Π°Ρ‡Π΅Π½ΠΈΠ΅ b. Для этого ΠΈΡΠΏΠΎΠ»ΡŒΠ·ΡƒΠ΅ΠΌ Ρ‚ΠΎΡ‚ Ρ„Π°ΠΊΡ‚, Ρ‡Ρ‚ΠΎ функция ΠΏΡ€ΠΎΡ…ΠΎΠ΄ΠΈΡ‚ Ρ‡Π΅Ρ€Π΅Π· Ρ‚ΠΎΡ‡ΠΊΡƒ А.

f(x0) = f’(x0)*x0 + b, ΠΎΡ‚ΡΡŽΠ΄Π° Π²Ρ‹Ρ€Π°ΠΆΠ°Π΅ΠΌ b ΠΈ ΠΏΠΎΠ»ΡƒΡ‡ΠΈΠΌ b = f(x0) — f’(x0)*x0.

ΠŸΠΎΠ΄ΡΡ‚Π°Π²Π»ΡΠ΅ΠΌ ΠΏΠΎΠ»ΡƒΡ‡Π΅Π½Π½ΠΎΠ΅ Π·Π½Π°Ρ‡Π΅Π½ΠΈΠ΅ Π² ΡƒΡ€Π°Π²Π½Π΅Π½ΠΈΠ΅ ΠΊΠ°ΡΠ°Ρ‚Π΅Π»ΡŒΠ½ΠΎΠΉ:

y = f’(x0)*x + b = f’(x0)*x + f(x0) — f’(x0)*x0 = f(x0) + f’(x0)*(x — x0).

y = f(x0) + f’(x0)*(x — x0).

Рассмотрим ΡΠ»Π΅Π΄ΡƒΡŽΡ‰ΠΈΠΉ ΠΏΡ€ΠΈΠΌΠ΅Ρ€: Π½Π°ΠΉΡ‚ΠΈ ΡƒΡ€Π°Π²Π½Π΅Π½ΠΈΠ΅ ΠΊΠ°ΡΠ°Ρ‚Π΅Π»ΡŒΠ½ΠΎΠΉ ΠΊ Π³Ρ€Π°Ρ„ΠΈΠΊΡƒ Ρ„ΡƒΠ½ΠΊΡ†ΠΈΠΈ f(x) = x 3 — 2*x 2 + 1 Π² Ρ‚ΠΎΡ‡ΠΊΠ΅ Ρ… = 2.

2. f(x0) = f(2) = 2 2 — 2*2 2 + 1 = 1.

3. f’(x) = 3*x 2 — 4*x.

4. f’(x0) = f’(2) = 3*2 2 — 4*2 = 4.

5. ΠŸΠΎΠ΄ΡΡ‚Π°Π²ΠΈΠΌ ΠΏΠΎΠ»ΡƒΡ‡Π΅Π½Π½Ρ‹Π΅ значСния Π² Ρ„ΠΎΡ€ΠΌΡƒΠ»Ρƒ ΠΊΠ°ΡΠ°Ρ‚Π΅Π»ΡŒΠ½ΠΎΠΉ, ΠΏΠΎΠ»ΡƒΡ‡ΠΈΠΌ: y = 1 + 4*(x — 2). Раскрыв скобки ΠΈ привСдя ΠΏΠΎΠ΄ΠΎΠ±Π½Ρ‹Π΅ слагаСмыС ΠΏΠΎΠ»ΡƒΡ‡ΠΈΠΌ: y = 4*x — 7.

ΠžΡ‚Π²Π΅Ρ‚: y = 4*x — 7.

ΠžΠ±Ρ‰Π°Ρ схСма составлСния уравнСния ΠΊΠ°ΡΠ°Ρ‚Π΅Π»ΡŒΠ½ΠΎΠΉ ΠΊ Π³Ρ€Π°Ρ„ΠΈΠΊΡƒ Ρ„ΡƒΠ½ΠΊΡ†ΠΈΠΈ y = f(x):

1. ΠžΠΏΡ€Π΅Π΄Π΅Π»ΠΈΡ‚ΡŒ Ρ…0.

2. Π’Ρ‹Ρ‡ΠΈΡΠ»ΠΈΡ‚ΡŒ f(x0).

3. Π’Ρ‹Ρ‡ΠΈΡΠ»ΠΈΡ‚ΡŒ f’(x)

Π˜Π½ΡΡ‚Ρ€ΡƒΠΊΡ†ΠΈΡ

ΠžΠΏΡ€Π΅Π΄Π΅Π»ΡΠ΅ΠΌ ΡƒΠ³Π»ΠΎΠ²ΠΎΠΉ коэффициСнт ΠΊΠ°ΡΠ°Ρ‚Π΅Π»ΡŒΠ½ΠΎΠΉ ΠΊ ΠΊΡ€ΠΈΠ²ΠΎΠΉ Π² Ρ‚ΠΎΡ‡ΠΊΠ΅ М.
ΠšΡ€ΠΈΠ²Π°Ρ, ΠΏΡ€Π΅Π΄ΡΡ‚Π°Π²Π»ΡΡŽΡ‰Π°Ρ собой Π³Ρ€Π°Ρ„ΠΈΠΊ Ρ„ΡƒΠ½ΠΊΡ†ΠΈΠΈ y = f(x), Π½Π΅ΠΏΡ€Π΅Ρ€Ρ‹Π²Π½Π° Π² Π½Π΅ΠΊΠΎΡ‚ΠΎΡ€ΠΎΠΉ окрСстности Ρ‚ΠΎΡ‡ΠΊΠΈ М (Π²ΠΊΠ»ΡŽΡ‡Π°Ρ саму Ρ‚ΠΎΡ‡ΠΊΡƒ М).

Если значСния fβ€˜(x0) Π½Π΅ сущСствуСт, Ρ‚ΠΎ Π»ΠΈΠ±ΠΎ ΠΊΠ°ΡΠ°Ρ‚Π΅Π»ΡŒΠ½ΠΎΠΉ Π½Π΅Ρ‚, Π»ΠΈΠ±ΠΎ ΠΎΠ½Π° ΠΏΡ€ΠΎΡ…ΠΎΠ΄ΠΈΡ‚ Π²Π΅Ρ€Ρ‚ΠΈΠΊΠ°Π»ΡŒΠ½ΠΎ. Π’Π²ΠΈΠ΄Ρƒ этого, Π½Π°Π»ΠΈΡ‡ΠΈΠ΅ ΠΏΡ€ΠΎΠΈΠ·Π²ΠΎΠ΄Π½ΠΎΠΉ Ρ„ΡƒΠ½ΠΊΡ†ΠΈΠΈ Π² Ρ‚ΠΎΡ‡ΠΊΠ΅ Ρ…0 обусловлСно сущСствованиСм Π½Π΅Π²Π΅Ρ€Ρ‚ΠΈΠΊΠ°Π»ΡŒΠ½ΠΎΠΉ ΠΊΠ°ΡΠ°Ρ‚Π΅Π»ΡŒΠ½ΠΎΠΉ, ΡΠΎΠΏΡ€ΠΈΠΊΠ°ΡΠ°ΡŽΡ‰Π΅ΠΉΡΡ с Π³Ρ€Π°Ρ„ΠΈΠΊΠΎΠΌ Ρ„ΡƒΠ½ΠΊΡ†ΠΈΠΈ Π² Ρ‚ΠΎΡ‡ΠΊΠ΅ (Ρ…0, f(Ρ…0)). Π’ этом случаС ΡƒΠ³Π»ΠΎΠ²ΠΎΠΉ коэффициСнт ΠΊΠ°ΡΠ°Ρ‚Π΅Π»ΡŒΠ½ΠΎΠΉ Ρ€Π°Π²Π΅Π½ Π±ΡƒΠ΄Π΅Ρ‚ f»(Ρ…0). Π’Π°ΠΊΠΈΠΌ ΠΎΠ±Ρ€Π°Π·ΠΎΠΌ, становится ясСн гСомСтричСский смысл ΠΏΡ€ΠΎΠΈΠ·Π²ΠΎΠ΄Π½ΠΎΠΉ – расчСт ΡƒΠ³Π»ΠΎΠ²ΠΎΠ³ΠΎ коэффициСнта ΠΊΠ°ΡΠ°Ρ‚Π΅Π»ΡŒΠ½ΠΎΠΉ.

НайдитС Π·Π½Π°Ρ‡Π΅Π½ΠΈΠ΅ абсциссы Ρ‚ΠΎΡ‡ΠΊΠΈ касания, ΠΊΠΎΡ‚ΠΎΡ€ΡƒΡŽ ΠΎΠ±ΠΎΠ·Π½Π°Ρ‡Π°ΡŽΡ‚ΡΡ Π±ΡƒΠΊΠ²ΠΎΠΉ Β«Π°Β». Если ΠΎΠ½Π° совпадаСт с Π·Π°Π΄Π°Π½Π½ΠΎΠΉ Ρ‚ΠΎΡ‡ΠΊΠΎΠΉ ΠΊΠ°ΡΠ°Ρ‚Π΅Π»ΡŒΠ½ΠΎΠΉ, Ρ‚ΠΎ Β«Π°Β» Π±ΡƒΠ΄Π΅Ρ‚ Π΅Π΅ Ρ…-ΠΊΠΎΠΎΡ€Π΄ΠΈΠ½Π°Ρ‚Π΅. ΠžΠΏΡ€Π΅Π΄Π΅Π»ΠΈΡ‚Π΅ Π·Π½Π°Ρ‡Π΅Π½ΠΈΠ΅ Ρ„ΡƒΠ½ΠΊΡ†ΠΈΠΈ f(a), подставив Π² ΡƒΡ€Π°Π²Π½Π΅Π½ΠΈΠ΅ Ρ„ΡƒΠ½ΠΊΡ†ΠΈΠΈ Π²Π΅Π»ΠΈΡ‡ΠΈΠ½Ρƒ абсциссы.

ΠžΠΏΡ€Π΅Π΄Π΅Π»ΠΈΡ‚Π΅ ΠΏΠ΅Ρ€Π²ΡƒΡŽ ΠΏΡ€ΠΎΠΈΠ·Π²ΠΎΠ΄Π½ΡƒΡŽ уравнСния Ρ„ΡƒΠ½ΠΊΡ†ΠΈΠΈ f’(x) ΠΈ ΠΏΠΎΠ΄ΡΡ‚Π°Π²ΡŒΡ‚Π΅ Π² Π½Π΅Π³ΠΎ Π·Π½Π°Ρ‡Π΅Π½ΠΈΠ΅ Ρ‚ΠΎΡ‡ΠΊΠΈ Β«Π°Β».

Π’ΠΎΠ·ΡŒΠΌΠΈΡ‚Π΅ ΠΎΠ±Ρ‰Π΅Π΅ ΡƒΡ€Π°Π²Π½Π΅Π½ΠΈΠ΅ ΠΊΠ°ΡΠ°Ρ‚Π΅Π»ΡŒΠ½ΠΎΠΉ, ΠΊΠΎΡ‚ΠΎΡ€ΠΎΠ΅ опрСдСляСтся ΠΊΠ°ΠΊ y = f(a) = f (a)(x – a), ΠΈ ΠΏΠΎΠ΄ΡΡ‚Π°Π²ΡŒΡ‚Π΅ Π² Π½Π΅Π³ΠΎ Π½Π°ΠΉΠ΄Π΅Π½Π½Ρ‹Π΅ значСния a, f(a), f «(a). Π’ Ρ€Π΅Π·ΡƒΠ»ΡŒΡ‚Π°Ρ‚Π΅ Π±ΡƒΠ΄Π΅Ρ‚ Π½Π°ΠΉΠ΄Π΅Π½ΠΎ Ρ€Π΅ΡˆΠ΅Π½ΠΈΠ΅ Π³Ρ€Π°Ρ„ΠΈΠΊΠ° ΠΈ ΠΊΠ°ΡΠ°Ρ‚Π΅Π»ΡŒΠ½ΠΎΠΉ.

Π Π΅ΡˆΠΈΡ‚Π΅ Π·Π°Π΄Π°Ρ‡Ρƒ ΠΈΠ½Ρ‹ΠΌ способом, Ссли заданная Ρ‚ΠΎΡ‡ΠΊΠ° ΠΊΠ°ΡΠ°Ρ‚Π΅Π»ΡŒΠ½ΠΎΠΉ Π½Π΅ совпала с Ρ‚ΠΎΡ‡ΠΊΠΎΠΉ касания. Π’ этом случаС Π½Π΅ΠΎΠ±Ρ…ΠΎΠ΄ΠΈΠΌΠΎ Π² ΡƒΡ€Π°Π²Π½Π΅Π½ΠΈΠ΅ ΠΊΠ°ΡΠ°Ρ‚Π΅Π»ΡŒΠ½ΠΎΠΉ вмСсто Ρ†ΠΈΡ„Ρ€ ΠΏΠΎΠ΄ΡΡ‚Π°Π²ΠΈΡ‚ΡŒ Β«Π°Β». ПослС этого вмСсто Π±ΡƒΠΊΠ² Β«Ρ…Β» ΠΈ Β«ΡƒΒ» ΠΏΠΎΠ΄ΡΡ‚Π°Π²ΡŒΡ‚Π΅ Π·Π½Π°Ρ‡Π΅Π½ΠΈΠ΅ ΠΊΠΎΠΎΡ€Π΄ΠΈΠ½Π°Ρ‚ Π·Π°Π΄Π°Π½Π½ΠΎΠΉ Ρ‚ΠΎΡ‡ΠΊΠΈ. Π Π΅ΡˆΠΈΡ‚Π΅ ΠΏΠΎΠ»ΡƒΡ‡ΠΈΠ²ΡˆΠ΅Π΅ΡΡ ΡƒΡ€Π°Π²Π½Π΅Π½ΠΈΠ΅, Π² ΠΊΠΎΡ‚ΠΎΡ€ΠΎΠΌ Β«Π°Β» являСтся нСизвСстной. ΠŸΠΎΡΡ‚Π°Π²ΡŒΡ‚Π΅ ΠΏΠΎΠ»ΡƒΡ‡Π΅Π½Π½ΠΎΠ΅ Π·Π½Π°Ρ‡Π΅Π½ΠΈΠ΅ Π² ΡƒΡ€Π°Π²Π½Π΅Π½ΠΈΠ΅ ΠΊΠ°ΡΠ°Ρ‚Π΅Π»ΡŒΠ½ΠΎΠΉ.

Π‘ΠΎΡΡ‚Π°Π²ΡŒΡ‚Π΅ ΡƒΡ€Π°Π²Π½Π΅Π½ΠΈΠ΅ ΠΊΠ°ΡΠ°Ρ‚Π΅Π»ΡŒΠ½ΠΎΠΉ с Π±ΡƒΠΊΠ²ΠΎΠΉ Β«Π°Β», Ссли Π² условии Π·Π°Π΄Π°Ρ‡ΠΈ Π·Π°Π΄Π°Π½ΠΎ ΡƒΡ€Π°Π²Π½Π΅Π½ΠΈΠ΅ Ρ„ΡƒΠ½ΠΊΡ†ΠΈΠΈ ΠΈ ΡƒΡ€Π°Π²Π½Π΅Π½ΠΈΠ΅ ΠΏΠ°Ρ€Π°Π»Π»Π΅Π»ΡŒΠ½ΠΎΠΉ Π»ΠΈΠ½ΠΈΠΈ ΠΎΡ‚Π½ΠΎΡΠΈΡ‚Π΅Π»ΡŒΠ½ΠΎ искомой ΠΊΠ°ΡΠ°Ρ‚Π΅Π»ΡŒΠ½ΠΎΠΉ. ПослС этого Π½Π΅ΠΎΠ±Ρ…ΠΎΠ΄ΠΈΠΌΠΎ ΠΏΡ€ΠΎΠΈΠ·Π²ΠΎΠ΄Π½ΡƒΡŽ Ρ„ΡƒΠ½ΠΊΡ†ΠΈΠΈ , Ρ‡Ρ‚ΠΎΠ±Ρ‹ ΠΊΠΎΠΎΡ€Π΄ΠΈΠ½Π°Ρ‚Ρƒ Ρƒ Ρ‚ΠΎΡ‡ΠΊΠΈ Β«Π°Β». ΠŸΠΎΠ΄ΡΡ‚Π°Π²ΡŒΡ‚Π΅ ΡΠΎΠΎΡ‚Π²Π΅Ρ‚ΡΡ‚Π²ΡƒΡŽΡ‰Π΅Π΅ Π·Π½Π°Ρ‡Π΅Π½ΠΈΠ΅ Π² ΡƒΡ€Π°Π²Π½Π΅Π½ΠΈΠ΅ ΠΊΠ°ΡΠ°Ρ‚Π΅Π»ΡŒΠ½ΠΎΠΉ ΠΈ Ρ€Π΅ΡˆΠΈΡ‚Π΅ Ρ„ΡƒΠ½ΠΊΡ†ΠΈΡŽ.

Π’ этой ΡΡ‚Π°Ρ‚ΡŒΠ΅ ΠΌΡ‹ Ρ€Π°Π·Π±Π΅Ρ€Π΅ΠΌ всС Ρ‚ΠΈΠΏΡ‹ Π·Π°Π΄Π°Ρ‡ Π½Π° Π½Π°Ρ…ΠΎΠΆΠ΄Π΅Π½ΠΈΠ΅

Вспомним гСомСтричСский смысл ΠΏΡ€ΠΎΠΈΠ·Π²ΠΎΠ΄Π½ΠΎΠΉ : Ссли ΠΊ Π³Ρ€Π°Ρ„ΠΈΠΊΡƒ Ρ„ΡƒΠ½ΠΊΡ†ΠΈΠΈ Π² Ρ‚ΠΎΡ‡ΠΊΠ΅ ΠΏΡ€ΠΎΠ²Π΅Π΄Π΅Π½Π° ΠΊΠ°ΡΠ°Ρ‚Π΅Π»ΡŒΠ½Π°Ρ, Ρ‚ΠΎ коэффициСнт Π½Π°ΠΊΠ»ΠΎΠ½Π° ΠΊΠ°ΡΠ°Ρ‚Π΅Π»ΡŒΠ½ΠΎΠΉ (Ρ€Π°Π²Π½Ρ‹ΠΉ тангСнсу ΡƒΠ³Π»Π° ΠΌΠ΅ΠΆΠ΄Ρƒ ΠΊΠ°ΡΠ°Ρ‚Π΅Π»ΡŒΠ½ΠΎΠΉ ΠΈ ΠΏΠΎΠ»ΠΎΠΆΠΈΡ‚Π΅Π»ΡŒΠ½Ρ‹ΠΌ Π½Π°ΠΏΡ€Π°Π²Π»Π΅Π½ΠΈΠ΅ΠΌ оси ) Ρ€Π°Π²Π΅Π½ ΠΏΡ€ΠΎΠΈΠ·Π²ΠΎΠ΄Π½ΠΎΠΉ Ρ„ΡƒΠ½ΠΊΡ†ΠΈΠΈ Π² Ρ‚ΠΎΡ‡ΠΊΠ΅ .


Π’ΠΎΠ·ΡŒΠΌΠ΅ΠΌ Π½Π° ΠΊΠ°ΡΠ°Ρ‚Π΅Π»ΡŒΠ½ΠΎΠΉ ΠΏΡ€ΠΎΠΈΠ·Π²ΠΎΠ»ΡŒΠ½ΡƒΡŽ Ρ‚ΠΎΡ‡ΠΊΡƒ с ΠΊΠΎΠΎΡ€Π΄ΠΈΠ½Π°Ρ‚Π°ΠΌΠΈ :


И рассмотрим ΠΏΡ€ΡΠΌΠΎΡƒΠ³ΠΎΠ»ΡŒΠ½Ρ‹ΠΉ Ρ‚Ρ€Π΅ΡƒΠ³ΠΎΠ»ΡŒΠ½ΠΈΠΊ :


Π’ этом Ρ‚Ρ€Π΅ΡƒΠ³ΠΎΠ»ΡŒΠ½ΠΈΠΊΠ΅

ΠžΡ‚ΡΡŽΠ΄Π°

Π­Ρ‚ΠΎ ΠΈ Π΅ΡΡ‚ΡŒ ΡƒΡ€Π°Π²Π½Π΅Π½ΠΈΠ΅ ΠΊΠ°ΡΠ°Ρ‚Π΅Π»ΡŒΠ½ΠΎΠΉ, ΠΏΡ€ΠΎΠ²Π΅Π΄Π΅Π½Π½ΠΎΠΉ ΠΊ Π³Ρ€Π°Ρ„ΠΈΠΊΡƒ Ρ„ΡƒΠ½ΠΊΡ†ΠΈΠΈ Π² Ρ‚ΠΎΡ‡ΠΊΠ΅ .

Π§Ρ‚ΠΎΠ±Ρ‹ Π½Π°ΠΏΠΈΡΠ°Ρ‚ΡŒ ΡƒΡ€Π°Π²Π½Π΅Π½ΠΈΠ΅ ΠΊΠ°ΡΠ°Ρ‚Π΅Π»ΡŒΠ½ΠΎΠΉ, Π½Π°ΠΌ достаточно Π·Π½Π°Ρ‚ΡŒ ΡƒΡ€Π°Π²Π½Π΅Π½ΠΈΠ΅ Ρ„ΡƒΠ½ΠΊΡ†ΠΈΠΈ ΠΈ Ρ‚ΠΎΡ‡ΠΊΡƒ, Π² ΠΊΠΎΡ‚ΠΎΡ€ΠΎΠΉ ΠΏΡ€ΠΎΠ²Π΅Π΄Π΅Π½Π° ΠΊΠ°ΡΠ°Ρ‚Π΅Π»ΡŒΠ½Π°Ρ. Π’ΠΎΠ³Π΄Π° ΠΌΡ‹ смоТСм Π½Π°ΠΉΡ‚ΠΈ ΠΈ .

Π•ΡΡ‚ΡŒ Ρ‚Ρ€ΠΈ основных Ρ‚ΠΈΠΏΠ° Π·Π°Π΄Π°Ρ‡ Π½Π° составлСниС уравнСния ΠΊΠ°ΡΠ°Ρ‚Π΅Π»ΡŒΠ½ΠΎΠΉ.

1. Π”Π°Π½Π° Ρ‚ΠΎΡ‡ΠΊΠ° касания

2. Π”Π°Π½ коэффициСнт Π½Π°ΠΊΠ»ΠΎΠ½Π° ΠΊΠ°ΡΠ°Ρ‚Π΅Π»ΡŒΠ½ΠΎΠΉ, Ρ‚ΠΎ Π΅ΡΡ‚ΡŒ Π·Π½Π°Ρ‡Π΅Π½ΠΈΠ΅ ΠΏΡ€ΠΎΠΈΠ·Π²ΠΎΠ΄Π½ΠΎΠΉ Ρ„ΡƒΠ½ΠΊΡ†ΠΈΠΈ Π² Ρ‚ΠΎΡ‡ΠΊΠ΅ .

3. Π”Π°Π½Ρ‹ ΠΊΠΎΠΎΡ€Π΄ΠΈΠ½Π°Ρ‚Ρ‹ Ρ‚ΠΎΡ‡ΠΊΠΈ, Ρ‡Π΅Ρ€Π΅Π· ΠΊΠΎΡ‚ΠΎΡ€ΡƒΡŽ ΠΏΡ€ΠΎΠ²Π΅Π΄Π΅Π½Π° ΠΊΠ°ΡΠ°Ρ‚Π΅Π»ΡŒΠ½Π°Ρ, Π½ΠΎ которая Π½Π΅ являСтся Ρ‚ΠΎΡ‡ΠΊΠΎΠΉ касания.

Рассмотрим ΠΊΠ°ΠΆΠ΄Ρ‹ΠΉ Ρ‚ΠΈΠΏ Π·Π°Π΄Π°Ρ‡.

1 . ΠΠ°ΠΏΠΈΡΠ°Ρ‚ΡŒ ΡƒΡ€Π°Π²Π½Π΅Π½ΠΈΠ΅ ΠΊΠ°ΡΠ°Ρ‚Π΅Π»ΡŒΠ½ΠΎΠΉ ΠΊ Π³Ρ€Π°Ρ„ΠΈΠΊΡƒ Ρ„ΡƒΠ½ΠΊΡ†ΠΈΠΈ Π² Ρ‚ΠΎΡ‡ΠΊΠ΅ .

.

Π±) НайдСм Π·Π½Π°Ρ‡Π΅Π½ΠΈΠ΅ ΠΏΡ€ΠΎΠΈΠ·Π²ΠΎΠ΄Π½ΠΎΠΉ Π² Ρ‚ΠΎΡ‡ΠΊΠ΅ . Π‘Π½Π°Ρ‡Π°Π»Π° Π½Π°ΠΉΠ΄Π΅ΠΌ ΠΏΡ€ΠΎΠΈΠ·Π²ΠΎΠ΄Π½ΡƒΡŽ Ρ„ΡƒΠ½ΠΊΡ†ΠΈΠΈ

ΠŸΠΎΠ΄ΡΡ‚Π°Π²ΠΈΠΌ Π½Π°ΠΉΠ΄Π΅Π½Π½Ρ‹Π΅ значСния Π² ΡƒΡ€Π°Π²Π½Π΅Π½ΠΈΠ΅ ΠΊΠ°ΡΠ°Ρ‚Π΅Π»ΡŒΠ½ΠΎΠΉ:

РаскроСм скобки Π² ΠΏΡ€Π°Π²ΠΎΠΉ части уравнСния. ΠŸΠΎΠ»ΡƒΡ‡ΠΈΠΌ:

ΠžΡ‚Π²Π΅Ρ‚: .

2 . Найти абсциссы Ρ‚ΠΎΡ‡Π΅ΠΊ, Π² ΠΊΠΎΡ‚ΠΎΡ€Ρ‹Ρ… ΠΊΠ°ΡΠ°Ρ‚Π΅Π»ΡŒΠ½Ρ‹Π΅ ΠΊ Π³Ρ€Π°Ρ„ΠΈΠΊΡƒ Ρ„ΡƒΠ½ΠΊΡ†ΠΈΠΈ ΠΏΠ°Ρ€Π°Π»Π»Π΅Π»ΡŒΠ½Ρ‹ оси абсцисс.

Если ΠΊΠ°ΡΠ°Ρ‚Π΅Π»ΡŒΠ½Π°Ρ ΠΏΠ°Ρ€Π°Π»Π»Π΅Π»ΡŒΠ½Π° оси абсцисс, ΡΠ»Π΅Π΄ΠΎΠ²Π°Ρ‚Π΅Π»ΡŒΠ½ΠΎ ΡƒΠ³ΠΎΠ» ΠΌΠ΅ΠΆΠ΄Ρƒ ΠΊΠ°ΡΠ°Ρ‚Π΅Π»ΡŒΠ½ΠΎΠΉ ΠΈ ΠΏΠΎΠ»ΠΎΠΆΠΈΡ‚Π΅Π»ΡŒΠ½Ρ‹ΠΌ Π½Π°ΠΏΡ€Π°Π²Π»Π΅Π½ΠΈΠ΅ΠΌ оси Ρ€Π°Π²Π΅Π½ Π½ΡƒΠ»ΡŽ, ΡΠ»Π΅Π΄ΠΎΠ²Π°Ρ‚Π΅Π»ΡŒΠ½ΠΎ тангСнс ΡƒΠ³Π»Π° Π½Π°ΠΊΠ»ΠΎΠ½Π° ΠΊΠ°ΡΠ°Ρ‚Π΅Π»ΡŒΠ½ΠΎΠΉ Ρ€Π°Π²Π΅Π½ Π½ΡƒΠ»ΡŽ. Π—Π½Π°Ρ‡ΠΈΡ‚, Π·Π½Π°Ρ‡Π΅Π½ΠΈΠ΅ ΠΏΡ€ΠΎΠΈΠ·Π²ΠΎΠ΄Π½ΠΎΠΉ Ρ„ΡƒΠ½ΠΊΡ†ΠΈΠΈ Π² Ρ‚ΠΎΡ‡ΠΊΠ°Ρ… касания Ρ€Π°Π²Π½ΠΎ Π½ΡƒΠ»ΡŽ.

Π°) НайдСм ΠΏΡ€ΠΎΠΈΠ·Π²ΠΎΠ΄Π½ΡƒΡŽ Ρ„ΡƒΠ½ΠΊΡ†ΠΈΠΈ .

Π±) ΠŸΡ€ΠΈΡ€Π°Π²Π½ΡΠ΅ΠΌ ΠΏΡ€ΠΎΠΈΠ·Π²ΠΎΠ΄Π½ΡƒΡŽ ΠΊ Π½ΡƒΠ»ΡŽ ΠΈ Π½Π°ΠΉΠ΄Π΅ΠΌ значСния , Π² ΠΊΠΎΡ‚ΠΎΡ€Ρ‹Ρ… ΠΊΠ°ΡΠ°Ρ‚Π΅Π»ΡŒΠ½Π°Ρ ΠΏΠ°Ρ€Π°Π»Π»Π΅Π»ΡŒΠ½Π° оси :

ΠŸΡ€ΠΈΡ€Π°Π²Π½ΡΠ΅ΠΌ ΠΊΠ°ΠΆΠ΄Ρ‹ΠΉ ΠΌΠ½ΠΎΠΆΠΈΡ‚Π΅Π»ΡŒ ΠΊ Π½ΡƒΠ»ΡŽ, ΠΏΠΎΠ»ΡƒΡ‡ΠΈΠΌ:

ΠžΡ‚Π²Π΅Ρ‚: 0;3;5

3 . ΠΠ°ΠΏΠΈΡΠ°Ρ‚ΡŒ уравнСния ΠΊΠ°ΡΠ°Ρ‚Π΅Π»ΡŒΠ½Ρ‹Ρ… ΠΊ Π³Ρ€Π°Ρ„ΠΈΠΊΡƒ Ρ„ΡƒΠ½ΠΊΡ†ΠΈΠΈ , ΠΏΠ°Ρ€Π°Π»Π»Π΅Π»ΡŒΠ½Ρ‹Ρ… прямой .

ΠšΠ°ΡΠ°Ρ‚Π΅Π»ΡŒΠ½Π°Ρ ΠΏΠ°Ρ€Π°Π»Π»Π΅Π»ΡŒΠ½Π° прямой . ΠšΠΎΡΡ„Ρ„ΠΈΡ†ΠΈΠ΅Π½Ρ‚ Π½Π°ΠΊΠ»ΠΎΠ½Π° этой прямой Ρ€Π°Π²Π΅Π½ -1. Π’Π°ΠΊ ΠΊΠ°ΠΊ ΠΊΠ°ΡΠ°Ρ‚Π΅Π»ΡŒΠ½Π°Ρ ΠΏΠ°Ρ€Π°Π»Π»Π΅Π»ΡŒΠ½Π° этой прямой, ΡΠ»Π΅Π΄ΠΎΠ²Π°Ρ‚Π΅Π»ΡŒΠ½ΠΎ, коэффициСнт Π½Π°ΠΊΠ»ΠΎΠ½Π° ΠΊΠ°ΡΠ°Ρ‚Π΅Π»ΡŒΠ½ΠΎΠΉ Ρ‚ΠΎΠΆΠ΅ Ρ€Π°Π²Π΅Π½ -1. Π’ΠΎ Π΅ΡΡ‚ΡŒ ΠΌΡ‹ Π·Π½Π°Π΅ΠΌ коэффициСнт Π½Π°ΠΊΠ»ΠΎΠ½Π° ΠΊΠ°ΡΠ°Ρ‚Π΅Π»ΡŒΠ½ΠΎΠΉ , Π°, Ρ‚Π΅ΠΌ самым, Π·Π½Π°Ρ‡Π΅Π½ΠΈΠ΅ ΠΏΡ€ΠΎΠΈΠ·Π²ΠΎΠ΄Π½ΠΎΠΉ Π² Ρ‚ΠΎΡ‡ΠΊΠ΅ касания . 2}»>. ΠœΡ‹ ΠΏΠΎΠ»ΡƒΡ‡ΠΈΠ»ΠΈ ΠΏΠΎΠ΄ ΠΊΠΎΡ€Π½Π΅ΠΌ ΠΎΡ‚Ρ€ΠΈΡ†Π°Ρ‚Π΅Π»ΡŒΠ½ΠΎΠ΅ число, равСнство Π½Π΅ Π²Π΅Ρ€Π½ΠΎ, ΠΈ Ρ‚ΠΎΡ‡ΠΊΠ° Π½Π΅ ΠΏΡ€ΠΈΠ½Π°Π΄Π»Π΅ΠΆΠΈΡ‚ Π³Ρ€Π°Ρ„ΠΈΠΊΡƒ Ρ„ΡƒΠ½ΠΊΡ†ΠΈΠΈ ΠΈ Π½Π΅ являСтся Ρ‚ΠΎΡ‡ΠΊΠΎΠΉ касания.

Π­Ρ‚ΠΎ послСдний Ρ‚ΠΈΠΏ Π·Π°Π΄Π°Ρ‡ Π½Π° Π½Π°Ρ…ΠΎΠΆΠ΄Π΅Π½ΠΈΠ΅ уравнСния ΠΊΠ°ΡΠ°Ρ‚Π΅Π»ΡŒΠ½ΠΎΠΉ. ΠŸΠ΅Ρ€Π²Ρ‹ΠΌ Π΄Π΅Π»ΠΎΠΌ Π½Π°ΠΌ Π½ΡƒΠΆΠ½ΠΎ Π½Π°ΠΉΡ‚ΠΈ абсциссу Ρ‚ΠΎΡ‡ΠΊΠΈ касания .

НайдСм Π·Π½Π°Ρ‡Π΅Π½ΠΈΠ΅ .

ΠŸΡƒΡΡ‚ΡŒ — Ρ‚ΠΎΡ‡ΠΊΠ° касания. Π’ΠΎΡ‡ΠΊΠ° ΠΏΡ€ΠΈΠ½Π°Π΄Π»Π΅ΠΆΠΈΡ‚ ΠΊΠ°ΡΠ°Ρ‚Π΅Π»ΡŒΠ½ΠΎΠΉ ΠΊ Π³Ρ€Π°Ρ„ΠΈΠΊΡƒ Ρ„ΡƒΠ½ΠΊΡ†ΠΈΠΈ . Если ΠΌΡ‹ подставим ΠΊΠΎΠΎΡ€Π΄ΠΈΠ½Π°Ρ‚Ρ‹ этой Ρ‚ΠΎΡ‡ΠΊΠΈ Π² ΡƒΡ€Π°Π²Π½Π΅Π½ΠΈΠ΅ ΠΊΠ°ΡΠ°Ρ‚Π΅Π»ΡŒΠ½ΠΎΠΉ, Ρ‚ΠΎ ΠΏΠΎΠ»ΡƒΡ‡ΠΈΠΌ Π²Π΅Ρ€Π½ΠΎΠ΅ равСнство:

.

Π—Π½Π°Ρ‡Π΅Π½ΠΈΠ΅ Ρ„ΡƒΠ½ΠΊΡ†ΠΈΠΈ Π² Ρ‚ΠΎΡ‡ΠΊΠ΅ Ρ€Π°Π²Π½ΠΎ .

НайдСм Π·Π½Π°Ρ‡Π΅Π½ΠΈΠ΅ ΠΏΡ€ΠΎΠΈΠ·Π²ΠΎΠ΄Π½ΠΎΠΉ Ρ„ΡƒΠ½ΠΊΡ†ΠΈΠΈ Π² Ρ‚ΠΎΡ‡ΠΊΠ΅ .

Π‘Π½Π°Ρ‡Π°Π»Π° Π½Π°ΠΉΠ΄Π΅ΠΌ ΠΏΡ€ΠΎΠΈΠ·Π²ΠΎΠ΄Π½ΡƒΡŽ Ρ„ΡƒΠ½ΠΊΡ†ΠΈΠΈ . Π­Ρ‚ΠΎ .

ΠŸΡ€ΠΎΠΈΠ·Π²ΠΎΠ΄Π½Π°Ρ Π² Ρ‚ΠΎΡ‡ΠΊΠ΅ Ρ€Π°Π²Π½Π° .

ΠŸΠΎΠ΄ΡΡ‚Π°Π²ΠΈΠΌ выраТСния для ΠΈ Π² ΡƒΡ€Π°Π²Π½Π΅Π½ΠΈΠ΅ ΠΊΠ°ΡΠ°Ρ‚Π΅Π»ΡŒΠ½ΠΎΠΉ. ΠŸΠΎΠ»ΡƒΡ‡ΠΈΠΌ ΡƒΡ€Π°Π²Π½Π΅Π½ΠΈΠ΅ ΠΎΡ‚Π½ΠΎΡΠΈΡ‚Π΅Π»ΡŒΠ½ΠΎ :

РСшим это ΡƒΡ€Π°Π²Π½Π΅Π½ΠΈΠ΅.

Π‘ΠΎΠΊΡ€Π°Ρ‚ΠΈΠΌ Ρ‡ΠΈΡΠ»ΠΈΡ‚Π΅Π»ΡŒ ΠΈ Π·Π½Π°ΠΌΠ΅Π½Π°Ρ‚Π΅Π»ΡŒ Π΄Ρ€ΠΎΠ±ΠΈ Π½Π° 2:

ΠŸΡ€ΠΈΠ²Π΅Π΄Π΅ΠΌ ΠΏΡ€Π°Π²ΡƒΡŽ Ρ‡Π°ΡΡ‚ΡŒ уравнСния ΠΊ ΠΎΠ±Ρ‰Π΅ΠΌΡƒ Π·Π½Π°ΠΌΠ΅Π½Π°Ρ‚Π΅Π»ΡŽ. ΠŸΠΎΠ»ΡƒΡ‡ΠΈΠΌ:

Упростим Ρ‡ΠΈΡΠ»ΠΈΡ‚Π΅Π»ΡŒ Π΄Ρ€ΠΎΠ±ΠΈ ΠΈ ΡƒΠΌΠ½ΠΎΠΆΠΈΠΌ ΠΎΠ±Π΅ части Π½Π° — это Π²Ρ‹Ρ€Π°ΠΆΠ΅Π½ΠΈΠ΅ строго большС нуля. 2} {8-3x_0>=0} }}{ }»>

РСшим ΠΏΠ΅Ρ€Π²ΠΎΠ΅ ΡƒΡ€Π°Π²Π½Π΅Π½ΠΈΠ΅.

РСшим ΠΊΠ²Π°Π΄Ρ€Π°Ρ‚Π½ΠΎΠ΅ ΡƒΡ€Π°Π²Π½Π΅Π½ΠΈΠ΅, ΠΏΠΎΠ»ΡƒΡ‡ΠΈΠΌ

Π’Ρ‚ΠΎΡ€ΠΎΠΉ ΠΊΠΎΡ€Π΅Π½ΡŒ Π½Π΅ удовлСтворяСт ΡƒΡΠ»ΠΎΠ²ΠΈΡŽ title=»8-3x_0>=0″>, ΡΠ»Π΅Π΄ΠΎΠ²Π°Ρ‚Π΅Π»ΡŒΠ½ΠΎ, Ρƒ нас Ρ‚ΠΎΠ»ΡŒΠΊΠΎ ΠΎΠ΄Π½Π° Ρ‚ΠΎΡ‡ΠΊΠ° касания ΠΈ Π΅Ρ‘ абсцисса Ρ€Π°Π²Π½Π° .

НапишСм ΡƒΡ€Π°Π²Π½Π΅Π½ΠΈΠ΅ ΠΊΠ°ΡΠ°Ρ‚Π΅Π»ΡŒΠ½ΠΎΠΉ ΠΊ ΠΊΡ€ΠΈΠ²ΠΎΠΉ Π² Ρ‚ΠΎΡ‡ΠΊΠ΅ . Для этого подставим Π·Π½Π°Ρ‡Π΅Π½ΠΈΠ΅ Π² ΡƒΡ€Π°Π²Π½Π΅Π½ΠΈΠ΅ — ΠΌΡ‹ Π΅Π³ΠΎ ΡƒΠΆΠ΅ записывали.

ΠžΡ‚Π²Π΅Ρ‚:
.

Π£Ρ€Π°Π²Π½Π΅Π½ΠΈΠ΅ ΠΊΠ°ΡΠ°Ρ‚Π΅Π»ΡŒΠ½ΠΎΠΉ

Π’ этой ΡΡ‚Π°Ρ‚ΡŒΠ΅ ΠΌΡ‹ Ρ€Π°Π·Π±Π΅Ρ€Π΅ΠΌ всС Ρ‚ΠΈΠΏΡ‹ Π·Π°Π΄Π°Ρ‡ Π½Π° Π½Π°Ρ…ΠΎΠΆΠ΄Π΅Π½ΠΈΠ΅ уравнСния ΠΊΠ°ΡΠ°Ρ‚Π΅Π»ΡŒΠ½ΠΎΠΉ.

Вспомним гСомСтричСский смысл ΠΏΡ€ΠΎΠΈΠ·Π²ΠΎΠ΄Π½ΠΎΠΉ: Ссли ΠΊ Π³Ρ€Π°Ρ„ΠΈΠΊΡƒ Ρ„ΡƒΠ½ΠΊΡ†ΠΈΠΈ Π² Ρ‚ΠΎΡ‡ΠΊΠ΅ ΠΏΡ€ΠΎΠ²Π΅Π΄Π΅Π½Π° ΠΊΠ°ΡΠ°Ρ‚Π΅Π»ΡŒΠ½Π°Ρ, Ρ‚ΠΎ коэффициСнт Π½Π°ΠΊΠ»ΠΎΠ½Π° ΠΊΠ°ΡΠ°Ρ‚Π΅Π»ΡŒΠ½ΠΎΠΉ (Ρ€Π°Π²Π½Ρ‹ΠΉ тангСнсу ΡƒΠ³Π»Π° ΠΌΠ΅ΠΆΠ΄Ρƒ ΠΊΠ°ΡΠ°Ρ‚Π΅Π»ΡŒΠ½ΠΎΠΉ ΠΈ ΠΏΠΎΠ»ΠΎΠΆΠΈΡ‚Π΅Π»ΡŒΠ½Ρ‹ΠΌ Π½Π°ΠΏΡ€Π°Π²Π»Π΅Π½ΠΈΠ΅ΠΌ оси ) Ρ€Π°Π²Π΅Π½ ΠΏΡ€ΠΎΠΈΠ·Π²ΠΎΠ΄Π½ΠΎΠΉ Ρ„ΡƒΠ½ΠΊΡ†ΠΈΠΈ Π² Ρ‚ΠΎΡ‡ΠΊΠ΅ .

Π’ΠΎΠ·ΡŒΠΌΠ΅ΠΌ Π½Π° ΠΊΠ°ΡΠ°Ρ‚Π΅Π»ΡŒΠ½ΠΎΠΉ ΠΏΡ€ΠΎΠΈΠ·Π²ΠΎΠ»ΡŒΠ½ΡƒΡŽ Ρ‚ΠΎΡ‡ΠΊΡƒΒ  с ΠΊΠΎΠΎΡ€Π΄ΠΈΠ½Π°Ρ‚Π°ΠΌΠΈ :

И рассмотрим ΠΏΡ€ΡΠΌΠΎΡƒΠ³ΠΎΠ»ΡŒΠ½Ρ‹ΠΉ Ρ‚Ρ€Π΅ΡƒΠ³ΠΎΠ»ΡŒΠ½ΠΈΠΊ :

Π’ этом Ρ‚Ρ€Π΅ΡƒΠ³ΠΎΠ»ΡŒΠ½ΠΈΠΊΠ΅

ΠžΡ‚ΡΡŽΠ΄Π°

Или

Π­Ρ‚ΠΎ ΠΈ Π΅ΡΡ‚ΡŒ ΡƒΡ€Π°Π²Π½Π΅Π½ΠΈΠ΅ ΠΊΠ°ΡΠ°Ρ‚Π΅Π»ΡŒΠ½ΠΎΠΉ, ΠΏΡ€ΠΎΠ²Π΅Π΄Π΅Π½Π½ΠΎΠΉ ΠΊ Π³Ρ€Π°Ρ„ΠΈΠΊΡƒ Ρ„ΡƒΠ½ΠΊΡ†ΠΈΠΈ Π² Ρ‚ΠΎΡ‡ΠΊΠ΅Β .

Π§Ρ‚ΠΎΠ±Ρ‹ Π½Π°ΠΏΠΈΡΠ°Ρ‚ΡŒ ΡƒΡ€Π°Π²Π½Π΅Π½ΠΈΠ΅ ΠΊΠ°ΡΠ°Ρ‚Π΅Π»ΡŒΠ½ΠΎΠΉ, Π½Π°ΠΌ достаточно Π·Π½Π°Ρ‚ΡŒ ΡƒΡ€Π°Π²Π½Π΅Π½ΠΈΠ΅ Ρ„ΡƒΠ½ΠΊΡ†ΠΈΠΈ ΠΈ Ρ‚ΠΎΡ‡ΠΊΡƒ, Π² ΠΊΠΎΡ‚ΠΎΡ€ΠΎΠΉ ΠΏΡ€ΠΎΠ²Π΅Π΄Π΅Π½Π° ΠΊΠ°ΡΠ°Ρ‚Π΅Π»ΡŒΠ½Π°Ρ. Π’ΠΎΠ³Π΄Π° ΠΌΡ‹ смоТСм Π½Π°ΠΉΡ‚ΠΈ ΠΈ .

Π•ΡΡ‚ΡŒ Ρ‚Ρ€ΠΈ основных Ρ‚ΠΈΠΏΠ° Π·Π°Π΄Π°Ρ‡ Π½Π° составлСниС уравнСния ΠΊΠ°ΡΠ°Ρ‚Π΅Π»ΡŒΠ½ΠΎΠΉ.

1. Π”Π°Π½Π° Ρ‚ΠΎΡ‡ΠΊΠ° касания 

2. Π”Π°Π½ коэффициСнт Π½Π°ΠΊΠ»ΠΎΠ½Π° ΠΊΠ°ΡΠ°Ρ‚Π΅Π»ΡŒΠ½ΠΎΠΉ, Ρ‚ΠΎ Π΅ΡΡ‚ΡŒ Π·Π½Π°Ρ‡Π΅Π½ΠΈΠ΅ ΠΏΡ€ΠΎΠΈΠ·Π²ΠΎΠ΄Π½ΠΎΠΉ Ρ„ΡƒΠ½ΠΊΡ†ΠΈΠΈ Π² Ρ‚ΠΎΡ‡ΠΊΠ΅Β .

3. Π”Π°Π½Ρ‹ ΠΊΠΎΠΎΡ€Π΄ΠΈΠ½Π°Ρ‚Ρ‹ Ρ‚ΠΎΡ‡ΠΊΠΈ, Ρ‡Π΅Ρ€Π΅Π· ΠΊΠΎΡ‚ΠΎΡ€ΡƒΡŽ ΠΏΡ€ΠΎΠ²Π΅Π΄Π΅Π½Π° ΠΊΠ°ΡΠ°Ρ‚Π΅Π»ΡŒΠ½Π°Ρ, Π½ΠΎ которая Π½Π΅ являСтся Ρ‚ΠΎΡ‡ΠΊΠΎΠΉ касания.

Рассмотрим ΠΊΠ°ΠΆΠ΄Ρ‹ΠΉ Ρ‚ΠΈΠΏ Π·Π°Π΄Π°Ρ‡.

1. ΠΠ°ΠΏΠΈΡΠ°Ρ‚ΡŒ ΡƒΡ€Π°Π²Π½Π΅Π½ΠΈΠ΅ ΠΊΠ°ΡΠ°Ρ‚Π΅Π»ΡŒΠ½ΠΎΠΉ ΠΊ Π³Ρ€Π°Ρ„ΠΈΠΊΡƒ Ρ„ΡƒΠ½ΠΊΡ†ΠΈΠΈ Β  Π² Ρ‚ΠΎΡ‡ΠΊΠ΅ .

Π°) НайдСм Π·Π½Π°Ρ‡Π΅Π½ΠΈΠ΅ Ρ„ΡƒΠ½ΠΊΡ†ΠΈΠΈ Π² Ρ‚ΠΎΡ‡ΠΊΠ΅ .

.

Π±) НайдСм Π·Π½Π°Ρ‡Π΅Π½ΠΈΠ΅ ΠΏΡ€ΠΎΠΈΠ·Π²ΠΎΠ΄Π½ΠΎΠΉ Π² Ρ‚ΠΎΡ‡ΠΊΠ΅ . Π‘Π½Π°Ρ‡Π°Π»Π° Π½Π°ΠΉΠ΄Π΅ΠΌ ΠΏΡ€ΠΎΠΈΠ·Π²ΠΎΠ΄Π½ΡƒΡŽ Ρ„ΡƒΠ½ΠΊΡ†ΠΈΠΈ

ΠŸΠΎΠ΄ΡΡ‚Π°Π²ΠΈΠΌ Π½Π°ΠΉΠ΄Π΅Π½Π½Ρ‹Π΅ значСния Π² ΡƒΡ€Π°Π²Π½Π΅Π½ΠΈΠ΅ ΠΊΠ°ΡΠ°Ρ‚Π΅Π»ΡŒΠ½ΠΎΠΉ:

РаскроСм скобки Π² ΠΏΡ€Π°Π²ΠΎΠΉ части уравнСния. ΠŸΠΎΠ»ΡƒΡ‡ΠΈΠΌ:

ΠžΡ‚Π²Π΅Ρ‚: .

Β 

2. Найти абсциссы Ρ‚ΠΎΡ‡Π΅ΠΊ, Π² ΠΊΠΎΡ‚ΠΎΡ€Ρ‹Ρ… ΠΊΠ°ΡΠ°Ρ‚Π΅Π»ΡŒΠ½Ρ‹Π΅ ΠΊ Π³Ρ€Π°Ρ„ΠΈΠΊΡƒ Ρ„ΡƒΠ½ΠΊΡ†ΠΈΠΈ ΠΏΠ°Ρ€Π°Π»Π»Π΅Π»ΡŒΠ½Ρ‹ оси абсцисс.

Если ΠΊΠ°ΡΠ°Ρ‚Π΅Π»ΡŒΠ½Π°Ρ ΠΏΠ°Ρ€Π°Π»Π»Π΅Π»ΡŒΠ½Π° оси абсцисс, ΡΠ»Π΅Π΄ΠΎΠ²Π°Ρ‚Π΅Π»ΡŒΠ½ΠΎ ΡƒΠ³ΠΎΠ» ΠΌΠ΅ΠΆΠ΄Ρƒ ΠΊΠ°ΡΠ°Ρ‚Π΅Π»ΡŒΠ½ΠΎΠΉ ΠΈ ΠΏΠΎΠ»ΠΎΠΆΠΈΡ‚Π΅Π»ΡŒΠ½Ρ‹ΠΌ Π½Π°ΠΏΡ€Π°Π²Π»Π΅Π½ΠΈΠ΅ΠΌ оси Ρ€Π°Π²Π΅Π½ Π½ΡƒΠ»ΡŽ, ΡΠ»Π΅Π΄ΠΎΠ²Π°Ρ‚Π΅Π»ΡŒΠ½ΠΎ тангСнс ΡƒΠ³Π»Π° Π½Π°ΠΊΠ»ΠΎΠ½Π° ΠΊΠ°ΡΠ°Ρ‚Π΅Π»ΡŒΠ½ΠΎΠΉ Ρ€Π°Π²Π΅Π½ Π½ΡƒΠ»ΡŽ. Π—Π½Π°Ρ‡ΠΈΡ‚, Π·Π½Π°Ρ‡Π΅Π½ΠΈΠ΅ ΠΏΡ€ΠΎΠΈΠ·Π²ΠΎΠ΄Π½ΠΎΠΉ Ρ„ΡƒΠ½ΠΊΡ†ΠΈΠΈ Π² Ρ‚ΠΎΡ‡ΠΊΠ°Ρ… касания Ρ€Π°Π²Π½ΠΎ Π½ΡƒΠ»ΡŽ.

Π°) НайдСм ΠΏΡ€ΠΎΠΈΠ·Π²ΠΎΠ΄Π½ΡƒΡŽ Ρ„ΡƒΠ½ΠΊΡ†ΠΈΠΈ .

Π±) ΠŸΡ€ΠΈΡ€Π°Π²Π½ΡΠ΅ΠΌ ΠΏΡ€ΠΎΠΈΠ·Π²ΠΎΠ΄Π½ΡƒΡŽ ΠΊ Π½ΡƒΠ»ΡŽ ΠΈ Π½Π°ΠΉΠ΄Π΅ΠΌ значСния , Π² ΠΊΠΎΡ‚ΠΎΡ€Ρ‹Ρ… ΠΊΠ°ΡΠ°Ρ‚Π΅Π»ΡŒΠ½Π°Ρ ΠΏΠ°Ρ€Π°Π»Π»Π΅Π»ΡŒΠ½Π° оси :

ΠŸΡ€ΠΈΡ€Π°Π²Π½ΡΠ΅ΠΌ ΠΊΠ°ΠΆΠ΄Ρ‹ΠΉ ΠΌΠ½ΠΎΠΆΠΈΡ‚Π΅Π»ΡŒ ΠΊ Π½ΡƒΠ»ΡŽ, ΠΏΠΎΠ»ΡƒΡ‡ΠΈΠΌ:

ΠžΡ‚Π²Π΅Ρ‚: 0;3;5

Β 

3. ΠΠ°ΠΏΠΈΡΠ°Ρ‚ΡŒ уравнСния ΠΊΠ°ΡΠ°Ρ‚Π΅Π»ΡŒΠ½Ρ‹Ρ… ΠΊ Π³Ρ€Π°Ρ„ΠΈΠΊΡƒ Ρ„ΡƒΠ½ΠΊΡ†ΠΈΠΈ , ΠΏΠ°Ρ€Π°Π»Π»Π΅Π»ΡŒΠ½Ρ‹Ρ…Β  прямой .

ΠšΠ°ΡΠ°Ρ‚Π΅Π»ΡŒΠ½Π°Ρ ΠΏΠ°Ρ€Π°Π»Π»Π΅Π»ΡŒΠ½Π° прямой . ΠšΠΎΡΡ„Ρ„ΠΈΡ†ΠΈΠ΅Π½Ρ‚ Π½Π°ΠΊΠ»ΠΎΠ½Π° этой прямой Ρ€Π°Π²Π΅Π½ -1. Π’Π°ΠΊ ΠΊΠ°ΠΊ ΠΊΠ°ΡΠ°Ρ‚Π΅Π»ΡŒΠ½Π°Ρ ΠΏΠ°Ρ€Π°Π»Π»Π΅Π»ΡŒΠ½Π° этой прямой, ΡΠ»Π΅Π΄ΠΎΠ²Π°Ρ‚Π΅Π»ΡŒΠ½ΠΎ, коэффициСнт Π½Π°ΠΊΠ»ΠΎΠ½Π° ΠΊΠ°ΡΠ°Ρ‚Π΅Π»ΡŒΠ½ΠΎΠΉ Ρ‚ΠΎΠΆΠ΅ Ρ€Π°Π²Π΅Π½ -1. Π’ΠΎ Π΅ΡΡ‚ΡŒ ΠΌΡ‹ Π·Π½Π°Π΅ΠΌ коэффициСнт Π½Π°ΠΊΠ»ΠΎΠ½Π° ΠΊΠ°ΡΠ°Ρ‚Π΅Π»ΡŒΠ½ΠΎΠΉ, Π°, Ρ‚Π΅ΠΌ самым, Π·Π½Π°Ρ‡Π΅Π½ΠΈΠ΅ ΠΏΡ€ΠΎΠΈΠ·Π²ΠΎΠ΄Π½ΠΎΠΉ Π² Ρ‚ΠΎΡ‡ΠΊΠ΅ касания.

Π­Ρ‚ΠΎ Π²Ρ‚ΠΎΡ€ΠΎΠΉ Ρ‚ΠΈΠΏ Π·Π°Π΄Π°Ρ‡ Π½Π° Π½Π°Ρ…ΠΎΠΆΠ΄Π΅Π½ΠΈΠ΅ уравнСния ΠΊΠ°ΡΠ°Ρ‚Π΅Π»ΡŒΠ½ΠΎΠΉ.

Π˜Ρ‚Π°ΠΊ, Ρƒ нас Π΄Π°Π½Π° функция ΠΈ Π·Π½Π°Ρ‡Π΅Π½ΠΈΠ΅ ΠΏΡ€ΠΎΠΈΠ·Π²ΠΎΠ΄Π½ΠΎΠΉ Π² Ρ‚ΠΎΡ‡ΠΊΠ΅ касания.

Π°) НайдСм Ρ‚ΠΎΡ‡ΠΊΠΈ, Π² ΠΊΠΎΡ‚ΠΎΡ€Ρ‹Ρ… производная Ρ„ΡƒΠ½ΠΊΡ†ΠΈΠΈ Ρ€Π°Π²Π½Π° -1.

Π‘Π½Π°Ρ‡Π°Π»Π° Π½Π°ΠΉΠ΄Π΅ΠΌ ΡƒΡ€Π°Π²Π½Π΅Π½ΠΈΠ΅ ΠΏΡ€ΠΎΠΈΠ·Π²ΠΎΠ΄Π½ΠΎΠΉ.

Нам Π½ΡƒΠΆΠ½ΠΎ Π½Π°ΠΉΡ‚ΠΈ ΠΏΡ€ΠΎΠΈΠ·Π²ΠΎΠ΄Π½ΡƒΡŽ Π΄Ρ€ΠΎΠ±ΠΈ.

ΠŸΡ€ΠΈΡ€Π°Π²Π½ΡΠ΅ΠΌ ΠΏΡ€ΠΎΠΈΠ·Π²ΠΎΠ΄Π½ΡƒΡŽ ΠΊ числу -1.

ΠΈΠ»ΠΈ

ΠΈΠ»ΠΈ

Π±) НайдСм ΡƒΡ€Π°Π²Π½Π΅Π½ΠΈΠ΅ ΠΊΠ°ΡΠ°Ρ‚Π΅Π»ΡŒΠ½ΠΎΠΉ ΠΊ Π³Ρ€Π°Ρ„ΠΈΠΊΡƒ Ρ„ΡƒΠ½ΠΊΡ†ΠΈΠΈ Π² Ρ‚ΠΎΡ‡ΠΊΠ΅ .

НайдСм Π·Π½Π°Ρ‡Π΅Π½ΠΈΠ΅ Ρ„ΡƒΠ½ΠΊΡ†ΠΈΠΈ Π² Ρ‚ΠΎΡ‡ΠΊΠ΅ .

(ΠΏΠΎ ΡƒΡΠ»ΠΎΠ²ΠΈΡŽ)

ΠŸΠΎΠ΄ΡΡ‚Π°Π²ΠΈΠΌ эти значСния Π² ΡƒΡ€Π°Π²Π½Π΅Π½ΠΈΠ΅ ΠΊΠ°ΡΠ°Ρ‚Π΅Π»ΡŒΠ½ΠΎΠΉ:

.

Π±) НайдСм ΡƒΡ€Π°Π²Π½Π΅Π½ΠΈΠ΅ ΠΊΠ°ΡΠ°Ρ‚Π΅Π»ΡŒΠ½ΠΎΠΉ ΠΊ Π³Ρ€Π°Ρ„ΠΈΠΊΡƒ Ρ„ΡƒΠ½ΠΊΡ†ΠΈΠΈ Π² Ρ‚ΠΎΡ‡ΠΊΠ΅ .

НайдСм Π·Π½Π°Ρ‡Π΅Π½ΠΈΠ΅ Ρ„ΡƒΠ½ΠΊΡ†ΠΈΠΈ Π² Ρ‚ΠΎΡ‡ΠΊΠ΅ .

(ΠΏΠΎ ΡƒΡΠ»ΠΎΠ²ΠΈΡŽ).

ΠŸΠΎΠ΄ΡΡ‚Π°Π²ΠΈΠΌ эти значСния Π² ΡƒΡ€Π°Π²Π½Π΅Π½ΠΈΠ΅ ΠΊΠ°ΡΠ°Ρ‚Π΅Π»ΡŒΠ½ΠΎΠΉ:

.

ΠžΡ‚Π²Π΅Ρ‚:

Β 

4. ΠΠ°ΠΏΠΈΡΠ°Ρ‚ΡŒ ΡƒΡ€Π°Π²Π½Π΅Π½ΠΈΠ΅ ΠΊΠ°ΡΠ°Ρ‚Π΅Π»ΡŒΠ½ΠΎΠΉ ΠΊ ΠΊΡ€ΠΈΠ²ΠΎΠΉ , проходящСй Ρ‡Π΅Ρ€Π΅Π· Ρ‚ΠΎΡ‡ΠΊΡƒ

Π‘Π½Π°Ρ‡Π°Π»Π° ΠΏΡ€ΠΎΠ²Π΅Ρ€ΠΈΠΌ, Π½Π΅ являСтся Π»ΠΈ Ρ‚ΠΎΡ‡ΠΊΠ° Ρ‚ΠΎΡ‡ΠΊΠΎΠΉ касания. Если Ρ‚ΠΎΡ‡ΠΊΠ° являСтся Ρ‚ΠΎΡ‡ΠΊΠΎΠΉ касания, Ρ‚ΠΎ ΠΎΠ½Π° ΠΏΡ€ΠΈΠ½Π°Π΄Π»Π΅ΠΆΠΈΡ‚ Π³Ρ€Π°Ρ„ΠΈΠΊΡƒ Ρ„ΡƒΠ½ΠΊΡ†ΠΈΠΈ, ΠΈ Π΅Ρ‘ ΠΊΠΎΠΎΡ€Π΄ΠΈΠ½Π°Ρ‚Ρ‹ Π΄ΠΎΠ»ΠΆΠ½Ρ‹ ΡƒΠ΄ΠΎΠ²Π»Π΅Ρ‚Π²ΠΎΡ€ΡΡ‚ΡŒ ΡƒΡ€Π°Π²Π½Π΅Π½ΠΈΡŽ Ρ„ΡƒΠ½ΠΊΡ†ΠΈΠΈ. ΠŸΠΎΠ΄ΡΡ‚Π°Π²ΠΈΠΌ ΠΊΠΎΠΎΡ€Π΄ΠΈΠ½Π°Ρ‚Ρ‹Β  Ρ‚ΠΎΡ‡ΠΊΠΈ Β  Π² ΡƒΡ€Π°Π²Π½Π΅Π½ΠΈΠ΅ Ρ„ΡƒΠ½ΠΊΡ†ΠΈΠΈ.

. ΠœΡ‹ ΠΏΠΎΠ»ΡƒΡ‡ΠΈΠ»ΠΈ ΠΏΠΎΠ΄ ΠΊΠΎΡ€Π½Π΅ΠΌ ΠΎΡ‚Ρ€ΠΈΡ†Π°Ρ‚Π΅Π»ΡŒΠ½ΠΎΠ΅ число, равСнство Π½Π΅ Π²Π΅Ρ€Π½ΠΎ, ΠΈ Ρ‚ΠΎΡ‡ΠΊΠ° Π½Π΅ ΠΏΡ€ΠΈΠ½Π°Π΄Π»Π΅ΠΆΠΈΡ‚ Π³Ρ€Π°Ρ„ΠΈΠΊΡƒ Ρ„ΡƒΠ½ΠΊΡ†ΠΈΠΈ ΠΈ Π½Π΅ являСтся Ρ‚ΠΎΡ‡ΠΊΠΎΠΉ касания.

Π­Ρ‚ΠΎ послСдний Ρ‚ΠΈΠΏ Π·Π°Π΄Π°Ρ‡ Π½Π° Π½Π°Ρ…ΠΎΠΆΠ΄Π΅Π½ΠΈΠ΅ уравнСния ΠΊΠ°ΡΠ°Ρ‚Π΅Π»ΡŒΠ½ΠΎΠΉ. ΠŸΠ΅Ρ€Π²Ρ‹ΠΌ Π΄Π΅Π»ΠΎΠΌ Π½Π°ΠΌ Π½ΡƒΠΆΠ½ΠΎ Π½Π°ΠΉΡ‚ΠΈ абсциссу Ρ‚ΠΎΡ‡ΠΊΠΈ касания.

НайдСм Π·Π½Π°Ρ‡Π΅Π½ΠΈΠ΅ .

ΠŸΡƒΡΡ‚ΡŒ — Ρ‚ΠΎΡ‡ΠΊΠ° касания. Π’ΠΎΡ‡ΠΊΠ°Β  ΠΏΡ€ΠΈΠ½Π°Π΄Π»Π΅ΠΆΠΈΡ‚ ΠΊΠ°ΡΠ°Ρ‚Π΅Π»ΡŒΠ½ΠΎΠΉ ΠΊ Π³Ρ€Π°Ρ„ΠΈΠΊΡƒ Ρ„ΡƒΠ½ΠΊΡ†ΠΈΠΈ . Если ΠΌΡ‹ подставим ΠΊΠΎΠΎΡ€Π΄ΠΈΠ½Π°Ρ‚Ρ‹ этой Ρ‚ΠΎΡ‡ΠΊΠΈ Π² ΡƒΡ€Π°Π²Π½Π΅Π½ΠΈΠ΅ ΠΊΠ°ΡΠ°Ρ‚Π΅Π»ΡŒΠ½ΠΎΠΉ, Ρ‚ΠΎ ΠΏΠΎΠ»ΡƒΡ‡ΠΈΠΌ Π²Π΅Ρ€Π½ΠΎΠ΅ равСнство:

.

Π—Π½Π°Ρ‡Π΅Π½ΠΈΠ΅ Ρ„ΡƒΠ½ΠΊΡ†ΠΈΠΈ Π² Ρ‚ΠΎΡ‡ΠΊΠ΅ Ρ€Π°Π²Π½ΠΎ .

НайдСм Π·Π½Π°Ρ‡Π΅Π½ΠΈΠ΅ ΠΏΡ€ΠΎΠΈΠ·Π²ΠΎΠ΄Π½ΠΎΠΉ Ρ„ΡƒΠ½ΠΊΡ†ΠΈΠΈ Π² Ρ‚ΠΎΡ‡ΠΊΠ΅ .

Π‘Π½Π°Ρ‡Π°Π»Π° Π½Π°ΠΉΠ΄Π΅ΠΌ ΠΏΡ€ΠΎΠΈΠ·Π²ΠΎΠ΄Π½ΡƒΡŽ Ρ„ΡƒΠ½ΠΊΡ†ΠΈΠΈ . Π­Ρ‚ΠΎ слоТная функция.

ΠŸΡ€ΠΎΠΈΠ·Π²ΠΎΠ΄Π½Π°Ρ Π² Ρ‚ΠΎΡ‡ΠΊΠ΅ Ρ€Π°Π²Π½Π° .

ΠŸΠΎΠ΄ΡΡ‚Π°Π²ΠΈΠΌ выраТСния для ΠΈ Π² ΡƒΡ€Π°Π²Π½Π΅Π½ΠΈΠ΅ ΠΊΠ°ΡΠ°Ρ‚Π΅Π»ΡŒΠ½ΠΎΠΉ. ΠŸΠΎΠ»ΡƒΡ‡ΠΈΠΌ ΡƒΡ€Π°Π²Π½Π΅Π½ΠΈΠ΅ ΠΎΡ‚Π½ΠΎΡΠΈΡ‚Π΅Π»ΡŒΠ½ΠΎ :

РСшим это ΡƒΡ€Π°Π²Π½Π΅Π½ΠΈΠ΅.

Π‘ΠΎΠΊΡ€Π°Ρ‚ΠΈΠΌ Ρ‡ΠΈΡΠ»ΠΈΡ‚Π΅Π»ΡŒ ΠΈ Π·Π½Π°ΠΌΠ΅Π½Π°Ρ‚Π΅Π»ΡŒ Π΄Ρ€ΠΎΠ±ΠΈ Π½Π° 2:

ΠŸΡ€ΠΈΠ²Π΅Π΄Π΅ΠΌ ΠΏΡ€Π°Π²ΡƒΡŽ Ρ‡Π°ΡΡ‚ΡŒ уравнСния ΠΊ ΠΎΠ±Ρ‰Π΅ΠΌΡƒ Π·Π½Π°ΠΌΠ΅Π½Π°Ρ‚Π΅Π»ΡŽ. ΠŸΠΎΠ»ΡƒΡ‡ΠΈΠΌ:

Упростим Ρ‡ΠΈΡΠ»ΠΈΡ‚Π΅Π»ΡŒ Π΄Ρ€ΠΎΠ±ΠΈ ΠΈ ΡƒΠΌΠ½ΠΎΠΆΠΈΠΌ ΠΎΠ±Π΅ части Π½Π° — это Π²Ρ‹Ρ€Π°ΠΆΠ΅Π½ΠΈΠ΅ строго большС нуля.

ΠŸΠΎΠ»ΡƒΡ‡ΠΈΠΌ ΡƒΡ€Π°Π²Π½Π΅Π½ΠΈΠ΅

Π­Ρ‚ΠΎ ΠΈΡ€Ρ€Π°Ρ†ΠΈΠΎΠ½Π°Π»ΡŒΠ½ΠΎΠ΅ ΡƒΡ€Π°Π²Π½Π΅Π½ΠΈΠ΅.

РСшим Π΅Π³ΠΎ. Для этого Π²ΠΎΠ·Π²Π΅Π΄Π΅ΠΌ ΠΎΠ±Π΅ части Π² ΠΊΠ²Π°Π΄Ρ€Π°Ρ‚ ΠΈ ΠΏΠ΅Ρ€Π΅ΠΉΠ΄Π΅ΠΌ ΠΊ систСмС.

РСшим ΠΏΠ΅Ρ€Π²ΠΎΠ΅ ΡƒΡ€Π°Π²Π½Π΅Π½ΠΈΠ΅.

РСшим ΠΊΠ²Π°Π΄Ρ€Π°Ρ‚Π½ΠΎΠ΅ ΡƒΡ€Π°Π²Π½Π΅Π½ΠΈΠ΅, ΠΏΠΎΠ»ΡƒΡ‡ΠΈΠΌ

ΠΈΠ»ΠΈ

Π’Ρ‚ΠΎΡ€ΠΎΠΉ ΠΊΠΎΡ€Π΅Π½ΡŒ Π½Π΅ удовлСтворяСт ΡƒΡΠ»ΠΎΠ²ΠΈΡŽ , ΡΠ»Π΅Π΄ΠΎΠ²Π°Ρ‚Π΅Π»ΡŒΠ½ΠΎ, Ρƒ нас Ρ‚ΠΎΠ»ΡŒΠΊΠΎ ΠΎΠ΄Π½Π° Ρ‚ΠΎΡ‡ΠΊΠ° касания ΠΈ Π΅Ρ‘ абсцисса Ρ€Π°Π²Π½Π° .

НапишСм ΡƒΡ€Π°Π²Π½Π΅Π½ΠΈΠ΅ ΠΊΠ°ΡΠ°Ρ‚Π΅Π»ΡŒΠ½ΠΎΠΉ ΠΊ ΠΊΡ€ΠΈΠ²ΠΎΠΉ Π² Ρ‚ΠΎΡ‡ΠΊΠ΅ . Для этого подставим Π·Π½Π°Ρ‡Π΅Π½ΠΈΠ΅ Π² ΡƒΡ€Π°Π²Π½Π΅Π½ΠΈΠ΅ Β  — ΠΌΡ‹ Π΅Π³ΠΎ ΡƒΠΆΠ΅ записывали.

ΠŸΠΎΠ»ΡƒΡ‡ΠΈΠΌ:

ΠžΡ‚Π²Π΅Ρ‚:
И.Π’. ЀСльдман, Ρ€Π΅ΠΏΠ΅Ρ‚ΠΈΡ‚ΠΎΡ€ ΠΏΠΎ ΠΌΠ°Ρ‚Π΅ΠΌΠ°Ρ‚ΠΈΠΊΠ΅.

ΠšΠ°ΡΠ°Ρ‚Π΅Π»ΡŒΠ½Π°Ρ ΠΊ Π³Ρ€Π°Ρ„ΠΈΠΊΡƒ Ρ„ΡƒΠ½ΠΊΡ†ΠΈΠΈ, ΠΊΠ°ΠΊ ΡΠΎΡΡ‚Π°Π²ΠΈΡ‚ΡŒ ΡƒΡ€Π°Π²Π½Π΅Π½ΠΈΠ΅, свойства, ΡƒΠ³Π»ΠΎΠ²ΠΎΠΉ коэффициСнт ΠΊΠ°ΡΠ°Ρ‚Π΅Π»ΡŒΠ½ΠΎΠΉ ΠΏΡ€ΠΎΠ²Π΅Π΄Π΅Π½Π½ΠΎΠΉ ΠΊ Π³Ρ€Π°Ρ„ΠΈΠΊΡƒ Ρ„ΡƒΠ½ΠΊΡ†ΠΈΠΈ, Ρ„ΠΎΡ€ΠΌΡƒΠ»Π°, ΠΏΡ€ΠΈΠΌΠ΅Ρ€Ρ‹ Ρ€Π΅ΡˆΠ΅Π½ΠΈΡ

На экзамСнах ΠΏΠΎ дисциплинам с Ρ„ΠΈΠ·ΠΈΠΊΠΎ-матСматичСским ΡƒΠΊΠ»ΠΎΠ½ΠΎΠΌ ΠΈΠ»ΠΈ ΠΏΡ€ΠΈ расчСтах встрСчаСтся Ρ‚ΠΈΠΏ Π·Π°Π΄Π°Ρ‡ ΠΎ ΠΊΠ°ΡΠ°Ρ‚Π΅Π»ΡŒΠ½ΠΎΠΉ ΠΊ Π³Ρ€Π°Ρ„ΠΈΠΊΡƒ Ρ„ΡƒΠ½ΠΊΡ†ΠΈΠΈ.

Однако слСдуСт Ρ€Π°Π·ΠΎΠ±Ρ€Π°Ρ‚ΡŒΡΡ Π² основных Ρ‚Π΅Ρ€ΠΌΠΈΠ½Π°Ρ… ΠΈ ΡΠΎΠΎΡ‚Π½ΠΎΡˆΠ΅Π½ΠΈΡΡ….

БпСциалисты Ρ€Π΅ΠΊΠΎΠΌΠ΅Π½Π΄ΡƒΡŽΡ‚ ΠΏΠΎΠ»ΡŒΠ·ΠΎΠ²Π°Ρ‚ΡŒΡΡ ΡΠΏΠ΅Ρ†ΠΈΠ°Π»ΡŒΠ½Ρ‹ΠΌ Π°Π»Π³ΠΎΡ€ΠΈΡ‚ΠΌΠΎΠΌ, ΠΏΠΎΠ·Π²ΠΎΠ»ΡΡŽΡ‰ΠΈΠΌ ΠΏΡ€Π°Π²ΠΈΠ»ΡŒΠ½ΠΎ Π½Π°Ρ…ΠΎΠ΄ΠΈΡ‚ΡŒ Ρ‚ΠΎΡ‡ΠΊΡƒ касания прямой с ΠΊΠ°ΠΊΠΎΠΉ-Π»ΠΈΠ±ΠΎ Ρ„ΠΈΠ³ΡƒΡ€ΠΎΠΉ.

  • ΠžΠ±Ρ‰ΠΈΠ΅ свСдСния
    • ΠžΠΏΡ€Π΅Π΄Π΅Π»Π΅Π½ΠΈΡ ΠΈ понятия
    • ГСомСтричСский смысл
  • ΠšΠ°ΡΠ°Ρ‚Π΅Π»ΡŒΠ½Ρ‹Π΅ ΠΊ Ρ„ΠΈΠ³ΡƒΡ€Π°ΠΌ ΠΈ Π³Ρ€Π°Ρ„ΠΈΠΊΠ°ΠΌ
    • Одна ΠΈ нСсколько окруТностСй
    • Эллипс, Π³ΠΈΠΏΠ΅Ρ€Π±ΠΎΠ»Π° ΠΈ ΠΏΠ°Ρ€Π°Π±ΠΎΠ»Π°
  • ΠŸΡ€ΠΈΠΌΠ΅Ρ€Ρ‹ Ρ€Π΅ΡˆΠ΅Π½ΠΈΡ
    • Π Π΅ΠΊΠΎΠΌΠ΅Π½Π΄Π°Ρ†ΠΈΠΈ спСциалистов
    • УпраТнСния ΠΈ Ρ…ΠΎΠ΄ вычислСний

ΠžΠ±Ρ‰ΠΈΠ΅ свСдСния

ΠšΠ°ΡΠ°Ρ‚Π΅Π»ΡŒΠ½ΠΎΠΉ называСтся прямая, ΠΈΠΌΠ΅ΡŽΡ‰Π°Ρ с Ρ„ΠΈΠ³ΡƒΡ€ΠΎΠΉ ΠΈΠ»ΠΈ Π³Ρ€Π°Ρ„ΠΈΠΊΠΎΠΌ Π·Π°Π΄Π°Π½Π½ΠΎΠΉ Ρ„ΡƒΠ½ΠΊΡ†ΠΈΠΈ ΠΎΠ΄Π½Ρƒ ΠΎΠ±Ρ‰ΡƒΡŽ Ρ‚ΠΎΡ‡ΠΊΡƒ. Однако ΠΈΠ½ΠΎΠ³Π΄Π° ΠΎΠ½Π° ΠΏΡ€ΠΎΡ…ΠΎΠ΄ΠΈΡ‚ Ρ‡Π΅Ρ€Π΅Π· 2 Ρ‚ΠΎΡ‡ΠΊΠΈ. Π’ этом случаС Π΅Π΅ Π½Π°Π·Ρ‹Π²Π°ΡŽΡ‚ сСкущСй. ΠŸΡ€ΡΠΌΠ°Ρ задаСтся ΡΠ»Π΅Π΄ΡƒΡŽΡ‰ΠΈΠΌ ΡƒΡ€Π°Π²Π½Π΅Π½ΠΈΠ΅ΠΌ: y = kx + b. Π—Π½Π°Ρ‡Π΅Π½ΠΈΠ΅ Β«kΒ» β€” это ΡƒΠ³Π»ΠΎΠ²ΠΎΠΉ коэффициСнт.

Для Ρ€Π΅ΡˆΠ΅Π½ΠΈΡ Π·Π°Π΄Π°Ρ‡ слСдуСт Ρ€Π°Π·ΠΎΠ±Ρ€Π°Ρ‚ΡŒ основныС понятия, опрСдСлСния, Ρ„ΠΎΡ€ΠΌΡƒΠ»Ρ‹ ΠΈ свойства ΠΊΠ°ΡΠ°Ρ‚Π΅Π»ΡŒΠ½ΠΎΠΉ.

ΠšΡ€ΠΎΠΌΠ΅ Ρ‚ΠΎΠ³ΠΎ, ΠΎΡ‡Π΅Π½ΡŒ Π²Π°ΠΆΠ½ΠΎ ΠΏΠΎΠ½ΡΡ‚ΡŒ Π΅Π΅ гСомСтричСский смысл, ΠΏΠΎΡΠΊΠΎΠ»ΡŒΠΊΡƒ Π±Π΅Π· Π½Π΅Π³ΠΎ Π±ΡƒΠ΄Π΅Ρ‚ слоТно Ρ€Π°Π·ΠΎΠ±Ρ€Π°Ρ‚ΡŒΡΡ Π² Π±ΠΎΠ»Π΅Π΅ слоТных дисциплинах с Ρ„ΠΈΠ·ΠΈΠΊΠΎ-матСматичСским ΡƒΠΊΠ»ΠΎΠ½ΠΎΠΌ.

ΠžΠΏΡ€Π΅Π΄Π΅Π»Π΅Π½ΠΈΡ ΠΈ понятия

Π£ ΠΊΠ°ΡΠ°Ρ‚Π΅Π»ΡŒΠ½ΠΎΠΉ Π΅ΡΡ‚ΡŒ ΠΎΠΏΡ€Π΅Π΄Π΅Π»Π΅Π½Π½Ρ‹ΠΉ ΠΏΠ°Ρ€Π°ΠΌΠ΅Ρ‚Ρ€ β€” ΡƒΠ³ΠΎΠ» Π½Π°ΠΊΠ»ΠΎΠ½Π° (Π°).

Π•Π³ΠΎ Π½Π΅ΠΎΠ±Ρ…ΠΎΠ΄ΠΈΠΌΠΎ ΠΎΡ‚ΡΡ‡ΠΈΡ‚Ρ‹Π²Π°Ρ‚ΡŒ ΠΎΡ‚ оси абсцисс (Ρ‚ΠΎΠ»ΡŒΠΊΠΎ ΠΏΠΎΠ»ΠΎΠΆΠΈΡ‚Π΅Π»ΡŒΠ½ΠΎΠ΅ Π½Π°ΠΏΡ€Π°Π²Π»Π΅Π½ΠΈΠ΅) ΠΊ прямой, Π·Π°Π΄Π°Π½Π½ΠΎΠΉ Π³Ρ€Π°Ρ„ΠΈΠΊΠΎΠΌ y = kx + b.

ΠžΡ‚ Π½Π΅Π³ΠΎ зависит Π΅Π΅ располоТСниС.

ΠšΠΎΡΡ„Ρ„ΠΈΡ†ΠΈΠ΅Π½Ρ‚ Β«ΠΊΒ» Ρ€Π°Π²Π΅Π½ Π·Π½Π°Ρ‡Π΅Π½ΠΈΡŽ тангСнса ΡƒΠ³Π»Π° Π½Π°ΠΊΠ»ΠΎΠ½Π°, Ρ‚. Π΅. tg(a).

ΠœΠ°Ρ‚Π΅ΠΌΠ°Ρ‚ΠΈΠΊΠΈ сдСлали Π½Π΅ΠΊΠΎΡ‚ΠΎΡ€Ρ‹Π΅ Π²Ρ‹Π²ΠΎΠ΄Ρ‹, ΠΊΠΎΡ‚ΠΎΡ€Ρ‹Π΅ ΠΎΡΠ½ΠΎΠ²Ρ‹Π²Π°ΡŽΡ‚ΡΡ Π½Π° Π·Π½Π°Ρ‡Π΅Π½ΠΈΠΈ ΡƒΠ³Π»ΠΎΠ²ΠΎΠ³ΠΎ коэффициСнта:

  • ΠŸΡ€ΠΈ a = 0 прямая ΠΏΠ°Ρ€Π°Π»Π»Π΅Π»ΡŒΠ½Π° ОΠ₯: k = tg(0) = 0 (y = b).
  • Если ΡƒΠ³ΠΎΠ» острый (0<a<90), Ρ‚ΠΎ k>0. Π’ этом случаС Π³Ρ€Π°Ρ„ΠΈΠΊ возрастаСт (y = kx + b).
  • Π£Π³ΠΎΠ» являСтся прямым (a = ПИ/2): прямая располоТСна пСрпСндикулярно оси абсцисс.
  • Π’ случаС, ΠΊΠΎΠ³Π΄Π° выполняСтся нСравСнство 90<a<180, Π·Π½Π°Ρ‡Π΅Π½ΠΈΠ΅ k<0. Π“Ρ€Π°Ρ„ΠΈΠΊ являСтся ΡƒΠ±Ρ‹Π²Π°ΡŽΡ‰ΠΈΠΌ.
  • Π’ ΠΏΠ΅Ρ€Π²ΠΎΠΌ, Π²Ρ‚ΠΎΡ€ΠΎΠΌ ΠΈ Ρ‚Ρ€Π΅Ρ‚ΡŒΠ΅ΠΌ случаях коэффициСнт являСтся ΠΏΠΎΠ»ΠΎΠΆΠΈΡ‚Π΅Π»ΡŒΠ½Ρ‹ΠΌ, Π° Π² послСднСм β€” ΠΎΡ‚Ρ€ΠΈΡ†Π°Ρ‚Π΅Π»ΡŒΠ½Ρ‹ΠΌ. Π­Ρ‚ΠΈ Ρ„Π°ΠΊΡ‚Ρ‹ слСдуСт ΡƒΡ‡ΠΈΡ‚Ρ‹Π²Π°Ρ‚ΡŒ ΠΏΡ€ΠΈ Ρ€Π΅ΡˆΠ΅Π½ΠΈΠΈ Π·Π°Π΄Π°Ρ‡. ΠšΠ°ΡΠ°Ρ‚Π΅Π»ΡŒΠ½Π°Ρ прямая ΠΌΠΎΠΆΠ΅Ρ‚ ΡΠ²Π»ΡΡ‚ΡŒΡΡ ΠΈ сСкущСй, Ρ‚. Π΅. ΡΠΎΠΏΡ€ΠΈΠΊΠ°ΡΠ°Ρ‚ΡŒΡΡ с Π³Ρ€Π°Ρ„ΠΈΠΊΠΎΠΌ Ρ„ΡƒΠ½ΠΊΡ†ΠΈΠΈ сразу Π² Π΄Π²ΡƒΡ… ΠΈ Π±ΠΎΠ»Π΅Π΅ Ρ‚ΠΎΡ‡ΠΊΠ°Ρ…. Π‘Π»Π΅Π΄ΡƒΠ΅Ρ‚ ΠΎΡ‚ΠΌΠ΅Ρ‚ΠΈΡ‚ΡŒ, Ρ‡Ρ‚ΠΎ ΠΏΡ€ΠΈ ΠΏΠ°Ρ€Π°Π»Π»Π΅Π»ΡŒΠ½ΠΎΡΡ‚ΠΈ прямой оси ОΠ₯ (y = b), ΠΎΠ½Π° ΠΌΠΎΠΆΠ΅Ρ‚ ΠΏΠ΅Ρ€Π΅ΡΠ΅ΠΊΠ°Ρ‚ΡŒ Ρ„ΡƒΠ½ΠΊΡ†ΠΈΡŽ бСсконСчноС число Ρ€Π°Π·.

    БущСствуСт Π΅Ρ‰Π΅ ΠΎΠ΄Π½ΠΎ ΠΎΠΏΡ€Π΅Π΄Π΅Π»Π΅Π½ΠΈΠ΅: ΠΊΠ°ΡΠ°Ρ‚Π΅Π»ΡŒΠ½ΠΎΠΉ ΠΊ Ρ„ΡƒΠ½ΠΊΡ†ΠΈΠΈ Π²ΠΈΠ΄Π° y = f(x) Π² Ρ‚ΠΎΡ‡ΠΊΠ΅ (Ρ…0, f(x0)) являСтся прямая, которая ΠΏΡ€ΠΎΡ…ΠΎΠ΄ΠΈΡ‚ Ρ‡Π΅Ρ€Π΅Π· эту Ρ‚ΠΎΡ‡ΠΊΡƒ с Ρ‚Π΅ΠΌ условиСм, Ρ‡Ρ‚ΠΎ ΠΎΡ‚Ρ€Π΅Π·ΠΎΠΊ ΠΈΠΌΠ΅Π΅Ρ‚ мноТСство Π·Π½Π°Ρ‡Π΅Π½ΠΈΠΉ, Π±Π»ΠΈΠ·ΠΊΠΈΡ… ΠΊ Π½Π΅ΠΉ (Ρ… -> x0).

    ГСомСтричСский смысл

    ΠŸΡƒΡΡ‚ΡŒ Π΄Π°Π½Π° нСкоторая функция y = f(x) ΠΈ сСкущая АВ (рис. 1). ΠšΠΎΠΎΡ€Π΄ΠΈΠ½Π°Ρ‚Ρ‹ послСднСй Π² Ρ‚ΠΎΡ‡ΠΊΠ°Ρ… А ΠΈ Π’ ΡΠ»Π΅Π΄ΡƒΡŽΡ‰ΠΈΠ΅: А(Ρ…0;f(x0)) ΠΈ Π’(Ρ…0+zx;f(x0+zx)). Π’Π΅Π»ΠΈΡ‡ΠΈΠ½Π° Β«zxΒ» β€” ΠΏΡ€ΠΈΡ€Π°Ρ‰Π΅Π½ΠΈΠ΅ Π°Ρ€Π³ΡƒΠΌΠ΅Π½Ρ‚Π° ΠΏΠΎ Ρ…, ΠΊΠΎΡ‚ΠΎΡ€ΠΎΠ΅ ΠΏΠΎΠΊΠ°Π·Π°Π½ΠΎ стрСлками. Если ΠΏΠΎΠ΄ΡΡ‚Π°Π²ΠΈΡ‚ΡŒ ΠΊΠΎΠΎΡ€Π΄ΠΈΠ½Π°Ρ‚Ρ‹ Π² Ρ„ΡƒΠ½ΠΊΡ†ΠΈΡŽ, Ρ‚ΠΎ ΠΎΠ½Π° ΠΈΠΌΠ΅Π΅Ρ‚ Ρ‚Π°ΠΊΠΎΠΉ Π²ΠΈΠ΄: zy = zf(x) = f(x0+zx) β€” f(zx).

    Рисунок 1. ГСомСтричСский смысл.

    Π‘ΠΎΠΎΡ‚Π½ΠΎΡˆΠ΅Π½ΠΈΠ΅, ΠΊΠΎΡ‚ΠΎΡ€ΠΎΠ΅ Π±Ρ‹Π»ΠΎ ΠΏΠΎΠ»ΡƒΡ‡Π΅Π½ΠΎ Π²Ρ‹ΡˆΠ΅, называСтся ΠΏΡ€ΠΎΠΈΠ·Π²ΠΎΠ΄Π½ΠΎΠΉ. Если ΠΊ Π³Ρ€Π°Ρ„ΠΈΠΊΡƒ Π² Ρ‚ΠΎΡ‡ΠΊΠ΅ ΠΏΡ€ΠΎΠ²Π΅Π΄Π΅Π½Π° сСкущая ΠΈΠ»ΠΈ ΠΊΠ°ΡΠ°Ρ‚Π΅Π»ΡŒΠ½Π°Ρ, Ρ‚ΠΎ тангСнс ΡƒΠ³Π»Π° Π±ΡƒΠ΄Π΅Ρ‚ Ρ€Π°Π²Π΅Π½ самой ΠΏΡ€ΠΎΠΈΠ·Π²ΠΎΠ΄Π½ΠΎΠΉ Π·Π°Π΄Π°Π½Π½ΠΎΠΉ Ρ„ΡƒΠ½ΠΊΡ†ΠΈΠΈ Π² Ρ‚ΠΎΡ‡ΠΊΠ΅ с ΠΊΠΎΠΎΡ€Π΄ΠΈΠ½Π°Ρ‚ΠΎΠΉ Ρ…0.

    Из этого опрСдСлСния ΠΌΠΎΠΆΠ½ΠΎ ΡΠ΄Π΅Π»Π°Ρ‚ΡŒ Π²Ρ‹Π²ΠΎΠ΄ ΠΎ сущСствовании ΠΏΡ€ΠΎΠΈΠ·Π²ΠΎΠ΄Π½ΠΎΠΉ. Если Π·Π½Π°Ρ‡Π΅Π½ΠΈΠ΅ послСднСй Ρ€Π°Π²Π½ΠΎ 0, Ρ‚ΠΎ, ΡΠ»Π΅Π΄ΠΎΠ²Π°Ρ‚Π΅Π»ΡŒΠ½ΠΎ, Π½Π΅ сущСствуСт ΠΎΠ±Ρ‰ΠΈΡ… Ρ‚ΠΎΡ‡Π΅ΠΊ с Π·Π°Π΄Π°Π½Π½ΠΎΠΉ Ρ„ΠΈΠ³ΡƒΡ€ΠΎΠΉ.

    ΠšΠ°ΡΠ°Ρ‚Π΅Π»ΡŒΠ½Ρ‹Π΅ ΠΊ Ρ„ΠΈΠ³ΡƒΡ€Π°ΠΌ ΠΈ Π³Ρ€Π°Ρ„ΠΈΠΊΠ°ΠΌ

    ΠŸΡ€ΠΈ Ρ€Π΅ΡˆΠ΅Π½ΠΈΠΈ Π·Π°Π΄Π°Ρ‡ слСдуСт ΠΎΠ±Ρ€Π°Ρ‚ΠΈΡ‚ΡŒ Π²Π½ΠΈΠΌΠ°Π½ΠΈΠ΅ Π½Π° частныС случаи. НуТно произвСсти расчСты уравнСния прямой ΠΈΠ»ΠΈ Π½Π°ΠΉΡ‚ΠΈ Ρ‚ΠΎΡ‡ΠΊΠΈ соприкосновСния с ΠΎΠΊΡ€ΡƒΠΆΠ½ΠΎΡΡ‚ΡŒΡŽ, эллипсом, Π³ΠΈΠΏΠ΅Ρ€Π±ΠΎΠ»ΠΎΠΉ ΠΈΠ»ΠΈ ΠΏΠ°Ρ€Π°Π±ΠΎΠ»ΠΎΠΉ. ΠžΡ‡Π΅Π½ΡŒ распространСнная Π·Π°Π΄Π°Ρ‡Π° встрСчаСтся Ρ‚Π°ΠΊΠΆΠ΅ Π² ΠΌΠ΅Ρ…Π°Π½ΠΈΠΊΠ΅ ΠΎ Ρ€Π΅ΠΌΠ΅Π½Π½ΠΎΠΉ ΠΏΠ΅Ρ€Π΅Π΄Π°Ρ‡Π΅.

    ЧастныС случаи позволят Π½Π°ΠΉΡ‚ΠΈ ΠΎΠΏΡ‚ΠΈΠΌΠ°Π»ΡŒΠ½ΠΎΠ΅ Ρ€Π΅ΡˆΠ΅Π½ΠΈΠ΅ ΠΈ ΠΌΠ΅Ρ‚ΠΎΠ΄ расчСта, ΠΏΠΎΡΠΊΠΎΠ»ΡŒΠΊΡƒ экономия Π²Ρ€Π΅ΠΌΠ΅Π½ΠΈ являСтся Π²Π°ΠΆΠ½Ρ‹ΠΌ элСмСнтом ΠΏΡ€ΠΈ Π½Π°ΡƒΡ‡Π½Ρ‹Ρ… исслСдованиях, написании ΠΊΠΎΠ½Ρ‚Ρ€ΠΎΠ»ΡŒΠ½Ρ‹Ρ… Ρ€Π°Π±ΠΎΡ‚ ΠΈ сдачС экзамСнов. Π’Π°ΠΆΠ½Ρ‹ΠΉ этап β€” идСнтификация Ρ‚ΠΈΠΏΠ° Π·Π°Π΄Π°Ρ‡ΠΈ. ΠšΠ°ΡΠ°Ρ‚Π΅Π»ΡŒΠ½Π°Ρ ΠΊ Π²Ρ‹ΡˆΠ΅ΠΏΠ΅Ρ€Π΅Ρ‡ΠΈΡΠ»Π΅Π½Π½Ρ‹ΠΌ Ρ„ΠΈΠ³ΡƒΡ€Π°ΠΌ β€” основной Ρ‚ΠΈΠΏ Π·Π°Π΄Π°Π½ΠΈΠΉ, Π½ΠΎ ΡΡƒΡ‰Π΅ΡΡ‚Π²ΡƒΡŽΡ‚ ΠΈ Π±ΠΎΠ»Π΅Π΅ слоТныС Ρ„ΡƒΠ½ΠΊΡ†ΠΈΠΈ.

    НапримСр, слоТно ΡΠΎΡΡ‚Π°Π²ΠΈΡ‚ΡŒ ΡƒΡ€Π°Π²Π½Π΅Π½ΠΈΠ΅ прямой, которая ΠΈΠΌΠ΅Π΅Ρ‚ Ρ‚ΠΎΡ‡ΠΊΠΈ касания с ΠΊΠ°ΠΊΠΎΠΉ-Π»ΠΈΠ±ΠΎ слоТной Ρ„ΡƒΠ½ΠΊΡ†ΠΈΠ΅ΠΉ. 2] + yΡ†.

    Π”Π²Π΅ Ρ„ΡƒΠ½ΠΊΡ†ΠΈΠΈ ΡΠ²Π»ΡΡŽΡ‚ΡΡ ΠΏΠΎΠ»ΡƒΠΊΡ€ΡƒΠ³Π°ΠΌΠΈ ΠΈ вмСстС ΠΎΠ±Ρ€Π°Π·ΡƒΡŽΡ‚ ΠΎΠΊΡ€ΡƒΠΆΠ½ΠΎΡΡ‚ΡŒ. Π§Ρ‚ΠΎΠ±Ρ‹ ΡΠΎΡΡ‚Π°Π²ΠΈΡ‚ΡŒ Π³Ρ€Π°Ρ„ΠΈΠΊ ΠΊΡ€ΡƒΠ³Π° Π² Ρ‚ΠΎΡ‡ΠΊΠ΅ (Ρ…0;Ρƒ0), Π½ΡƒΠΆΠ½ΠΎ ΡƒΡ€Π°Π²Π½Π΅Π½ΠΈΠ΅ Π² этой Ρ‚ΠΎΡ‡ΠΊΠ΅. Π’ Ρ‚ΠΎΡ‡ΠΊΠ°Ρ… с ΠΊΠΎΠΎΡ€Π΄ΠΈΠ½Π°Ρ‚Π°ΠΌΠΈ (Ρ…Ρ†;yΡ†+R) ΠΈ (Ρ…Ρ†;yΡ†-R) уравнСния ΠΊΠ°ΡΠ°Ρ‚Π΅Π»ΡŒΠ½Ρ‹Ρ… ΠΊ окруТности Π·Π°Π΄Π°ΡŽΡ‚ΡΡ ΡΠ»Π΅Π΄ΡƒΡŽΡ‰ΠΈΠΌΠΈ уравнСниями: y = yΡ† + R ΠΈ y = yΡ† β€” R. Если Π²Π·ΡΡ‚ΡŒ Ρ‚ΠΎΡ‡ΠΊΠΈ (Ρ…Ρ†+R;yΡ†) ΠΈ (Ρ…Ρ†-R;yΡ†), ΠΎΠ½ΠΈ Π±ΡƒΠ΄ΡƒΡ‚ ΠΈΠΌΠ΅Ρ‚ΡŒ Ρ‚Π°ΠΊΡƒΡŽ Ρ„ΠΎΡ€ΠΌΡƒ: x = xΡ† + R ΠΈ x = xΡ† β€” R.

    Π’ случаС для Π΄Π²ΡƒΡ… окруТностСй всСго ΠΌΠΎΠΆΠ½ΠΎ провСсти Π΄ΠΎ 4 ΠΊΠ°ΡΠ°Ρ‚Π΅Π»ΡŒΠ½Ρ‹Ρ… (2 Π²Π½Π΅ΡˆΠ½ΠΈΡ… ΠΈ 2 Π²Π½ΡƒΡ‚Ρ€Π΅Π½Π½ΠΈΡ…). Π­Ρ‚ΠΎ зависит ΠΎΡ‚ случая располоТСния Ρ„ΠΈΠ³ΡƒΡ€. Π’ΠΎΡ‡ΠΊΠΎΠΉ пСрСсСчСния Π²Π½Π΅ΡˆΠ½ΠΈΡ… считаСтся внСшняя гомотСтия (ΠΏΠΎΠ΄ΠΎΠ±ΠΈΠ΅), Π° Π²Π½ΡƒΡ‚Ρ€Π΅Π½Π½ΠΈΡ… β€” Π² Ρ†Π΅Π½Ρ‚Ρ€Π΅ Π²Π½ΡƒΡ‚Ρ€Π΅Π½Π½Π΅Π³ΠΎ подобия. Π’Π½Π΅ΡˆΠ½ΠΈΠΌΠΈ Π½Π°Π·Ρ‹Π²Π°ΡŽΡ‚ΡΡ прямыС, ΠΊΠΎΡ‚ΠΎΡ€Ρ‹Π΅ ΠΊΠ°ΡΠ°ΡŽΡ‚ΡΡ Π²Π½Π΅ΡˆΠ½ΠΈΡ… Ρ‚ΠΎΡ‡Π΅ΠΊ ΠΊΡ€ΡƒΠ³Π°. Если ΠΊΠ°ΡΠ°Ρ‚Π΅Π»ΡŒΠ½Ρ‹Π΅ ΡΠ²Π»ΡΡŽΡ‚ΡΡ Π²Π½ΡƒΡ‚Ρ€Π΅Π½Π½ΠΈΠΌΠΈ, Ρ‚ΠΎ ΠΎΠ½ΠΈ ΠΏΠ΅Ρ€Π΅ΡΠ΅ΠΊΠ°ΡŽΡ‚ линию, ΡΠΎΠ΅Π΄ΠΈΠ½ΡΡŽΡ‰ΡƒΡŽ Ρ†Π΅Π½Ρ‚Ρ€Ρ‹ окруТностСй.

    Π‘Π»Π΅Π΄ΡƒΠ΅Ρ‚ ΠΎΡ‚ΠΌΠ΅Ρ‚ΠΈΡ‚ΡŒ, Ρ‡Ρ‚ΠΎ внСшний ΠΈ Π²Π½ΡƒΡ‚Ρ€Π΅Π½Π½ΠΈΠΉ Ρ†Π΅Π½Ρ‚Ρ€Ρ‹ Π³ΠΎΠΌΠΎΡ‚Π΅Ρ‚ΠΈΠΈ Π»Π΅ΠΆΠ°Ρ‚ Π½Π° Π½Π΅ΠΊΠΎΡ‚ΠΎΡ€ΠΎΠΉ прямой. Она ΠΏΡ€ΠΎΡ…ΠΎΠ΄ΠΈΡ‚ Ρ‡Π΅Ρ€Π΅Π· Ρ†Π΅Π½Ρ‚Ρ€Ρ‹ Π·Π°Π΄Π°Π½Π½Ρ‹Ρ… окруТностСй. 2 β€” 4a(c β€” x).

    Π’ зависимости ΠΎΡ‚ Π΅Π³ΠΎ значСния находятся ΠΊΠΎΡ€Π½ΠΈ:

  • D>0: y = [-b + sqrt(D)] / 2a ΠΈ y = [-b β€” sqrt(D)] / 2a.
  • D=0: y = -b / 2a.
  • D<0: Π½Π΅Ρ‚ Ρ‚ΠΎΡ‡Π΅ΠΊ касания.
  • Π‘ΡƒΡ‚ΡŒ сводится ΠΊ Ρ€Π΅ΡˆΠ΅Π½ΠΈΡŽ ΠΎΠ±Ρ‹ΠΊΠ½ΠΎΠ²Π΅Π½Π½ΠΎΠ³ΠΎ ΠΊΠ²Π°Π΄Ρ€Π°Ρ‚Π½ΠΎΠ³ΠΎ уравнСния. Если коэффициСнт Π°=1, Ρ‚ΠΎ ΠΊΠΎΡ€Π½ΠΈ ΠΌΠΎΠΆΠ½ΠΎ Π½Π°ΠΉΡ‚ΠΈ ΠΏΠΎ Ρ‚Π΅ΠΎΡ€Π΅ΠΌΠ΅ Π’ΠΈΠ΅Ρ‚Π°: x1 + x2 = β€” b ΠΈ x1 * x2 = c.

    ΠŸΡ€ΠΈΠΌΠ΅Ρ€Ρ‹ Ρ€Π΅ΡˆΠ΅Π½ΠΈΡ

    БущСствуСт нСсколько Ρ‚ΠΈΠΏΠΎΠ² Π·Π°Π΄Π°Ρ‡ Π½Π° Π½Π°Ρ…ΠΎΠΆΠ΄Π΅Π½ΠΈΠ΅ уравнСния прямой, которая соприкасаСтся с Π·Π°Π΄Π°Π½Π½Ρ‹ΠΌ Π³Ρ€Π°Ρ„ΠΈΠΊΠΎΠΌ Ρ„ΡƒΠ½ΠΊΡ†ΠΈΠΈ. Π‘Π°ΠΌΠΎΠΉ простой являСтся Π·Π°Π΄Π°Ρ‡Π° со ΡΠ»Π΅Π΄ΡƒΡŽΡ‰Π΅ΠΉ Ρ„ΠΎΡ€ΠΌΡƒΠ»ΠΈΡ€ΠΎΠ²ΠΊΠΎΠΉ: прямая являСтся ΠΊΠ°ΡΠ°Ρ‚Π΅Π»ΡŒΠ½ΠΎΠΉ ΠΊ Π³Ρ€Π°Ρ„ΠΈΠΊΡƒ Ρ„ΡƒΠ½ΠΊΡ†ΠΈΠΈ. НайдитС всС Ρ‚ΠΎΡ‡ΠΊΠΈ касания. Π’ этом случаС задаСтся ΡƒΡ€Π°Π²Π½Π΅Π½ΠΈΠ΅ Π³Ρ€Π°Ρ„ΠΈΠΊΠ° Ρ„ΡƒΠ½ΠΊΡ†ΠΈΠΈ ΠΈ прямой. НСкоторыС задания ΡΡ‡ΠΈΡ‚Π°ΡŽΡ‚ΡΡ Π±ΠΎΠ»Π΅Π΅ слоТными. Π’ Π½ΠΈΡ… Π½Π΅ΠΎΠ±Ρ…ΠΎΠ΄ΠΈΠΌΠΎ Π½Π°ΠΏΠΈΡΠ°Ρ‚ΡŒ ΡƒΡ€Π°Π²Π½Π΅Π½ΠΈΠ΅ ΠΊΠ°ΡΠ°Ρ‚Π΅Π»ΡŒΠ½ΠΎΠΉ ΠΈΠ»ΠΈ ΠΊΠ°ΡΠ°Ρ‚Π΅Π»ΡŒΠ½Ρ‹Ρ….

    Π Π΅ΠΊΠΎΠΌΠ΅Π½Π΄Π°Ρ†ΠΈΠΈ спСциалистов

    Для Ρ€Π΅ΡˆΠ΅Π½ΠΈΡ Π·Π°Π΄Π°Ρ‡ΠΈ Π½ΡƒΠΆΠ½ΠΎ Π²Π½ΠΈΠΌΠ°Ρ‚Π΅Π»ΡŒΠ½ΠΎ ΠΏΡ€ΠΎΡ‡ΠΈΡ‚Π°Ρ‚ΡŒ условиС ΠΈ Π²Ρ‹ΡΡΠ½ΠΈΡ‚ΡŒ Π²Π΅Π»ΠΈΡ‡ΠΈΠ½Ρ‹, ΠΊΠΎΡ‚ΠΎΡ€Ρ‹Π΅ слСдуСт Π½Π°ΠΉΡ‚ΠΈ. ВсС построСно Π½Π° Π½Π°Ρ…ΠΎΠΆΠ΄Π΅Π½ΠΈΠΈ ΠΏΡ€ΠΎΠΈΠ·Π²ΠΎΠ΄Π½ΠΎΠΉ Ρ„ΡƒΠ½ΠΊΡ†ΠΈΠΈ. ПослС этого Π½ΡƒΠΆΠ½ΠΎ ΠΏΠΎΠ΄ΡΡ‚Π°Π²ΠΈΡ‚ΡŒ Π·Π½Π°Ρ‡Π΅Π½ΠΈΠ΅ ΠΊΠΎΠΎΡ€Π΄ΠΈΠ½Π°Ρ‚ Ρ‚ΠΎΡ‡ΠΊΠΈ Π² Π²Ρ‹Ρ€Π°ΠΆΠ΅Π½ΠΈΠ΅ ΠΏΠ΅Ρ€Π²ΠΎΠΎΠ±Ρ€Π°Π·Π½ΠΎΠΉ. Π’ Π½Π΅ΠΊΠΎΡ‚ΠΎΡ€Ρ‹Ρ… случаях функция задаСтся парамСтричСски. Для удобства Π΅Π΅ рСкомСндуСтся пСрСвСсти Π² ΠΊΠ°Π½ΠΎΠ½ΠΈΡ‡Π΅ΡΠΊΡƒΡŽ Ρ„ΠΎΡ€ΠΌΡƒ.

    РСкомСндуСтся Ρ€Π°Π·Π±ΠΈΠ²Π°Ρ‚ΡŒ Π·Π°Π΄Π°Ρ‡Ρƒ Π½Π° нСсколько ΠΏΠΎΠ΄Π·Π°Π΄Π°Ρ‡, ΠΏΠΎΡΠΊΠΎΠ»ΡŒΠΊΡƒ Π±ΡƒΠ΄Π΅Ρ‚ ΠΎΡ‡Π΅Π½ΡŒ просто Π²Ρ‹ΠΏΠΎΠ»Π½ΠΈΡ‚ΡŒ ΠΏΡ€ΠΎΠ²Π΅Ρ€ΠΊΡƒ ΠΈ ΠΈΡΠΏΡ€Π°Π²ΠΈΡ‚ΡŒ Π½Π°ΠΉΠ΄Π΅Π½Π½Ρ‹Π΅ ошибки. БущСствуСт нСсколько способов нахоТдСния уравнСния ΠΊΠ°ΡΠ°Ρ‚Π΅Π»ΡŒΠ½ΠΎΠΉ: Π°Π²Ρ‚ΠΎΠΌΠ°Ρ‚ΠΈΠ·ΠΈΡ€ΠΎΠ²Π°Π½Π½Ρ‹ΠΉ ΠΈ Ρ€ΡƒΡ‡Π½ΠΎΠΉ. Π’ ΠΏΠ΅Ρ€Π²ΠΎΠΌ случаС Π½ΡƒΠΆΠ½ΠΎ ΠΈΡΠΏΠΎΠ»ΡŒΠ·ΠΎΠ²Π°Ρ‚ΡŒ ΠΏΡ€ΠΎΠ³Ρ€Π°ΠΌΠΌΠ½ΠΎΠ΅ обСспСчСниС. ΠžΠΏΡ‚ΠΈΠΌΠ°Π»ΡŒΠ½Ρ‹ΠΌ Ρ€Π΅ΡˆΠ΅Π½ΠΈΠ΅ΠΌ ΠΏΡ€ΠΎΠ±Π»Π΅ΠΌΡ‹ являСтся ΠΎΠ½Π»Π°ΠΉΠ½-ΠΊΠ°Π»ΡŒΠΊΡƒΠ»ΡΡ‚ΠΎΡ€.

    ΠŸΡ€ΠΈ Ρ€ΡƒΡ‡Π½ΠΎΠΌ Ρ€Π΅ΠΆΠΈΠΌΠ΅ Π½ΡƒΠΆΠ½ΠΎ Ρ€Π΅ΡˆΠ°Ρ‚ΡŒ, Π° ΠΈΠ½ΠΎΠ³Π΄Π° Π²Ρ‹ΠΏΠΎΠ»Π½ΠΈΡ‚ΡŒ построСниС Π³Ρ€Π°Ρ„ΠΈΠΊΠ°. Для ΠΎΠΏΡ‚ΠΈΠΌΠΈΠ·Π°Ρ†ΠΈΠΈ вычислСний ΠΌΠΎΠΆΠ½ΠΎ ΠΈΡΠΏΠΎΠ»ΡŒΠ·ΠΎΠ²Π°Ρ‚ΡŒ Excel. Π“Ρ€Π°Ρ„ΠΈΠΊ Π΄ΠΎΠ»ΠΆΠ΅Π½ Π±Ρ‹Ρ‚ΡŒ качСствСнно построСн ΠΈ ΠΏΡ€Π΅Π΄Π΅Π»ΡŒΠ½ΠΎ понятСн. Π’ Π½Π΅ΠΊΠΎΡ‚ΠΎΡ€Ρ‹Ρ… случая Π½ΡƒΠΆΠ½ΠΎ Π±ΡƒΠ΄Π΅Ρ‚ Π²Ρ‹Ρ‡ΠΈΡΠ»ΡΡ‚ΡŒ ΠΏΡ€Π΅Π΄Π΅Π»ΡŒΠ½Ρ‹Π΅ значСния ΠΈΡΠΏΠΎΠ»ΡŒΠ·ΡƒΡ Π³Ρ€Π°Π½ΠΈΡ†Ρ‹ (lim).

    УпраТнСния ΠΈ Ρ…ΠΎΠ΄ вычислСний

    НуТно Π½Π°ΠΏΠΈΡΠ°Ρ‚ΡŒ ΡƒΡ€Π°Π²Π½Π΅Π½ΠΈΠ΅ прямой-ΠΊΠ°ΡΠ°Ρ‚Π΅Π»ΡŒΠ½ΠΎΠΉ ΠΊ y(x) = x^3 β€” 2x^2 + 3 Π² Ρ‚. 2 β€” 4ac = 4 β€” 4 * 3 * (-3) = 40.

  • Π£Ρ€Π°Π²Π½Π΅Π½ΠΈΠ΅ ΠΈΠΌΠ΅Π΅Ρ‚ 2 корня: Ρ…1 = (2 β€” sqr(40)) / 6 = (1 β€” sqr(10)) / 3 ΠΈ x2 = (1 + sqr(10)) / 3.
  • РСкомСндуСтся ΠΎΡΡ‚Π°Π²ΠΈΡ‚ΡŒ Π² Ρ‚Π°ΠΊΠΎΠΌ Π²ΠΈΠ΄Π΅, ΠΏΠΎΡΠΊΠΎΠ»ΡŒΠΊΡƒ ΠΏΡ€ΠΈ вычислСнии кубичСского корня появятся Π½Π΅ΠΊΠΎΡ‚ΠΎΡ€Ρ‹Π΅ ΠΏΠΎΠ³Ρ€Π΅ΡˆΠ½ΠΎΡΡ‚ΠΈ. Π’ этих ΠΏΡ€ΠΈΠΌΠ΅Ρ€Π°Ρ… Π½Π΅ΠΎΠ±ΡΠ·Π°Ρ‚Π΅Π»ΡŒΠ½ΠΎ составлСниС Π³Ρ€Π°Ρ„ΠΈΠΊΠ°.

    Π’Π°ΠΊΠΈΠΌ ΠΎΠ±Ρ€Π°Π·ΠΎΠΌ, гСомСтричСский смысл уравнСния ΠΊΠ°ΡΠ°Ρ‚Π΅Π»ΡŒΠ½ΠΎΠΉ ΠΊ Ρ„ΡƒΠ½ΠΊΡ†ΠΈΠΈ β€” производная. Π‘Π»Π΅Π΄ΡƒΠ΅Ρ‚ ΠΈΠ·ΡƒΡ‡ΠΈΡ‚ΡŒ основныС понятия, Ρ„ΠΎΡ€ΠΌΡƒΠ»Ρ‹ ΠΈ Ρ€Π°Π·ΠΎΠ±Ρ€Π°Ρ‚ΡŒ Ρ€Π΅ΡˆΠ΅Π½ΠΈΠ΅ Ρ‚ΠΈΠΏΠΎΠ²Ρ‹Ρ… Π·Π°Π΄Π°Ρ‡. Π’Π°ΠΊΠΆΠ΅ Π½ΡƒΠΆΠ½ΠΎ ΠΏΠΎΠ²Ρ‚ΠΎΡ€ΠΈΡ‚ΡŒ Ρ‚Π°Π±Π»ΠΈΡ†Ρƒ ΠΏΡ€ΠΎΠΈΠ·Π²ΠΎΠ΄Π½Ρ‹Ρ… Ρ„ΡƒΠ½ΠΊΡ†ΠΈΠΉ.

    ΠœΡΡ‚ΡƒΡΠΉ | ΠŸΠΎΠΏΡƒΠ»ΡΡ€Π½Ρ‹Π΅ Π·Π°Π΄Π°Ρ‡ΠΈ

    92) 9(3x) ΠΏΠΎ ΠΎΡ‚Π½ΠΎΡˆΠ΅Π½ΠΈΡŽ ΠΊ x 92+1
    1 Найти ΠΏΡ€ΠΎΠΈΠ·Π²ΠΎΠ΄Π½ΡƒΡŽ — d/dx Π±Ρ€Π΅Π²Π½ΠΎ Π½Π°Ρ‚ΡƒΡ€Π°Π»ΡŒΠ½ΠΎΠ΅ Ρ…
    2 ΠžΡ†Π΅Π½ΠΈΡ‚ΡŒ ΠΈΠ½Ρ‚Π΅Π³Ρ€Π°Π» ΠΈΠ½Ρ‚Π΅Π³Ρ€Π°Π» Π½Π°Ρ‚ΡƒΡ€Π°Π»ΡŒΠ½ΠΎΠ³ΠΎ Π»ΠΎΠ³Π°Ρ€ΠΈΡ„ΠΌΠ° x ΠΎΡ‚Π½ΠΎΡΠΈΡ‚Π΅Π»ΡŒΠ½ΠΎ x
    3 Найти ΠΏΡ€ΠΎΠΈΠ·Π²ΠΎΠ΄Π½ΡƒΡŽ — d/dx
    21 ΠžΡ†Π΅Π½ΠΈΡ‚ΡŒ ΠΈΠ½Ρ‚Π΅Π³Ρ€Π°Π» ΠΈΠ½Ρ‚Π΅Π³Ρ€Π°Π» ΠΎΡ‚ 0 Π΄ΠΎ 1 кубичСского корня ΠΈΠ· 1+7x ΠΎΡ‚Π½ΠΎΡΠΈΡ‚Π΅Π»ΡŒΠ½ΠΎ x
    22 Найти ΠΏΡ€ΠΎΠΈΠ·Π²ΠΎΠ΄Π½ΡƒΡŽ — d/dx Π³Ρ€Π΅Ρ…(2x)
    23 Найти ΠΏΡ€ΠΎΠΈΠ·Π²ΠΎΠ΄Π½ΡƒΡŽ — d/dx
    41 ΠžΡ†Π΅Π½ΠΈΡ‚ΡŒ ΠΈΠ½Ρ‚Π΅Π³Ρ€Π°Π» ΠΈΠ½Ρ‚Π΅Π³Ρ€Π°Π» ΠΎΡ‚ cos(2x) ΠΎΡ‚Π½ΠΎΡΠΈΡ‚Π΅Π»ΡŒΠ½ΠΎ x
    42 Найти ΠΏΡ€ΠΎΠΈΠ·Π²ΠΎΠ΄Π½ΡƒΡŽ — d/dx 1/(ΠΊΠΎΡ€Π΅Π½ΡŒ ΠΊΠ²Π°Π΄Ρ€Π°Ρ‚Π½Ρ‹ΠΉ ΠΈΠ· Ρ…)
    43 ΠžΡ†Π΅Π½ΠΊΠ° ΠΈΠ½Ρ‚Π΅Π³Ρ€Π°Π»Π° 9Π±Π΅ΡΠΊΠΎΠ½Π΅Ρ‡Π½ΠΎΡΡ‚ΡŒ
    45 Найти ΠΏΡ€ΠΎΠΈΠ·Π²ΠΎΠ΄Π½ΡƒΡŽ — d/dx Ρ…/2
    46 Найти ΠΏΡ€ΠΎΠΈΠ·Π²ΠΎΠ΄Π½ΡƒΡŽ — d/dx -cos(x)
    47 Найти ΠΏΡ€ΠΎΠΈΠ·Π²ΠΎΠ΄Π½ΡƒΡŽ — d/dx Π³Ρ€Π΅Ρ…(3x)
    68 ΠžΡ†Π΅Π½ΠΈΡ‚ΡŒ ΠΈΠ½Ρ‚Π΅Π³Ρ€Π°Π» ΠΈΠ½Ρ‚Π΅Π³Ρ€Π°Π» ΠΎΡ‚ sin(x) ΠΏΠΎ x
    69 Найти ΠΏΡ€ΠΎΠΈΠ·Π²ΠΎΠ΄Π½ΡƒΡŽ — d/dx ΡƒΠ³Π»ΠΎΠ²ΠΎΠΉ синус(Ρ…)
    70 ΠžΡ†Π΅Π½ΠΈΡ‚ΡŒ ΠΏΡ€Π΅Π΄Π΅Π» ΠΎΠ³Ρ€Π°Π½ΠΈΡ‡Π΅Π½ΠΈΠ΅, ΠΊΠΎΠ³Π΄Π° x приблиТаСтся ΠΊ 0 ΠΈΠ· (sin(x))/x 92 ΠΏΠΎ ΠΎΡ‚Π½ΠΎΡˆΠ΅Π½ΠΈΡŽ ΠΊ Ρ…
    85 Найти ΠΏΡ€ΠΎΠΈΠ·Π²ΠΎΠ΄Π½ΡƒΡŽ — d/dx Π»ΠΎΠ³ Ρ…
    86 Найти ΠΏΡ€ΠΎΠΈΠ·Π²ΠΎΠ΄Π½ΡƒΡŽ — d/dx Π°Ρ€ΠΊΡ‚Π°Π½(Ρ…)
    87 Найти ΠΏΡ€ΠΎΠΈΠ·Π²ΠΎΠ΄Π½ΡƒΡŽ — d/dx Π±Ρ€Π΅Π²Π½ΠΎ Π½Π°Ρ‚ΡƒΡ€Π°Π»ΡŒΠ½ΠΎΠ΅ 5Ρ…92

    Найти ΡƒΡ€Π°Π²Π½Π΅Π½ΠΈΠ΅ прямой, ΠΊΠ°ΡΠ°Ρ‚Π΅Π»ΡŒΠ½ΠΎΠΉ ΠΊ ΠΊΡ€ΠΈΠ²ΠΎΠΉ Π² Π·Π°Π΄Π°Π½Π½ΠΎΠΉ Ρ‚ΠΎΡ‡ΠΊΠ΅

    ВсС рСсурсы ΠΏΠΎ ΠΏΡ€Π΅Π΄Π²Π°Ρ€ΠΈΡ‚Π΅Π»ΡŒΠ½ΠΎΠΌΡƒ ΠΈΡΡ‡ΠΈΡΠ»Π΅Π½ΠΈΡŽ

    12 ДиагностичСских тСстов 380 практичСских тСстов Вопрос дня ΠšΠ°Ρ€Ρ‚ΠΎΡ‡ΠΊΠΈ Learn by Concept

    Precalculus Help Β» Π’Π²ΠΎΠ΄Π½Ρ‹ΠΉ расчСт Β» ΠšΠ°ΡΠ°Ρ‚Π΅Π»ΡŒΠ½Ρ‹Π΅ ΠΊ ΠΊΡ€ΠΈΠ²ΠΎΠΉ Β» НайдитС ΡƒΡ€Π°Π²Π½Π΅Π½ΠΈΠ΅ прямой, ΠΊΠ°ΡΠ°Ρ‚Π΅Π»ΡŒΠ½ΠΎΠΉ ΠΊ ΠΊΡ€ΠΈΠ²ΠΎΠΉ Π² Π·Π°Π΄Π°Π½Π½ΠΎΠΉ Ρ‚ΠΎΡ‡ΠΊΠ΅

    НайдитС ΡƒΡ€Π°Π²Π½Π΅Π½ΠΈΠ΅ Π»ΠΈΠ½ΠΈΠΈ, ΠΊΠ°ΡΠ°Ρ‚Π΅Π»ΡŒΠ½ΠΎΠΉ ΠΊ Π³Ρ€Π°Ρ„ΠΈΠΊΡƒ

    Π² Ρ‚ΠΎΡ‡ΠΊΠ΅Β , Π² Ρ„ΠΎΡ€ΠΌΠ΅ пСрСсСчСния Π½Π°ΠΊΠ»ΠΎΠ½Π°.

    Π’ΠΎΠ·ΠΌΠΎΠΆΠ½Ρ‹Π΅ ΠΎΡ‚Π²Π΅Ρ‚Ρ‹:

    ΠŸΡ€Π°Π²ΠΈΠ»ΡŒΠ½Ρ‹ΠΉ ΠΎΡ‚Π²Π΅Ρ‚:

    ОбъяснСниС:

    НачнСм с Ρ‚ΠΎΠ³ΠΎ, Ρ‡Ρ‚ΠΎ вспомним, Ρ‡Ρ‚ΠΎ ΠΎΠ΄Π½ΠΈΠΌ ΠΈΠ· способов опрСдСлСния ΠΏΡ€ΠΎΠΈΠ·Π²ΠΎΠ΄Π½ΠΎΠΉ Ρ„ΡƒΠ½ΠΊΡ†ΠΈΠΈ являСтся Π½Π°ΠΊΠ»ΠΎΠ½ ΠΊΠ°ΡΠ°Ρ‚Π΅Π»ΡŒΠ½ΠΎΠΉ Ρ„ΡƒΠ½ΠΊΡ†ΠΈΠΈ Π² Π΄Π°Π½Π½ΠΎΠΉ Ρ‚ΠΎΡ‡ΠΊΠ΅. Π‘Π»Π΅Π΄ΠΎΠ²Π°Ρ‚Π΅Π»ΡŒΠ½ΠΎ, Π½Π°Ρ…ΠΎΠΆΠ΄Π΅Π½ΠΈΠ΅ ΠΏΡ€ΠΎΠΈΠ·Π²ΠΎΠ΄Π½ΠΎΠΉ нашСго уравнСния ΠΏΠΎΠ·Π²ΠΎΠ»ΠΈΡ‚ Π½Π°ΠΌ Π½Π°ΠΉΡ‚ΠΈ Π½Π°ΠΊΠ»ΠΎΠ½ ΠΊΠ°ΡΠ°Ρ‚Π΅Π»ΡŒΠ½ΠΎΠΉ. ΠŸΠΎΡΠΊΠΎΠ»ΡŒΠΊΡƒ Π΄Π²Π΅ Π²Π΅Ρ‰ΠΈ, Π½Π΅ΠΎΠ±Ρ…ΠΎΠ΄ΠΈΠΌΡ‹Π΅ для нахоТдСния уравнСния прямой, β€” это Π½Π°ΠΊΠ»ΠΎΠ½ ΠΈ Ρ‚ΠΎΡ‡ΠΊΠ°, ΠΌΡ‹ Π½Π° ΠΏΠΎΠ»ΠΏΡƒΡ‚ΠΈ.

    ΠœΡ‹ вычисляСм ΠΏΡ€ΠΎΠΈΠ·Π²ΠΎΠ΄Π½ΡƒΡŽ ΠΏΠΎ стСпСнному ΠΏΡ€Π°Π²ΠΈΠ»Ρƒ.

    Однако Π½Π°ΠΌ Π½ΡƒΠΆΠ΅Π½ Π½Π°ΠΊΠ»ΠΎΠ½ ΠΊΠ°ΡΠ°Ρ‚Π΅Π»ΡŒΠ½ΠΎΠΉ Π½Π΅ Π² любой Ρ‚ΠΎΡ‡ΠΊΠ΅, Π° ΠΈΠΌΠ΅Π½Π½ΠΎ Π² этой Ρ‚ΠΎΡ‡ΠΊΠ΅. Π§Ρ‚ΠΎΠ±Ρ‹ ΠΏΠΎΠ»ΡƒΡ‡ΠΈΡ‚ΡŒ это, ΠΌΡ‹ просто подставляСм нашС Π·Π½Π°Ρ‡Π΅Π½ΠΈΠ΅ x 1 Π² ΠΏΡ€ΠΎΠΈΠ·Π²ΠΎΠ΄Π½ΡƒΡŽ.

    Π‘Π»Π΅Π΄ΠΎΠ²Π°Ρ‚Π΅Π»ΡŒΠ½ΠΎ, Π½Π°ΠΊΠ»ΠΎΠ½ нашСй ΠΊΠ°ΡΠ°Ρ‚Π΅Π»ΡŒΠ½ΠΎΠΉ Ρ€Π°Π²Π΅Π½Β .

    Π’Π΅ΠΏΠ΅Ρ€ΡŒ Π½Π°ΠΌ Π½ΡƒΠΆΠ½Π° Ρ‚ΠΎΡ‡ΠΊΠ° Π½Π° нашСй ΠΊΠ°ΡΠ°Ρ‚Π΅Π»ΡŒΠ½ΠΎΠΉ. Наш Π²Ρ‹Π±ΠΎΡ€ довольно ΠΎΠ³Ρ€Π°Π½ΠΈΡ‡Π΅Π½, Ρ‚Π°ΠΊ ΠΊΠ°ΠΊ СдинствСнная Ρ‚ΠΎΡ‡ΠΊΠ° Π½Π° ΠΊΠ°ΡΠ°Ρ‚Π΅Π»ΡŒΠ½ΠΎΠΉ, ΠΊΠΎΡ‚ΠΎΡ€ΡƒΡŽ ΠΌΡ‹ Π·Π½Π°Π΅ΠΌ, β€” это Ρ‚ΠΎΡ‡ΠΊΠ°, Π² ΠΊΠΎΡ‚ΠΎΡ€ΠΎΠΉ ΠΎΠ½Π° пСрСсСкаСт наш исходный Π³Ρ€Π°Ρ„ΠΈΠΊ, Π° ΠΈΠΌΠ΅Π½Π½ΠΎ Ρ‚ΠΎΡ‡ΠΊΠ°Β .

    Π’Π°ΠΊΠΈΠΌ ΠΎΠ±Ρ€Π°Π·ΠΎΠΌ, ΠΌΡ‹ ΠΌΠΎΠΆΠ΅ΠΌ ΠΏΠΎΠ΄ΡΡ‚Π°Π²ΠΈΡ‚ΡŒ эти ΠΊΠΎΠΎΡ€Π΄ΠΈΠ½Π°Ρ‚Ρ‹ вмСстС с нашим Π½Π°ΠΊΠ»ΠΎΠ½ΠΎΠΌ Π² ΠΎΠ±Ρ‰ΡƒΡŽ Ρ„ΠΎΡ€ΠΌΡƒ Ρ‚ΠΎΡ‡ΠΊΠ°-Π½Π°ΠΊΠ»ΠΎΠ½, Ρ‡Ρ‚ΠΎΠ±Ρ‹ Π½Π°ΠΉΡ‚ΠΈ ΡƒΡ€Π°Π²Π½Π΅Π½ΠΈΠ΅.

    РСшСниС для даст Π½Π°ΠΌ Ρ„ΠΎΡ€ΠΌΡƒ пСрСсСчСния Π½Π°ΠΊΠ»ΠΎΠ½Π°.

    Β 

    Π‘ΠΎΠΎΠ±Ρ‰ΠΈΡ‚ΡŒ ΠΎΠ± ошибкС

    Найти ΡƒΡ€Π°Π²Π½Π΅Π½ΠΈΠ΅ ΠΊΠ°ΡΠ°Ρ‚Π΅Π»ΡŒΠ½ΠΎΠΉ прямой ΠΊ Ρ„ΡƒΠ½ΠΊΡ†ΠΈΠΈ

    at .

    Π’ΠΎΠ·ΠΌΠΎΠΆΠ½Ρ‹Π΅ ΠΎΡ‚Π²Π΅Ρ‚Ρ‹:

    ΠŸΡ€Π°Π²ΠΈΠ»ΡŒΠ½Ρ‹ΠΉ ΠΎΡ‚Π²Π΅Ρ‚:

    ОбъяснСниС:

    Π£Ρ€Π°Π²Π½Π΅Π½ΠΈΠ΅ ΠΊΠ°ΡΠ°Ρ‚Π΅Π»ΡŒΠ½ΠΎΠΉ Π² Ρ‚ΠΎΡ‡ΠΊΠ΅  зависит ΠΎΡ‚ ΠΏΡ€ΠΎΠΈΠ·Π²ΠΎΠ΄Π½ΠΎΠΉ Π² этой Ρ‚ΠΎΡ‡ΠΊΠ΅ ΠΈ значСния Ρ„ΡƒΠ½ΠΊΡ†ΠΈΠΈ.

    ΠŸΡ€ΠΎΠΈΠ·Π²ΠΎΠ΄Π½Π°Ρ Π² этой Ρ‚ΠΎΡ‡ΠΊΠ΅ Ρ€Π°Π²Π½Π°

     с использованиСм стСпСнного ΠΏΡ€Π°Π²ΠΈΠ»Π°

    , Ρ‡Ρ‚ΠΎ ΠΎΠ·Π½Π°Ρ‡Π°Π΅Ρ‚

    ΠŸΡ€ΠΎΠΈΠ·Π²ΠΎΠ΄Π½Π°Ρ Ρ€Π°Π²Π½Π° Π½ΡƒΠ»ΡŽ, поэтому ΠΊΠ°ΡΠ°Ρ‚Π΅Π»ΡŒΠ½Π°Ρ Π±ΡƒΠ΄Π΅Ρ‚ Π³ΠΎΡ€ΠΈΠ·ΠΎΠ½Ρ‚Π°Π»ΡŒΠ½ΠΎΠΉ.

    Он пСрСсСкаСт Π΅Π΅ Π² Ρ‚ΠΎΡ‡ΠΊΠ΅Β Β sinceΒ , Ρ‚Π°ΠΊ Ρ‡Ρ‚ΠΎ эта линия .

    Π‘ΠΎΠΎΠ±Ρ‰ΠΈΡ‚ΡŒ ΠΎΠ± ошибкС

    По Π·Π°Π΄Π°Π½Π½ΠΎΠΉ Ρ„ΡƒΠ½ΠΊΡ†ΠΈΠΈ Π½Π°ΠΉΠ΄ΠΈΡ‚Π΅ ΡƒΡ€Π°Π²Π½Π΅Π½ΠΈΠ΅ ΠΊΠ°ΡΠ°Ρ‚Π΅Π»ΡŒΠ½ΠΎΠΉ Π² Ρ‚ΠΎΡ‡ΠΊΠ΅ .

    Π’ΠΎΠ·ΠΌΠΎΠΆΠ½Ρ‹Π΅ ΠΎΡ‚Π²Π΅Ρ‚Ρ‹:

    ΠŸΡ€Π°Π²ΠΈΠ»ΡŒΠ½Ρ‹ΠΉ ΠΎΡ‚Π²Π΅Ρ‚:

    ОбъяснСниС:

    ΠŸΠ΅Ρ€Π΅ΠΏΠΈΡˆΠΈΡ‚Π΅Β Β Π² Ρ„ΠΎΡ€ΠΌΠ΅ пСрСсСчСния Π½Π°ΠΊΠ»ΠΎΠ½Π°, ,Β , Ρ‡Ρ‚ΠΎΠ±Ρ‹ ΠΎΠΏΡ€Π΅Π΄Π΅Π»ΠΈΡ‚ΡŒ Π½Π°ΠΊΠ»ΠΎΠ½.

    Наклон Π΄Π°Π½Π½ΠΎΠΉ Ρ„ΡƒΠ½ΠΊΡ†ΠΈΠΈ Ρ€Π°Π²Π΅Π½ 2.

    ΠŸΠΎΠ΄ΡΡ‚Π°Π²ΡŒΡ‚Π΅ Π½Π°ΠΊΠ»ΠΎΠ½ ΠΈ Π·Π°Π΄Π°Π½Π½ΡƒΡŽ Ρ‚ΠΎΡ‡ΠΊΡƒ Π² Ρ„ΠΎΡ€ΠΌΡƒ Π½Π°ΠΊΠ»ΠΎΠ½Π° ΠΈ Ρ‚ΠΎΡ‡ΠΊΠΈ пСрСсСчСния, Ρ‡Ρ‚ΠΎΠ±Ρ‹ ΠΎΠΏΡ€Π΅Π΄Π΅Π»ΠΈΡ‚ΡŒ Ρ‚ΠΎΡ‡ΠΊΡƒ пСрСсСчСния ΠΏΠΎ оси y.

    ΠŸΠΎΠ΄ΡΡ‚Π°Π²ΡŒΡ‚Π΅ это ΠΈ Π½Π°ΠΊΠ»ΠΎΠ½ ΠΎΠ±Ρ€Π°Ρ‚Π½ΠΎ Π² ΡƒΡ€Π°Π²Π½Π΅Π½ΠΈΠ΅ пСрСсСчСния Π½Π°ΠΊΠ»ΠΎΠ½Π°.

    Π£Ρ€Π°Π²Π½Π΅Π½ΠΈΠ΅ ΠΊΠ°ΡΠ°Ρ‚Π΅Π»ΡŒΠ½ΠΎΠΉ: Β 

    Π‘ΠΎΠΎΠ±Ρ‰ΠΈΡ‚ΡŒ ΠΎΠ± ошибкС

    Π˜ΡΠΏΠΎΠ»ΡŒΠ·ΡƒΡ ΠΏΡ€Π΅Π΄Π΅Π»ΡŒΠ½ΠΎΠ΅ ΠΎΠΏΡ€Π΅Π΄Π΅Π»Π΅Π½ΠΈΠ΅ ΠΏΡ€ΠΎΠΈΠ·Π²ΠΎΠ΄Π½ΠΎΠΉ, Π½Π°ΠΉΠ΄ΠΈΡ‚Π΅ ΡƒΡ€Π°Π²Π½Π΅Π½ΠΈΠ΅ ΠΊΠ°ΡΠ°Ρ‚Π΅Π»ΡŒΠ½ΠΎΠΉ ΠΊ ΠΊΡ€ΠΈΠ²ΠΎΠΉ Β Π² Ρ‚ΠΎΡ‡ΠΊΠ΅ .

    Π’ΠΎΠ·ΠΌΠΎΠΆΠ½Ρ‹Π΅ ΠΎΡ‚Π²Π΅Ρ‚Ρ‹:

    ΠŸΡ€Π°Π²ΠΈΠ»ΡŒΠ½Ρ‹ΠΉ ΠΎΡ‚Π²Π΅Ρ‚:

    ОбъяснСниС:

    НачнСм с нахоТдСния уравнСния ΠΏΡ€ΠΎΠΈΠ·Π²ΠΎΠ΄Π½ΠΎΠΉ, ΠΈΡΠΏΠΎΠ»ΡŒΠ·ΡƒΡ ΠΏΡ€Π΅Π΄Π΅Π»ΡŒΠ½ΠΎΠ΅ ΠΎΠΏΡ€Π΅Π΄Π΅Π»Π΅Π½ΠΈΠ΅:

    ΠžΠΏΡ€Π΅Π΄Π΅Π»ΠΈΠΌ Β ΠΈ Β ΡΠ»Π΅Π΄ΡƒΡŽΡ‰ΠΈΠΌ ΠΎΠ±Ρ€Π°Π·ΠΎΠΌ:

    Π—Π°Ρ‚Π΅ΠΌ ΠΌΡ‹ ΠΌΠΎΠΆΠ΅ΠΌ ΠΎΠΏΡ€Π΅Π΄Π΅Π»ΠΈΡ‚ΡŒ ΠΈΡ… Ρ€Π°Π·Π½ΠΎΡΡ‚ΡŒ:

    Π—Π°Ρ‚Π΅ΠΌ ΠΌΡ‹ Π΄Π΅Π»ΠΈΠΌ Π½Π° h, Ρ‡Ρ‚ΠΎΠ±Ρ‹ ΠΏΠΎΠ΄Π³ΠΎΡ‚ΠΎΠ²ΠΈΡ‚ΡŒΡΡ ΠΊ ΠΏΡ€Π΅Π΄Π΅Π»Ρƒ:

    Π’ΠΎΠ³Π΄Π° ΡƒΡ€Π°Π²Π½Π΅Π½ΠΈΠ΅ ΠΏΡ€Π΅Π΄Π΅Π»Π° даст Π½Π°ΠΌ ΠΏΡ€Π΅Π΄Π΅Π», производная .

    Π’Π΅ΠΏΠ΅Ρ€ΡŒ ΠΌΡ‹ Π΄ΠΎΠ»ΠΆΠ½Ρ‹ ΠΏΠΎΠ½ΡΡ‚ΡŒ, Ρ‡Ρ‚ΠΎ Π½Π°ΠΊΠ»ΠΎΠ½ Π»ΠΈΠ½ΠΈΠΈ, ΠΊΠ°ΡΠ°Ρ‚Π΅Π»ΡŒΠ½ΠΎΠΉ ΠΊ ΠΊΡ€ΠΈΠ²ΠΎΠΉ Π² Π΄Π°Π½Π½ΠΎΠΉ Ρ‚ΠΎΡ‡ΠΊΠ΅, эквивалСнтСн ΠΏΡ€ΠΎΠΈΠ·Π²ΠΎΠ΄Π½ΠΎΠΉ Π² этой Ρ‚ΠΎΡ‡ΠΊΠ΅. Π˜Ρ‚Π°ΠΊ, Ссли ΠΌΡ‹ ΠΎΠΏΡ€Π΅Π΄Π΅Π»ΠΈΠΌ Π½Π°ΡˆΡƒ ΠΊΠ°ΡΠ°Ρ‚Π΅Π»ΡŒΠ½ΡƒΡŽ линию ΠΊΠ°ΠΊ: , Ρ‚ΠΎ это m опрСдСляСтся ΡΠ»Π΅Π΄ΡƒΡŽΡ‰ΠΈΠΌ ΠΎΠ±Ρ€Π°Π·ΠΎΠΌ:

    Π‘Π»Π΅Π΄ΠΎΠ²Π°Ρ‚Π΅Π»ΡŒΠ½ΠΎ, ΡƒΡ€Π°Π²Π½Π΅Π½ΠΈΠ΅ ΠΊΠ°ΡΠ°Ρ‚Π΅Π»ΡŒΠ½ΠΎΠΉ ΠΊ ΠΊΡ€ΠΈΠ²ΠΎΠΉ Π² Π΄Π°Π½Π½ΠΎΠΉ Ρ‚ΠΎΡ‡ΠΊΠ΅:

    Π‘ΠΎΠΎΠ±Ρ‰ΠΈΡ‚ΡŒ ΠΎΠ± ошибкС

    ΠΠ°ΠΏΠΈΡˆΠΈΡ‚Π΅ ΡƒΡ€Π°Π²Π½Π΅Π½ΠΈΠ΅ ΠΊΠ°ΡΠ°Ρ‚Π΅Π»ΡŒΠ½ΠΎΠΉ ΠΊ ΠΊΡ€ΠΈΠ²ΠΎΠΉ Π² Ρ‚ΠΎΡ‡ΠΊΠ΅ .

    Π’ΠΎΠ·ΠΌΠΎΠΆΠ½Ρ‹Π΅ ΠΎΡ‚Π²Π΅Ρ‚Ρ‹:

    ΠŸΡ€Π°Π²ΠΈΠ»ΡŒΠ½Ρ‹ΠΉ ΠΎΡ‚Π²Π΅Ρ‚:

    ОбъяснСниС:

    Π‘Π½Π°Ρ‡Π°Π»Π° Π½Π°ΠΉΠ΄ΠΈΡ‚Π΅ Π½Π°ΠΊΠ»ΠΎΠ½ этой ΠΊΠ°ΡΠ°Ρ‚Π΅Π»ΡŒΠ½ΠΎΠΉ, взяв ΠΏΡ€ΠΎΠΈΠ·Π²ΠΎΠ΄Π½ΡƒΡŽ:

    ΠŸΠΎΠ΄ΡΡ‚Π°Π²ΠΈΠ² 1 вмСсто x:

    Π˜Ρ‚Π°ΠΊ, Π½Π°ΠΊΠ»ΠΎΠ½ Ρ€Π°Π²Π΅Π½ 4

    Π’Π΅ΠΏΠ΅Ρ€ΡŒ Π½Π°ΠΌ Π½ΡƒΠΆΠ½ΠΎ Π½Π°ΠΉΡ‚ΠΈ ΠΊΠΎΠΎΡ€Π΄ΠΈΠ½Π°Ρ‚Ρƒ y, ΠΊΠΎΠ³Π΄Π° x Ρ€Π°Π²Π΅Π½ 1, поэтому ΠΏΠΎΠ΄ΡΡ‚Π°Π²ΡŒΡ‚Π΅ 1 ΠΊ исходному ΡƒΡ€Π°Π²Π½Π΅Π½ΠΈΡŽ:

    Π²Π°Ρ€ΠΈΠ°Π½Ρ‚Ρ‹ ΠΎΡ‚Π²Π΅Ρ‚Π°:

    распрСдСлитС 4

    Π΄ΠΎΠ±Π°Π²ΡŒΡ‚Π΅ 2 ΠΊ ΠΎΠ±Π΅ΠΈΠΌ сторонам

    Π‘ΠΎΠΎΠ±Ρ‰ΠΈΡ‚Π΅ ΠΎΠ± ошибкС

    ΠΠ°ΠΏΠΈΡˆΠΈΡ‚Π΅ ΡƒΡ€Π°Π²Π½Π΅Π½ΠΈΠ΅ для ΠΊΠ°ΡΠ°Ρ‚Π΅Π»ΡŒΠ½ΠΎΠΉ ΠΊ Ρ‚ΠΎΡ‡ΠΊΠ΅ .

    Π’ΠΎΠ·ΠΌΠΎΠΆΠ½Ρ‹Π΅ ΠΎΡ‚Π²Π΅Ρ‚Ρ‹:

    ΠŸΡ€Π°Π²ΠΈΠ»ΡŒΠ½Ρ‹ΠΉ ΠΎΡ‚Π²Π΅Ρ‚:

    ОбъяснСниС:

    Π‘Π½Π°Ρ‡Π°Π»Π° Π½Π°ΠΉΠ΄ΠΈΡ‚Π΅ Π½Π°ΠΊΠ»ΠΎΠ½ ΠΊΠ°ΡΠ°Ρ‚Π΅Π»ΡŒΠ½ΠΎΠΉ, взяв ΠΏΠ΅Ρ€Π²ΡƒΡŽ ΠΏΡ€ΠΎΠΈΠ·Π²ΠΎΠ΄Π½ΡƒΡŽ:

    Π§Ρ‚ΠΎΠ±Ρ‹ Π·Π°ΠΊΠΎΠ½Ρ‡ΠΈΡ‚ΡŒ ΠΎΠΏΡ€Π΅Π΄Π΅Π»Π΅Π½ΠΈΠ΅ Π½Π°ΠΊΠ»ΠΎΠ½Π°, ΠΏΠΎΠ΄ΡΡ‚Π°Π²ΡŒΡ‚Π΅ Π·Π½Π°Ρ‡Π΅Π½ΠΈΠ΅ x, 2:

    , Π½Π°ΠΊΠ»ΠΎΠ½ Ρ€Π°Π²Π΅Π½ 6

    Π’Π΅ΠΏΠ΅Ρ€ΡŒ Π½Π°ΠΉΠ΄ΠΈΡ‚Π΅ y -ΠΊΠΎΠΎΡ€Π΄ΠΈΠ½Π°Ρ‚Π°, Π³Π΄Π΅ x Ρ€Π°Π²Π½ΠΎ 2, подставив 2 ΠΊ исходному ΡƒΡ€Π°Π²Π½Π΅Π½ΠΈΡŽ:

    Π§Ρ‚ΠΎΠ±Ρ‹ Π½Π°ΠΏΠΈΡΠ°Ρ‚ΡŒ ΡƒΡ€Π°Π²Π½Π΅Π½ΠΈΠ΅, Π½Π°Ρ‡Π½ΠΈΡ‚Π΅ Π² Ρ„ΠΎΡ€ΠΌΠ΅ Ρ‚ΠΎΡ‡ΠΊΠΈ-Π½Π°ΠΊΠ»ΠΎΠ½Π°, Π° Π·Π°Ρ‚Π΅ΠΌ ΠΈΡΠΏΠΎΠ»ΡŒΠ·ΡƒΠΉΡ‚Π΅ Π°Π»Π³Π΅Π±Ρ€Ρƒ, Ρ‡Ρ‚ΠΎΠ±Ρ‹ ΠΏΡ€Π΅ΠΎΠ±Ρ€Π°Π·ΠΎΠ²Π°Ρ‚ΡŒ Π΅Π³ΠΎ Π² Π½Π°ΠΊΠ»ΠΎΠ½-пСрСсСчСниС, ΠΊΠ°ΠΊ Π²Π°Ρ€ΠΈΠ°Π½Ρ‚Ρ‹ ΠΎΡ‚Π²Π΅Ρ‚Π°.

    распрСдСлитС 6

    ΠΏΡ€ΠΈΠ±Π°Π²ΡŒΡ‚Π΅ 8 ΠΊ ΠΎΠ±Π΅ΠΈΠΌ сторонам

    Π‘ΠΎΠΎΠ±Ρ‰ΠΈΡ‚Π΅ ΠΎΠ± ошибкС

    ΠΠ°ΠΏΠΈΡˆΠΈΡ‚Π΅ ΡƒΡ€Π°Π²Π½Π΅Π½ΠΈΠ΅ ΠΊΠ°ΡΠ°Ρ‚Π΅Π»ΡŒΠ½ΠΎΠΉ для Ρ‚ΠΎΡ‡ΠΊΠΈ .

    Π’ΠΎΠ·ΠΌΠΎΠΆΠ½Ρ‹Π΅ ΠΎΡ‚Π²Π΅Ρ‚Ρ‹:

    ΠŸΡ€Π°Π²ΠΈΠ»ΡŒΠ½Ρ‹ΠΉ ΠΎΡ‚Π²Π΅Ρ‚:

    ОбъяснСниС:

    Π‘Π½Π°Ρ‡Π°Π»Π° Π²ΠΎΠ·ΡŒΠΌΠΈΡ‚Π΅ ΠΏΠ΅Ρ€Π²ΡƒΡŽ ΠΏΡ€ΠΎΠΈΠ·Π²ΠΎΠ΄Π½ΡƒΡŽ, Ρ‡Ρ‚ΠΎΠ±Ρ‹ Π½Π°ΠΉΡ‚ΠΈ Π½Π°ΠΊΠ»ΠΎΠ½:

    Β 

    Π§Ρ‚ΠΎΠ±Ρ‹ ΠΏΡ€ΠΎΠ΄ΠΎΠ»ΠΆΠΈΡ‚ΡŒ поиск Π½Π°ΠΊΠ»ΠΎΠ½Π°, ΠΏΠΎΠ΄ΡΡ‚Π°Π²ΡŒΡ‚Π΅ Π·Π½Π°Ρ‡Π΅Π½ΠΈΠ΅ x, -2:

    Π—Π°Ρ‚Π΅ΠΌ Π½Π°ΠΉΠ΄ΠΈΡ‚Π΅ ΠΊΠΎΠΎΡ€Π΄ΠΈΠ½Π°Ρ‚Ρƒ y, подставив — 2 Π² исходноС ΡƒΡ€Π°Π²Π½Π΅Π½ΠΈΠ΅:

    ΠšΠΎΠΎΡ€Π΄ΠΈΠ½Π°Ρ‚Π° y:

    Π’Π΅ΠΏΠ΅Ρ€ΡŒ Π·Π°ΠΏΠΈΡˆΠΈΡ‚Π΅ ΡƒΡ€Π°Π²Π½Π΅Π½ΠΈΠ΅ Π² Ρ„ΠΎΡ€ΠΌΠ΅ Ρ‚ΠΎΡ‡ΠΊΠΈ-Π½Π°ΠΊΠ»ΠΎΠ½Π°, Π° Π·Π°Ρ‚Π΅ΠΌ алгСбраичСски ΠΎΠ±Ρ€Π°Π±ΠΎΡ‚Π°ΠΉΡ‚Π΅ Π΅Π³ΠΎ, Ρ‡Ρ‚ΠΎΠ±Ρ‹ ΠΎΠ½ΠΎ соотвСтствовало ΠΎΠ΄Π½ΠΎΠΉ ΠΈΠ· Ρ„ΠΎΡ€ΠΌ пСрСсСчСния Π½Π°ΠΊΠ»ΠΎΠ½Π° Π²Π°Ρ€ΠΈΠ°Π½Ρ‚ΠΎΠ² ΠΎΡ‚Π²Π΅Ρ‚Π°.

    Ρ€Π°ΡΠΏΡ€Π΅Π΄Π΅Π»ΠΈΡ‚ΡŒ -5

    Π΄ΠΎΠ±Π°Π²ΠΈΡ‚ΡŒ Π² ΠΎΠ±Π΅ стороны

    Β 

    Β 

    Π‘ΠΎΠΎΠ±Ρ‰ΠΈΡ‚ΡŒ ΠΎΠ± ошибкС

    ΠΠ°ΠΏΠΈΡˆΠΈΡ‚Π΅ ΡƒΡ€Π°Π²Π½Π΅Π½ΠΈΠ΅ для ΠΊΠ°ΡΠ°Ρ‚Π΅Π»ΡŒΠ½ΠΎΠΉ ΠΊ Ρ‚ΠΎΡ‡ΠΊΠ΅ .

    Π’ΠΎΠ·ΠΌΠΎΠΆΠ½Ρ‹Π΅ ΠΎΡ‚Π²Π΅Ρ‚Ρ‹:

    ΠŸΡ€Π°Π²ΠΈΠ»ΡŒΠ½Ρ‹ΠΉ ΠΎΡ‚Π²Π΅Ρ‚:

    ОбъяснСниС:

    Π‘Π½Π°Ρ‡Π°Π»Π° распространитС Ρ„Π°ΠΉΠ» . Π­Ρ‚ΠΎ ΠΎΠ±Π»Π΅Π³Ρ‡ΠΈΡ‚ ΠΏΠΎΠ»ΡƒΡ‡Π΅Π½ΠΈΠ΅ ΠΏΡ€ΠΎΠΈΠ·Π²ΠΎΠ΄Π½ΠΎΠΉ:

    Π’Π΅ΠΏΠ΅Ρ€ΡŒ Π²ΠΎΠ·ΡŒΠΌΠΈΡ‚Π΅ ΠΏΡ€ΠΎΠΈΠ·Π²ΠΎΠ΄Π½ΡƒΡŽ уравнСния:

    Π§Ρ‚ΠΎΠ±Ρ‹ Π½Π°ΠΉΡ‚ΠΈ Π½Π°ΠΊΠ»ΠΎΠ½, ΠΏΠΎΠ΄ΡΡ‚Π°Π²ΡŒΡ‚Π΅ Π·Π½Π°Ρ‡Π΅Π½ΠΈΠ΅ x -3:

    Π§Ρ‚ΠΎΠ±Ρ‹ Π½Π°ΠΉΡ‚ΠΈ ΠΊΠΎΠΎΡ€Π΄ΠΈΠ½Π°Ρ‚Ρƒ y Ρ‚ΠΎΡ‡ΠΊΠΈ, ΠΏΠΎΠ΄ΡΡ‚Π°Π²ΡŒΡ‚Π΅ Π·Π½Π°Ρ‡Π΅Π½ΠΈΠ΅ x Π² исходноС ΡƒΡ€Π°Π²Π½Π΅Π½ΠΈΠ΅:

    Π’Π΅ΠΏΠ΅Ρ€ΡŒ Π½Π°ΠΏΠΈΡˆΠΈΡ‚Π΅ ΡƒΡ€Π°Π²Π½Π΅Π½ΠΈΠ΅ Π² Π²ΠΈΠ΄Π΅ Ρ‚ΠΎΡ‡ΠΊΠ°-Π½Π°ΠΊΠ»ΠΎΠ½, Π·Π°Ρ‚Π΅ΠΌ ΠΈΡΠΏΠΎΠ»ΡŒΠ·ΡƒΠΉΡ‚Π΅ Π°Π»Π³Π΅Π±Ρ€Ρƒ, Ρ‡Ρ‚ΠΎΠ±Ρ‹ ΠΏΡ€Π΅ΠΎΠ±Ρ€Π°Π·ΠΎΠ²Π°Ρ‚ΡŒ Π΅Π³ΠΎ Π² Π½Π°ΠΊΠ»ΠΎΠ½-пСрСсСчСниС, ΠΊΠ°ΠΊ Π²Π°Ρ€ΠΈΠ°Π½Ρ‚Ρ‹ ΠΎΡ‚Π²Π΅Ρ‚Π°: 9

    Β 

    Π‘ΠΎΠΎΠ±Ρ‰ΠΈΡ‚ΡŒ ΠΎΠ± ошибкС

    Π£Π²Π΅Π΄ΠΎΠΌΠ»Π΅Π½ΠΈΠ΅ ΠΎΠ± авторских ΠΏΡ€Π°Π²Π°Ρ… 380 практичСских тСстов Вопрос дня ΠšΠ°Ρ€Ρ‚ΠΎΡ‡ΠΊΠΈ Learn by Concept

    Π’ΠΈΠ΄Π΅ΠΎ с вопросами: Π½Π°Ρ…ΠΎΠΆΠ΄Π΅Π½ΠΈΠ΅ Ρ‚ΠΎΡ‡Π΅ΠΊ Π½Π° ΠΊΡ€ΠΈΠ²ΠΎΠΉ кубичСской Ρ„ΡƒΠ½ΠΊΡ†ΠΈΠΈ, Π² ΠΊΠΎΡ‚ΠΎΡ€Ρ‹Ρ… ΠΊΠ°ΡΠ°Ρ‚Π΅Π»ΡŒΠ½Π°Ρ ΠΊ ΠΊΡ€ΠΈΠ²ΠΎΠΉ ΠΏΠ°Ρ€Π°Π»Π»Π΅Π»ΡŒΠ½Π° Π·Π°Π΄Π°Π½Π½ΠΎΠΉ прямой

    Π‘Ρ‚Π΅Π½ΠΎΠ³Ρ€Π°ΠΌΠΌΠ° Π²ΠΈΠ΄Π΅ΠΎ

    НайдитС Ρ‚ΠΎΡ‡ΠΊΠΈ Π½Π° ΠΊΡ€ΠΈΠ²ΠΎΠΉ 𝑦 Ρ€Π°Π²Π½ΠΎ Ρ‚Ρ€Π΅ΠΌ π‘₯ Π² ΠΊΡƒΠ±Π΅ минус ΠΏΡΡ‚ΡŒ π‘₯ плюс сСмь, Π² ΠΊΠΎΡ‚ΠΎΡ€Ρ‹Ρ… ΠΊΠ°ΡΠ°Ρ‚Π΅Π»ΡŒΠ½Ρ‹Π΅ ΠΏΠ°Ρ€Π°Π»Π»Π΅Π»ΡŒΠ½Ρ‹ прямой Ρ‡Π΅Ρ‚Ρ‹Ρ€Π΅ π‘₯ плюс 𝑦 минус Π΄Π²Π° Ρ€Π°Π²Π½ΠΎ Π½ΡƒΠ»ΡŽ.

    Π’ΠΎ-ΠΏΠ΅Ρ€Π²Ρ‹Ρ…, Π½Π°ΠΏΠΎΠΌΠ½ΠΈΠΌ, Ρ‡Ρ‚ΠΎ Π΄Π²Π΅ прямыС ΠΏΠ°Ρ€Π°Π»Π»Π΅Π»ΡŒΠ½Ρ‹, Ссли ΠΎΠ½ΠΈ ΠΈΠΌΠ΅ΡŽΡ‚ ΠΎΠ΄ΠΈΠ½Π°ΠΊΠΎΠ²Ρ‹ΠΉ Π½Π°ΠΊΠ»ΠΎΠ½. ΠœΡ‹ ΠΌΠΎΠΆΠ΅ΠΌ Π²Ρ‹Ρ‡ΠΈΡΠ»ΠΈΡ‚ΡŒ Π½Π°ΠΊΠ»ΠΎΠ½ Π»ΠΈΠ½ΠΈΠΈ Ρ‡Π΅Ρ‚Ρ‹Ρ€Π΅ π‘₯ плюс 𝑦 минус Π΄Π²Π° Ρ€Π°Π²Π½ΠΎ Π½ΡƒΠ»ΡŽ, сначала ΠΏΡ€Π΅ΠΎΠ±Ρ€Π°Π·ΠΎΠ²Π°Π² Π΅Π΅ ΡƒΡ€Π°Π²Π½Π΅Π½ΠΈΠ΅ Π² Ρ„ΠΎΡ€ΠΌΡƒ Π½Π°ΠΊΠ»ΠΎΠ½Π° ΠΈ Ρ‚ΠΎΡ‡ΠΊΠΈ пСрСсСчСния. ΠŸΠΎΠΌΠ½ΠΈΡ‚Π΅, Ρ‡Ρ‚ΠΎ Ρ„ΠΎΡ€ΠΌΠ° уравнСния Π½Π°ΠΊΠ»ΠΎΠ½Π° ΠΈ пСрСсСчСния прямой Π»ΠΈΠ½ΠΈΠΈ: 𝑦 Ρ€Π°Π²Π½ΠΎ π‘šπ‘₯ плюс 𝑏, Π³Π΄Π΅ π‘š, коэффициСнт ΠΏΡ€ΠΈ π‘₯, прСдставляСт собой Π½Π°ΠΊΠ»ΠΎΠ½ Π»ΠΈΠ½ΠΈΠΈ, Π° 𝑏, постоянный Ρ‡Π»Π΅Π½, прСдставляСт 𝑦-ΠΎΡ‚Ρ€Π΅Π·ΠΎΠΊ. Π§Ρ‚ΠΎΠ±Ρ‹ ΠΏΡ€Π΅ΠΎΠ±Ρ€Π°Π·ΠΎΠ²Π°Ρ‚ΡŒ ΡƒΡ€Π°Π²Π½Π΅Π½ΠΈΠ΅ этой Π»ΠΈΠ½ΠΈΠΈ Π² Ρ„ΠΎΡ€ΠΌΡƒ Π½Π°ΠΊΠ»ΠΎΠ½Π° ΠΈ пСрСсСчСния, Π½Π°ΠΌ Π½ΡƒΠΆΠ½ΠΎ Π²Ρ‹Ρ‡Π΅ΡΡ‚ΡŒ Ρ‡Π΅Ρ‚Ρ‹Ρ€Π΅ π‘₯ с ΠΊΠ°ΠΆΠ΄ΠΎΠΉ стороны, Π° Ρ‚Π°ΠΊΠΆΠ΅ Π΄ΠΎΠ±Π°Π²ΠΈΡ‚ΡŒ Π΄Π²Π° ΠΊ ΠΊΠ°ΠΆΠ΄ΠΎΠΉ сторонС уравнСния. Π­Ρ‚ΠΎ Π΄Π°Π΅Ρ‚ 𝑦 Ρ€Π°Π²Π½ΠΎ минус Ρ‡Π΅Ρ‚Ρ‹Ρ€Π΅ΠΌ π‘₯ плюс Π΄Π²Π°. Π’Π΅ΠΏΠ΅Ρ€ΡŒ ΠΌΡ‹ ΠΌΠΎΠΆΠ΅ΠΌ ΡΡ€Π°Π²Π½ΠΈΡ‚ΡŒ это с Ρ„ΠΎΡ€ΠΌΠΎΠΉ уравнСния прямой Π»ΠΈΠ½ΠΈΠΈ с Π½Π°ΠΊΠ»ΠΎΠ½ΠΎΠΌ ΠΈ Ρ‚ΠΎΡ‡ΠΊΠΎΠΉ пСрСсСчСния, ΠΈ ΠΌΡ‹ Π²ΠΈΠ΄ΠΈΠΌ, Ρ‡Ρ‚ΠΎ Π½Π°ΠΊΠ»ΠΎΠ½ этой Π»ΠΈΠ½ΠΈΠΈ, коэффициСнт ΠΏΡ€ΠΈ π‘₯, Ρ€Π°Π²Π΅Π½ минус Ρ‡Π΅Ρ‚Ρ‹Ρ€Π΅ΠΌ.

    Π’Π΅ΠΏΠ΅Ρ€ΡŒ ΠΌΡ‹ Π·Π½Π°Π΅ΠΌ, Ρ‡Ρ‚ΠΎ ΠΊΠ°ΡΠ°Ρ‚Π΅Π»ΡŒΠ½Ρ‹Π΅, ΠΊΠΎΡ‚ΠΎΡ€Ρ‹Π΅ ΠΌΡ‹ ΠΈΡ‰Π΅ΠΌ ΠΊ этой ΠΊΡ€ΠΈΠ²ΠΎΠΉ 𝑦, Ρ€Π°Π²Π½Ρ‹ Ρ‚Ρ€Π΅ΠΌ π‘₯ Π² ΠΊΡƒΠ±Π΅ минус ΠΏΡΡ‚ΡŒ π‘₯ плюс сСмь, Ρ‚Π°ΠΊΠΆΠ΅ Π΄ΠΎΠ»ΠΆΠ½Ρ‹ ΠΈΠΌΠ΅Ρ‚ΡŒ ΠΎΡ‚Ρ€ΠΈΡ†Π°Ρ‚Π΅Π»ΡŒΠ½Ρ‹ΠΉ Π½Π°ΠΊΠ»ΠΎΠ½ Ρ‡Π΅Ρ‚Ρ‹Ρ€Π΅, Ссли ΠΎΠ½ΠΈ Π΄ΠΎΠ»ΠΆΠ½Ρ‹ Π±Ρ‹Ρ‚ΡŒ ΠΏΠ°Ρ€Π°Π»Π»Π΅Π»ΡŒΠ½Ρ‹ прямой Ρ‡Π΅Ρ‚Ρ‹Ρ€Π΅ π‘₯ плюс 𝑦 минус Π΄Π²Π° Ρ€Π°Π²Π΅Π½ Π½ΡƒΠ»ΡŽ. Наклон ΠΊΡ€ΠΈΠ²ΠΎΠΉ задаСтся Π΅Π΅ ΠΏΠ΅Ρ€Π²ΠΎΠΉ ΠΏΡ€ΠΎΠΈΠ·Π²ΠΎΠ΄Π½ΠΎΠΉ ΠΈΠ»ΠΈ Ρ„ΡƒΠ½ΠΊΡ†ΠΈΠ΅ΠΉ Π½Π°ΠΊΠ»ΠΎΠ½Π°, ΠΊΠΎΡ‚ΠΎΡ€ΡƒΡŽ ΠΌΡ‹ Π½Π°Π·Ρ‹Π²Π°Π΅ΠΌ d𝑦 Ρ‡Π΅Ρ€Π΅Π· dπ‘₯, ΠΈ Π΅Π΅ ΠΌΠΎΠΆΠ½ΠΎ Π½Π°ΠΉΡ‚ΠΈ ΠΏΡƒΡ‚Π΅ΠΌ диффСрСнцирования уравнСния ΠΊΡ€ΠΈΠ²ΠΎΠΉ ΠΏΠΎ π‘₯. Наша функция 𝑦 являСтся ΠΏΠΎΠ»ΠΈΠ½ΠΎΠΌΠΎΠΌ ΠΎΡ‚ π‘₯. Π˜Ρ‚Π°ΠΊ, ΠΌΡ‹ вспоминаСм ΠΎΠ±Ρ‰Π΅Π΅ стСпСнноС ΠΏΡ€Π°Π²ΠΈΠ»ΠΎ диффСрСнцирования, ΠΊΠΎΡ‚ΠΎΡ€ΠΎΠ΅ Π³ΠΎΠ²ΠΎΡ€ΠΈΡ‚ Π½Π°ΠΌ, Ρ‡Ρ‚ΠΎ производная ΠΏΠΎ π‘₯ ΠΎΡ‚ π‘Žπ‘₯ Π² 𝑛-ΠΉ стСпСни, Π³Π΄Π΅ π‘Ž ΠΈ 𝑛 β€” вСщСствСнныС константы, Ρ€Π°Π²Π½Π° π‘Ž, ΡƒΠΌΠ½ΠΎΠΆΠ΅Π½Π½ΠΎΠΌΡƒ Π½Π° 𝑛, ΡƒΠΌΠ½ΠΎΠΆΠ΅Π½Π½ΠΎΠΌΡƒ Π½Π° π‘₯ Π² стСпСни 𝑛 минус ΠΎΠ΄ΠΈΠ½. Π£ΠΌΠ½ΠΎΠΆΠ°Π΅ΠΌ Π½Π° ΠΌΠΎΡ‰Π½ΠΎΡΡ‚ΡŒ ΠΈ Π·Π°Ρ‚Π΅ΠΌ ΡƒΠΌΠ΅Π½ΡŒΡˆΠ°Π΅ΠΌ ΠΌΠΎΡ‰Π½ΠΎΡΡ‚ΡŒ Π½Π° Π΅Π΄ΠΈΠ½ΠΈΡ†Ρƒ.

    Π˜Ρ‚Π°ΠΊ, Π΄Π°Π²Π°ΠΉΡ‚Π΅ Ρ‚Π΅ΠΏΠ΅Ρ€ΡŒ ΠΏΡ€ΠΈΠΌΠ΅Π½ΠΈΠΌ это ΠΏΡ€Π°Π²ΠΈΠ»ΠΎ, Ρ‡Ρ‚ΠΎΠ±Ρ‹ Π½Π°ΠΉΡ‚ΠΈ Π²Ρ‹Ρ€Π°ΠΆΠ΅Π½ΠΈΠ΅ для d𝑦 с ΠΏΠΎΠΌΠΎΡ‰ΡŒΡŽ dπ‘₯. ΠŸΠ΅Ρ€Π²Ρ‹ΠΉ Ρ‡Π»Π΅Π½ Π² 𝑦 Ρ€Π°Π²Π΅Π½ Ρ‚Ρ€Π΅ΠΌ π‘₯ Π² ΠΊΡƒΠ±Π΅. Π˜Ρ‚Π°ΠΊ, Π΄ΠΈΡ„Ρ„Π΅Ρ€Π΅Π½Ρ†ΠΈΡ€ΡƒΠ΅ΠΌ, ΠΈΠΌΠ΅Π΅ΠΌ Ρ‚Ρ€ΠΈ ΡƒΠΌΠ½ΠΎΠΆΠΈΡ‚ΡŒ Π½Π° Ρ‚Ρ€ΠΈ. Π—Π°Ρ‚Π΅ΠΌ ΠΌΡ‹ ΡƒΠΌΠ΅Π½ΡŒΡˆΠ°Π΅ΠΌ ΡΡ‚Π΅ΠΏΠ΅Π½ΡŒ π‘₯ Π½Π° Π΅Π΄ΠΈΠ½ΠΈΡ†Ρƒ, давая π‘₯ Π² ΠΊΠ²Π°Π΄Ρ€Π°Ρ‚Π΅. Π˜Ρ‚Π°ΠΊ, Ρƒ нас Π΅ΡΡ‚ΡŒ Ρ‚Ρ€ΠΈ ΡƒΠΌΠ½ΠΎΠΆΠΈΡ‚ΡŒ Π½Π° Ρ‚Ρ€ΠΈ π‘₯ Π² ΠΊΠ²Π°Π΄Ρ€Π°Ρ‚Π΅. Π§Ρ‚ΠΎΠ±Ρ‹ Ρ€Π°Π·Π»ΠΈΡ‡Π°Ρ‚ΡŒ Ρ‚Π΅Ρ€ΠΌΠΈΠ½ ΠΎΡ‚Ρ€ΠΈΡ†Π°Ρ‚Π΅Π»ΡŒΠ½Ρ‹Π΅ ΠΏΡΡ‚ΡŒ π‘₯, ΠΌΡ‹ ΠΌΠΎΠΆΠ΅ΠΌ Π΄ΡƒΠΌΠ°Ρ‚ΡŒ ΠΎΠ± этом ΠΊΠ°ΠΊ ΠΎ ΠΎΡ‚Ρ€ΠΈΡ†Π°Ρ‚Π΅Π»ΡŒΠ½Ρ‹Ρ… пяти π‘₯ Π² ΠΏΠ΅Ρ€Π²ΠΎΠΉ стСпСни. Π’Π°ΠΊΠΈΠΌ ΠΎΠ±Ρ€Π°Π·ΠΎΠΌ, примСняя стСпСнноС ΠΏΡ€Π°Π²ΠΈΠ»ΠΎ диффСрСнцирования, ΠΌΡ‹ ΠΈΠΌΠ΅Π΅ΠΌ ΠΎΡ‚Ρ€ΠΈΡ†Π°Ρ‚Π΅Π»ΡŒΠ½Ρ‹Π΅ ΠΏΡΡ‚ΡŒ, ΡƒΠΌΠ½ΠΎΠΆΠ΅Π½Π½Ρ‹Π΅ Π½Π° Π΅Π΄ΠΈΠ½ΠΈΡ†Ρƒ, ΡƒΠΌΠ½ΠΎΠΆΠ΅Π½Π½Ρ‹Π΅ Π½Π° π‘₯ Π² Π½ΡƒΠ»Π΅Π²ΠΎΠΉ стСпСни. Но Ссли Π²ΡΠΏΠΎΠΌΠ½ΠΈΡ‚ΡŒ, Ρ‡Ρ‚ΠΎ π‘₯ Π² Π½ΡƒΠ»Π΅Π²ΠΎΠΉ стСпСни Ρ€Π°Π²Π½ΠΎ Π΅Π΄ΠΈΠ½ΠΈΡ†Π΅, Ρ‚ΠΎ эту Ρ‡Π°ΡΡ‚ΡŒ Π·Π°ΠΏΠΈΡΡ‹Π²Π°Ρ‚ΡŒ Π½Π΅ Π½ΡƒΠΆΠ½ΠΎ.

    НаконСц, Π΄ΠΈΡ„Ρ„Π΅Ρ€Π΅Π½Ρ†ΠΈΡ€ΡƒΠ΅ΠΌ ΠΏΠΎΠ»ΠΎΠΆΠΈΡ‚Π΅Π»ΡŒΠ½ΡƒΡŽ сСмСрку ΠΈ вспоминаСм, Ρ‡Ρ‚ΠΎ производная константы ΠΏΠΎ π‘₯ Ρ€Π°Π²Π½Π° Π½ΡƒΠ»ΡŽ. Π§Ρ‚ΠΎΠ±Ρ‹ ΠΏΠΎΠ½ΡΡ‚ΡŒ, ΠΏΠΎΡ‡Π΅ΠΌΡƒ это Ρ‚Π°ΠΊ, ΠΌΡ‹ ΠΌΠΎΠΆΠ΅ΠΌ Π΄ΡƒΠΌΠ°Ρ‚ΡŒ ΠΎ константС сСмь ΠΊΠ°ΠΊ ΠΎ сСми, ΡƒΠΌΠ½ΠΎΠΆΠ΅Π½Π½Ρ‹Ρ… Π½Π° π‘₯ Π² стСпСни нуля. Как ΠΌΡ‹ ΡƒΠΆΠ΅ Π³ΠΎΠ²ΠΎΡ€ΠΈΠ»ΠΈ, π‘₯ Π² Π½ΡƒΠ»Π΅Π²ΠΎΠΉ стСпСни Ρ€Π°Π²Π½ΠΎ Π΅Π΄ΠΈΠ½ΠΈΡ†Π΅. Если Π±Ρ‹ ΠΌΡ‹ Π·Π°Ρ‚Π΅ΠΌ ΠΏΡ€ΠΈΠΌΠ΅Π½ΠΈΠ»ΠΈ ΠΎΠ±Ρ‰Π΅Π΅ ΠΏΡ€Π°Π²ΠΈΠ»ΠΎ стСпСни, Ρƒ нас Π±Ρ‹Π»ΠΎ Π±Ρ‹ сСмь, ΡƒΠΌΠ½ΠΎΠΆΠ΅Π½Π½ΠΎΠ΅ Π½Π° ноль, ΡƒΠΌΠ½ΠΎΠΆΠ΅Π½Π½ΠΎΠ΅ Π½Π° π‘₯ Π² ΠΎΡ‚Ρ€ΠΈΡ†Π°Ρ‚Π΅Π»ΡŒΠ½ΠΎΠΉ стСпСни. Но, ΠΊΠΎΠ½Π΅Ρ‡Π½ΠΎ, ΡƒΠΌΠ½ΠΎΠΆΠ΅Π½ΠΈΠ΅ Ρ‡Π΅Π³ΠΎ-Π»ΠΈΠ±ΠΎ Π½Π° ноль просто Π΄Π°Π΅Ρ‚ ноль. Π’Π°ΠΊΠΈΠΌ ΠΎΠ±Ρ€Π°Π·ΠΎΠΌ, Ρƒ нас остаСтся Ρ‚Ρ€ΠΈ, ΡƒΠΌΠ½ΠΎΠΆΠ΅Π½Π½Ρ‹Π΅ Π½Π° Ρ‚Ρ€ΠΈ π‘₯ Π² ΠΊΠ²Π°Π΄Ρ€Π°Ρ‚Π΅ минус ΠΏΡΡ‚ΡŒ, ΡƒΠΌΠ½ΠΎΠΆΠ΅Π½Π½Ρ‹Π΅ Π½Π° Π΅Π΄ΠΈΠ½ΠΈΡ†Ρƒ, Ρ‡Ρ‚ΠΎ упрощаСтся Π΄ΠΎ дСвяти π‘₯ Π² ΠΊΠ²Π°Π΄Ρ€Π°Ρ‚Π΅ минус ΠΏΡΡ‚ΡŒ.

    Π’Π΅ΠΏΠ΅Ρ€ΡŒ ΠΌΡ‹ нашли Π²Ρ‹Ρ€Π°ΠΆΠ΅Π½ΠΈΠ΅ для Ρ„ΡƒΠ½ΠΊΡ†ΠΈΠΈ Π½Π°ΠΊΠ»ΠΎΠ½Π° этой ΠΊΡ€ΠΈΠ²ΠΎΠΉ. ΠŸΠΎΠΌΠ½ΠΈΡ‚Π΅, ΠΎΠ΄Π½Π°ΠΊΠΎ, Ρ‡Ρ‚ΠΎ ΠΌΡ‹ пытаСмся ΠΎΠΏΡ€Π΅Π΄Π΅Π»ΠΈΡ‚ΡŒ Ρ‚ΠΎΡ‡ΠΊΠΈ, Π² ΠΊΠΎΡ‚ΠΎΡ€Ρ‹Ρ… ΠΊΠ°ΡΠ°Ρ‚Π΅Π»ΡŒΠ½Ρ‹Π΅ ΠΏΠ°Ρ€Π°Π»Π»Π΅Π»ΡŒΠ½Ρ‹ прямой Ρ‡Π΅Ρ‚Ρ‹Ρ€Π΅ π‘₯ плюс 𝑦 минус Π΄Π²Π° Ρ€Π°Π²Π½ΠΎ Π½ΡƒΠ»ΡŽ, ΠΊΠΎΡ‚ΠΎΡ€Ρ‹Π΅, ΠΊΠ°ΠΊ ΠΌΡ‹ ΡƒΠΆΠ΅ установили, ΡΠ²Π»ΡΡŽΡ‚ΡΡ Ρ‚ΠΎΡ‡ΠΊΠ°ΠΌΠΈ, Π² ΠΊΠΎΡ‚ΠΎΡ€Ρ‹Ρ… Π½Π°ΠΊΠ»ΠΎΠ½ Ρ€Π°Π²Π΅Π½ минус Ρ‡Π΅Ρ‚Ρ‹Ρ€Π΅ΠΌ. Π§Ρ‚ΠΎΠ±Ρ‹ Π½Π°ΠΉΡ‚ΠΈ π‘₯-ΠΊΠΎΠΎΡ€Π΄ΠΈΠ½Π°Ρ‚Ρ‹ этих Ρ‚ΠΎΡ‡Π΅ΠΊ, ΠΌΡ‹ ΠΌΠΎΠΆΠ΅ΠΌ Π²Π·ΡΡ‚ΡŒ нашС Π²Ρ‹Ρ€Π°ΠΆΠ΅Π½ΠΈΠ΅ для Ρ„ΡƒΠ½ΠΊΡ†ΠΈΠΈ Π½Π°ΠΊΠ»ΠΎΠ½Π° ΠΈ ΡƒΡΡ‚Π°Π½ΠΎΠ²ΠΈΡ‚ΡŒ Π΅Π³ΠΎ Ρ€Π°Π²Π½Ρ‹ΠΌ ΠΎΡ‚Ρ€ΠΈΡ†Π°Ρ‚Π΅Π»ΡŒΠ½Ρ‹ΠΌ Ρ‡Π΅Ρ‚Ρ‹Ρ€Π΅ΠΌ, Ρ‡Ρ‚ΠΎ Π΄Π°Π΅Ρ‚ Π΄Π΅Π²ΡΡ‚ΡŒ π‘₯ Π² ΠΊΠ²Π°Π΄Ρ€Π°Ρ‚Π΅ минус ΠΏΡΡ‚ΡŒ Ρ€Π°Π²Π½ΠΎ ΠΎΡ‚Ρ€ΠΈΡ†Π°Ρ‚Π΅Π»ΡŒΠ½Ρ‹ΠΌ Ρ‡Π΅Ρ‚Ρ‹Ρ€Π΅ΠΌ. Π’Π΅ΠΏΠ΅Ρ€ΡŒ ΠΌΡ‹ Ρ€Π΅ΡˆΠΈΠΌ это ΡƒΡ€Π°Π²Π½Π΅Π½ΠΈΠ΅, Ρ‡Ρ‚ΠΎΠ±Ρ‹ ΠΎΠΏΡ€Π΅Π΄Π΅Π»ΠΈΡ‚ΡŒ значСния π‘₯. Π’ΠΎ-ΠΏΠ΅Ρ€Π²Ρ‹Ρ…, ΠΌΡ‹ добавляСм ΠΏΡΡ‚ΡŒ ΠΊ ΠΊΠ°ΠΆΠ΄ΠΎΠΉ части уравнСния, Ρ‡Ρ‚ΠΎ Π΄Π°Π΅Ρ‚ Π΄Π΅Π²ΡΡ‚ΡŒ π‘₯ Π² ΠΊΠ²Π°Π΄Ρ€Π°Ρ‚Π΅ Ρ€Π°Π²Π½ΠΎ Π΅Π΄ΠΈΠ½ΠΈΡ†Π΅. Π—Π°Ρ‚Π΅ΠΌ Π΄Π΅Π»ΠΈΠΌ Π½Π° Π΄Π΅Π²ΡΡ‚ΡŒ, Ρ‡Ρ‚ΠΎ Π΄Π°Π΅Ρ‚ π‘₯ Π² ΠΊΠ²Π°Π΄Ρ€Π°Ρ‚Π΅ Ρ€Π°Π²Π½ΠΎ ΠΎΠ΄Π½ΠΎΠΉ дСвятой.

    Π§Ρ‚ΠΎΠ±Ρ‹ Π½Π°ΠΉΡ‚ΠΈ π‘₯, ΠΌΡ‹ Π±Π΅Ρ€Π΅ΠΌ ΠΊΠ²Π°Π΄Ρ€Π°Ρ‚Π½Ρ‹ΠΉ ΠΊΠΎΡ€Π΅Π½ΡŒ, π‘₯ Ρ€Π°Π²Π½ΠΎ плюс ΠΈΠ»ΠΈ минус ΠΊΠ²Π°Π΄Ρ€Π°Ρ‚Π½Ρ‹ΠΉ ΠΊΠΎΡ€Π΅Π½ΡŒ ΠΈΠ· ΠΎΠ΄Π½ΠΎΠΉ дСвятой. Π­Ρ‚ΠΎ эквивалСнтно плюс-минус ΠΊΠ²Π°Π΄Ρ€Π°Ρ‚Π½ΠΎΠΌΡƒ ΠΊΠΎΡ€Π½ΡŽ ΠΈΠ· Π΅Π΄ΠΈΠ½ΠΈΡ†Ρ‹ Π½Π° ΠΊΠ²Π°Π΄Ρ€Π°Ρ‚Π½Ρ‹ΠΉ ΠΊΠΎΡ€Π΅Π½ΡŒ ΠΈΠ· дСвяти. И ΠΏΠΎΡΠΊΠΎΠ»ΡŒΠΊΡƒ ΠΎΠ±Π° эти числа ΡΠ²Π»ΡΡŽΡ‚ΡΡ ΠΊΠ²Π°Π΄Ρ€Π°Ρ‚Π½Ρ‹ΠΌΠΈ, это упрощаСтся Π΄ΠΎ плюс-минус ΠΎΠ΄ΠΈΠ½ Π½Π° Ρ‚Ρ€ΠΈ ΠΈΠ»ΠΈ плюс-минус Ρ‚Ρ€Π΅Ρ‚ΡŒ. Π’Π΅ΠΏΠ΅Ρ€ΡŒ ΠΌΡ‹ нашли π‘₯-ΠΊΠΎΠΎΡ€Π΄ΠΈΠ½Π°Ρ‚Ρ‹ Ρ‚ΠΎΡ‡Π΅ΠΊ Π½Π° этой ΠΊΡ€ΠΈΠ²ΠΎΠΉ, Π³Π΄Π΅ ΠΊΠ°ΡΠ°Ρ‚Π΅Π»ΡŒΠ½Ρ‹Π΅ ΠΏΠ°Ρ€Π°Π»Π»Π΅Π»ΡŒΠ½Ρ‹ Π΄Π°Π½Π½ΠΎΠΉ прямой. ПослСдний шаг β€” Ρ‚Π°ΠΊΠΆΠ΅ Π½Π°ΠΉΡ‚ΠΈ ΠΈΡ… 𝑦-ΠΊΠΎΠΎΡ€Π΄ΠΈΠ½Π°Ρ‚Ρ‹, Ρ‡Ρ‚ΠΎ ΠΌΡ‹ ΠΌΠΎΠΆΠ΅ΠΌ ΡΠ΄Π΅Π»Π°Ρ‚ΡŒ, подставив ΠΊΠ°ΠΆΠ΄ΠΎΠ΅ ΠΈΠ· Π½Π°ΡˆΠΈΡ… π‘₯-Π·Π½Π°Ρ‡Π΅Π½ΠΈΠΉ Π² ΡƒΡ€Π°Π²Π½Π΅Π½ΠΈΠ΅ ΠΊΡ€ΠΈΠ²ΠΎΠΉ. Когда π‘₯ Ρ€Π°Π²Π½ΠΎ ΠΏΠΎΠ»ΠΎΠΆΠΈΡ‚Π΅Π»ΡŒΠ½ΠΎΠΉ Ρ‚Ρ€Π΅Ρ‚ΠΈ, ΠΏΡ€Π΅ΠΆΠ΄Π΅ всСго, 𝑦 Ρ€Π°Π²Π½ΠΎ Ρ‚Ρ€Π΅ΠΌ, ΡƒΠΌΠ½ΠΎΠΆΠ΅Π½Π½Ρ‹ΠΌ Π½Π° Ρ‚Ρ€Π΅Ρ‚ΡŒ Π² ΠΊΡƒΠ±Π΅ минус ΠΏΡΡ‚ΡŒ, ΡƒΠΌΠ½ΠΎΠΆΠ΅Π½Π½Ρ‹ΠΌ Π½Π° Ρ‚Ρ€Π΅Ρ‚ΡŒ плюс сСмь. Π­Ρ‚ΠΎ Ρ‚Ρ€ΠΈ Π½Π° 27 минус ΠΏΡΡ‚ΡŒ Π½Π° Ρ‚Ρ€ΠΈ плюс сСмь, Ρ‡Ρ‚ΠΎ упрощаСтся Π΄ΠΎ 49.большС дСвяти.

    ΠœΡ‹ ΠΌΠΎΠ³Π»ΠΈ Π±Ρ‹ ΠΏΠΎΠΊΠ°Π·Π°Ρ‚ΡŒ это, сократив Π΄Ρ€ΠΎΠ±ΡŒ Ρ‚Ρ€ΠΈ большС 27 Π΄ΠΎ Π΅Π΄ΠΈΠ½ΠΈΡ†Ρ‹ большС дСвяти, Π° Π·Π°Ρ‚Π΅ΠΌ записав ΠΊΠ°ΠΆΠ΄Ρ‹ΠΉ ΠΈΠ· Π΄Π²ΡƒΡ… Π΄Ρ€ΡƒΠ³ΠΈΡ… Ρ‡Π»Π΅Π½ΠΎΠ² Ρ‚Π°ΠΊΠΆΠ΅ ΠΊΠ°ΠΊ Π΄Ρ€ΠΎΠ±ΠΈ со Π·Π½Π°ΠΌΠ΅Π½Π°Ρ‚Π΅Π»Π΅ΠΌ Π΄Π΅Π²ΡΡ‚ΡŒ. Когда π‘₯ Ρ€Π°Π²Π½ΠΎ ΠΎΡ‚Ρ€ΠΈΡ†Π°Ρ‚Π΅Π»ΡŒΠ½ΠΎΠΉ ΠΎΠ΄Π½ΠΎΠΉ Ρ‚Ρ€Π΅Ρ‚ΠΈ, 𝑦 Ρ€Π°Π²Π½ΠΎ Ρ‚Ρ€Π΅ΠΌ, ΡƒΠΌΠ½ΠΎΠΆΠ΅Π½Π½Ρ‹ΠΌ Π½Π° ΠΎΡ‚Ρ€ΠΈΡ†Π°Ρ‚Π΅Π»ΡŒΠ½ΡƒΡŽ ΠΎΠ΄Π½Ρƒ Ρ‚Ρ€Π΅Ρ‚ΡŒ Π² ΠΊΡƒΠ±Π΅ минус ΠΏΡΡ‚ΡŒ, ΡƒΠΌΠ½ΠΎΠΆΠ΅Π½Π½ΠΎΠΌΡƒ Π½Π° ΠΎΡ‚Ρ€ΠΈΡ†Π°Ρ‚Π΅Π»ΡŒΠ½ΡƒΡŽ ΠΎΠ΄Π½Ρƒ Ρ‚Ρ€Π΅Ρ‚ΡŒ плюс сСмь. Π­Ρ‚ΠΎ Ρ€Π°Π²Π½ΠΎ минус Ρ‚Ρ€ΠΈ Π½Π° 27 плюс ΠΏΡΡ‚ΡŒ Π½Π° Ρ‚Ρ€ΠΈ плюс сСмь, Ρ‡Ρ‚ΠΎ Ρ€Π°Π²Π½ΠΎ 77 Π½Π° Π΄Π΅Π²ΡΡ‚ΡŒ. Π’Π°ΠΊΠΈΠΌ ΠΎΠ±Ρ€Π°Π·ΠΎΠΌ, ΠΌΡ‹ нашли, Ρ‡Ρ‚ΠΎ ΠΊΠΎΠΎΡ€Π΄ΠΈΠ½Π°Ρ‚Ρ‹ Π΄Π²ΡƒΡ… Ρ‚ΠΎΡ‡Π΅ΠΊ Π½Π° ΠΊΡ€ΠΈΠ²ΠΎΠΉ 𝑦 Ρ€Π°Π²Π½Ρ‹ Ρ‚Ρ€Π΅ΠΌ π‘₯ Π² ΠΊΡƒΠ±Π΅ минус ΠΏΡΡ‚ΡŒ π‘₯ плюс сСмь, Π² ΠΊΠΎΡ‚ΠΎΡ€Ρ‹Ρ… ΠΊΠ°ΡΠ°Ρ‚Π΅Π»ΡŒΠ½Ρ‹Π΅ ΠΏΠ°Ρ€Π°Π»Π»Π΅Π»ΡŒΠ½Ρ‹ прямой Ρ‡Π΅Ρ‚Ρ‹Ρ€Π΅ π‘₯ плюс 𝑦 минус Π΄Π²Π° Ρ€Π°Π²Π½ΠΎ Π½ΡƒΠ»ΡŽ. Π­Ρ‚ΠΈ ΠΎΡ‡ΠΊΠΈ ΡΠΎΡΡ‚Π°Π²Π»ΡΡŽΡ‚ ΠΎΠ΄Π½Ρƒ Ρ‚Ρ€Π΅Ρ‚ΡŒ, 49большС дСвяти ΠΈ минус ΠΎΠ΄Π½Π° Ρ‚Ρ€Π΅Ρ‚ΡŒ, 77 большС дСвяти.

    6.4 Π£Ρ€Π°Π²Π½Π΅Π½ΠΈΠ΅ ΠΊΠ°ΡΠ°Ρ‚Π΅Π»ΡŒΠ½ΠΎΠΉ ΠΊ ΠΊΡ€ΠΈΠ²ΠΎΠΉ | Π”ΠΈΡ„Ρ„Π΅Ρ€Π΅Π½Ρ†ΠΈΠ°Π»ΡŒΠ½ΠΎΠ΅ исчислСниС

    ΠŸΡ€Π΅Π΄Ρ‹Π΄ΡƒΡ‰ΠΈΠΉ

    6.3 ΠŸΡ€Π°Π²ΠΈΠ»Π° Π΄ΠΈΡ„Ρ„Π΅Ρ€Π΅Π½Ρ†ΠΈΠ°Ρ†ΠΈΠΈ

    Π‘Π»Π΅Π΄ΡƒΡŽΡ‰ΠΈΠΉ

    6,5 Вторая производная

    6.

    4 Π£Ρ€Π°Π²Π½Π΅Π½ΠΈΠ΅ ΠΊΠ°ΡΠ°Ρ‚Π΅Π»ΡŒΠ½ΠΎΠΉ ΠΊ ΠΊΡ€ΠΈΠ²ΠΎΠΉ (EMCH8) temp text

    Π’ Π΄Π°Π½Π½ΠΎΠΉ Ρ‚ΠΎΡ‡ΠΊΠ΅ ΠΊΡ€ΠΈΠ²ΠΎΠΉ Π³Ρ€Π°Π΄ΠΈΠ΅Π½Ρ‚ ΠΊΡ€ΠΈΠ²ΠΎΠΉ Ρ€Π°Π²Π΅Π½ Π³Ρ€Π°Π΄ΠΈΠ΅Π½Ρ‚Ρƒ ΠΊΠ°ΡΠ°Ρ‚Π΅Π»ΡŒΠ½ΠΎΠΉ ΠΊ ΠΈΠ·Π³ΠΈΠ±.

    ΠŸΡ€ΠΎΠΈΠ·Π²ΠΎΠ΄Π½Π°Ρ (ΠΈΠ»ΠΈ функция Π³Ρ€Π°Π΄ΠΈΠ΅Π½Ρ‚Π°) описываСт Π³Ρ€Π°Π΄ΠΈΠ΅Π½Ρ‚ ΠΊΡ€ΠΈΠ²ΠΎΠΉ Π² любой Ρ‚ΠΎΡ‡ΠΊΠ΅ ΠΊΡ€ΠΈΠ²ΠΎΠΉ. Π’ΠΎΡ‡Π½ΠΎ Ρ‚Π°ΠΊ ΠΆΠ΅ ΠΎΠ½ Ρ‚Π°ΠΊΠΆΠ΅ описываСт Π³Ρ€Π°Π΄ΠΈΠ΅Π½Ρ‚ ΠΊΠ°ΡΠ°Ρ‚Π΅Π»ΡŒΠ½ΠΎΠΉ ΠΊ ΠΊΡ€ΠΈΠ²ΠΎΠΉ Π² любой Ρ‚ΠΎΡ‡ΠΊΠ΅ ΠΊΡ€ΠΈΠ²ΠΎΠΉ.

    Для опрСдСлСния уравнСния ΠΊΠ°ΡΠ°Ρ‚Π΅Π»ΡŒΠ½ΠΎΠΉ ΠΊ ΠΊΡ€ΠΈΠ²ΠΎΠΉ:

    1. НайдитС ΠΏΡ€ΠΎΠΈΠ·Π²ΠΎΠ΄Π½ΡƒΡŽ ΠΏΠΎ ΠΏΡ€Π°Π²ΠΈΠ»Π°ΠΌ диффСрСнцирования.
    2. ΠŸΠΎΠ΄ΡΡ‚Π°Π²ΡŒΡ‚Π΅ \(x\)-ΠΊΠΎΠΎΡ€Π΄ΠΈΠ½Π°Ρ‚Ρƒ Π΄Π°Π½Π½ΠΎΠΉ Ρ‚ΠΎΡ‡ΠΊΠΈ Π² ΠΏΡ€ΠΎΠΈΠ·Π²ΠΎΠ΄Π½ΡƒΡŽ, Ρ‡Ρ‚ΠΎΠ±Ρ‹ Π²Ρ‹Ρ‡ΠΈΡΠ»ΠΈΡ‚ΡŒ Π³Ρ€Π°Π΄ΠΈΠ΅Π½Ρ‚ ΠΊΠ°ΡΠ°Ρ‚Π΅Π»ΡŒΠ½ΠΎΠΉ.
    3. ΠŸΠΎΠ΄ΡΡ‚Π°Π²ΠΈΡ‚ΡŒ Π³Ρ€Π°Π΄ΠΈΠ΅Π½Ρ‚ ΠΊΠ°ΡΠ°Ρ‚Π΅Π»ΡŒΠ½ΠΎΠΉ ΠΈ ΠΊΠΎΠΎΡ€Π΄ΠΈΠ½Π°Ρ‚Ρ‹ Π·Π°Π΄Π°Π½Π½ΠΎΠΉ Ρ‚ΠΎΡ‡ΠΊΠΈ Π² ΡΠΎΠΎΡ‚Π²Π΅Ρ‚ΡΡ‚Π²ΡƒΡŽΡ‰ΡƒΡŽ Ρ„ΠΎΡ€ΠΌΡƒ уравнСния прямой. 9{2} \\ & \\ \поэтому \frac{dy}{dx} &= 3 \left( 2x \right) \\ &= 6x \ΠΊΠΎΠ½Π΅Ρ†{Π²Ρ‹Ρ€Π°Π²Π½ΠΈΠ²Π°Π½ΠΈΠ΅*}

      Π’Ρ‹Ρ‡ΠΈΡΠ»ΠΈΡ‚ΡŒ Π³Ρ€Π°Π΄ΠΈΠ΅Π½Ρ‚ ΠΊΠ°ΡΠ°Ρ‚Π΅Π»ΡŒΠ½ΠΎΠΉ

      Π§Ρ‚ΠΎΠ±Ρ‹ ΠΎΠΏΡ€Π΅Π΄Π΅Π»ΠΈΡ‚ΡŒ Π³Ρ€Π°Π΄ΠΈΠ΅Π½Ρ‚ ΠΊΠ°ΡΠ°Ρ‚Π΅Π»ΡŒΠ½ΠΎΠΉ Π² Ρ‚ΠΎΡ‡ΠΊΠ΅ \(\left(1;3\right)\), ΠΌΡ‹ ΠΏΠΎΠ΄ΡΡ‚Π°Π²ΡŒΡ‚Π΅ \(Ρ…\)-Π·Π½Π°Ρ‡Π΅Π½ΠΈΠ΅ Π² ΡƒΡ€Π°Π²Π½Π΅Π½ΠΈΠ΅ для ΠΏΡ€ΠΎΠΈΠ·Π²ΠΎΠ΄Π½ΠΎΠΉ.

      \Π½Π°Ρ‡Π°Ρ‚ΡŒ{Π²Ρ‹Ρ€Π°Π²Π½ΠΈΠ²Π°Ρ‚ΡŒ*} \frac{dy}{dx} &= 6x \\ \поэтому m &= 6(1) \\ &= 6 \ΠΊΠΎΠ½Π΅Ρ†{Π²Ρ‹Ρ€Π°Π²Π½ΠΈΠ²Π°Π½ΠΈΠ΅*}

      ΠžΠΏΡ€Π΅Π΄Π΅Π»ΠΈΡ‚ΡŒ ΡƒΡ€Π°Π²Π½Π΅Π½ΠΈΠ΅ ΠΊΠ°ΡΠ°Ρ‚Π΅Π»ΡŒΠ½ΠΎΠΉ

      ΠŸΠΎΠ΄ΡΡ‚Π°Π²ΠΈΡ‚ΡŒ Π³Ρ€Π°Π΄ΠΈΠ΅Π½Ρ‚ ΠΊΠ°ΡΠ°Ρ‚Π΅Π»ΡŒΠ½ΠΎΠΉ ΠΈ ΠΊΠΎΠΎΡ€Π΄ΠΈΠ½Π°Ρ‚Ρ‹ Π·Π°Π΄Π°Π½Π½ΠΎΠΉ Ρ‚ΠΎΡ‡ΠΊΠΈ Π² Ρ„ΠΎΡ€ΠΌΠ° Ρ‚ΠΎΡ‡ΠΊΠΈ Π³Ρ€Π°Π΄ΠΈΠ΅Π½Ρ‚Π° уравнСния прямой Π»ΠΈΠ½ΠΈΠΈ.

      \Π½Π°Ρ‡Π°Ρ‚ΡŒ{Π²Ρ‹Ρ€Π°Π²Π½ΠΈΠ²Π°Ρ‚ΡŒ*} y-{y}_{1} & = m\left(x-{x}_{1}\right) \\ Ρƒ-3 & = 6\Π²Π»Π΅Π²ΠΎ(Ρ…-1\Π²ΠΏΡ€Π°Π²ΠΎ) \\ Ρƒ & = 6Ρ…-6+3 \\ Ρƒ & = 6x-3 \ΠΊΠΎΠ½Π΅Ρ†{Π²Ρ‹Ρ€Π°Π²Π½ΠΈΠ²Π°Π½ΠΈΠ΅*}

      Эскиз ΠΊΡ€ΠΈΠ²ΠΎΠΉ ΠΈ ΠΊΠ°ΡΠ°Ρ‚Π΅Π»ΡŒΠ½ΠΎΠΉ

      Π Π°Π±ΠΎΡ‡ΠΈΠΉ ΠΏΡ€ΠΈΠΌΠ΅Ρ€ 14: НахоТдСниС уравнСния ΠΊΠ°ΡΠ°Ρ‚Π΅Π»ΡŒΠ½ΠΎΠΉ ΠΊ ΠΊΡ€ΠΈΠ²ΠΎΠΉ 9{2} + 24(-1) + 9 \\ \поэтому m &= 12 — 24 + 9 \\ &= -3 \ΠΊΠΎΠ½Π΅Ρ†{Π²Ρ‹Ρ€Π°Π²Π½ΠΈΠ²Π°Π½ΠΈΠ΅*}

      ΠžΠΏΡ€Π΅Π΄Π΅Π»ΠΈΡ‚Π΅ ΡƒΡ€Π°Π²Π½Π΅Π½ΠΈΠ΅ ΠΊΠ°ΡΠ°Ρ‚Π΅Π»ΡŒΠ½ΠΎΠΉ

      ΠŸΠΎΠ΄ΡΡ‚Π°Π²ΡŒΡ‚Π΅ Π³Ρ€Π°Π΄ΠΈΠ΅Π½Ρ‚ ΠΊΠ°ΡΠ°Ρ‚Π΅Π»ΡŒΠ½ΠΎΠΉ ΠΈ ΠΊΠΎΠΎΡ€Π΄ΠΈΠ½Π°Ρ‚Ρ‹ Ρ‚ΠΎΡ‡ΠΊΠΈ Π² ΡƒΡ€Π°Π²Π½Π΅Π½ΠΈΠ΅ Π³Ρ€Π°Π΄ΠΈΠ΅Π½Ρ‚Π½ΠΎ-точСчная Ρ„ΠΎΡ€ΠΌΠ° уравнСния прямой Π»ΠΈΠ½ΠΈΠΈ.

      \Π½Π°Ρ‡Π°Ρ‚ΡŒ{Π²Ρ‹Ρ€Π°Π²Π½ΠΈΠ²Π°Ρ‚ΡŒ*} y-{y}_{1} & = m\left(x-{x}_{1}\right) \\ y-1 & = -3\Π²Π»Π΅Π²ΠΎ(x-(-1)\Π²ΠΏΡ€Π°Π²ΠΎ) \\ Ρƒ & = -3Ρ… — 3 + 1 \ Ρƒ & = -3x — 2 \ΠΊΠΎΠ½Π΅Ρ†{Π²Ρ‹Ρ€Π°Π²Π½ΠΈΠ²Π°Π½ΠΈΠ΅*}

      Π Π°Π±ΠΎΡ‡ΠΈΠΉ ΠΏΡ€ΠΈΠΌΠ΅Ρ€ 15: НахоТдСниС уравнСния Π½ΠΎΡ€ΠΌΠ°Π»ΠΈ ΠΊ ΠΊΡ€ΠΈΠ²ΠΎΠΉ

      1. ΠžΠΏΡ€Π΅Π΄Π΅Π»ΠΈΡ‚ΡŒ ΡƒΡ€Π°Π²Π½Π΅Π½ΠΈΠ΅ Π½ΠΎΡ€ΠΌΠ°Π»ΠΈ ΠΊ ΠΊΡ€ΠΈΠ²ΠΎΠΉ \(xy = -4\) Π² Ρ‚ΠΎΡ‡ΠΊΠ΅ \(\Π²Π»Π΅Π²ΠΎ(-1;4\Π²ΠΏΡ€Π°Π²ΠΎ)\).
      2. НарисуйтС Π³Ρ€ΡƒΠ±Ρ‹ΠΉ набросок.

      НайдитС ΠΏΡ€ΠΎΠΈΠ·Π²ΠΎΠ΄Π½ΡƒΡŽ

      Π‘Π΄Π΅Π»Π°ΠΉΡ‚Π΅ \(y\) ΠΏΡ€Π΅Π΄ΠΌΠ΅Ρ‚ΠΎΠΌ Ρ„ΠΎΡ€ΠΌΡƒΠ»Ρ‹ ΠΈ ΠΏΡ€ΠΎΠ΄ΠΈΡ„Ρ„Π΅Ρ€Π΅Π½Ρ†ΠΈΡ€ΡƒΠΉΡ‚Π΅ ΠΏΠΎ \(x\): 9{2}} \\ \поэтому m &= 4 \ΠΊΠΎΠ½Π΅Ρ†{Π²Ρ‹Ρ€Π°Π²Π½ΠΈΠ²Π°Π½ΠΈΠ΅*}

      Π˜ΡΠΏΠΎΠ»ΡŒΠ·ΡƒΠΉΡ‚Π΅ Π³Ρ€Π°Π΄ΠΈΠ΅Π½Ρ‚ ΠΊΠ°ΡΠ°Ρ‚Π΅Π»ΡŒΠ½ΠΎΠΉ для расчСта Π³Ρ€Π°Π΄ΠΈΠ΅Π½Ρ‚Π° Π½ΠΎΡ€ΠΌΠ°Π»ΠΈ:

      \Π½Π°Ρ‡Π°Ρ‚ΡŒ{Π²Ρ‹Ρ€Π°Π²Π½ΠΈΠ²Π°Ρ‚ΡŒ*} m_{\text{тангСнс}} \times m_{\text{Π½ΠΎΡ€ΠΌΠ°Π»ΡŒ}} &= -1 \\ 4 \times m_{\text{Π½ΠΎΡ€ΠΌΠ°Π»ΡŒ}} &= -1 \\ \поэтому m_{\text{Π½ΠΎΡ€ΠΌΠ°Π»ΡŒ}} &= -\frac{1}{4} \ΠΊΠΎΠ½Π΅Ρ†{Π²Ρ‹Ρ€Π°Π²Π½ΠΈΠ²Π°Π½ΠΈΠ΅*}

      Найти ΡƒΡ€Π°Π²Π½Π΅Π½ΠΈΠ΅ Π½ΠΎΡ€ΠΌΠ°Π»ΠΈ

      ΠŸΠΎΠ΄ΡΡ‚Π°Π²ΠΈΡ‚ΡŒ Π³Ρ€Π°Π΄ΠΈΠ΅Π½Ρ‚ Π½ΠΎΡ€ΠΌΠ°Π»ΠΈ ΠΈ ΠΊΠΎΠΎΡ€Π΄ΠΈΠ½Π°Ρ‚Ρ‹ Π·Π°Π΄Π°Π½Π½ΠΎΠΉ Ρ‚ΠΎΡ‡ΠΊΠΈ Π² ΡƒΡ€Π°Π²Π½Π΅Π½ΠΈΠ΅ Π³Ρ€Π°Π΄ΠΈΠ΅Π½Ρ‚Π½ΠΎ-точСчная Ρ„ΠΎΡ€ΠΌΠ° уравнСния прямой Π»ΠΈΠ½ΠΈΠΈ.

      \Π½Π°Ρ‡Π°Ρ‚ΡŒ{Π²Ρ‹Ρ€Π°Π²Π½ΠΈΠ²Π°Ρ‚ΡŒ*} y-{y}_{1} & = m\left(x-{x}_{1}\right) \\ y-4 & = -\frac{1}{4}\left(x-(-1)\right) \\ y & = -\frac{1}{4}x — \frac{1}{4} + 4\\ y & = -\frac{1}{4}x + \frac{15}{4} \ΠΊΠΎΠ½Π΅Ρ†{Π²Ρ‹Ρ€Π°Π²Π½ΠΈΠ²Π°Π½ΠΈΠ΅*} 9{2} + (4)(2) -7 \\ &=13 \\ \поэтому \text{ΠšΠ°ΡΠ°Ρ‚Π΅Π»ΡŒΠ½Π°Ρ: } y &=13x +c \ΠΊΠΎΠ½Π΅Ρ†{Π²Ρ‹Ρ€Π°Π²Π½ΠΈΠ²Π°Π½ΠΈΠ΅*}

      Π³Π΄Π΅ \(c\) — это \(y\)-ΠΎΡ‚Ρ€Π΅Π·ΠΎΠΊ.

      ΠšΠ°ΡΠ°Ρ‚Π΅Π»ΡŒΠ½Π°Ρ пСрСсСкаСт \(F(x)\) Π² Ρ‚ΠΎΡ‡ΠΊΠ΅ \((2;F(2))\)

      \Π½Π°Ρ‡Π°Ρ‚ΡŒ{Π²Ρ‹Ρ€Π°Π²Π½ΠΈΠ²Π°Ρ‚ΡŒ*} F(2) &=(2)^{3} + 2(2)^{2} — 7(2) +1 \\ &=8+8-14+1\ &=3 \\ \text{ΠšΠ°ΡΠ°Ρ‚Π΅Π»ΡŒΠ½Π°Ρ: } 3 &=13(2) + c \\ \поэтому с &= — 23 \\ Ρƒ & = 13x — 23 \ΠΊΠΎΠ½Π΅Ρ†{Π²Ρ‹Ρ€Π°Π²Π½ΠΈΠ²Π°Π½ΠΈΠ΅*} 9{2}\) Ρ€Π°Π²Π½ΠΎ \(\text{5}\). {2} \\ &=1-3 \Π²Π»Π΅Π²ΠΎ( \frac{25}{36} \Π²ΠΏΡ€Π°Π²ΠΎ) \\ &=1 — \фракция{25}{12} \\ &= — \фракция{13}{12} \\ \поэтому & \left( — \frac{5}{6};- \frac{13}{12} \right) \end{Π²Ρ‹Ρ€Π°Π²Π½ΠΈΠ²Π°Π½ΠΈΠ΅*} 9{2}+2x+1\) Ρ€Π°Π²Π½ΠΎ \(\text{0}\).

      \begin{Π²Ρ‹Ρ€Π°Π²Π½ΠΈΠ²Π°Π½ΠΈΠ΅*} \text{Π“Ρ€Π°Π΄ΠΈΠ΅Π½Ρ‚ ΠΊΠ°ΡΠ°Ρ‚Π΅Π»ΡŒΠ½ΠΎΠΉ } = g'(x) = \frac{2}{3}x+2 \\ \поэтому \frac{2}{3}x+2 &=0 \\ \frac{2}{3}x &= -2\\ \поэтому x&=-2 \times \frac{3}{2} \\ &=-3 \\ \text{И} g(-3) &= \frac{1}{3}(-3)^{2}+2(-3)+1 \\ &= \фракция{1}{3}(9)-6+1\ &= 3-6+1 \\ &= -2 \\ \поэтому & (-3;-2) \end{align*}

      ΠΏΠ°Ρ€Π°Π»Π»Π΅Π»ΡŒΠ½ΠΎ Π»ΠΈΠ½ΠΈΠΈ \(y=4x-2\). {2} \\ & = 1 \ΠΊΠΎΠ½Π΅Ρ†{Π²Ρ‹Ρ€Π°Π²Π½ΠΈΠ²Π°Π½ΠΈΠ΅*}

      Π‘Π»Π΅Π΄ΠΎΠ²Π°Ρ‚Π΅Π»ΡŒΠ½ΠΎ, ΠΊΠ°ΡΠ°Ρ‚Π΅Π»ΡŒΠ½Π°Ρ ΠΏΠ°Ρ€Π°Π»Π»Π΅Π»ΡŒΠ½Π° Π΄Π°Π½Π½ΠΎΠΉ прямой Π² Ρ‚ΠΎΡ‡ΠΊΠ΅ Ρ‚ΠΎΡ‡ΠΊΠ° \((1;1)\).

      пСрпСндикулярно прямой \(2y+x-4=0\).

      \Π½Π°Ρ‡Π°Ρ‚ΡŒ{Π²Ρ‹Ρ€Π°Π²Π½ΠΈΠ²Π°Ρ‚ΡŒ*} \text{ΠŸΠ΅Ρ€ΠΏΠ΅Π½Π΄ΠΈΠΊΡƒΠ»ΡΡ€Π½ΠΎ } 2y + x — 4 &= 0 \\ y&= -\frac{1}{2}x+2\\ \поэтому \text{ Π³Ρ€Π°Π΄ΠΈΠ΅Π½Ρ‚ } \perp \text{ линия } & = 2 \quad (m_1 \times m_2 = -1) \\ \поэтому f'(x) &= 8x-4 \\ \поэтому 8x-4 &=2\\ 8x&=6\\ Ρ…&=\фракция{3}{4} \\ \поэтому y&=\left[2\left(\frac{3}{4}\right)-1\right]^{2} \\ &=\фракция{1}{4} \\ \поэтому \Π²Π»Π΅Π²ΠΎ(\frac{3}{4};\frac{1}{4}\right) \ΠΊΠΎΠ½Π΅Ρ†{Π²Ρ‹Ρ€Π°Π²Π½ΠΈΠ²Π°Π½ΠΈΠ΅*}

      Π‘Π»Π΅Π΄ΠΎΠ²Π°Ρ‚Π΅Π»ΡŒΠ½ΠΎ, ΠΊΠ°ΡΠ°Ρ‚Π΅Π»ΡŒΠ½Π°Ρ пСрпСндикулярна Π΄Π°Π½Π½ΠΎΠΉ прямой Π² Ρ‚ΠΎΡ‡ΠΊΠ΅ Ρ‚ΠΎΡ‡ΠΊΠ° \(\left(\frac{3}{4};\frac{1}{4}\right)\). {2} — 4Ρ… + 3 = 0 \\ (Ρ…-3)(Ρ…-1) = 0 \\ Ρ…=3 \тСкст{ ΠΈΠ»ΠΈ } Ρ…=1 \\ \text{Π€ΠΎΡ€ΠΌΠ°: Β«Ρ…ΠΌΡƒΡ€Ρ‹ΠΉΒ» } (a < 0) \\\)

      НайдитС уравнСния ΠΊΠ°ΡΠ°Ρ‚Π΅Π»ΡŒΠ½Ρ‹Ρ… ΠΊ \(f\) Π² Ρ‚ΠΎΡ‡ΠΊΠ΅:

      1. \(y\)-пСрСсСчСниС \(f\).
      2. ΠΏΠΎΠ²ΠΎΡ€ΠΎΡ‚Π½Ρ‹ΠΉ ΠΌΠΎΠΌΠ΅Π½Ρ‚ \(f\).
      3. Ρ‚ΠΎΡ‡ΠΊΠ°, Π³Π΄Π΅ \(x = \text{4,25}\).
      1. \begin{Π²Ρ‹Ρ€Π°Π²Π½ΠΈΠ²Π°Π½ΠΈΠ΅*} Ρƒ _ {\ тСкст {Ρ†Π΅Π»ΠΎΠ΅}}: (0; -3) \\ m _ {\ text {тангСнс}} = f ‘(x) &= -2x + 4 \\ f'(0) &=-2(0) + 4 \\ \поэтому m &=4\\ \text{ΠšΠ°ΡΠ°Ρ‚Π΅Π»ΡŒΠ½Π°Ρ}y&=4x+c\\ \text{Π§Π΅Ρ€Π΅Π· }(0;-3) \поэтому y&=4x-3 \end{Π²Ρ‹Ρ€Π°Π²Π½ΠΈΠ²Π°Π½ΠΈΠ΅*}
      2. \begin{Π²Ρ‹Ρ€Π°Π²Π½ΠΈΠ²Π°Π½ΠΈΠ΅*} \text{ΠŸΠΎΠ²ΠΎΡ€ΠΎΡ‚Π½Ρ‹ΠΉ ΠΌΠΎΠΌΠ΅Π½Ρ‚: } (2;1) \\ m _ {\ text {тангСнс}} = f ‘(2) &= -2 (2) + 4 \\ &=0\\ \text{ΠšΠ°ΡΠ°Ρ‚Π΅Π»ΡŒΠ½ΠΎΠ΅ ΡƒΡ€Π°Π²Π½Π΅Π½ΠΈΠ΅ } y &= 1 \end{Π²Ρ‹Ρ€Π°Π²Π½ΠΈΠ²Π°Π½ΠΈΠ΅*} 9{2}+4(\тСкст{4,25})-3 \\ &= -\тСкст{4,0625} \\ m _ {\ text {тангСнс}} \ text { Π² } x & = \тСкст{4,25} \\ m&=-2(\text{4,25})+4\\ &=-\тСкст{4,5} \\ \text{ΠšΠ°ΡΠ°Ρ‚Π΅Π»ΡŒΠ½Π°Ρ}y&=-\text{4,5}x+c\\ \text{Π§Π΅Ρ€Π΅Π·}(\text{4,25};-\text{4,0625}) \\ -\text{4,0625}&=-\text{4,5}(\text{4,25})+c\\ \поэтому c&= \text{15,0625} \\ Ρƒ&=-\тСкст{4,5}Ρ…+\тСкст{15,0625} \end{Π²Ρ‹Ρ€Π°Π²Π½ΠΈΠ²Π°Π½ΠΈΠ΅*}

      НарисуйтС Ρ‚Ρ€ΠΈ ΠΊΠ°ΡΠ°Ρ‚Π΅Π»ΡŒΠ½Ρ‹Π΅ Π²Ρ‹ΡˆΠ΅ Π½Π° Π³Ρ€Π°Ρ„ΠΈΠΊΠ΅ \(Ρ„\).

      Π—Π°ΠΏΠΈΡˆΠΈΡ‚Π΅ всС наблюдСния ΠΎ Ρ‚Ρ€Π΅Ρ… ΠΊΠ°ΡΠ°Ρ‚Π΅Π»ΡŒΠ½Ρ‹Π΅ ΠΊ \(f\).

      ВангСнс Π² Ρ‚ΠΎΡ‡ΠΊΠ΅ \(y_{\text{int}}\) (синяя линия): Π³Ρ€Π°Π΄ΠΈΠ΅Π½Ρ‚ Ρ€Π°Π²Π΅Π½ ΠΏΠΎΠ»ΠΎΠΆΠΈΡ‚Π΅Π»ΡŒΠ½Π°, функция Π² этой Ρ‚ΠΎΡ‡ΠΊΠ΅ возрастаСт.

      ΠšΠ°ΡΠ°Ρ‚Π΅Π»ΡŒΠ½Π°Ρ Π² Ρ‚ΠΎΡ‡ΠΊΠ΅ ΠΏΠΎΠ²ΠΎΡ€ΠΎΡ‚Π° (зСлСная линия): ΡƒΠΊΠ»ΠΎΠ½ Ρ€Π°Π²Π΅Π½ Π½ΡƒΠ»ΡŽ, ΠΊΠ°ΡΠ°Ρ‚Π΅Π»ΡŒΠ½Π°Ρ — это Π³ΠΎΡ€ΠΈΠ·ΠΎΠ½Ρ‚Π°Π»ΡŒΠ½Π°Ρ линия, ΠΏΠ°Ρ€Π°Π»Π»Π΅Π»ΡŒΠ½Π°Ρ оси \(x\).

      ΠšΠ°ΡΠ°Ρ‚Π΅Π»ΡŒΠ½Π°Ρ Π² Ρ‚ΠΎΡ‡ΠΊΠ΅ \(x=\text{4,25}\) (фиолСтовая линия): Π³Ρ€Π°Π΄ΠΈΠ΅Π½Ρ‚ ΠΎΡ‚Ρ€ΠΈΡ†Π°Ρ‚Π΅Π»ΡŒΠ½ΠΎ, функция Π² этой Ρ‚ΠΎΡ‡ΠΊΠ΅ ΡƒΠ±Ρ‹Π²Π°Π΅Ρ‚.

      ΠŸΡ€Π΅Π΄Ρ‹Π΄ΡƒΡ‰ΠΈΠΉ

      6.3 ΠŸΡ€Π°Π²ΠΈΠ»Π° Π΄ΠΈΡ„Ρ„Π΅Ρ€Π΅Π½Ρ†ΠΈΠ°Ρ†ΠΈΠΈ

      ОглавлСниС

      Π‘Π»Π΅Π΄ΡƒΡŽΡ‰ΠΈΠΉ

      6,5 Вторая производная

      ΠŸΠΎΡΡ‚Ρ€ΠΎΠ΅Π½ΠΈΠ΅ ΠΊΠ°ΡΠ°Ρ‚Π΅Π»ΡŒΠ½Ρ‹Ρ… ΠΊ функциям ΠΈ вычислСниС ΠΏΡ€ΠΎΠΈΠ·Π²ΠΎΠ΄Π½ΠΎΠΉ

      Π“Π»Π°Π²Π° 11. ДинамичСская гСомСтрия исчислСния

      Β 

      Π’ Π³Π»Π°Π²Π΅ 6 ΠΌΡ‹ ΠΏΠΎΠ·Π½Π°ΠΊΠΎΠΌΠΈΠ»ΠΈ вас с Π±Π»ΠΎΠΊΠ½ΠΎΡ‚Π°ΠΌΠΈ. возмоТности построСния Π³Ρ€Π°Ρ„ΠΈΠΊΠΎΠ² Ρ„ΡƒΠ½ΠΊΡ†ΠΈΠΉ. Π’ Π’ этой Π³Π»Π°Π²Π΅ ΠΌΡ‹ Π²ΠΎΡΠΏΠΎΠ»ΡŒΠ·ΡƒΠ΅ΠΌΡΡ этими возмоТностями для изучСния Π³Π΅ΠΎΠΌΠ΅Ρ‚Ρ€ΠΈΠΈ исчислСниС, начиная с ΠΎΡ‚Π½ΠΎΡˆΠ΅Π½ΠΈΡ сСкущих ΠΊ ΠΊΡ€ΠΈΠ²ΠΎΠΉ, ΠΊΠ°ΡΠ°Ρ‚Π΅Π»ΡŒΠ½Ρ‹Ρ… Ρ‚ΠΎΠΉ ΠΆΠ΅ ΠΊΡ€ΠΈΠ²ΠΎΠΉ ΠΈ ΠΏΡ€ΠΎΠΈΠ·Π²ΠΎΠ΄Π½ΠΎΠΉ Ρ„ΡƒΠ½ΠΊΡ†ΠΈΠΈ, ΠΏΠΎΡ€ΠΎΠΆΠ΄Π°ΡŽΡ‰Π΅ΠΉ ΠΈΠ·Π³ΠΈΠ±. ΠœΡ‹ Ρ‚Π°ΠΊΠΆΠ΅ прСдставили Π² Π“Π»Π°Π²Π° 6 идСя использования ΠΏΠΎΠ»Π·ΡƒΠ½ΠΊΠΎΠ² (ΠΏΠ΅Ρ€Π΅ΠΌΠ΅Π½Π½Ρ‹Ρ… сСгмСнтов Π³ΠΎΡ€ΠΈΠ·ΠΎΠ½Ρ‚Π°Π»ΡŒΠ½ΠΎΠΉ Π»ΠΈΠ½ΠΈΠΈ) для измСнСния ΠΏΠ°Ρ€Π°ΠΌΠ΅Ρ‚Ρ€Ρ‹ Ρ„ΡƒΠ½ΠΊΡ†ΠΈΠΈ. ΡΠ»Π΅Π΄ΡƒΡŽΡ‰ΠΈΠΉ Ρ€Π°Π·Π΄Π΅Π» ΠΏΡ€ΠΎΠ²Π΅Π΄Π΅Ρ‚ вас Ρ‡Π΅Ρ€Π΅Π· шаги ΠΏΠΎ созданию собствСнного инструмСнта слайдСра ΠΊΠΎΡ‚ΠΎΡ€Ρ‹ΠΉ Π²Ρ‹ Π±ΡƒΠ΄Π΅Ρ‚Π΅ ΠΈΡΠΏΠΎΠ»ΡŒΠ·ΠΎΠ²Π°Ρ‚ΡŒ для создания ΠΌΠ½ΠΎΠ³ΠΈΡ… Ρ„ΡƒΠ½ΠΊΡ†ΠΈΠΉ Π² этой Π³Π»Π°Π²Π΅.

      Β Β Β Β Β Β Β Β Β Β Β  Π­Ρ‚ΠΎ довольно Π»Π΅Π³ΠΊΠΎ ΡΠΎΠ·Π΄Π°Ρ‚ΡŒ ΠΏΠΎΠ»Π·ΡƒΠ½ΠΊΠΈ (динамичСскиС Π»ΠΈΠ½Π΅ΠΉΠ½Ρ‹Π΅ сСгмСнты) для прСдоставлСния Π·Π½Π°Ρ‡Π΅Π½ΠΈΠΉ, ΠΊΠΎΡ‚ΠΎΡ€Ρ‹Π΅ ΠΌΠΎΠΆΠ½ΠΎ ΠΌΠ°Π½ΠΈΠΏΡƒΠ»ΠΈΡ€ΠΎΠ²Π°Ρ‚ΡŒ, просто пСрСмСщая ΠΊΠΎΠ½Π΅Ρ‡Π½ΡƒΡŽ Ρ‚ΠΎΡ‡ΠΊΡƒ ΠΏΠΎΠ»Π·ΡƒΠ½ΠΊΠ°.

      • Π’ Π½ΠΎΠ²ΠΎΠΌ эскизС ΠΏΠ΅Ρ€Π΅ΠΉΠ΄ΠΈΡ‚Π΅ Π² мСню Graph ΠΈ Π²Ρ‹Π±Π΅Ρ€ΠΈΡ‚Π΅ ΠžΠΏΡ€Π΅Π΄Π΅Π»ΠΈΡ‚ΡŒ систСму ΠΊΠΎΠΎΡ€Π΄ΠΈΠ½Π°Ρ‚ .
      • Π‘ΠΎΠ·Π΄Π°ΠΉΡ‚Π΅ Π½Π° эскизС ΡΠ²ΠΎΠ±ΠΎΠ΄Π½ΡƒΡŽ Ρ‚ΠΎΡ‡ΠΊΡƒ ΠΈ Π½Π°Π·ΠΎΠ²ΠΈΡ‚Π΅ Π΅Π΅ A.
      • Π’Ρ‹Π±Π΅Ρ€ΠΈΡ‚Π΅ Ρ‚ΠΎΡ‡ΠΊΡƒ A ΠΈ ось x ΠΈ постройтС линию ΠΏΠ°Ρ€Π°Π»Π»Π΅Π»ΡŒΠ½ΠΎ оси x Ρ‡Π΅Ρ€Π΅Π· Ρ‚ΠΎΡ‡ΠΊΡƒ A.
      • ΠŸΠΎΠΌΠ΅ΡΡ‚ΠΈΡ‚Π΅ ΡΠ²ΠΎΠ±ΠΎΠ΄Π½ΡƒΡŽ Ρ‚ΠΎΡ‡ΠΊΡƒ B Π½Π° этой Π»ΠΈΠ½ΠΈΠΈ.
      • Π‘ΠΊΡ€Ρ‹Ρ‚ΡŒ линию Ρ‡Π΅Ρ€Π΅Π· Ρ‚ΠΎΡ‡ΠΊΠΈ A ΠΈ B.
      • ΠŸΠΎΡΡ‚Ρ€ΠΎΠΈΡ‚ΡŒ сСгмСнт ΠΌΠ΅ΠΆΠ΄Ρƒ A ΠΈ B.
      • Π’Ρ‹Π±Π΅Ρ€ΠΈΡ‚Π΅ Ρ‚ΠΎΡ‡ΠΊΠΈ A ΠΈ B ΠΈ ΠΈΠ·ΠΌΠ΅Ρ€ΡŒΡ‚Π΅ ΠΈΡ… абсциссу .
      • ВычислитС x B -x A ΠΈ ΠΏΠ΅Ρ€Π΅ΠΈΠΌΠ΅Π½ΡƒΠΉΡ‚Π΅ это вычислСниС b .
      • Π‘ΠΊΡ€ΠΎΠΉΡ‚Π΅ значСния для x B ΠΈ x A ΠΈ ΠΎΡ‚ΠΊΠ»ΡŽΡ‡ΠΈΡ‚Π΅ ΠΌΠ΅Ρ‚ΠΊΡƒ для Ρ‚ΠΎΡ‡ΠΊΠΈ A (Π½Π°ΠΆΠΌΠΈΡ‚Π΅ Π½Π° Ρ‚ΠΎΡ‡ΠΊΡƒ A с ΠΏΠΎΠΌΠΎΡ‰ΡŒΡŽ тСкстового инструмСнта).

      Π’Π΅ΠΏΠ΅Ρ€ΡŒ Π²Ρ‹ Π³ΠΎΡ‚ΠΎΠ²Ρ‹ ΡΠΎΠ·Π΄Π°Ρ‚ΡŒ слайдСр ΠΈΠ· этой конструкции. Π’Ρ‹ΠΏΠΎΠ»Π½ΠΈΡ‚Π΅ ΡΠ»Π΅Π΄ΡƒΡŽΡ‰ΠΈΠ΅ шаги:

      • Π’Ρ‹Π±Π΅Ρ€ΠΈΡ‚Π΅ ось X, сСгмСнт ΠΈ Π΄Π²Π΅ Π΅Π³ΠΎ ΠΊΠΎΠ½Π΅Ρ‡Π½Ρ‹Π΅ Ρ‚ΠΎΡ‡ΠΊΠΈ, ΠΈ Π·Π½Π°Ρ‡Π΅Π½ΠΈΠ΅, ΠΏΠΎΠΌΠ΅Ρ‡Π΅Π½Π½ΠΎΠ΅ b .
      • НаТмитС послСднюю ΠΊΠ½ΠΎΠΏΠΊΡƒ Π½Π° ΠΏΠ°Π½Π΅Π»ΠΈ инструмСнтов ΠΈ ΡƒΠ΄Π΅Ρ€ΠΆΠΈΠ²Π°ΠΉΡ‚Π΅ ΠΊΠ½ΠΎΠΏΠΊΡƒ ΠΌΡ‹ΡˆΠΈ Π½Π°ΠΆΠ°Ρ‚ΠΎΠΉ.
      • Π’Ρ‹Π±Π΅Ρ€ΠΈΡ‚Π΅ Create Tool Π² появившСмся подмСню.
      • НазовитС инструмСнт slider_tool ΠΈ установитС Ρ„Π»Π°ΠΆΠΎΠΊ Show 9. 1501 Script View ΠΊΠΎΡ€ΠΎΠ±ΠΊΠ°.

      Окно, ΠΏΠΎΠ΄ΠΎΠ±Π½ΠΎΠ΅ рис. 11.1. Π½ΠΈΠΆΠ΅ Π΄ΠΎΠ»ΠΆΠ½ΠΎ ΠΏΠΎΡΠ²ΠΈΡ‚ΡŒΡΡ:

      Рисунок 11.1: Окно просмотра сцСнария для Slider_Tool

      • Π”Π²ΠΎΠΉΠ½ΠΎΠΉ Ρ‰Π΅Π»Ρ‡ΠΎΠΊ ΠΏΠΎ ΠΏΠ΅Ρ€Π²ΠΎΠΌΡƒ Π·Π°Π΄Π°Π½Π½ΠΎΠΌΡƒ (1. Straight Object Икс).
      • УстановитС Ρ„Π»Π°ΠΆΠΎΠΊ АвтоматичСски ΡΠΎΠΏΠΎΡΡ‚Π°Π²Π»ΡΡ‚ΡŒ ΠΎΠ±ΡŠΠ΅ΠΊΡ‚ эскиза Π² появившССся Π΄ΠΈΠ°Π»ΠΎΠ³ΠΎΠ²ΠΎΠ΅ ΠΎΠΊΠ½ΠΎ. Straight Object x Π΄ΠΎΠ»ΠΆΠ΅Π½ ΠΏΠ΅Ρ€Π΅ΠΌΠ΅ΡΡ‚ΠΈΡ‚ΡŒΡΡ Π² Ρ‡Π°ΡΡ‚ΡŒ Assassining ΠΎΠΊΠ½Π° Script View.
      • Π‘ΠΎΡ…Ρ€Π°Π½ΠΈΡ‚Π΅ свой эскиз ΠΊΠ°ΠΊ Slider_tool Π² инструмСнт Папка.

      Π’Π°Ρˆ slider_tool Ρ‚Π΅ΠΏΠ΅Ρ€ΡŒ Π±ΡƒΠ΄Π΅Ρ‚ доступСн для использования ΠΏΡ€ΠΈ ΠΊΠ°ΠΆΠ΄ΠΎΠΌ ΠΎΡ‚ΠΊΡ€Ρ‹Ρ‚ΠΈΠΈ GSP 4.

      Β 

      Β Β Β Β Β Β Β Β Β Β Β  Π˜ΡΠΏΠΎΠ»ΡŒΠ·ΠΎΠ²Π°Ρ‚ΡŒ ваш инструмСнт ΠΏΠΎΠ»Π·ΡƒΠ½ΠΊΠ°, Ρ‡Ρ‚ΠΎΠ±Ρ‹ ΡΠΎΠ·Π΄Π°Ρ‚ΡŒ Ρ‡Π΅Ρ‚Ρ‹Ρ€Π΅ ΠΏΠΎΠ»Π·ΡƒΠ½ΠΊΠ° Π² Π½ΠΎΠ²ΠΎΠΌ эскизС. Оси ΠΊΠΎΠΎΡ€Π΄ΠΈΠ½Π°Ρ‚ Π±ΡƒΠ΄ΡƒΡ‚ созданы автоматичСски. Π’Π°ΡˆΠΈ слайдСры (сСгмСнты) Π±ΡƒΠ΄ΡƒΡ‚ ΠΈΠΌΠ΅Ρ‚ΡŒ Ρ‚ΠΎΠ»ΡŒΠΊΠΎ ΠΎΠ΄Π½Ρƒ ΠΊΠΎΠ½Π΅Ρ‡Π½ΡƒΡŽ Ρ‚ΠΎΡ‡ΠΊΡƒ. ΠŸΠΎΡΠ²ΡΡ‚ΡΡ Ρ‡Π΅Ρ‚Ρ‹Ρ€Π΅ ΠΌΠ΅Ρ€Ρ‹. ΠŸΠ΅Ρ€Π΅ΠΌΠ°Ρ€ΠΊΠΈΡ€ΡƒΠΉΡ‚Π΅ эти ΠΌΠ΅Ρ€Ρ‹ Π² соотвСтствии с этикСтками Π½Π° вашСм ΠΏΠΎΠ»Π·ΡƒΠ½ΠΊΠΈ. Π§Ρ‚ΠΎΠ± Π½Π΅ ΠΏΡƒΡ‚Π°Ρ‚ΡŒ какая конСчная Ρ‚ΠΎΡ‡ΠΊΠ° являСтся ΠΏΠΎΠ΄Π²ΠΈΠΆΠ½ΠΎΠΉ, Π²Ρ‹ ΠΌΠΎΠΆΠ΅Ρ‚Π΅ ΡΠΊΡ€Ρ‹Ρ‚ΡŒ Π½Π΅ΠΌΠ°Ρ€ΠΊΠΈΡ€ΠΎΠ²Π°Π½Π½ΡƒΡŽ ΠΊΠΎΠ½Π΅Ρ‡Π½ΡƒΡŽ Ρ‚ΠΎΡ‡ΠΊΡƒ ΠΊΠ°ΠΆΠ΄ΠΎΠ³ΠΎ слайдСр. Π˜ΡΠΏΠΎΠ»ΡŒΠ·ΡƒΠΉΡ‚Π΅ ΠΌΠ΅Ρ€Ρ‹ ΠΊΠ°ΠΆΠ΄ΠΎΠ³ΠΎ ΠΏΠΎΠ»Π·ΡƒΠ½ΠΎΠΊ для создания ΠΈ построСния Π½ΠΎΠ²ΠΎΠΉ Ρ„ΡƒΠ½ΠΊΡ†ΠΈΠΈ : f(x) = ax 3 +bx 2 +cx+d . Π’Ρ‹ Π΄ΠΎΠ»ΠΆΠ½ΠΎ ΠΈΠΌΠ΅Ρ‚ΡŒ Ρ‡Ρ‚ΠΎ-Ρ‚ΠΎ Π²Ρ€ΠΎΠ΄Π΅ рисунка 11.2 Π½ΠΈΠΆΠ΅.

      Рисунок 11.2: Π“Ρ€Π°Ρ„ΠΈΠΊ кубичСская функция, ΠΈΡΠΏΠΎΠ»ΡŒΠ·ΡƒΡŽΡ‰Π°Ρ 4 ΠΏΠΎΠ»Π·ΡƒΠ½ΠΊΠ° Π² качСствС ΠΏΠ°Ρ€Π°ΠΌΠ΅Ρ‚Ρ€ΠΎΠ².

      Β 

      Β Β Β Β Β Β Β Β Β Β Β  Π£Π·Π½Π°Ρ‚ΡŒ большС Ρ€ΠΎΠ»ΠΈ Ρ‡Π΅Ρ‚Ρ‹Ρ€Π΅Ρ… ΠΏΠ°Ρ€Π°ΠΌΠ΅Ρ‚Ρ€ΠΎΠ² Π² кубичСской Ρ„ΡƒΠ½ΠΊΡ†ΠΈΠΈ, рСгулируя ΠΊΠ°ΠΆΠ΄Ρ‹ΠΉ ΠΈΠ· ΠΏΠΎΠ»Π·ΡƒΠ½ΠΊΠΈ. Π‘ΠΎΠ·Π΄Π°Ρ‚ΡŒ Π½ΠΎΠ²ΡƒΡŽ ΠΊΡƒΠ±ΠΈΡ‡Π΅ΡΠΊΡƒΡŽ ΠΈΡΠΏΠΎΠ»ΡŒΠ·ΡƒΠΉΡ‚Π΅ Ρ‚Ρ€ΠΈ ΠΏΠΎΠ»Π·ΡƒΠ½ΠΊΠ° Π² качСствС ΠΊΠΎΡ€Π½Π΅ΠΉ Ρ„ΡƒΠ½ΠΊΡ†ΠΈΠΈ: Π½Π°ΠΏΡ€ΠΈΠΌΠ΅Ρ€. Π³(Ρ…) = Π°(Ρ…-Π±)(Ρ…-с)(Ρ…-d) . Π£Π·Π½Π°ΠΉΡ‚Π΅, ΠΊΠ°ΠΊ эти Π΄Π²Π΅ кубичСскиС Ρ„ΡƒΠ½ΠΊΡ†ΠΈΠΈ ΠΎΡ‚Π»ΠΈΡ‡Π°ΡŽΡ‚ΡΡ ΠΈ Ρ‡Π΅ΠΌ ΠΏΠΎΡ…ΠΎΠΆΠΈ.

      Β 

      Π’ этом ΡƒΠΏΡ€Π°ΠΆΠ½Π΅Π½ΠΈΠΈ ΠΌΡ‹ Π±ΡƒΠ΄Π΅ΠΌ ΠΈΡΠΏΠΎΠ»ΡŒΠ·ΠΎΠ²Π°Ρ‚ΡŒ наш ΠΏΠΎΠ»Π·ΡƒΠ½ΠΎΠΊ . инструмСнт для создания ΠΊΠΎΠ½Ρ‚Ρ€ΠΎΠ»ΡŒΠ½ΠΎΠΉ Ρ‚ΠΎΡ‡ΠΊΠΈ, которая Π±ΡƒΠ΄Π΅Ρ‚ ΠΈΡΠΏΠΎΠ»ΡŒΠ·ΠΎΠ²Π°Ρ‚ΡŒΡΡ для измСрСния Π½Π΅Π±ΠΎΠ»ΡŒΡˆΠΈΡ… ΠΈΠ·ΠΌΠ΅Π½Π΅Π½ΠΈΠΉ Π² Ρ€Π°Π·Ρ€Π΅ΡˆΠ΅Π½ΠΈΠΈ x . НазовитС эту ΠΊΠΎΠ½Ρ‚Ρ€ΠΎΠ»ΡŒΠ½ΡƒΡŽ Ρ‚ΠΎΡ‡ΠΊΡƒ d x . ΠœΠ΅ΡΡ‚ΠΎ свободная Ρ‚ΠΎΡ‡ΠΊΠ° Π½Π° оси x Π³Ρ€Π°Ρ„ΠΈΠΊΠ° вашСй Ρ„ΡƒΠ½ΠΊΡ†ΠΈΠΈ для кубичСской Ρ„ΡƒΠ½ΠΊΡ†ΠΈΠΈ f(x) = ax 3 +bx 2 +cx+d . НазовитС эту ΡΠ²ΠΎΠ±ΠΎΠ΄Π½ΡƒΡŽ Ρ‚ΠΎΡ‡ΠΊΡƒ x . Π˜Π·ΠΌΠ΅Ρ€ΡŒΡ‚Π΅ абсциссу этой Ρ‚ΠΎΡ‡ΠΊΠΈ ΠΈ ΠΎΠ±ΠΎΠ·Π½Π°Ρ‡ΡŒΡ‚Π΅ это ΠΈΠ·ΠΌΠ΅Ρ€Π΅Π½ΠΈΠ΅ ΠΊΠ°ΠΊ x 9.1501 . Π’ ΠΊΠ°Π»ΡŒΠΊΡƒΠ»ΡΡ‚ΠΎΡ€ создаСт ΠΌΠ΅Ρ€Ρƒ (x — d x/2) ΠΈ (x + Π΄ Ρ…/2) . ИспользованиС ΠΊΠ°Π»ΡŒΠΊΡƒΠ»ΡΡ‚ΠΎΡ€Π° Π’Π‘ΠŸ ΠΈ ΠΊΠ°ΠΆΠ΄ΠΎΠ³ΠΎ ΠΈΠ· этих ΠΏΠΎΠΊΠ°Π·Π°Ρ‚Π΅Π»Π΅ΠΉ ΠΈ функция f(x) , создайтС значСния f(x — d x/2) ΠΈ f(x + d x/2) соотвСтствСнно. Π‘Π»Π΅Π΄ΡƒΡŽΡ‰ΠΈΠ΅ шаги ΠΎΡ‚ΠΎΠ±Ρ€Π°Π·ΠΈΡ‚ эти значСния f(x) Π² Π²ΠΈΠ΄Π΅ Ρ‚ΠΎΡ‡Π΅ΠΊ Π½Π° Π³Ρ€Π°Ρ„ΠΈΠΊΠ΅ ΠΈ сСкущСй ΠΌΠ΅ΠΆΠ΄Ρƒ Π½ΠΈΠΌΠΈ. ΠΈΡ….

      1. Π’Ρ‹Π±Π΅Ρ€ΠΈΡ‚Π΅ ΠΌΠ΅Ρ€Ρ‹ (x — d x/2) ΠΈ f(x — d x/2) Π² ΡƒΠΊΠ°Π·Π°Π½Π½ΠΎΠΌ порядкС ΠΈ Plot as ( x , y Π² мСню Graph. Π’ΠΎΡ‡ΠΊΠ° Π΄ΠΎΠ»ΠΆΠ½Π° появится Π½Π° вашСм Ρ„ΡƒΠ½ΠΊΡ†ΠΈΠΎΠ½Π°Π»ΡŒΠ½ΠΎΠΌ Π³Ρ€Π°Ρ„ΠΈΠΊΠ΅.

      2. Π‘Π΄Π΅Π»Π°ΠΉΡ‚Π΅ Ρ‚ΠΎ ΠΆΠ΅ самоС для (x + d x/2) ΠΈ f(x + d x/2) Π‘Π΅ΠΊΡƒΠ½Π΄Π° Ρ‚ΠΎΡ‡ΠΊΠ° Π΄ΠΎΠ»ΠΆΠ½Π° ΠΏΠΎΡΠ²ΠΈΡ‚ΡŒΡΡ Π½Π° вашСм Ρ„ΡƒΠ½ΠΊΡ†ΠΈΠΎΠ½Π°Π»ΡŒΠ½ΠΎΠΌ Π³Ρ€Π°Ρ„ΠΈΠΊΠ΅.

      3. Π’Ρ‹Π΄Π΅Π»ΠΈΡ‚Π΅ эти Π΄Π²Π΅ Π½ΠΎΠ²Ρ‹Π΅ Ρ‚ΠΎΡ‡ΠΊΠΈ Π½Π° Π³Ρ€Π°Ρ„ΠΈΠΊΠ΅ ΠΈ ΠΏΡ€ΠΎΠ²Π΅Π΄ΠΈΡ‚Π΅ Ρ‡Π΅Ρ€Π΅Π· Π½ΠΈΡ… линию .

      4. Π˜Π·ΠΌΠ΅Π½ΠΈΡ‚Π΅ Π΄Π»ΠΈΠ½Ρƒ d x , Ρ‡Ρ‚ΠΎΠ±Ρ‹ ΡƒΠ²ΠΈΠ΄Π΅Ρ‚ΡŒ, Ρ‡Ρ‚ΠΎ происходит с этой сСкущСй.

      Β 

      ΠŸΠΎΡΠΊΠΎΠ»ΡŒΠΊΡƒ d x становится ΠΎΡ‡Π΅Π½ΡŒ малСньким, эта сСкущая линия каТСтся ΡΡ‚Π°Ρ‚ΡŒ ΠΊΠ°ΡΠ°Ρ‚Π΅Π»ΡŒΠ½ΠΎΠΉ ΠΊ ΠΊΡ€ΠΈΠ²ΠΎΠΉ, ΠΏΡ€Π΅Π΄ΡΡ‚Π°Π²Π»ΡΡŽΡ‰Π΅ΠΉ ΠΊΡƒΠ±ΠΈΡ‡Π΅ΡΠΊΡƒΡŽ Ρ„ΡƒΠ½ΠΊΡ†ΠΈΡŽ. Π‘ Π΄ Ρ… ΠΎΡ‡Π΅Π½ΡŒ малСнький (<0,1) ΠΏΠ΅Ρ€Π΅ΠΌΠ΅ΡΡ‚ΠΈΡ‚ΡŒ бСсплатно x Ρ‚ΠΎΡ‡Π΅ΠΊ вдоль оси x Ρ‡Ρ‚ΠΎΠ±Ρ‹ ΡƒΠ²ΠΈΠ΄Π΅Ρ‚ΡŒ, ΠΊΠ°ΠΊ сСкущая ΠΏΡ€ΠΎΡ…ΠΎΠ΄ΠΈΡ‚ вдоль ΠΊΡ€ΠΈΠ²ΠΎΠΉ (см. рис. 11.3).

      Рисунок 11.3

      Β 

      Β Β Β Β Β Β Β Β Β Β Β  Π£Π΄Π»ΠΈΠ½ΠΈΡ‚ΡŒ d x , Ρ‡Ρ‚ΠΎΠ±Ρ‹ Π²Ρ‹ ΠΌΠΎΠ³Π»ΠΈ Ρ‡Π΅Ρ‚ΠΊΠΎ Π²ΠΈΠ΄Π΅Ρ‚ΡŒ Π΄Π²Π΅ Ρ‚ΠΎΡ‡ΠΊΠΈ Π½Π° Π³Ρ€Π°Ρ„ΠΈΠΊΠ΅ функция. НанСситС Π½Π° Π³Ρ€Π°Ρ„ΠΈΠΊ Ρ‚ΠΎΡ‡ΠΊΠΈ x , f(x) . Π­Ρ‚Π° Ρ‚ΠΎΡ‡ΠΊΠ° Π΄ΠΎΠ»ΠΆΠ΅Π½ ΠΏΠΎΡΠ²ΠΈΡ‚ΡŒΡΡ ΠΌΠ΅ΠΆΠ΄Ρƒ двумя Ρ‚ΠΎΡ‡ΠΊΠ°ΠΌΠΈ, ΠΎΠΏΡ€Π΅Π΄Π΅Π»ΡΡŽΡ‰ΠΈΠΌΠΈ ΡΠ΅ΠΊΡƒΡ‰ΡƒΡŽ линию. Π’Ρ‹Π±Π΅Ρ€ΠΈΡ‚Π΅ ΡΠ΅ΠΊΡƒΡ‰ΡƒΡŽ ΠΈ Ρ‚ΠΎΡ‡ΠΊΡƒ ΠΌΠ΅ΠΆΠ΄Ρƒ двумя Ρ‚ΠΎΡ‡ΠΊΠ°ΠΌΠΈ, ΠΎΠΏΡ€Π΅Π΄Π΅Π»ΡΡŽΡ‰ΠΈΠΌΠΈ линию. ΠŸΠΎΡΡ‚Ρ€ΠΎΠΉΡ‚Π΅ ΠΏΡ€ΡΠΌΡƒΡŽ, ΠΏΠ°Ρ€Π°Π»Π»Π΅Π»ΡŒΠ½ΡƒΡŽ сСкущая Ρ‡Π΅Ρ€Π΅Π· Ρ‚ΠΎΡ‡ΠΊΡƒ Ρ… , Ρ„(Ρ…) . Π˜Π·ΠΌΠ΅Ρ€ΡŒΡ‚Π΅ Π½Π°ΠΊΠ»ΠΎΠ½ этой ΠΊΠ°ΡΠ°Ρ‚Π΅Π»ΡŒΠ½Π°Ρ линия. Когда Π²Ρ‹ Π΄Π΅Π»Π°Π΅Ρ‚Π΅ d x ΠΎΡ‡Π΅Π½ΡŒ малСнькими, сСкущая линия становится ΠΊΠ°ΡΠ°Ρ‚Π΅Π»ΡŒΠ½ΠΎΠΉ. Если Π²Ρ‹ сдСлаСтС d x большими (d x > 2.0) ΠΈ пСрСмСститС ΡΠ²ΠΎΠ±ΠΎΠ΄Π½ΡƒΡŽ Ρ‚ΠΎΡ‡ΠΊΡƒ Π½Π° x , станСт ΠΎΡ‡Π΅Π²ΠΈΠ΄Π½ΠΎ, Ρ‡Ρ‚ΠΎ линия, ΠΏΠ°Ρ€Π°Π»Π»Π΅Π»ΡŒΠ½Π°Ρ сСкущая линия Ρ‡Π΅Ρ€Π΅Π· f(x) Π½Π΅ всСгда касаСтся кубичСской Ρ„ΡƒΠ½ΠΊΡ†ΠΈΠΈ. Π‘Π½ΠΎΠ²Π° сдСлайтС d x малСнькими. Π’ настоящСС врСмя постройтС Ρ‚ΠΎΡ‡ΠΊΡƒ, ΠΎΠΏΡ€Π΅Π΄Π΅Π»ΡΠ΅ΠΌΡƒΡŽ ΠΌΠ΅Ρ€ΠΎΠΉ x ΠΈ Π½Π°ΠΊΠ»ΠΎΠ½ Π»ΠΈΠ½ΠΈΠΈ Ρ‡Π΅Ρ€Π΅Π· Ρ‚ΠΎΡ‡ΠΊΡƒ f(x) [Π’Ρ‹Π±Π΅Ρ€ΠΈΡ‚Π΅ ΠΌΠ΅Ρ€Ρƒ x ΠΈ Π½Π°ΠΊΠ»ΠΎΠ½ ΠΊΠ°ΡΠ°Ρ‚Π΅Π»ΡŒΠ½ΠΎΠΉ Π² этот порядок ΠΈ Plot as ( x , y ) ΠΈΠ· мСню Graph.] Π‘ этим Π½ΠΎΠ²Ρ‹ΠΌ Ρ‚ΠΎΡ‡ΠΊΠ° всС Π΅Ρ‰Π΅ Π²Ρ‹Π±Ρ€Π°Π½Π°, ΠΏΠΎΡΡ‚Ρ€ΠΎΠΈΡ‚ΡŒ Π΅Π΅ гСомСтричСскоС мСсто ΠΎΡ‚Π½ΠΎΡΠΈΡ‚Π΅Π»ΡŒΠ½ΠΎ свободной Ρ‚ΠΎΡ‡ΠΊΠΈ x Π½Π° оси x . Π’Π΅ΠΏΠ΅Ρ€ΡŒ Ρƒ вас Π΄ΠΎΠ»ΠΆΠ΅Π½ Π±Ρ‹Ρ‚ΡŒ эскиз, ΠΊΠΎΡ‚ΠΎΡ€Ρ‹ΠΉ выглядит Ρ‡Ρ‚ΠΎ-Ρ‚ΠΎ Π²Ρ€ΠΎΠ΄Π΅ рисунка 11.4. Какой Ρ„ΠΎΡ€ΠΌΡ‹ эта гСомСтричСская кривая? Как эта кривая относятся ΠΊ ΠΏΡ€ΠΎΠΈΠ·Π²ΠΎΠ΄Π½ΠΎΠΉ кубичСской Ρ„ΡƒΠ½ΠΊΡ†ΠΈΠΈ?

      Рисунок 11.4: ΠŸΠΎΡΡ‚Ρ€ΠΎΠ΅Π½ΠΈΠ΅ ΠΊΠ°ΡΠ°Ρ‚Π΅Π»ΡŒΠ½ΠΎΠΉ ΠΊ кубичСской Ρ„ΡƒΠ½ΠΊΡ†ΠΈΠΈ ΠΈ построСниС Π΅Π΅ Π½Π°ΠΊΠ»ΠΎΠ½Π°

      Β 

      Β Β Β Β Β Β Β Β Β Β Β  Π‘ΠΎΠ·Π΄Π°Ρ‚ΡŒ Π²Ρ‹Ρ€Π°ΠΆΠ΅Π½ΠΈΠ΅ для ΠΏΡ€ΠΎΠΈΠ·Π²ΠΎΠ΄Π½ΠΎΠΉ вашСй кубичСской Ρ„ΡƒΠ½ΠΊΡ†ΠΈΠΈ, Π²Ρ‹Π±Ρ€Π°Π² Π²Ρ‹Ρ€Π°ΠΆΠ΅Π½ΠΈΠ΅ для f(x) ΠΈ Π²Ρ‹Π±Ρ€Π°Π² Derivative Π² мСню Graph. Π”ΠΎΠ»ΠΆΠ½ΠΎ ΠΏΠΎΠ»ΡƒΡ‡ΠΈΡ‚ΡŒΡΡ ΡΠ»Π΅Π΄ΡƒΡŽΡ‰Π΅Π΅ Π²Ρ‹Ρ€Π°ΠΆΠ΅Π½ΠΈΠ΅: f(x) = 3ax 2 +2bx+c . Π˜ΡΠΏΠΎΠ»ΡŒΠ·ΡƒΠΉΡ‚Π΅ ΠΊΠ°Π»ΡŒΠΊΡƒΠ»ΡΡ‚ΠΎΡ€, Ρ‡Ρ‚ΠΎΠ±Ρ‹ ΠΏΠΎΠ»ΡƒΡ‡ΠΈΡ‚ΡŒ Π·Π½Π°Ρ‡Π΅Π½ΠΈΠ΅ этого выраТСния для Ρ‚ΠΎΡ‡ΠΊΠΈ x . Π‘Π΄Π΅Π»Π°Ρ‚ΡŒ d x ΠΎΡ‡Π΅Π½ΡŒ малСнький. Π§Ρ‚ΠΎ Π²Ρ‹ Π·Π°ΠΌΠ΅Ρ‡Π°Π΅Ρ‚Π΅ ΠΎ Π½Π°ΠΊΠ»ΠΎΠ½ ΠΊΠ°ΡΠ°Ρ‚Π΅Π»ΡŒΠ½ΠΎΠΉ ΠΈ Π·Π½Π°Ρ‡Π΅Π½ΠΈΠ΅ этого выраТСния Π² Ρ‚ΠΎΡ‡ΠΊΠ΅ x ? ΠŸΠ΅Ρ€Π΅ΠΌΠ΅ΡΡ‚ΠΈΡ‚Π΅ ΡΠ²ΠΎΠ±ΠΎΠ΄Π½ΡƒΡŽ Ρ‚ΠΎΡ‡ΠΊΡƒ x . Π­Ρ‚ΠΈ значСния ΠΎΡΡ‚Π°ΡŽΡ‚ΡΡ ΠΏΡ€Π΅ΠΆΠ½ΠΈΠΌΠΈ? ΠžΡΡ‚Π°ΡŽΡ‚ΡΡ Π»ΠΈ ΠΎΠ½ΠΈ Ρ€Π°Π²Π½Ρ‹ Π΄Ρ€ΡƒΠ³ Π΄Ρ€ΡƒΠ³Ρƒ?

      Β Β Β Β Β Β Β Β Β Β Β  Π·Π½Π°Ρ‡Π΅Π½ΠΈΠ΅ ΠΏΡ€ΠΎΠΈΠ·Π²ΠΎΠ΄Π½ΠΎΠΉ мСняСтся Π² зависимости ΠΎΡ‚ полоТСния x , поэтому ΠΌΡ‹ ΠΌΠΎΠΆΠ΅ΠΌ ΠΈΠ·ΠΎΠ±Ρ€Π°Π·ΠΈΡ‚ΡŒ эту Π·Π°Π²ΠΈΡΠΈΠΌΠΎΡΡ‚ΡŒ графичСски. Π’Ρ‹Π±Π΅Ρ€ΠΈΡ‚Π΅ Π²Ρ‹Ρ€Π°ΠΆΠ΅Π½ΠΈΠ΅ для ΠΏΡ€ΠΎΠΈΠ·Π²ΠΎΠ΄Π½ΠΎΠΉ ( f(x) = 3ax 2 +2bx+c ) ΠΈ Π²Ρ‹Π±Π΅Ρ€ΠΈΡ‚Π΅ Ѐункция Π³Ρ€Π°Ρ„ΠΈΠΊΠ° Π½Π° Π³Ρ€Π°Ρ„ΠΈΠΊΠ΅ мСню. Какой Ρ„ΠΎΡ€ΠΌΡ‹ Π³Ρ€Π°Ρ„ΠΈΠΊ ΠΏΡ€ΠΎΠΈΠ·Π²ΠΎΠ΄Π½ΠΎΠΉ Ρ„ΡƒΠ½ΠΊΡ†ΠΈΠΈ? Π­Ρ‚Π° кривая совпадаСт Π²Π½ΡƒΡ‚Ρ€ΠΈ с гСомСтричСским мСстом Π½Π°ΠΊΠ»ΠΎΠ½Π° ΠΊΠ°ΡΠ°Ρ‚Π΅Π»ΡŒΠ½ΠΎΠΉ, проходящСй Ρ‡Π΅Ρ€Π΅Π· Ρ‚ΠΎΡ‡ΠΊΡƒ f(x)? Β Π£Π²Π΅Π»ΠΈΡ‡ΠΈΠ²Π°ΠΉΡ‚Π΅ d x , ΠΏΠΎΠΊΠ° Π½Π΅ ΡƒΠ²ΠΈΠ΄ΠΈΡ‚Π΅ Π½Π΅ΠΊΠΎΡ‚ΠΎΡ€ΠΎΠ΅ Ρ€Π°Π·Π΄Π΅Π»Π΅Π½ΠΈΠ΅ ΠΌΠ΅ΠΆΠ΄Ρƒ этими двумя ΠΊΡ€ΠΈΠ²Ρ‹Π΅. ΠžΡΡ‚Π°ΡŽΡ‚ΡΡ Π»ΠΈ Ρ„ΠΎΡ€ΠΌΡ‹ ΠΊΡ€ΠΈΠ²Ρ‹Ρ… ΠΎΠ΄ΠΈΠ½Π°ΠΊΠΎΠ²Ρ‹ΠΌΠΈ, Π΄Π°ΠΆΠ΅ Ссли ΠΎΠ½ΠΈ Ρ€Π°Π·Π΄Π΅Π»Π΅Π½Ρ‹? Π£ΠΌΠ΅Π½ΡŒΡˆΠΈΡ‚ΡŒ d x ΠΏΠΎΠΊΠ° Π΄Π²Π΅ ΠΊΡ€ΠΈΠ²Ρ‹Π΅ снова Π½Π΅ совпадут.

      ΠœΠ΅Π½ΡΠΉΡ‚Π΅ коэффициСнты, Π° Π·Π°Ρ‚Π΅ΠΌ Π½Π°ΠΏΠΈΡˆΠΈΡ‚Π΅ Ρ‚Ρ€ΠΈ Π²Π΅Ρ‰ΠΈ ΠΎ связи ΠΌΠ΅ΠΆΠ΄Ρƒ Π³Ρ€Π°Ρ„ΠΈΠΊΠ°ΠΌΠΈ кубичСского функция ΠΈ Π΅Π΅ производная (см. рис. 11.5).

      Β 

      Рисунок 11.5: Π“Ρ€Π°Ρ„ΠΈΠΊΠΈ кубичСской Ρ„ΡƒΠ½ΠΊΡ†ΠΈΠΈ ΠΈ Π΅Π΅ ΠΏΡ€ΠΎΠΈΠ·Π²ΠΎΠ΄Π½ΠΎΠΉ

      Β 

      ИсслСдованиС ΠΏΡ€ΠΎΠΈΠ·Π²ΠΎΠ΄Π½ΠΎΠΉ ΠΊΠ²Π°Π΄Ρ€Π°Ρ‚Π½ΠΎΠ³ΠΎ числа

      ΠΠ°Ρ‡Π°Ρ‚ΡŒ Π½ΠΎΠ²Ρ‹ΠΉ эскиз, создайтС оси ΠΈ помСститС ΡΠ²ΠΎΠ±ΠΎΠ΄Π½ΡƒΡŽ Ρ‚ΠΎΡ‡ΠΊΡƒ Π½Π° оси x . ΠŸΠ΅Ρ€Π΅ΠΈΠΌΠ΅Π½ΡƒΠΉΡ‚Π΅ эту Ρ‚ΠΎΡ‡ΠΊΡƒ Π² x . Π˜ΡΠΏΠΎΠ»ΡŒΠ·ΡƒΠΉΡ‚Π΅ свои 9Π˜Π½ΡΡ‚Ρ€ΡƒΠΌΠ΅Π½Ρ‚ ΠΏΠΎΠ»Π·ΡƒΠ½ΠΊΠ° 1500 для создания Ρ‚Ρ€Π΅Ρ… ΠΊΠΎΠ½Ρ‚Ρ€ΠΎΠ»ΡŒΠ½Ρ‹Π΅ Ρ‚ΠΎΡ‡ΠΊΠΈ Π°, Π± ΠΈ Π². Π‘Π»Π΅Π΄ΠΎΠ²Π°Ρ‚ΡŒ ΠΌΠ΅Ρ‚ΠΎΠ΄, ΠΊΠΎΡ‚ΠΎΡ€Ρ‹ΠΉ Π²Ρ‹ использовали для кубичСской Ρ„ΡƒΠ½ΠΊΡ†ΠΈΠΈ, Ρ‡Ρ‚ΠΎΠ±Ρ‹ ΡΠΎΠ·Π΄Π°Ρ‚ΡŒ Ρ„ΡƒΠ½ΠΊΡ†ΠΈΡŽ f(x) = x 2 +bx+c ΠΈ ΠΏΠΎΡΡ‚Ρ€ΠΎΠΈΡ‚ΡŒ Π΅Π΅ Π³Ρ€Π°Ρ„ΠΈΠΊ. Π‘ΠΎΠ·Π΄Π°ΠΉΡ‚Π΅ ΠΏΠΎΠ»Π·ΡƒΠ½ΠΎΠΊ dx ΠΈ ΠΈΡΠΏΠΎΠ»ΡŒΠ·ΡƒΠΉΡ‚Π΅ Π΅Π³ΠΎ для построСния сСкущСй ΠΊ Π²Π°ΡˆΠ΅ΠΌΡƒ квадратичная функция. НанСситС Π½Π° Π³Ρ€Π°Ρ„ΠΈΠΊ Ρ‚ΠΎΡ‡ΠΊΠΈ x , f(x) . ΠŸΠΎΡΡ‚Ρ€ΠΎΠΈΡ‚ΡŒ ΠΏΡ€ΡΠΌΡƒΡŽ Ρ‡Π΅Ρ€Π΅Π· f(x) ΠΏΠ°Ρ€Π°Π»Π»Π΅Π»ΡŒΠ½ΠΎ вашСй сСкущСй. ΠŸΠ΅Ρ€Π΅ΠΌΠ΅ΡΡ‚ΠΈΡ‚Π΅ Ρ‚ΠΎΡ‡ΠΊΡƒ x , Ρ‡Ρ‚ΠΎΠ±Ρ‹ ΡƒΠ²ΠΈΠ΄Π΅Ρ‚ΡŒ ΠΊΠ°ΡΠ°Ρ‚Π΅Π»ΡŒΠ½ΡƒΡŽ ΠΈ сСкущиС Π»ΠΈΠ½ΠΈΠΈ двиТутся Π²ΠΎΠΊΡ€ΡƒΠ³ вашСй ΠΏΠ°Ρ€Π°Π±ΠΎΠ»Ρ‹. Π’Π°Ρ€ΡŒΠΈΡ€ΡƒΠΉΡ‚Π΅ Π΄Ρ…. Π‘Π΄Π΅Π»Π°ΠΉΡ‚Π΅ Π΅Π³ΠΎ Π½Π°ΡΡ‚ΠΎΠ»ΡŒΠΊΠΎ большим, насколько смоТСтС. ΠŸΠ΅Ρ€Π΅ΠΌΠ΅ΡΡ‚ΠΈΡ‚ΡŒ Ρ… ΠΎΠΏΡΡ‚ΡŒ Ρ‚Π°ΠΊΠΈ. ΠŸΠΎΡΠ²Π»ΡΠ΅Ρ‚ΡΡ Π»ΠΈ ΠΊΠΎΠ³Π΄Π°-Π½ΠΈΠ±ΡƒΠ΄ΡŒ ΠΊΠ°ΡΠ°Ρ‚Π΅Π»ΡŒΠ½Π°Ρ , Π° Π½Π΅ , Ρ‡Ρ‚ΠΎΠ±Ρ‹ ΠΊΠ°ΡΠ°Ρ‚ΡŒΡΡ ΠΈΠ·Π³ΠΈΠ±? Π’Π΅Ρ€Π½ΠΈΡ‚Π΅ΡΡŒ ΠΊ своСму кубичСскому эскизу ΠΈ ΠΏΠΎΠΏΡ€ΠΎΠ±ΡƒΠΉΡ‚Π΅ Ρ‚ΠΎ ΠΆΠ΅ самоС. Π§Ρ‚ΠΎ каТСтся ΡΠΏΠ΅Ρ†ΠΈΠ°Π»ΡŒΠ½Ρ‹Π΅ ΠΎ ΠΊΠ²Π°Π΄Ρ€Π°Ρ‚ΠΈΡ‡Π½ΠΎΠΉ Ρ„ΡƒΠ½ΠΊΡ†ΠΈΠΈ?

      НазначСниС 11.1 : Π‘ΠΎΠ·Π΄Π°ΠΉΡ‚Π΅ Π²Ρ‹Ρ€Π°ΠΆΠ΅Π½ΠΈΠ΅ для ΠΏΡ€ΠΎΠΈΠ·Π²ΠΎΠ΄Π½ΠΎΠΉ вашСго ΠΊΠ²Π°Π΄Ρ€Π°Ρ‚ΠΈΡ‡Π½ΠΎΠ³ΠΎ f'(x)=2ax+b (ΠΈΠ»ΠΈ спроситС GSP для ΠΏΡ€ΠΎΠΈΠ·Π²ΠΎΠ΄Π½ΠΎΠΉ вашСй Ρ„ΡƒΠ½ΠΊΡ†ΠΈΠΈ) ΠΈ постройтС Π΅Π΅ Π³Ρ€Π°Ρ„ΠΈΠΊ. Π—Π½Π°Ρ‡Π΅Π½ΠΈΠ΅ f'(x) Ρ€Π°Π²Π½ΠΎ Π½Π°ΠΊΠ»ΠΎΠ½Ρƒ сСкущСй для любого x?

      Π”ΠΎΠΊΠ°ΠΆΠΈΡ‚Π΅ (алгСбраичСски) Ρ‡Ρ‚ΠΎ сСкущая Ρ‡Π΅Ρ€Π΅Π· Ρ‚ΠΎΡ‡ΠΊΠΈ ((x-dx/2), f(x-dx/2)) ΠΈ ((x+dx/2), f(x+dx/2)) всСгда Π±ΡƒΠ΄ΡƒΡ‚ ΠΏΠ°Ρ€Π°Π»Π»Π΅Π»ΡŒΠ½Ρ‹ ΠΊΠ°ΡΠ°Ρ‚Π΅Π»ΡŒΠ½ΠΎΠΉ Π² Ρ‚ΠΎΡ‡ΠΊΠ΅ (x, f(x)) для любой ΠΊΠ²Π°Π΄Ρ€Π°Ρ‚ΠΈΡ‡Π½ΠΎΠΉ Ρ„ΡƒΠ½ΠΊΡ†ΠΈΠΈ!

      Β 

      Β Β Β Β Β Β Β Β Β Β Β  ИспользованиС описанныС Π²Ρ‹ΡˆΠ΅ ΠΌΠ΅Ρ‚ΠΎΠ΄Ρ‹ построСния ΡΠΎΠ·Π΄Π°ΡŽΡ‚ Π³Ρ€Π°Ρ„ΠΈΠΊ Ρ„ΡƒΠ½ΠΊΡ†ΠΈΠΈ ΠΎΠ±Ρ‰Π΅Π³ΠΎ синуса: f(x)= asin(bx+c ) ΠΈ постройтС Ρ‚ΠΎΡ‡ΠΊΠΈ (x+ d x/2, f(x+ d x/2)) ΠΈ (x- d , d 2 f(x- d x/2)) для построСния сСкущСй синусоиды. [ΠŸΡ€ΠΈΠΌΠ΅Ρ‡Π°Π½ΠΈΠ΅. Π’Π°ΠΌ потрСбуСтся ΠΈΠ·ΠΌΠ΅Π½ΠΈΡ‚ΡŒ Preferences для измСрСния ΡƒΠ³Π»Π° Π² мСню Edit Π΄ΠΎ Ρ€Π°Π΄ΠΈΠ°Π½ .] Π‘Π΄Π΅Π»Π°ΠΉΡ‚Π΅ d x ΠΎΡ‡Π΅Π½ΡŒ малСньким ΠΈ ΠΈΠ·ΠΌΠ΅Ρ€ΡŒΡ‚Π΅ Π½Π°ΠΊΠ»ΠΎΠ½ сСкущая-ΠΊΠ°ΡΠ°Ρ‚Π΅Π»ΡŒΠ½Π°Ρ линия. Участок Ρ… ΠΈ Π½Π°ΠΊΠ»ΠΎΠ½ ΠΊΠ°ΠΊ ( x , y ). ΠŸΠΎΡΡ‚Ρ€ΠΎΠΉΡ‚Π΅ гСомСтричСскоС мСсто этой Π½ΠΎΠ²ΠΎΠΉ Ρ‚ΠΎΡ‡ΠΊΠΈ Π½Π° Π³Ρ€Π°Ρ„ΠΈΠΊΠ΅ ΠΏΡ€ΠΈ ΠΈΠ·ΠΌΠ΅Π½Π΅Π½ΠΈΠΈ x . Π§Π΅ΠΌ эта новая кривая ΠΏΠΎΡ…ΠΎΠΆΠ° Π½Π° Π’Π°ΡˆΠ° синусоида? Π’Π°Ρ€ΡŒΠΈΡ€ΡƒΠΉΡ‚Π΅ Π° ΠΈ с. Как Π²Π»ΠΈΡΡŽΡ‚ Π»ΠΈ эти коэффициСнты Π½Π° Π΄Π²Π΅ ΠΊΡ€ΠΈΠ²Ρ‹Π΅? Π’Π°Ρ€ΡŒΠΈΡ€ΠΎΠ²Π°Ρ‚ΡŒ Π±. Как этот коэффициСнт влияСт Π½Π° Π΄Π²Π΅ ΠΊΡ€ΠΈΠ²Ρ‹Π΅? Π‘Π΄Π΅Π»Π°ΠΉΡ‚Π΅ b = 1,00. Π§Ρ‚ΠΎ Π²Ρ‹ Π·Π°ΠΌΠ΅Ρ‚ΠΈΠ»ΠΈ Π² Π΄Π²ΡƒΡ… ΠΊΡ€ΠΈΠ²Ρ‹Π΅? Π§Π΅ΠΌ ΠΎΠ½ΠΈ ΠΎΡ‚Π»ΠΈΡ‡Π°ΡŽΡ‚ΡΡ ΠΈ Ρ‡Π΅ΠΌ ΠΎΠ½ΠΈ ΠΏΠΎΡ…ΠΎΠΆΠΈ? Π˜Π·ΠΌΠ΅Π½ΠΈΡ‚Π΅ a ΠΈ c снова. Π§Ρ‚ΠΎ Ρ‚Ρ‹ Π·Π°ΠΌΠ΅Ρ‚ΠΈΠ»? УстановитС ΠΎΠ±Π° ΠΏΠ°Ρ€Π°ΠΌΠ΅Ρ‚Ρ€Π° a ΠΈ b Ρ€Π°Π²Π΅Π½ 1; ΡƒΡΡ‚Π°Π½ΠΎΠ²ΠΈΡ‚ΡŒ с Ρ€Π°Π²Π½Ρ‹ΠΌ Π½ΡƒΠ»ΡŽ. Π‘Π΄Π΅Π»Π°ΠΉΡ‚Π΅ ΠΏΡ€Π΅Π΄ΠΏΠΎΠ»ΠΎΠΆΠ΅Π½ΠΈΠ΅ ΠΎ Ρ„ΡƒΠ½ΠΊΡ†ΠΈΠΈ, которая создал Π±Ρ‹ ΠΊΡ€ΠΈΠ²ΡƒΡŽ, ΠΈΠ·ΠΎΠ±Ρ€Π°ΠΆΠ°ΡŽΡ‰ΡƒΡŽ Π½Π°ΠΊΠ»ΠΎΠ½ ΠΊΠ°ΡΠ°Ρ‚Π΅Π»ΡŒΠ½ΠΎΠΉ ΠΊ синусу Ρ… .

      Β Β Β Β Β Β Β Β Β Β Β  НазначСниС 11.2 : Π‘ΠΎΠ·Π΄Π°ΠΉΡ‚Π΅ ΠΏΡ€ΠΎΠΈΠ·Π²ΠΎΠ΄Π½ΡƒΡŽ Ρ„ΡƒΠ½ΠΊΡ†ΠΈΡŽ g(x) asin(bx+c) , ΠΈΡΠΏΠΎΠ»ΡŒΠ·ΡƒΡ измСрСния ΠΏΠ°Ρ€Π°ΠΌΠ΅Ρ‚Ρ€ΠΎΠ² ΠΈ Π½ΠΎΠ²ΡƒΡŽ Ρ„ΡƒΠ½ΠΊΡ†ΠΈΡŽ . ΠΏΡƒΠ½ΠΊΡ‚ мСню Β«Π“Ρ€Π°Ρ„ΠΈΠΊΒ». Π‘ΡŽΠΆΠ΅Ρ‚ Π³(Ρ…). Π“Ρ€Π°Ρ„ΠΈΠΊ вашСй ΠΏΡ€ΠΎΠΈΠ·Π²ΠΎΠ΄Π½ΠΎΠΉ Ρ„ΡƒΠ½ΠΊΡ†ΠΈΠΈ Π΄ΠΎΠ»ΠΆΠ΅Π½ ΡΠΎΠ²ΠΏΠ°Π΄Π°Ρ‚ΡŒ с Π³Ρ€Π°Ρ„ΠΈΠΊΠΎΠΌ Π½Π°ΠΊΠ»ΠΎΠ½ ΠΊΠ°ΡΠ°Ρ‚Π΅Π»ΡŒΠ½ΠΎΠΉ ΠΊ f(x) для ΠΎΡ‡Π΅Π½ΡŒ малСнький d x . Если это Π½Π΅ измСняСтся коэффициСнт b. ΠšΠ°ΠΊΡƒΡŽ Ρ€ΠΎΠ»ΡŒ ΠΈΠ³Ρ€Π°Π΅Ρ‚ b Π² Π½Π°ΠΊΠ»ΠΎΠ½Π΅ функция?

      Β 

      Рисунок 11.6

      ΠŸΠΎΡΡ‚Ρ€ΠΎΠ΅Π½ΠΈΠ΅ ΠΏΠΎΠ»ΠΈΠ³ΠΎΠ½ΠΎΠ² для ΠŸΡ€ΠΈΠ±Π»ΠΈΠ·ΠΈΡ‚Π΅Π»ΡŒΠ½ΠΎ Π Π°ΠΉΠΎΠ½

      Β Β Β Β Β  слово Π˜Π½Ρ‚Π΅Π³Ρ€Π°Ρ†ΠΈΡ нСсСт Π² сСбС понятиС , ΠΎΠ±ΡŠΠ΅Π΄ΠΈΠ½ΡΡŽΡ‰ΠΈΡ…ΡΡ вмСстС. Если ΠΌΡ‹ рассмотрим ΡΠΈΡ‚ΡƒΠ°Ρ†ΠΈΡŽ воТдСния автомобиля Π² ΠΏΡƒΡ‚ΠΈ Π² Ρ‚Π΅Ρ‡Π΅Π½ΠΈΠ΅ Π΄Π²ΡƒΡ… часов, для НапримСр, ΠΎΠ±Ρ‰Π΅Π΅ расстояниС, ΠΏΡ€ΠΎΠΉΠ΄Π΅Π½Π½ΠΎΠ΅ Π·Π° Π΄Π²Π° часа, Π±ΡƒΠ΄Π΅Ρ‚ нашим срСдним ΡΠΊΠΎΡ€ΠΎΡΡ‚ΡŒ умноТаСтся Π½Π° ΠΏΡ€ΠΎΠΉΠ΄Π΅Π½Π½ΠΎΠ΅ врСмя. Если Π±Ρ‹ ΠΌΡ‹ ΠΌΠΎΠ³Π»ΠΈ ΠΏΡ€ΠΎΠΉΡ‚ΠΈ вСсь ΠΏΡƒΡ‚ΡŒ с ΠΎΠ΄Π½ΠΎΠΉ ΡΠΊΠΎΡ€ΠΎΡΡ‚ΡŒΡŽ (скаТСм, 50 миль Π² час), Ρ‚ΠΎ Π³Ρ€Π°Ρ„ΠΈΠΊ зависимости скорости ΠΎΡ‚ Π²Ρ€Π΅ΠΌΠ΅Π½ΠΈ Π±ΡƒΠ΄Π΅Ρ‚ ΠΏΡ€Π΅Π΄ΡΡ‚Π°Π²Π»ΡΡ‚ΡŒ собой Π³ΠΎΡ€ΠΈΠ·ΠΎΠ½Ρ‚Π°Π»ΡŒΠ½ΡƒΡŽ ΠΏΡ€ΡΠΌΡƒΡŽ линию, Π° ΠΏΡ€ΠΎΠΉΠ΄Π΅Π½Π½ΠΎΠ΅ расстояниС Π±ΡƒΠ΄Π΅Ρ‚ Ρ€Π°Π²Π½ΠΎ ΠΏΠ»ΠΎΡ‰Π°Π΄ΠΈ ΠΏΡ€ΡΠΌΠΎΡƒΠ³ΠΎΠ»ΡŒΠ½ΠΈΠΊΠ°, ΠΏΠΎΠΊΠ°Π·Π°Π½Π½ΠΎΠ³ΠΎ Π½Π° рис. 11.7. это 100 миль (50×2).

      Π’ΠΎΠ·Π²Ρ€Π°Ρ‚ ΠΊ квадратичная функция, управляСмая вашими ΠΏΠΎΠ»Π·ΡƒΠ½ΠΊΠ°ΠΌΠΈ ΠΈΠ»ΠΈ ΠΏΠ°Ρ€Π°ΠΌΠ΅Ρ‚Ρ€Π°ΠΌΠΈ для a, b ΠΈ c. ΠŸΠΎΡΡ‚Ρ€ΠΎΠΈΡ‚ΡŒ ΠΎΡ‚Ρ€Π°ΠΆΠ΅Π½Π½ΠΎΠ΅ ΠΈΠ·ΠΎΠ±Ρ€Π°ΠΆΠ΅Π½ΠΈΠ΅ Ρ‚ΠΎΡ‡ΠΊΠΈ LL ΠΎΡ‚Π½ΠΎΡΠΈΡ‚Π΅Π»ΡŒΠ½ΠΎ Π²Π΅Ρ€Ρ‚ΠΈΠΊΠ°Π»ΡŒΠ½ΠΎΠΉ оси. ΠŸΠ΅Ρ€Π΅ΠΌΠ΅ΡΡ‚ΠΈΡ‚Π΅ Ρ‚ΠΎΡ‡ΠΊΡƒ UL Π² ΠΎΡ‚Ρ€Π°ΠΆΠ΅Π½ΠΈΠ΅ LL с ΠΏΠΎΠΌΠΎΡ‰ΡŒΡŽ ΠΊΠ½ΠΎΠΏΠΊΠΈ двиТСния. Π’Π΅ΠΏΠ΅Ρ€ΡŒ ΠΈΠ·ΠΌΠ΅Π½ΠΈΡ‚Π΅ ΠΏΠ°Ρ€Π°ΠΌΠ΅Ρ‚Ρ€ (b) для коэффициСнт x -Ρ‡Π»Π΅Π½Π° Π² вашСм функция. Π§Ρ‚ΠΎ Π²Ρ‹ Π·Π°ΠΌΠ΅Ρ‚ΠΈΠ»ΠΈ Π² ΠΏΡ€ΡΠΌΠΎΡƒΠ³ΠΎΠ»ΡŒΠ½ΠΈΠΊΠ΅ накоплСния ΠΈΠ»ΠΈ Π·Π½Π°Ρ‡Π΅Π½ΠΈΠΈ для ΠΏΠ»ΠΎΡ‰Π°Π΄ΡŒ ΠΏΠΎΠ΄ ΠΊΡ€ΠΈΠ²ΠΎΠΉ? Π¦ΠΈΡ„Ρ€Ρ‹ 11.16 ΠΈ 11.17 ΠΈΠ»Π»ΡŽΡΡ‚Ρ€ΠΈΡ€ΡƒΡŽΡ‚ ΡΠΈΡ‚ΡƒΠ°Ρ†ΠΈΡŽ для Π΄Π²ΡƒΡ… Ρ€Π°Π·Π»ΠΈΡ‡Π½Ρ‹Ρ… Π·Π½Π°Ρ‡Π΅Π½ΠΈΠΉ b.

      Рисунок 11.16: ΠŸΠ»ΠΎΡ‰Π°Π΄ΡŒ ΠΏΠΎΠ΄ ΠΊΠ²Π°Π΄Ρ€Π°Ρ‚ΠΈΡ‡Π½Ρ‹ΠΌ ΡƒΡ€Π°Π²Π½Π΅Π½ΠΈΠ΅ΠΌ с Π½ΠΈΠΆΠ½Π΅ΠΉ Π³Ρ€Π°Π½ΠΈΡ†Π΅ΠΉ = -ВСрхняя Π³Ρ€Π°Π½ΠΈΡ†Π°

      ΠšΠ°ΡΠ°Ρ‚Π΅Π»ΡŒΠ½Ρ‹Π΅ Π»ΠΈΠ½ΠΈΠΈ: ΡƒΡ€Π°Π²Π½Π΅Π½ΠΈΠ΅, Π½Π°ΠΊΠ»ΠΎΠ½ ΠΈ Π³Ρ€Π°Ρ„ΠΈΠΊ

      На Π»Π°Ρ‚Ρ‹Π½ΠΈ слово ΠΊΠ°ΡΠ°Ρ‚Π΅Π»ΡŒΠ½Π°Ρ ΠΎΠ·Π½Π°Ρ‡Π°Π΅Ρ‚ Β«ΠΊΠ°ΡΠ°Ρ‚ΡŒΡΡΒ». Π˜Ρ‚Π°ΠΊ, ΠΊΠ°ΡΠ°Ρ‚Π΅Π»ΡŒΠ½Π°Ρ β€” это линия, которая касаСтся. Рассмотрим вСлосипСд, двиТущийся ΠΏΠΎ Ρ€ΠΎΠ²Π½ΠΎΠΌΡƒ Ρ‚Ρ€ΠΎΡ‚ΡƒΠ°Ρ€Ρƒ. Π”ΠΎΡ€ΠΎΠ³Π° ΠΏΠΎ сущСству касаСтся вСлосипСдного колСса, ΠΏΠΎΡΠΊΠΎΠ»ΡŒΠΊΡƒ ΠΎΠ½Π° касаСтся колСса Π² ΠΎΠ΄Π½ΠΎΠΉ Ρ‚ΠΎΡ‡ΠΊΠ΅. Π’ этой ΡΡ‚Π°Ρ‚ΡŒΠ΅ ΠΌΡ‹ Π΄ΠΎΠΏΠΎΠ»Π½ΠΈΡ‚Π΅Π»ΡŒΠ½ΠΎ обсудим Π·Π½Π°Ρ‡Π΅Π½ΠΈΠ΅ ΠΊΠ°ΡΠ°Ρ‚Π΅Π»ΡŒΠ½ΠΎΠΉ, Ρ„ΠΎΡ€ΠΌΡƒΠ»Ρƒ ΠΊΠ°ΡΠ°Ρ‚Π΅Π»ΡŒΠ½ΠΎΠΉ ΠΈ Π·Π½Π°Ρ‡Π΅Π½ΠΈΠ΅ Π½Π°ΠΊΠ»ΠΎΠ½Π° ΠΊΠ°ΡΠ°Ρ‚Π΅Π»ΡŒΠ½ΠΎΠΉ.

      ΠžΠΏΡ€Π΅Π΄Π΅Π»Π΅Π½ΠΈΠ΅ ΠΈ Ρ„ΠΎΡ€ΠΌΡƒΠ»Π° ΠΊΠ°ΡΠ°Ρ‚Π΅Π»ΡŒΠ½ΠΎΠΉ Π»ΠΈΠ½ΠΈΠΈ

      ΠšΠ°ΡΠ°Ρ‚Π΅Π»ΡŒΠ½Π°Ρ линия β€” это линия, которая «просто касаСтся» Ρ‚ΠΎΡ‡ΠΊΠΈ. Π•Π³ΠΎ Ρ‚Π°ΠΊΠΆΠ΅ ΠΌΠΎΠΆΠ½ΠΎ ΠΎΠΏΡ€Π΅Π΄Π΅Π»ΠΈΡ‚ΡŒ ΠΊΠ°ΠΊ линию, ΡΠΎΠ΅Π΄ΠΈΠ½ΡΡŽΡ‰ΡƒΡŽ Π΄Π²Π΅ бСсконСчно Π±Π»ΠΈΠ·ΠΊΠΈΠ΅ Ρ‚ΠΎΡ‡ΠΊΠΈ Π½Π° ΠΊΡ€ΠΈΠ²ΠΎΠΉ.

      ΠšΠ°ΡΠ°Ρ‚Π΅Π»ΡŒΠ½Π°Ρ ΠΊ ΠΊΡ€ΠΈΠ²ΠΎΠΉ Π² Ρ‚ΠΎΡ‡ΠΊΠ΅ с ΠΊΠΎΠΎΡ€Π΄ΠΈΠ½Π°Ρ‚Π°ΠΌΠΈ , β€” это линия, проходящая Ρ‡Π΅Ρ€Π΅Π· с Π½Π°ΠΊΠ»ΠΎΠ½ΠΎΠΌ

      , Ссли ΠΏΡ€Π΅Π΄Π΅Π» сущСствуСт.

      Π£Ρ€Π°Π²Π½Π΅Π½ΠΈΠ΅ ΠΊΠ°ΡΠ°Ρ‚Π΅Π»ΡŒΠ½ΠΎΠΉ

      Один Π½Π°ΠΊΠ»ΠΎΠ½ , ΡƒΡ€Π°Π²Π½Π΅Π½ΠΈΠ΅ ΠΊΠ°ΡΠ°Ρ‚Π΅Π»ΡŒΠ½ΠΎΠΉ Π»ΠΈΠ½ΠΈΠΈ Ρ‚Π°ΠΊΠΎΠ΅ ΠΆΠ΅, ΠΊΠ°ΠΊ ΠΈ любая другая линия Π² Ρ„ΠΎΡ€ΠΌΠ΅ Ρ‚ΠΎΡ‡ΠΊΠ°-Π½Π°ΠΊΠ»ΠΎΠ½ Ρ‡Π΅Ρ€Π΅Π· Ρ‚ΠΎΡ‡ΠΊΡƒ:

      ΠšΠ°ΡΠ°Ρ‚Π΅Π»ΡŒΠ½Ρ‹Π΅ Π»ΠΈΠ½ΠΈΠΈ Π½Π° Π³Ρ€Π°Ρ„ΠΈΠΊΠ΅

      На ΠΏΡ€ΠΈΠ²Π΅Π΄Π΅Π½Π½ΠΎΠΌ Π½ΠΈΠΆΠ΅ Π³Ρ€Π°Ρ„ΠΈΠΊΠ΅ ΠΌΡ‹ Π³ΠΎΠ²ΠΎΡ€ΠΈΠΌ, Ρ‡Ρ‚ΠΎ являСтся ΠΊΠ°ΡΠ°Ρ‚Π΅Π»ΡŒΠ½ΠΎΠΉ линия ΠΊ ΠΊΡ€ΠΈΠ²ΠΎΠΉ Π² Ρ‚ΠΎΡ‡ΠΊΠ΅ . Или ΠΌΡ‹ ΠΌΠΎΠΆΠ΅ΠΌ ΡΠΊΠ°Π·Π°Ρ‚ΡŒ, Ρ‡Ρ‚ΠΎ касаСтся ΠΊΡ€ΠΈΠ²ΠΎΠΉ Π² Ρ‚ΠΎΡ‡ΠΊΠ΅ .

      ΠšΠ°ΡΠ°Ρ‚Π΅Π»ΡŒΠ½Π°Ρ, выдСлСнная Π·Π΅Π»Π΅Π½Ρ‹ΠΌ Ρ†Π²Π΅Ρ‚ΠΎΠΌ, просто касаСтся ΠΊΡ€ΠΈΠ²ΠΎΠΉ f Π² Ρ‚ΠΎΡ‡ΠΊΠ΅ P — StudySmarter Original

      ΠžΠ±Ρ€Π°Ρ‚ΠΈΡ‚Π΅ Π²Π½ΠΈΠΌΠ°Π½ΠΈΠ΅, ΠΊΠ°ΠΊ ΠΊΠ°ΡΠ°Ρ‚Π΅Π»ΡŒΠ½Π°Ρ «просто касаСтся» ΠΊΡ€ΠΈΠ²ΠΎΠΉ Π² Ρ‚ΠΎΡ‡ΠΊΠ΅ P.

      Наклон ΠΊΠ°ΡΠ°Ρ‚Π΅Π»ΡŒΠ½ΠΎΠΉ

      ГСомСтрия

      Наклон ΠΊΠ°ΡΠ°Ρ‚Π΅Π»ΡŒΠ½ΠΎΠΉ Π² Ρ‚ΠΎΡ‡ΠΊΠ΅ Π½Π° ΠΊΡ€ΠΈΠ²ΠΎΠΉ Ρ€Π°Π²Π΅Π½ Π½Π°ΠΊΠ»ΠΎΠ½Ρƒ ΠΊΡ€ΠΈΠ²ΠΎΠΉ Π² Ρ‚Π°ΠΊΠΎΠΌ случаС. ΠŸΡ€Π΅Π΄ΠΏΠΎΠ»ΠΎΠΆΠ΅Π½ΠΈΠ΅, ΠΊΠ°ΡΠ°ΡŽΡ‰Π΅Π΅ΡΡ ΠΊΠ°ΡΠ°Ρ‚Π΅Π»ΡŒΠ½Ρ‹Ρ… Π»ΠΈΠ½ΠΈΠΉ, Π·Π°ΠΊΠ»ΡŽΡ‡Π°Π΅Ρ‚ΡΡ Π² Ρ‚ΠΎΠΌ, Ρ‡Ρ‚ΠΎ ΠΏΡ€ΠΈ просмотрС Π³Ρ€Π°Ρ„ΠΈΠΊΠ° ΠΊΡ€ΠΈΠ²ΠΎΠΉ, Ссли Π²Ρ‹ ΡƒΠ²Π΅Π»ΠΈΡ‡ΠΈΡ‚Π΅ ΠΌΠ°ΡΡˆΡ‚Π°Π± достаточно Π±Π»ΠΈΠ·ΠΊΠΎ ΠΊ сСгмСнту ΠΊΡ€ΠΈΠ²ΠΎΠΉ, кривая Π±ΡƒΠ΄Π΅Ρ‚ Π²Ρ‹Π³Π»ΡΠ΄Π΅Ρ‚ΡŒ Π½Π΅ΠΎΡ‚Π»ΠΈΡ‡ΠΈΠΌΠΎΠΉ ΠΎΡ‚ ΠΊΠ°ΡΠ°Ρ‚Π΅Π»ΡŒΠ½ΠΎΠΉ.

      НапримСр, Π΄Π°Π²Π°ΠΉΡ‚Π΅ ΡƒΠ²Π΅Π»ΠΈΡ‡ΠΈΠΌ Π³Ρ€Π°Ρ„ΠΈΠΊ Π²Ρ‹ΡˆΠ΅.

      Π£Π²Π΅Π»ΠΈΡ‡Π΅Π½ΠΈΠ΅ Π² Ρ‚ΠΎΡ‡ΠΊΠ΅, Π³Π΄Π΅ ΠΊΠ°ΡΠ°Ρ‚Π΅Π»ΡŒΠ½Π°Ρ линия касаСтся ΠΊΡ€ΠΈΠ²ΠΎΠΉ — StudySmarter Original

      Π£Π²Π΅Π»ΠΈΡ‡Π΅Π½ΠΈΠ΅ Π΅Ρ‰Π΅ Π½Π΅ΠΌΠ½ΠΎΠ³ΠΎ…

      Π—Π΄Π΅ΡΡŒ ΠΌΡ‹ Π²ΠΈΠ΄ΠΈΠΌ, Ρ‡Ρ‚ΠΎ ΠΊΠ°ΡΠ°Ρ‚Π΅Π»ΡŒΠ½Π°Ρ ΠΈ Ρ‚ΠΎΡ‡ΠΊΠ° ΠΊΡ€ΠΈΠ²ΠΎΠΉ, Π³Π΄Π΅ касаСтся ΠΊΠ°ΡΠ°Ρ‚Π΅Π»ΡŒΠ½Π°Ρ, Π½Π΅Ρ€Π°Π·Π»ΠΈΡ‡ΠΈΠΌΡ‹ — StudySmarter Original

      ΠžΠ±Ρ€Π°Ρ‚ΠΈΡ‚Π΅ Π²Π½ΠΈΠΌΠ°Π½ΠΈΠ΅ Π½Π° Ρ‚ΠΎ, ΠΊΠ°ΠΊ линия ΠΊΠ°ΡΠ°Ρ‚Π΅Π»ΡŒΠ½ΠΎΠΉ получаСтся ΠΏΡ€ΠΈ соСдинСнии Π΄Π²ΡƒΡ… бСсконСчно Π±Π»ΠΈΠ·ΠΊΠΈΡ… Ρ‚ΠΎΡ‡Π΅ΠΊ Π½Π° ΠΊΡ€ΠΈΠ²ΠΎΠΉ.

      Π”ΠΎΠΏΠΎΠ»Π½ΠΈΡ‚Π΅Π»ΡŒΠ½ΠΎΠ΅ ΡƒΡ€Π°Π²Π½Π΅Π½ΠΈΠ΅ Π½Π°ΠΊΠ»ΠΎΠ½Π° ΠΊΠ°ΡΠ°Ρ‚Π΅Π»ΡŒΠ½ΠΎΠΉ

      БущСствуСт Π΅Ρ‰Π΅ ΠΎΠ΄Π½Π° вСрсия уравнСния для Π½Π°ΠΊΠ»ΠΎΠ½Π° ΠΊΠ°ΡΠ°Ρ‚Π΅Π»ΡŒΠ½ΠΎΠΉ, с ΠΊΠΎΡ‚ΠΎΡ€ΠΎΠΉ ΠΏΡ€ΠΎΡ‰Π΅ Ρ€Π°Π±ΠΎΡ‚Π°Ρ‚ΡŒ. Π­Ρ‚ΠΎ ΡƒΡ€Π°Π²Π½Π΅Π½ΠΈΠ΅ Π³ΠΎΠ²ΠΎΡ€ΠΈΡ‚ ΠΎ Ρ‚ΠΎΠΌ, Ρ‡Ρ‚ΠΎ Π½Π°ΠΊΠ»ΠΎΠ½ ΠΊΠ°ΡΠ°Ρ‚Π΅Π»ΡŒΠ½ΠΎΠΉ Ρ€Π°Π²Π½ΠΎ

      Π­Ρ‚ΠΎ ΡƒΡ€Π°Π²Π½Π΅Π½ΠΈΠ΅ устанавливаСт ΠΈ . ΠŸΠΎΡΠΊΠΎΠ»ΡŒΠΊΡƒ приблиТаСтся ΠΊ , приблиТаСтся ΠΊ 0. Π’Π°ΠΊΠΈΠΌ ΠΎΠ±Ρ€Π°Π·ΠΎΠΌ, формируСтся Π΄ΠΎΠΏΠΎΠ»Π½ΠΈΡ‚Π΅Π»ΡŒΠ½ΠΎΠ΅ ΡƒΡ€Π°Π²Π½Π΅Π½ΠΈΠ΅ ΠΊΠ°ΡΠ°Ρ‚Π΅Π»ΡŒΠ½ΠΎΠΉ.

      Π­Ρ‚ΠΎ ΡƒΡ€Π°Π²Π½Π΅Π½ΠΈΠ΅ для Π½Π°ΠΊΠ»ΠΎΠ½Π° ΠΊΠ°ΡΠ°Ρ‚Π΅Π»ΡŒΠ½ΠΎΠΉ Π΄ΠΎΠ»ΠΆΠ½ΠΎ ΠΏΠΎΠΊΠ°Π·Π°Ρ‚ΡŒΡΡ Π²Π°ΠΌ Π·Π½Π°ΠΊΠΎΠΌΡ‹ΠΌ… Π­Ρ‚ΠΎ ΡƒΡ€Π°Π²Π½Π΅Π½ΠΈΠ΅ для ΠΏΡ€ΠΎΠΈΠ·Π²ΠΎΠ΄Π½ΠΎΠΉ Ρ„ΡƒΠ½ΠΊΡ†ΠΈΠΈ Π² Ρ‚ΠΎΡ‡ΠΊΠ΅ (a, f(a)). Π˜Ρ‚Π°ΠΊ, ΠΌΠΎΠΆΠ½ΠΎ ΡΠΊΠ°Π·Π°Ρ‚ΡŒ, Ρ‡Ρ‚ΠΎ Π½Π°ΠΊΠ»ΠΎΠ½ ΠΊΠ°ΡΠ°Ρ‚Π΅Π»ΡŒΠ½ΠΎΠΉ ΠΊ ΠΊΡ€ΠΈΠ²ΠΎΠΉ Π² Ρ‚ΠΎΡ‡ΠΊΠ΅ P Ρ€Π°Π²Π΅Π½ ΠΏΡ€ΠΎΠΈΠ·Π²ΠΎΠ΄Π½ΠΎΠΉ ΠΊΡ€ΠΈΠ²ΠΎΠΉ Π² Ρ‚ΠΎΡ‡ΠΊΠ΅ P !

      ΠŸΡ€ΠΈΠΌΠ΅Ρ€Ρ‹ нахоТдСния уравнСния ΠΊΠ°ΡΠ°Ρ‚Π΅Π»ΡŒΠ½ΠΎΠΉ

      ΠŸΡ€ΠΈΠΌΠ΅Ρ€ 1

      НайдитС ΡƒΡ€Π°Π²Π½Π΅Π½ΠΈΠ΅ для ΠΊΠ°ΡΠ°Ρ‚Π΅Π»ΡŒΠ½ΠΎΠΉ ΠΊ Π² Ρ‚ΠΎΡ‡ΠΊΠ΅ (2, 4).

      ΠŸΠΎΡΠΊΠΎΠ»ΡŒΠΊΡƒ Π½Π°ΠΌ Π΄Π°Π½Π° Ρ‚ΠΎΡ‡ΠΊΠ°, всС, Ρ‡Ρ‚ΠΎ Π½Π°ΠΌ Π½ΡƒΠΆΠ½ΠΎ, Ρ‡Ρ‚ΠΎΠ±Ρ‹ ΡΠΎΡΡ‚Π°Π²ΠΈΡ‚ΡŒ ΡƒΡ€Π°Π²Π½Π΅Π½ΠΈΠ΅ ΠΊΠ°ΡΠ°Ρ‚Π΅Π»ΡŒΠ½ΠΎΠΉ, это Π½Π°ΠΊΠ»ΠΎΠ½. Π§Ρ‚ΠΎΠ±Ρ‹ Π½Π°ΠΉΡ‚ΠΈ Π½Π°ΠΊΠ»ΠΎΠ½, Π²ΠΎΡΠΏΠΎΠ»ΡŒΠ·ΡƒΠ΅ΠΌΡΡ Π΄ΠΎΠΏΠΎΠ»Π½ΠΈΡ‚Π΅Π»ΡŒΠ½Ρ‹ΠΌ ΡƒΡ€Π°Π²Π½Π΅Π½ΠΈΠ΅ΠΌ ΠΊΠ°ΡΠ°Ρ‚Π΅Π»ΡŒΠ½ΠΎΠΉ.

      Наклон ΠΊΠ°ΡΠ°Ρ‚Π΅Π»ΡŒΠ½ΠΎΠΉ Π² Ρ‚ΠΎΡ‡ΠΊΠ°Ρ… (2, 4) Ρ€Π°Π²Π΅Π½

      Π˜Ρ‚Π°ΠΊ, ΡƒΡ€Π°Π²Π½Π΅Π½ΠΈΠ΅ ΠΊΠ°ΡΠ°Ρ‚Π΅Π»ΡŒΠ½ΠΎΠΉ ΠΊ f(x) Π² (2, 4) Π΅ΡΡ‚ΡŒ.

      ВспомнитС, ΠΊΠ°ΠΊ Π½Π°ΠΊΠ»ΠΎΠ½ ΠΊΠ°ΡΠ°Ρ‚Π΅Π»ΡŒΠ½ΠΎΠΉ совпадаСт с ΠΏΡ€ΠΎΠΈΠ·Π²ΠΎΠ΄Π½ΠΎΠΉ. Π’Π°ΠΊΠΈΠΌ ΠΎΠ±Ρ€Π°Π·ΠΎΠΌ, ΠΌΡ‹ Ρ‚Π°ΠΊΠΆΠ΅ ΠΌΠΎΠΆΠ΅ΠΌ просто Π²Π·ΡΡ‚ΡŒ ΠΏΡ€ΠΎΠΈΠ·Π²ΠΎΠ΄Π½ΡƒΡŽ ΠΈ ΠΏΠΎΠ΄ΡΡ‚Π°Π²ΠΈΡ‚ΡŒ Π΅Π΅, Ρ‡Ρ‚ΠΎΠ±Ρ‹ Π½Π°ΠΉΡ‚ΠΈ Π½Π°ΠΊΠ»ΠΎΠ½ ΠΊΠ°ΡΠ°Ρ‚Π΅Π»ΡŒΠ½ΠΎΠΉ Π² Ρ‚ΠΎΡ‡ΠΊΠ΅ .

      Π“Ρ€Π°Ρ„ΠΈΠΊ Ρ„ΡƒΠ½ΠΊΡ†ΠΈΠΈ f(x) ΠΈ ΠΊΠ°ΡΠ°Ρ‚Π΅Π»ΡŒΠ½ΠΎΠΉ ΠΊ f(x) Π² Ρ‚ΠΎΡ‡ΠΊΠ°Ρ… (2, 4) — StudySmarter Original

      ΠŸΡ€ΠΈΠΌΠ΅Ρ€ 2

      Ρ‚ΠΎΡ‡ΠΊΠ° (1, 0).

      ΠžΠΏΡΡ‚ΡŒ ΠΆΠ΅, ΠΏΠΎΡΠΊΠΎΠ»ΡŒΠΊΡƒ Π½Π°ΠΌ Π΄Π°Π½Ρ‹ ΠΈ Ρ‚ΠΎΡ‡ΠΊΠ°, всС, Ρ‡Ρ‚ΠΎ Π½Π°ΠΌ Π½ΡƒΠΆΠ½ΠΎ, Ρ‡Ρ‚ΠΎΠ±Ρ‹ ΡΠΎΡΡ‚Π°Π²ΠΈΡ‚ΡŒ ΡƒΡ€Π°Π²Π½Π΅Π½ΠΈΠ΅ ΠΊΠ°ΡΠ°Ρ‚Π΅Π»ΡŒΠ½ΠΎΠΉ Π»ΠΈΠ½ΠΈΠΈ, это Π½Π°ΠΊΠ»ΠΎΠ½. Π§Ρ‚ΠΎΠ±Ρ‹ Π½Π°ΠΉΡ‚ΠΈ Π½Π°ΠΊΠ»ΠΎΠ½, Π²ΠΎΡΠΏΠΎΠ»ΡŒΠ·ΡƒΠ΅ΠΌΡΡ Π΄ΠΎΠΏΠΎΠ»Π½ΠΈΡ‚Π΅Π»ΡŒΠ½Ρ‹ΠΌ ΡƒΡ€Π°Π²Π½Π΅Π½ΠΈΠ΅ΠΌ ΠΊΠ°ΡΠ°Ρ‚Π΅Π»ΡŒΠ½ΠΎΠΉ.

      Π‘ , Π½Π°ΠΊΠ»ΠΎΠ½ ΠΊΠ°ΡΠ°Ρ‚Π΅Π»ΡŒΠ½ΠΎΠΉ Π² Ρ‚ΠΎΡ‡ΠΊΠ΅ (1, 0) Ρ€Π°Π²Π΅Π½:

      Π˜Ρ‚Π°ΠΊ, ΡƒΡ€Π°Π²Π½Π΅Π½ΠΈΠ΅ ΠΊΠ°ΡΠ°Ρ‚Π΅Π»ΡŒΠ½ΠΎΠΉ ΠΊ f(x) Π² Ρ‚ΠΎΡ‡ΠΊΠ΅ (1, 0) Ρ€Π°Π²Π½ΠΎ .

      ΠžΠΏΡΡ‚ΡŒ ΠΆΠ΅, вспомним, Ρ‡Ρ‚ΠΎ Π½Π°ΠΊΠ»ΠΎΠ½ ΠΊΠ°ΡΠ°Ρ‚Π΅Π»ΡŒΠ½ΠΎΠΉ совпадаСт с ΠΏΡ€ΠΎΠΈΠ·Π²ΠΎΠ΄Π½ΠΎΠΉ. Π’Π°ΠΊΠΈΠΌ ΠΎΠ±Ρ€Π°Π·ΠΎΠΌ, ΠΌΡ‹ Ρ‚Π°ΠΊΠΆΠ΅ ΠΌΠΎΠΆΠ΅ΠΌ просто Π²Π·ΡΡ‚ΡŒ ΠΏΡ€ΠΎΠΈΠ·Π²ΠΎΠ΄Π½ΡƒΡŽ ΠΈ ΠΏΠΎΠ΄ΡΡ‚Π°Π²ΠΈΡ‚ΡŒ 1, Ρ‡Ρ‚ΠΎΠ±Ρ‹ Π½Π°ΠΉΡ‚ΠΈ Π½Π°ΠΊΠ»ΠΎΠ½ ΠΊΠ°ΡΠ°Ρ‚Π΅Π»ΡŒΠ½ΠΎΠΉ Π² Ρ‚ΠΎΡ‡ΠΊΠ΅ .

      Π“Ρ€Π°Ρ„ΠΈΠΊ f(x) ΠΈ ΠΊΠ°ΡΠ°Ρ‚Π΅Π»ΡŒΠ½Π°Ρ ΠΊ f(x) Π² Ρ‚ΠΎΡ‡ΠΊΠ΅ (1, 0) — StudySmarter Original

      ΠšΠ°ΡΠ°Ρ‚Π΅Π»ΡŒΠ½Ρ‹Π΅ Π»ΠΈΠ½ΠΈΠΈ Π² окруТности

      Говорят, Ρ‡Ρ‚ΠΎ прямая касаСтся окруТности , Ссли ΠΎΠ½Π° касаСтся окруТности Ρ€ΠΎΠ²Π½ΠΎ Π² ΠΎΠ΄Π½ΠΎΠΉ Ρ‚ΠΎΡ‡ΠΊΠ΅. Если линия касаСтся окруТности Π² Ρ‚ΠΎΡ‡ΠΊΠ΅, Ρ‚ΠΎ ΠΊΠ°ΡΠ°Ρ‚Π΅Π»ΡŒΠ½Π°Ρ линия пСрпСндикулярна радиусу, ΠΏΡ€ΠΎΠ²Π΅Π΄Π΅Π½Π½ΠΎΠΌΡƒ ΠΊ Ρ‚ΠΎΡ‡ΠΊΠ΅.

    Π”ΠΎΠ±Π°Π²ΠΈΡ‚ΡŒ ΠΊΠΎΠΌΠΌΠ΅Π½Ρ‚Π°Ρ€ΠΈΠΉ

    Π’Π°Ρˆ адрСс email Π½Π΅ Π±ΡƒΠ΄Π΅Ρ‚ ΠΎΠΏΡƒΠ±Π»ΠΈΠΊΠΎΠ²Π°Π½. ΠžΠ±ΡΠ·Π°Ρ‚Π΅Π»ΡŒΠ½Ρ‹Π΅ поля ΠΏΠΎΠΌΠ΅Ρ‡Π΅Π½Ρ‹ *